#Proviamoci - Soluzioni commentate OH6

Informazioni sulle gare, come allenarsi, chi corrompere.
Denistusk
Messaggi: 1
Iscritto il: 17 dic 2018, 07:59

Re: #Proviamoci - Soluzioni commentate OH6

Messaggio da Denistusk »

Problema 1 [3]
$n$ il numero di commensali e $d$ la distanza tra loro, i commensali sono i punti ${A_1, A_2,..., A_n}$ mettiamo $A_1$ in un punto qualsiasi del piano. $A_2$ potrà stare ovunque nella circonferenza di raggio $d$ e centro $A_1$. Il terzo punto ha solo $2$ posizioni in cui poter stare, che sono le intersezioni delle $2$ circonferenze di centro $A_1$ e $A_2$, entrambe di raggio $d$. Posizionato il terzo punto e tracciata la terza circonferenza noteremo che non ci sarà nessun punto in comune a tutte e $3$ le circonferenze. Di conseguenza non ci può essere un quarto punto che rispetta le condizioni.
(Alessandro Avellino)

Problema 2 [6]
Si tratta di disporre tre diversi ingredienti, che si può fare semplicemente in $3!=6$ modi diversi. La risposta è quindi $6$.
(Matteo Salicandro)

Problema 3 [228]
Il massimo numero ottenibile lanciando gli $n$ dadi è $n\cdot k$, il minimo è $n\cdot 1=n$. Pertanto $n\cdot k-n=168$, cioè $n(k-1)=168$. Questo vuol dire che $n$ è un qualsiasi divisore positivo di $168$, esclusi $168$ e $84$ visto che $k \geq 4$. $168=2^3\cdot 3\cdot 7$, quindi la somma dei suoi divisori è $(2^4-1)(3+1)(7+1)=480$. La somma dei possibili valori di $n$ è pertanto $480-168-84=228$.
(Federico Borasio)

Problema 4

Problema 5

Problema 6 [36]
Un rettangolo con le diagonali perpendicolari è un quadrato. L'area è quindi $6^2=36$.
(Filippo Prandina)

Problema 7 [18]
La successione è definita, per $n \geq 1$, come
$\begin{cases}
x_{n+2}=x_{n+1}+x_n\\
x_6=76\\
x_5=47
\end{cases}$
Da qui ricaviamo che $x_n=x_{n+2}-x_{n+1}$, pertanto $x_4=76-49=29$ e $x_3=47-29=18$, che è il terzo termine della successione.
(Federico Borasio)

Problema 8 [2022]
Supponiamo che ci siano $2022$ fisici. Tutti dicono la verità, cioè che non sono matematici: non c'è alcuna contraddizione. $2022$ è anche il numero di abitanti dell'isola, quindi è il massimo numero di fisici.

Problema 9 [8]
$\frac{4n}{32}=k$ con $k\in N\geq 0$. Quindi $n=8k$. Il minimo si ha quando $k=1$ e $n=8$.
(Filippo Prandina)

Problema 10

Problema 11 [9999]
Ci sono più soluzioni in quanto i primi due termini possono essere $1,3$, oppure $2,2$ oppure $3,1$. La risposta è quindi $9999$.
(Matteo Salicandro)

Problema 12

Problema 13 [28]
$X$ appartiene alla perpendicolare di $BC$ passante per $A$. Inoltre $X$ e $2$ punti su $BC$ formano lo stesso angolo che forma $A$ con quei $2$ punti, di conseguenza $X$ è il simmetrico di $A$. Da questo deduciamo che $AX=2AD$, dove $AD$ è l'altezza.
(Alessandro Avellino)

Problema 14 [1011]
La quantità in questione, $(3n+2)^{5n+8}$, è un quadrato perfetto se lo è la base, cioè $3n+2$, oppure se l'esponente, $5n+8$, è pari. Tuttavia la prima possibilità non si verifica mai poiché un quadrato non è mai congruo a $2\,(\mathrm{mod}\,3)$. Dunque la risposta è semplicemente il numero di interi positivi $1\leq n\leq 2022$ tali che la quantità $5n+8$ sia pari, ovvero tutti e soli i numeri pari in questo intervallo, che sono $\frac{2022}{2}=1011$.
(Lorenzo Weiss)

Problema 15

Problema 16 [2019]
Il problema chiede per quanti interi positivi minori o uguali a $2022$ accade che $\lfloor \frac{\pi +n}{n}\rfloor =\lfloor \frac{\pi}{n}+1\rfloor =\lfloor \frac{\pi}{n}\rfloor +1=1\Longrightarrow \lfloor \frac{\pi}{n}\rfloor =0$, il che succede solo quando $\pi\approx 3,14\leq n$, i.e. in $2022-(4-1)=2019$ casi.
(Lorenzo Weiss)

Problema 17

Problema 18

Problema 19 [14]
Disegnando la circonferenza $x^2+y^2=5$ risulta evidente che gli unici punti a coordinate intere che vi appartengono, che sono $k=8$, sono quelli della forma $P_i(\pm 1,\pm 2)$, $P_i(\pm 2,\pm 1)$, $P_i(\pm 1,\mp 2)$, $P_i(\pm 2,\mp 1)$. In alternativa, è possibile risolvere l'equazione della stessa circonferenza sugli interi $x,y$. Da ciò si ricava l'ottagono $P_1P_2\dots P_8$, la cui area, come si vede facilmente, è $16-4\cdot\frac{1}{2}=14$.
(Lorenzo Weiss)

Problema 20

Problema 21 [210]
Notiamo che, comunque presi $6$ elementi distinti dall'insieme $X=\{1, 2, 3, 4, 5, 6, 7, 8, 9, 10\}$, c'è uno e un solo modo di disporli in ordine crescente, dunque la richiesta del problema è equivalente al trovare il numero di sottoinsiemi di $6$ elementi di $X$, che sono $\displaystyle\binom{10}{6}=210$.
(Valeria Martinelli)

Problema 22 [2022]
Si può ottenere $2$ solo quando una delle cifre è $2$ e le altre sono nulle, oppure quando due cifre sono uguali a $1$ e le altre sono nulle. Nel primo caso, la prima cifra sarà necessariamente $2$, altrimenti il numero non avrà $2022$ cifre: si ottiene solo una possibilità.
Nel secondo caso, invece, la prima cifra sarà $1$, mentre l'altro $1$ occuperà una delle altre $2021$ posizioni disponibili, ottenendo $2021$ possibilità.
Sommando le possibilità, si arriva a $2022$.
(Daniele Prisco)

Problema 23

Problema 24 [4042]
Se $x$ è la lunghezza del segmento $PA$ e $r$ è il raggio della circonferenza,
$OP=OA+PA=r+x=2021$; $PB=OA+OB+PA=r+r+x$. Di conseguenza, $PA+PB=x+r+r+x=2(r+x)=2×2021=4042$.
(Daniele Prisco)

Problema 25 [66]
Poniamo $a+b=x$ e $c+d=y$.
La disuguaglianza adesso è
$x+y\leq \sqrt{x^2+y^2} \Rightarrow x^2+2xy+y^2\leq x^2+y^2$
Da cui $2xy\leq0$ ma siccome $x$ e $y$ sono somme di interi maggiori o uguali a $0$, l'unico modo per cui $xy=0$ si ha quando uno dei $2$, o entrambi, è uguale a $0$. Se $c+d=y=0$ da cui $c=d=0$, quindi $(a,b,0,0)$. Ricordiamo che $a \geq b$ quindi i possibili valori sono $66$.
(Alessandro Avellino)

Problema 26 [6856]
Sia $n$ il numero di circonferenze sulla prima riga. Avendo le circonferenze diametro $2$, la base del rettangolo è uguale a $2n$. Invece, essendo tutte le circonferenze tangenti, i centri di quella più alta e delle due agli estremi della base sono disposte sui vertici di un triangolo equilatero, di lato $2n-2$. L'altezza del rettangolo è quindi uguale all'altezza del triangolo aumentata di $2$, ossia $\sqrt{3}(n-1)+2$.
Quindi vogliamo $4n+2\sqrt{3}(n-1)+4>2022$, che ci dà immediatamente $n=271$, quindi $k=\frac{271\cdot 272}{2}=36856$.
La risposta voluta sono le ultime quattro cifre di $k$.
(Valeria Martinelli)

Problema 27 [250]
Intanto notiamo che $78125$ è $5^7$.
La somma di $M$ termini consecutivi con primo termine $a$ è del tipo:
$a+(a+1)+(a+2)+...+(a+M-1)$ nel quale compaiono infatti $M$ termini consecutivi.
È quindi possibile raccogliere come $a \cdot M + (1+2+3+...+M-1)=a \cdot M+ \frac{M(M-1)}{2}=M(a+\frac{M-1}{2})=78125 \Rightarrow M(2a-1+M)=5^7\cdot2$, $M<2a-1+M$ quindi al massimo $M=2 \cdot 5^3$.
(Alessandro Avellino)

Problema 28

Problema 29

Problema 30

Problema 31 [20]
Il doppio delle cifre di $n$ dovrà necessariamente essere un numero di $3$ cifre ognuna delle quali minore di $5$. Una volta individuato il numero più piccolo $n$ come $101\,(2n=202)$ e il massimo $n$ come $444\,(2n=888)$ i restanti saranno compresi tra essi. Ci saranno $18$ numeri palindromi tra questi due estremi, con la cifra intermedia contente al più il $4$ e gli estremi uguali da $1$ a $4$. I numeri saranno $101$, $111$, $121$, $131$, $141$, $202$, $222$… fino a $444$. Il totale è di $20$ numeri bipalindromi.
(Irene Mancone)


Problema 32

Problema 33 [74]
Il costo di un pallone ($C$) può essere pari o dispari. Se è pari, può essere pagato con $\frac{C}{2}$ monete da $2$ orue. Poiché $\frac{C}{2}$ è pari, $C$ sarà un multiplo di $4$, quindi potrà assumere $37$ valori.
Se $C$ è dispari, invece, il pallone può essere pagato con $\frac{C-1}{2}$ monete da $2$ orue e una moneta da un orue. Di conseguenza, $C-1$ è un multiplo di $2$, ma $\frac{C-1}{2}$ è dispari (aggiungendo una moneta da un orue, il numero di monete usate è pari). Di conseguenza, $C-1$ è multiplo di $2$ ma non di $4$, quindi può assumere altri $37$ valori.
La somma dei valori possibili è dunque $74$.
(Daniele Prisco)

Problema 34 [2023]
Notiamo che possiamo riscrivere il testo nella seguente maniera: $P((x-y)(x+y))=P(x-y)P(x+y)$.
Notiamo anche che sostituendo $x-y \mapsto z$ e $x+y \mapsto w$ la coppia $(z,w)$ può assumere qualsiasi coppia di valori in $\mathbb{R}^2$ poiché il sistema
$\begin{cases}
x-y=z\\
x+y=w
\end{cases}$
ha soluzioni per ogni coppia $(z,w)$ fissata.
Dunque il nostro problema è equivalente a $P(zw)=P(z)P(y)$ per ogni $z$ e $w$ in $\mathbb{R}$. Questa relazione è molto simile ad una delle famose equazioni funzionali di Cauchy, $f(xy)=f(x)f(y)$. Quest'ultima equazione citata ha soluzione $f(x)=x^n$ o $f(x)=0$ (con $n$ naturale) se la funzione è continua (la continuità è una delle tante condizioni sufficienti), ma un polinomio come $P$ è continuo dunque le possibili soluzioni sono quelle riportate con $n$ che va da $0$ a $2021$ per $P(x)=x^n$ più $P(x)=0$. La soluzione è quindi $2023$ polinomi possibili.
(Lorenzo Bastioni)

Problema 35

Problema 36 [2020]
Si noti che $abc=2022-ab-a$, quindi per massimizzare il prodotto bisogna minimizzare $a$ e $b$. $a,b,c$ sono $\geq 1$ per ipotesi, dunque si pone $a=b=1$ dando come soluzione $c=2020$. Quindi $(a,b,c)=(1,1,2020)$ e $abc=2020$.
(Filippo Prandina)

Problema 37 [201]
Poniamo [math] e notiamo che [math] per [math]. Ora possiamo costruire il seguente polinomio: [math] di cui conosciamo gli zeri [math] e poiché [math] è chiaramente monico possiamo affermare che [math]. Sostituendo infine [math] con [math] e ricavando [math] otteniamo che [math] da cui possiamo trovare la soluzione al problema calcolando [math]. La risposta è quindi [math].
(Lorenzo Bastioni)

Problema 38

Problema 39

Problema 40

Problema 41

Problema 42

Problema 43 [4238]
Se si devono prendere almeno $n$ dolci per essere sicuri di prenderne almeno $1$ di un determinato tipo, questo vuol dire che ci sono $n-1$ dolci non di quel tipo. In particolare:
$\begin{cases}
S+B=55\\
B+C=70\\
S+C=91
\end{cases}$
Il sistema è di immediata risoluzione e porta come soluzione $(S,C,B)=(38,53,17)$ da cui $SCB=34238$. Sono richieste solo le ultime $4$ cifre.
(Filippo Prandina)

Problema 44 [4]
La configurazione è unica, infatti se prendiamo il punto $A_1$ e lo collegassimo con un punto diverso da $B_{2022}$, come per esempio $B_{2021}$, $B_{2022}$ sarà collegato con un punto diverso da $A_1$, per esempio $A_2$. Allora $A_1B_{2021}$ non interseca $A_2B_{2022}$. Iterando questo ragionamento ad ogni punto $A_i$, si ottiene un'unica configurazione: segmenti di estremi $A_iB_{2023-i}$. Quindi i coefficienti angolari saranno $\pm 2/(2023-2i)$. Per il prodotto, il numeratore è sempre $2$ mentre il denominatore è sempre dispari. Trovare quindi ciò che il problema richiede equivale a trovare le ultime $2$ cifre di $2^{2022}$.
(Alessandro Avellino)

Problema 45 [18]
Notiamo che, comunque presa una casella, il numero di giorni che Alberto o Barbara impiegano per raggiungerla non dipende dal percorso scelto, e le uniche caselle che hanno la stessa distanza sia da Alberto che da Barbara sono quelle sulla diagonale del quadrato diversa da quella di partenza, dunque i due si incontrano se e solo se entrambi passano per una di queste tre caselle.
Dato che un percorso è valido se e solo se fa $2$ passi in una direzione e $2$ nell'altra, tutte le coppie di percorsi possibili sono $\frac{4!}{2!\cdot 2!}\cdot \frac{4!}{2!\cdot 2!}=36$, quelli in cui entrambi passano per il centro sono $(2!\cdot 2!)(2!\cdot 2!)=16$, mentre quelli in cui entrambi passano in uno dei due angoli sono $2$.
Le coppie di percorsi con cui Alberto e Barbara non si incontrano sono quindi $36-16-2=18$.
(Valeria Martinelli)

Problema 46

Problema 47 [171]
Poiché $wxy+2021$ deve dare un numero pari, $wxy$ dev'essere dispari, e di conseguenza $w$,$x$ e $y$ devono esserlo.

$w=2a+1, x=2b+1, z=2c+1$ quindi $2a+1+2b+1+2c+1=37 \Rightarrow 2(a+b+c)=34 \Rightarrow a+b+c=17$
Adesso è un semplice stars and bars, infatti $a$, $b$ e $c$ possono essere anche nulli. Distribuire $17$ elementi in $3$ contenitori, $\binom{17+3-1}{3-1}=\binom{19}{2}$.
(Alessandro Avellino)

Problema 48 [42]
Notiamo che $CQB~APB$, in quanto hanno $2$ angoli congruenti, entrambi pari a $74$. Da questo consegue anche che sono triangoli isosceli. $\measuredangle CQB=\measuredangle APB=180-2(74)=32$. Inoltre $S$ sta nell'asse di $BC$, di conseguenza anche nella bisettrice di $CBQ$. Ragionamento analogo per $APB$. Ora osserviamo il quadrilatero $PCAQ$. Questo ha gli angoli $APB$ e $CQB$ congruenti, da questo possiamo asserire che $PCAQ$ è ciclico. Questo permette di stabilire che $\measuredangle DAC=\measuredangle PQC$ e che $\measuredangle DCA=\measuredangle QPD$. Infine osserviamo il triangolo $QSP$. $\measuredangle QSP=180-\measuredangle QPS-\measuredangle PQS= 180-(16+\measuredangle PQC)-(16+\measuredangle QPA)=148-(\measuredangle PQC+\measuredangle APQ)=148-(\measuredangle DCA+\measuredangle DAC)=148-(180-\measuredangle ADC)=\measuredangle ADC-32.$ $\measuredangle ADC=360-(\measuredangle DCB+\measuredangle CBA+\measuredangle BAD)=360-3 \cdot 74=138$ da cui $\measuredangle QSP=138-132=106$. Essendo infine $S$ il circocentro di $ABC$, ne consegue che $\measuredangle ASC=2 \measuredangle ABC=148$. Quindi $\measuredangle ASQ + \measuredangle PSC= \measuredangle ASC- \measuredangle QSP=148-106=42$.
(Alessandro Avellino)

Problema 49 [15]
Dovendo esistere almeno un numero $k>1$ che elevato al cubo sia un divisore di $n$ si possono escludere i numeri da $2$ a $7$ in quanto l’unico loro divisore al cubo è $1$ e $k$ non può assumere tale valore. L’$8$ si può vedere come $2^3$ ed è un prigioniero di cubi, così come lo sono tutti i suoi multipli fino a $96$. È valido lo stesso discorso anche per i multipli di $27$ (essendo $3^3$) fino a $81$. Contandoli si arriva al risultato richiesto, $15$.
(Irene Mancone)

Problema 50

Problema 51

Problema 52[1872]
Chiamiamo le tre radici di p(x) a,b,c dove a<b<c . Essendoa,b e c in progressione aritmetica si ha che a = b-r e c = b+r dove r è la ragione della progressione aritmetica. Per le formule di Vietè a+b+c= 39 b-r + b+ b+ r= 39 da cui si ricava che b =13 . Sempre applicando le formule di Vietè, ab + ac + bc = 482 sostituendo i risultati ricavati in precedenza si trova che 13(13-r) + (13-r)(13+r) + 13(13+r) = 482 da cui sviluppando le parentesi 169 -13r +169 -r2 + 169 +13r = 482 ottenendo che r2 = 25 quindi poiché r è maggiore di 0 si ha che r=5. Quindi a=8 c=18 .
Applicando nuovamente le formule di Vietè, -m = -8*13*18 ovvero m= 1872
(Mattia Zunino)

Problema 53 [24]
Poniamo l'età di Marco al momento della prima affermazione uguale a $x$, quindi l'età del padre Luca è $4x$. Allora, $4$ anni dopo l'età di Marco è $x+4$ e quella del padre è $4x+4$, quindi vale l'equazione $3(x+4)=4x+4$; $x=12-4=8$ e infine $4x=32$.
La differenza di età tra i $2$ è $24$, quindi per rispondere alla domanda risolviamo $2y=y+24$, con $y$ l'età di Marco e $y+24$ l'età di Luca, $y=24$ che è quindi la risposta.
(Niso Cicalò)

Problema 54

Problema 55

Problema 56

Problema 57 [183]
Si può considerare il triangolo isoscele con i vertici di coordinate $B (0;0), C (20;0)$ e A$ (10; y)$ con $y > 0$
Il punto $H$ è l’intersezione tra le rette perpendicolari ai lati passanti per $A, B, C$. Sapendo che le distanze tra
$AH$ e $AD$ sono segmenti di lunghezza intera il punto $H$ si potrà trovare al minimo nella coordinata $(10;1)$. Calcolando il coefficiente angolare tra $H$ e i punti $B$ e $C$, le rette passanti per $B$ e $C$ con coefficiente angolare antireciproco si può ottenere l’ordinata di $A$ sostituendo in una delle due equazioni.
$m_{BH}$= $\frac{1}{10}$ $m^1$=$-10$ retta r passante per A e C: $y=-10x+200$
$m_{CH}$=$\frac{-1}{10}$ $m^1$=10 retta s passante per B e A : $y=10x$

Per $H (10,1)$ $A$ vale $(10, 100)$ e la distanza è intera ($99$).
Una volta stabilita l’ordinata massima di $A$ è sufficiente ripetere il procedimento. Per $H (10,2)$ $A (10, 50)$, per $H (10,4)$ $A(10,25)$. Nelle coordinate $(10, 10)$ $H$ e $A$ si sovrappongono. Poi si avrà che $A$ si trova in $(10,20)$, in $(10,2)$ e $(10,5)$. La soluzione si ottiene sommando le distanze trovate tra $H$ e $A$ senza ripetizioni.
(Irene Mancone)

Soluzione alternativa:
Essendo $ABC$ un triangolo isoscele la mediana $AD$ coincide con l'altezza rispetto a $BC$, chiamando $K$ il piede dell'altezza relativa ad $AC$ otteniamo che il triangolo $BKC$ è simile al triangolo $ABD$ poiché $\widehat{BKC} \cong \widehat{BDA} = 90^{\circ}$ e $\widehat{BCK} \cong \widehat{ABD}$ (essendo $ABC$ isoscele). Consideriamo ora i triangoli $BKC$ e $BDH$, anch'essi sono simili poiché $\widehat{BKC} \cong \widehat{BDH} = 90^{\circ}$ e $\widehat{CBK}$ è in comune. Per la proprietà transitiva $ABD \sim BKC$ e $BKC \sim BDH$ $\rightarrow$ $ABD \sim BDH$. Avendo dimostrato che i triangoli $ABD$ e $BDH$ sono simili vale la seguente uguaglianza: $\frac{BD}{AD}=\frac{HD}{BD} \rightarrow BD^2 = AD \cdot HD \rightarrow 100=AD \cdot HD$ (essendo $BD$ la metà di un segmento lungo $20$). Avendo che $AH$ e $AD$ sono di lunghezza intera anche $HD$ sarà di lunghezza intera (Poiché $HD$ è la differenza tra i due segmenti) ed avremo soluzioni limitate a $100=AD \cdot HD$ ovvero $(AD,HD)=(100,1),(50,2),(25,4),(20,5),(10,10),(5,20),(4,25),(2,50),(1,100)$. La soluzione $(AD,HD)=(10,10)$ non ci piace poiché $\widehat{ABC}$ varrebbe $90^{\circ}$ ($D$ sarebbe il centro della circonferenza circoscritta ad $ABC$ e $\widehat{ABC}$ sottenderebbe un diametro) dunque i possibili valori di $AH$ sono le diverse differenze fra le soluzioni ottenute ($AH=|AD-AH|$): $AH = 100-1= 99$, $AH = 50-2 = 48$, $AH = 25-4= 21$, $AH =20-5=15$ (le altre danno gli stessi valori poiché sono soluzioni "specchiate"). La risposta è quindi $99+48+21+15=183$.
(Lorenzo Bastioni)

Problema 58 [46]
La somma $S$ dei quadrati di delle radici un polinomio del tipo $ x^n + \alpha_{n-1} \cdot x^{n-1} + ... + \alpha_1 \cdot x + \alpha_0$ è

$S=\alpha_{n-1}^2-2 \cdot \alpha_{n-2}$

$$Q_n(x)=x^n+\sum_{i=0}^{n-1} \sqrt{i} \cdot x^i$$

Per cui, definiamo $S_n$ come la somma dei quadrati delle radici di $Q_n(x)$. Allora $S_n=n-1-2\sqrt{n-2}$ che è razionale solo se anche $2\sqrt{n-2}$ lo è. Questo accade solo quando $n-2$ è un quadrato perfetto, quindi ci sono $45$ valori. A questo aggiungiamo $n=1$, il cui polinomio è quindi $x$, che ha come radici un numero razionale. Quindi la soluzione è $46$.
(Alessandro Avellino)

Problema 59

Problema 60

Problema 61[9098]
Per il criterio di divisibilità per 9, s(n) ≡ n mod 9 dove s(n) è la somma delle cifre di n. Iterando il ragionamento su s(n) si ottiene che A(n)≡n mod9 . Poiché A(n) è un intero compreso fra 0 e 9, A(n) è uguale al resto della divisione di n per 9. I possibili resti della divisione delle potenze di 2021 per 9 sono 5, 7, 8, 4, 2, 1 e poi si ripetono ciclicamente con periodo 6. Perciò la somma richiesta è 2016/6 *(5+7+8+4+2+1) +5+7+8+4+2 = 9098.
(Mattia Zunino)
Problema 62 [1158]
Dette $H$ e $K$, rispettivamente, le proiezioni di $D$ e $C$ su $AB$ e $AH=x$ possiamo calcolare l'altezza del trapezio in due modi diversi servendoci del teorema di Pitagora ed eguagliare le espressioni $DH^2=AD^2-AH^2=33^2-x^2=CK^2=BC^2-BK^2=60^2-(69-6-x)^2$.

Risolvendo l'equazione, si trova l'unico valore $x=\frac{81}{7}$ e l'altezza si può quindi calcolare, ad esempio, come $h=DH=\sqrt{AD^2-AH^2}=\frac{60\sqrt{13}}{7}$. L'area del trapezio vale $A=\frac{(b_1+b_2)\cdot h}{2}=\frac{(AB+CD)\cdot AH}{2}=\frac{(69+6)\cdot (60\sqrt13)}{7\cdot2}\approx 1158,93$; la sua parte intera è $1158$ ed è la soluzione.
(Federico Magnolfi)

Problema 63

Problema 64

Problema 65 [5877]
$X$ si trova sulla superficie del triangolo $AMC$: infatti, se così non fosse, $\widehat{ABC}$ e $\widehat{AXM}$ sarebbero due angoli congruenti che insistono su uno stesso segmento ($AM$) e che appartengono allo stesso semipiano rispetto ad esso: ma ciò non è possibile in quanto significherebbe che $X$ appartiene alla circonferenza circoscritta al trangolo $ABM$, alla quale ovviamente non appartengono punti interni al triangolo $ABM$.

$AXMY$ è ciclico per ipotesi e $\widehat{AYM}$ e $\widehat{AXM}$ sono supplementari in quanto angoli opposti (per quanto prima dimostrato); dunque $\widehat{BYM}$, essendo anch'esso supplementare di $\widehat{AYM}$, è congruente a $\widehat{AXM}$, il quale a sua volta è congruente a $\widehat{ABC}$ per ipotesi.
Avremo allora che il triangolo $MBY$ è isoscele su base $BY$; essendo $M$ punto medio di $BC$, $MB\cong MY\cong MC$ e ciò implica che $BYC$ è rettangolo in $Y$. $CY$ è dunque l'altezza uscente da $C$ e si può calcolare con la formula inversa $CY=\frac{2A}{AB}$ dopo aver ricavato l'area con la formula di Erone o per via trigonometrica.
Si trova che quest'ultima vale $A=1200\sqrt{5}$ e dunque $CY^2=\left(\frac{2A}{AB}\right)^2=\frac{4\cdot1200^2\cdot 5}{70^2}\approx 5877,55$. La sua parte intera è $5877$ ed è la soluzione.
(Federico Magnolfi)

Problema 66 [1152]
Prendiamo l'equazione del problema e consideriamola $(\text{mod}\,7)$: $x^2 \equiv 7y + z \,(\text{mod}\,7) \Rightarrow x^2 \equiv z \, (\text{mod}\,7)$. I quadrati come $x^2$ modulo $7$ possono assumere come valori solo $0,1,2,4$ quindi $z$ potrà assumere tutti e i soli valori $0,1,2,4$ modulo $7$. Nell'intervallo $[1,2022]$ ci sono esattamente $288$ numeri congrui a $0 \, (mod 7)$, $288$ congrui a $1$, $288$ congrui a $2$ e $288$ congrui a $4$. La risposta è quindi $288 \cdot 4=1152$.
(Lorenzo Bastioni)

Problema 67 [3026]
Scriviamo il numeratore come $\prod_{n=2}^{2021}((n-1)+n+(n+1))=\prod_{n=2}^{2021}(3n)=3^{2021}\prod_{n=2}^{2021}(n)$. Si può quindi raccogliere un $3^{2021}$ al numeratore. Inoltre si raccolgono anche una potenza di $3^{1005}$ dato che $n$ può avere $3$ come fattore primo (elevato alla $1,\,2,\,3,\,\dots,6$). Ci sono $ \Big \lfloor \frac{2021}{3} \Big \rfloor=673$ multipli di 3, $ \Big \lfloor \frac{2021}{9} \Big \rfloor =224$ multipli di $9$ e così via: $ \Big \lfloor \frac{2021}{3} \Big \rfloor+ \Big \lfloor \frac{2021}{9} \Big \rfloor + \Big \lfloor \frac{2021}{27} \Big \rfloor + \Big \lfloor \frac{2021}{81} \Big \rfloor + \Big \lfloor \frac{2021}{243} \Big \rfloor + \Big \lfloor \frac{2021}{729} \Big \rfloor =673+224+74+24+8+2=1005$. In tutto quindi si è raccolto $2021+1005=3026$.
(Filippo Prandina)

Problema 68 [1104]
Chiamiamo due insiemi $A$ e $B$ "gemelli di primo grado" se $|A \cap B|=1$ e $1 \in A,B$, in generale chiamiamo due insiemi $A$ e $B$ "gemelli di $n$-esimo grado" se $|A \cap B|=n$ e $n \in A,B$.
Possiamo ora affrontare il conteggio di tutte le coppie di sottoinsiemi gemelle di $\{1,2,3,...,2021,2022 \}$ considerando prima i gemelli di primo grado, di secondo grado fino a quelli di $2022$-esimo grado (Non ci sono coppie di sottoinsiemi di $2023$-esimo grado o di $0$-esimo grado).
Gemelli di primo grado:
se $A$ e $B$ sono di primo grado allora contengono entrambi l'elemento $1$ e non hanno nessun altro elemento in comune. Ci verra comodo per dopo scrivere che gli altri zero elementi in comune si possono scegliere in $\displaystyle \binom{2021}{0}$ modi. Ora tutti gli altri $2021$ elementi possono essere messi o in $A$ o in $B$ o in nessuno dei due (in questo caso la "o" è una o esclusiva), quindi abbiamo 3 possibilità per ognuno dei $2021$ elementi ovvero $3^{2021}$ possibilità. Avremo quindi $\displaystyle \binom{2021}{0} 3^{2021}$ coppie di sottoinsiemi gemelle di primo grado.
In generale siamo pronti ad affrontare il caso di gemelli di $n$-esimo grado:
se $A$ e $B$ sono gemelli di $n$-esimo grado allora contengono entrambi l'elemento $n$ e hanno altri $n-1$ elementi in comune. Gli altri $n-1$ elementi in comune si possono scegliere in $\displaystyle \binom{2021}{n-1}$ modi. E per i restanti $2021-(n-1)=2022-n$ elementi abbiamo $3^{2022-n}$ possibilità come spiegato precedentemente. Avremo quindi $\displaystyle \binom{2021}{n-1} 3^{2022-n}$ coppie di sottoinsiemi gemelle di $n$-esimo grado.
Sommando tutte le possibilità otteniamo:
$$\sum_{k=1}^{2022} \binom{2021}{k-1} 3^{2022-k}$$
Reimpostando la somma sostituendo $k-1 \mapsto k$ otteniamo:
$$\sum_{k=0}^{2021} \binom{2021}{k} 3^{2021-k}$$
Ma questo è esattamente uguale a $(3+1)^{2021}=4^{2021}$ (Per il binomio di Newton o teorema binomiale). Riducendo l'enorme numero $(\text{mod}\,10000)$ si trova che le ultime $4$ cifre che ci interessano sono $1104$, da cui la risposta.
(Lorenzo Bastioni)

Problema 69

Problema 70

Problema 71

Problema 72[247]
Disponiamo le 32 squadre in qualunque ordine, e costruiamo gli incontri nel seguente modo: ad ogni turno, le squadre in un posto dispari si scontrano con la successiva. Le squadre perdenti vengono eliminate. Disponiamo le squadre restanti nello stesso ordine in cui si trovano, e ripetiamo finché resta solo una squadra. Ad esempio, se ad un certo punto l'ordine è $(3, 5, 2, 1)$, la squadra $3$ vince contro $5$ e $1$ vince contro $2$, dunque al turno dopo l'ordine sarà $(3, 1)$. Poniamo per semplicità la squadra $S_3$ per prima, e consideriamo i restanti $31!$ ordinamenti possibili. La squadra $S_3$ arriva in semifinale se e solo se non viene eliminata nei 3 turni precedenti, e ciò accade se e solo se nessuna tra le squadre $S_1$ ed $S_2$ è presente tra le prime 8 squadre. Possiamo quindi scegliere le prime 8, inclusa $S_3$, in $\displaystyle\binom{29}{7}$ modi, e le restanti in $24!$ modi.
La probabilità cercata è quindi $\displaystyle\binom{29}{7}\cdot 24!\cdot \frac{1}{31!}=\frac{29 \cdot 28 \cdot 27 \cdot 26 \cdot 25 \cdot 24 \cdot 23}{31 \cdot 30 \cdot 29 \cdot 28 \cdot 27}=\frac{24 \cdot 23}{31 \cdot 30}=\frac{92}{155}$. La risposta è quindi $92+155=247$.
(Valeria Martinelli)

Problema 73 [6068]
Notiamo che
$(p+q+r)(pq+qr+pr)=p^2q+p^2r+pq^2+pr^2+q^2r+qr^2+3pqr=1+3\cdot 2022=6067$.
Dato che $p+q+r$ e $pq+qr+pr$ sono entrambi interi positivi, devono necessariamente dividere entrambi $6067$. $6067$ è un numero primo, quindi o $p+q+r=1$ o $p+q+r=6067$. La risposta è perciò $6067+1=6068$.
Si noti che non ci si deve preoccupare di trovare che esistano $p,q,r$ che rispettano le condizioni trovate, dato che questo è assicurato dal fatto che sono le tre soluzioni dell'equazione $x^3-(p+q+r)x^2+(pq+qr+pr)x-pqr=0$, che ha sempre tre soluzioni nei numeri complessi.
(Federico Borasio)

Problema 74

Problema 75 [78]
Elevando entrambi i termini al quadrato (lecito perché entrambi sicuramente positivi) si ottiene $x^2+y^2-x-y+1/2 \leq 1/4$ riarrangiando $(x-1/2)^2+(y-1/2)^2 \leq 1/4$. Si ha quindi l'equazione di un cerchio centrato in $(1/2,1/2)$ con raggio $1/2$. La probabilità che un punto interno al quadrato definito nel piano cartesiano che ha vertice nell'origine e lato $1$ sia dentro il cerchio è quindi il rapporto tra l'area del cerchio e quella del quadrato: $\pi (1/2)^2 \approx 78%$.
(Filippo Prandina)

Problema 76 [4253]
Iniziamo col dimostrare che $P$ si trova all'esterno di $ABC$.

Detto $K$ il piede dell'altezza uscente da $A$ si ha che, essendo $\widehat{APB}$ congruente a $\widehat{AKB}$ perché entrambi retti, e dunque $AKBP$ ciclico, $P$ appartiene alla circonferenza circoscritta a $AKB$ alla quale non appartengono punti interni a $AKB$; ma $P$ appartiene alla mediana $AM$, la quale è evidentemente contenuta nel triangolo $AKB$ (lo si potrebbe dimostrare con i dati del problema), e quindi $P$ si trova all'esterno di $AKB$ ma anche di $ABC$.

Ora, è noto che in un triangolo di lati di lunghezza $9k$, $8k$, $7k$, con $k$ intero positivo, la mediana relativa alla base di lunghezza $8k$ misura $7k$: questo è facilmente verificabile per via trigonometrica, col teorema della mediana o anche con Pitagora; in ogni caso, $AM\cong AC$ e in particolare $\widehat{AMC}\cong \widehat{ACM}=\gamma$.

Sia $\left\{D\right\}=BC\cap PT$. $BTD\sim BCH$ per il secondo criterio di similitudine in quanto hanno l'angolo in $B$ in comune e $\widehat{BDT}\cong \widehat{BHC}$ perché retti, in particolare $\widehat{ATD}=\gamma$.

$\widehat{BMP}\cong\widehat{AMC}$ perché opposi al vertice ed essendo $BPM$ rettangolo in $P$ e $PD$ l'altezza relativa all'ipotenusa, $BPD=\gamma$. Allora $BPT$ è isoscele sulla base $PT$, in altre parole $T$ è il simmetrico di $P$ rispetto a $BC$; da ciò deriva anche che $BMT\cong BMP$, in particolare che $\widehat{BTM}$ sia retto.

$BMT\sim BCH$ per il secondo criterio di similitudine in quanto hanno l'angolo in $B$ in comune e $\widehat{BTM}\cong \widehat{BHC}$ perché entrambi retti. Ma allora, essendo $M$ punto medio di $BC$, necessariamente $T$ è punto medio di $BH$.

Il rapporto $\frac{[ATB]}{[THC]}$ è uguale al rapporto fra le altezze $\frac{AH}{CH}$ in quanto i due triangoli hanno basi congruenti per quanto appena dimostrato. Esse si possono calcolare con pitagora, dopo aver ricavato $BH=\frac{2A}{AC}$ e l'area attraverso Erone o per via trigonometrica. Sviluppando i calcoli si trova $BH=\frac{24\sqrt{5}}{2}$, $CH=\sqrt{BC^2-BH^2}=\frac{16}{7}$ e $AH=AC-CH=\frac{33}{7}$
Il risultato cercato è $1000\cdot \left(\frac{AH}{CH}\right)^2=1000\cdot\left(\frac{33}{16}\right)^2\approx4253,90$ la cui parte intera è $4253$ che è la soluzione.
(Federico Magnolfi)

Problema 77

Problema 78 [4151]
Cominciamo col dimostrare che $BCPQ$ è un trapezio. Sappiamo che l'ortocentro e il circocentro sono coniugati isogonali: in pratica, le rette $AH$ e $AO$ formano coi lati $AB$ e $AC$ angoli congruenti. Avremo dunque $\widehat{BAH}\cong \widehat{CAO}$, ma anche $\widehat{HAI}\cong \widehat{OAI}$ essendo $AI$ bisettrice perché $I$ incentro di $ABC$.

Riferendoci ora alla circonferenza circoscritta ad $ABC$, che chiamiamo $\Gamma$, abbiamo che $\widehat{FCE}\cong \widehat{FAE}\cong\widehat{EAD}\cong \widehat{ECD}$ dove la prima e la terza uguaglianza sono date dal fatto che gli angoli insistono su uno stesso arco. Facendo attenzione, notiamo che $\widehat{QAP}$ e $\widehat{QCP}$ sono congruenti per l'uguaglianza precedente e insistono sullo stesso segmento $QP$, dunque $AQPC$ è ciclico e chiamiamo $\omega$ la sua circonferenza circoscritta. Essendo $\widehat{ACD}$ retto poiché insiste sul diametro $AD$ di $\Gamma$, $\widehat{AQP}$ sarà anch'esso retto poiché supplementare dell'angolo opposto $\widehat{ACP}$ (ricordando che ora abbiamo fatto rifereimento a $\omega$). $BC$ e $QP$ sono allora due rette perpendicolari alla stessa retta $AQ$, perciò sono parallele e $BCPQ$ è un trapezio.

Per calcolare la sua area abbiamo bisogno dell'altezza $QH$ e della base minore $QP$, dato che la base maggiore ci è nota ed è $BC$.

$\textbf{Prima soluzione (sintetica)}$
Detti $K$ e $J$ rispettivamente i piedi dell'altezza $AH$ e della bisettrice $AI$, analizziamo i triangoli $ABJ$ e $AEC$: essi hanno $\widehat{ABJ}\cong \widehat{AEC}$ perché angoli alla circonferenza che insistono su uno stesso arco; $\widehat{BAJ}\cong \widehat{EAC}$ perché $AI$ bisettrice: quindi sono simili per il secondo criterio di similitudine. Considerando ora i triangoli $AKJ$ e $AQP$: essi hanno: l'angolo in $A$ in comune; $\widehat{AKJ}\cong \widehat{AQP}$ perché retti: quindi sono anch'essi simili per il secondo criterio di similitudine. Impostiamo la proporzione $\frac{QP}{KJ}=\frac{AP}{AJ}$ $(1)$: troveremo $KJ$ e $AJ$ lavorando su $ABC$, mentre $AP$ attraverso il rapporto di similitudine fra i triangoli $ABJ$ e $AEC$. Calcoliamo $AK$ invertendo la formula dell'area di $ABC$ e $BK$ col teorema di Pitagora: $AK=\frac{2A}{BC}=\frac{2\cdot \sqrt{p(p-AB)(p-BC)(p-AC)}}{BC}=120 \quad$ $BK=\sqrt{AB^2-AK^2}=50$.
Applichiamo ora il teorema della bisettrice per trovare $BJ$: $\frac{BJ}{AB}=\frac{BC-BJ}{AC}$ da cui si ricava che $BJ=65$. Dunque $KJ=BJ-BK=15$ e $AJ=\sqrt{AK^2+KJ^2}=15\sqrt{65}$.

Impostiamo il rapporto di similitudine fra $ABJ$ e $AEC$: $\frac{AP}{AB}=\frac{AJ}{AC}$ da cui si ricava che $AP=\frac{150\sqrt{65}}{7}$ e per la proporzione $(1)$ $QP=\frac{AP\cdot KJ}{AJ}=\frac{150}{7}$. A questo punto si trova $AQ$ con pitagora e $QK$ per differenza: $AQ=\sqrt{AP^2-QP^2}=\frac{1200}{7} \quad QK=AQ-AK=\frac{360}{7}$
L'area del trapezio $BCPQ$ è dunque $$A=\frac{(140+\frac{150}{7})\cdot\frac{360}{7}}{2}\approx 4151,02$$.

$\textbf{Seconda soluzione (trigonometria)}$
Ponendo $\widehat{ABC}=\beta$, $\cos\beta=\frac{AB^2+BC^2-AC^2}{2AB\cdot BC}=\frac{5}{13}$ (teorema del coseno), $\sin\beta=\sqrt{1-\cos^2\beta}=\frac{12}{13}$ $\widehat{BAC}=\alpha$, $\widehat{CAI}=\frac{\alpha}{2}$ perché $AI$ bisettrice, $\cos\alpha=\frac{AB^2+AC^2-BC^2}{2AB\cdot AC}=\frac{33}{65}$ (teorema del coseno), $\cos\frac{\alpha}{2}=\sqrt{\frac{1+\cos\alpha}{2}}=\frac{7}{\sqrt{65}}$.

Adesso abbiamo abbastanza dati per calcolare $AP=\frac{AC}{\cos\frac{\alpha}{2}}=\frac{150\sqrt{65}}{7}$.
Ponendo $\widehat{HAI}=\widehat{OAI}=\theta$ si ha che, essendo $\widehat{BAH}=\frac{\pi}{2}-\beta$, $\theta=\frac{\alpha}{2}+\beta-\frac{\pi}{2}$; $\cos\theta=\sin\frac{\alpha}{2}\cos\beta+\cos\frac{\alpha}{2}\sin\beta=\frac{8}{\sqrt{65}}$

$AQ=AP\cos\theta=\frac{1200}{7}\quad QP=\sqrt{AP^2-AQ^2}=\frac{150}{7}$. Ci manca l'altezza $QK$ dove $K$ è il piede dell'altezza uscente da $A$. $AK=AB\sin\beta=120\qquad QK=AQ-AK=\frac{360}{7}$. A questo punto l'area si trova allo stesso modo della soluzione per via sintetica.
(Federico Magnolfi)

Problema 79 [2011]
Si ha che $p^{(2)}(x)$ ha lo stesso grado di $p^{(4)}(x)$ questo significa che $p(x)$ è di grado $1$ o grado $0$. Ma non può essere di grado $0$ perché non è costante. Allora è del tipo $p(x)=mx+q$. La funzione $p(x)$ è chiaramente iniettiva ed essendo $p^{(2)}(x)=p^{(2022)}(x)$ per ipotesi, si ha necessariamente che $p(x)=p^{(2021)}(x)$. Ma quindi $p(2021)=0$ e $p(p(2021))=p(0)=2021$. Da qui si ricava facilmente che il polinomio è $p(x)=-x+2021$ e quindi $p(10)=2011$.
(Filippo Prandina)

Problema 80[2025]
Innanzitutto notiamo che $f(n)\le 2n+9$ $\forall n \in \mathbb{N}$, poiché altrimenti l'equazione del testo non sarebbe verificata. Per cui, la funzione cercata ha sicuramente un grado minore di $2n+9$, in quanto altrimenti crescerebbe più velocemente. Inoltre, $f(n)$ non può ovviamente essere costante, poiché $2n+9$ non lo è. Poniamo quindi $f(n)=mn+q$, e notiamo che, se $|m|>1$, $f(f(n))>4n>2n+9$ per n sufficientemente grande. Per cui $m=1$ o $m=-1$, ma dato che la funzione non può assumere valori negativi solo $m=1$ è accettabile (altrimenti, qualunque sia la scelta di q, basterebbe porre n=q+1 per ottenere un valore negativo).

Per cui abbiamo che $f(n+q)+n+q=2n+9$, da cui $n+2q+n+q=2n+9$, e quindi $3q=9$ e $q=3$.

L'unica funzione possibile è dunque $f(n)=n+3$, che ci dà $f(2022)=2025$
(Valeria Martinelli)

Problema 81

Problema 82 [4]
$45=9\cdot 5$ quindi scomponiamo il modulo in modulo $9$ e modulo $5$. Modulo $5$ è semplicemente $4$, in quanto l'ultima cifra è proprio $4$. Definiamo $\overline{a_1a_2a_3...a_k}$ la scrittura decimale in successione di $a_1,a_2,...,a_k$, per esempio $\overline{4^2 5^2}=1625$. Inoltre, chiamato $N$ il nostro numero, $N=\overline{1^22^23^24^2...2022^2}$. $\overline{a_1a_2a_3...a_k}\equiv a_1+a_2+a_3+...+a_k\,(\mathrm{mod}\,9)$, dimostrarlo è facile. I residui quadratici di $9$ sono $0,1,4,7$ mentre la somma dei quadrati da $1$ a $9\,\mathrm{mod}\,9$ è $24$, ($-3$). Quindi ogni $9$ numeri vi è un $-3$ da addizionare, quindi è $(-3)\cdot 224$ (che sarebbe il numero di multipli di $9$ minori di $2022$). $-672$, a questo devi aggiungere i numeri da $2017$ a $2022$, $-672+1+4+7+7=-653\equiv -14\equiv 4$.
Siccome $N$ è congruo a $4$ sia $\mathrm{mod}\,5$ che $\mathrm{mod}\,9$, sarà congruo a $4$ anche $\mathrm{mod}\,45$.
(Alessandro Avellino)

Problema 83 [21]
Sia $H$ il piede dell'altezza uscente da $A$; $AY=AH-HY$. L'altezza $AH$ si può calcolare, ad esempio, invertendo la formula dell'area $A=\frac{b\cdot h}{2}$ dopo aver trovato quest'ultima con la formula di Erone, oppure trovando il coseno dell'angolo in $B$ col teorema del coseno, quindi il suo seno e applicando il primo teorema dei triangoli rettangoli a $ABH$. Eseguendo i calcoli, si trova che essa vale $AH=3\sqrt{5}$. $BH=\sqrt{AB^2-AH^2}=2$.

Detto $M$ il punto medio di$BC$ si ha che $HM=HX=2$ e quindi esso è punto medio anche di $HX$; ma $OM\perp BC$ in quanto in un triangolo isoscele la mediana è anche altezza: allora, essendo $HYX$ e $MOX$ simili per il secondo criterio di similitudine, e avendo rapporto di similitudine $2$, $HY=2HO$

Attraverso il teorema della corda, oppure attraverso la formula nota, possiamo calcolare il raggio della circonferenza circoscritta ad $ABC$, $R=\frac{BC}{2\sin\alpha}=\frac{abc}{4A}=\frac{21}{2\sqrt{5}}$.
Ora possiamo trovare $HY=2HO=2\sqrt{R^2-MC^2}=\frac{11\sqrt{5}}{5}$ e infine $AY=AH-HY=3\sqrt{5}-\frac{11\sqrt{5}}{5}=\frac{4\sqrt{5}}{5}$.

Stiamo cercando $AY^2=\frac{16}{5}$ e la risposta è infine $16+5=21$.
(Alessandro Lombardo & Federico Magnolfi)

Problema 84

Problema 85 [1136]

Iniziamo col calcolarci alcuni oggetti che in futuro potranno esserci utili.

Perimetro: 2p=14+16+18=48

l'Area: usando la formula di Erone, A=$\sqrt{p(p-a)(p-b)(p-c)}=\sqrt{24\cdot10\cdot8\cdot6}=48\sqrt{5}$

Raggio circonferenza circoscritta: $R=\frac{abc}{4A}=\frac{4032}{192\sqrt{5}}=\frac{21}{\sqrt{5}}=\frac{21\sqrt{5}}{5}$

Altezza relativa a BC: $AD=2A/BC=6\sqrt{5}$

Altezza relativa a AB, che incontra AB in F: $CF=2A/AB=\frac{48\sqrt{5}}{7}$

Detto L il punto di incontro tra BC e la perpendicolare passante per O, L è il punto medio di BC. Per cui CL=8, OC= R, $OL=\sqrt{\frac{441}{5}-\frac{320}{5}}=\sqrt{\frac{121}{5}}=\frac{11\sqrt{5}}{5}$

$CD=\sqrt{AC^2-AD^2}=\sqrt{144}=12$

$AF=\sqrt{AC^2-CF^2}=\frac{66}{7}$

LD=CD-CL=12-8=4

adesso calcoliamo OD:

$OD=\sqrt{OL^2+LD^2}=\sqrt{\frac{201}{5}}$

Infine calcoliamo AH, dove H è l'ortocentro. per calcolare AH notiamo che, AHF è simile a ABD, per cui $\frac{AH}{AB}=\frac{AF}{AD} \Rightarrow AH=\frac{AF \cdot AB}{AD}=\frac{66 \cdot 14}{42\sqrt{5}}=\frac{22\sqrt{5}}{5}$.

Adesso che abbiamo tutto ciò che ci serve proseguiamo. O è il punto di Miquel relativo ad XYD. questo vuol dire che O appartiene alla circonferenza passante per X,Y,A. Detto questo, abbiamo 2 quadrilateri ciclici, per definizione, BDOX e CDOY. A questi 2 aggiungiamo il quadrilatero AYOX, anch'esso ciclico.

1° Claim: XYD è simile ad ABC.
è sufficiente un po' di angle chasing:

ricordiamo che OA, OB e OC sono i raggi di (ABC).
Di conseguenza OBA=OAB, OBC=OCB e OCA=OAC. Inoltre, per via della ciclicità dei 3 quadrilateri sopra elencati,
OAB=OBA=OBX=ODX, OAC=OCA=OCY=ODY, da questo segue che YDX=ODY+ODX=CAO+OAB=BAC.

Analogamente DYX=CBA e DXY=BCA. Per cui i 2 triangoli sono simili

2° Claim: O è l'ortocentro di XYD.
Anche qui è sufficiente un po' di Angle chasing:

DYX+YDO=(DYO+OYX)+OCY=DCO+OAX+OCA=(DCO+OCA)+OAX=ACB+OAX=ACB+(OAH+HAX)=ACB+(OAH+HAB)=
ACB+(OAH+DAB)=ACB+(OAH+OAC)=ACB+DAC=90.
Ne consegue che OD è perpendicolare ad XY. Analogamente si ottiene che , XO e YO sono perpendicolari, rispettivamente, a DY e DX. Per cui O è l'ortocentro.

Di conseguenza, detto K il rapporto di similitudine tra ABC e XYD, $K=\frac{AH}{OD}=\frac{22}{201}$, il rapporto vale anche per i raggi r e R rispettivamente di XYD, e di ABC, $K=\frac{R}{r} \Rightarrow r=\frac{R}{K}=\frac{21\sqrt{1005}}{110}$
(Alessandro Avellino)

Problema 86 [1483]
Per prima cosa notiamo che, dato che $\frac{x_1+x_2}{x_3+x_4}$ è intero, abbiamo che $x_1+x_2 \ge x_3+x_4$, e similmente $x_3+x_4 \ge x_5+x_6 \ge x_1+x_2$.
Dunque $x_1+x_2= x_3+x_4=x_5+x_6$. Analogamente $x_2+x_3=x_4+x_5=x_6+x_1$.

Sia ora $S=x_1+x_2= x_3+x_4=x_5+x_6$ e $T=x_2+x_3= x_4+x_5=x_6+x_1$.
Abbiamo quindi che $\sum x_i = 3S=3T$, quindi $S=T$, ossia $x_1+x_2=x_2+x_3$, da cui $x_1=x_3$. In modo analogo otteniamo $x_1=x_3=x_5$ e $x_2=x_4=x_6$.

Possiamo quindi scrivere la sestupla come $(a, b, a, b, a, b)$, con $a, b$ interi positivi. Si verifica facilmente che ogni sestupla siffatta rispetta le condizioni del testo, difatti la somma di due termini consecutivi è costante, dunque le frazioni sono tutte uguali a $1$ e quindi intere.
Dunque, fissati $a, b$, c'è una e una sola sestupla valida.

Consideriamo ora due casi:

$\bullet$ $M$ non è multiplo di $3$. Allora, dato che $M=3S$, non ci sono sestuple valide, e quindi $f(M)=0$, che è un multiplo di $5$. In questo caso abbiamo quindi $2022-\frac{2022}{3}=1348$ possibili valori di $M$.

$\bullet$ $M=3k$, per qualche $k$ intero positivo. Dato che $k=a+b$, per ogni valore di $k$, ci sono $k-1$ sestuple valide, poiché abbiamo $k-1$ possibili valori di $a$, dunque $f(M)=k-1$. Vogliamo quindi che $k-1$ sia multiplo di $5$. Se $k=1$, non ci sono sestuple valide in quanto $a+b>1$, altrimenti, potendo scegliere $k$ nell'intervallo $[2, 1348]$, ci sono ${\lfloor \frac{1347}{5} \rfloor}=134$ valori accettabili.

In totale abbiamo quindi $1348+1+134=1483$ possibili valori di $M$.
(Valeria Martinelli)

Problema 87

Problema 88 [6418]
Osserviamo che, prese $3$ cifre distinte $a, b, c$ dall'insieme $X=\{1, 2, 3, 4, 5, 6, 7, 8, ,9 \}$, ci sono esattamente $6$ numeri compresi tra $100$ e $999$ che si scrivono con tutte e sole le cifre $a, b, c$, siano questi $x_1, ..., x_6$. Abbiamo che la loro somma $x_1+...+x_6$ vale $222a+222b+222c$, quindi $\sum f(x_i)=\frac{222(a+b+c)}{a+b+c}=222$.
Ci sono $\displaystyle\binom{9}{3}=84$ modi di scegliere $a, b, c$, quindi la somma in questo caso vale $222\cdot 84=18648$

Dobbiamo ora distinguere i casi in cui $a, b, c$ contengono uno o più zeri o cifre ripetute.
$\bullet$ $a=b=c$: per ognuna delle $9$ possibili scelte di $a, b, c$, c'è uno e un solo numero $x=111a$, e $f(x)=\frac{111a}{3a}=37$. Dunque la somma in questo caso vale $37 \cdot 9=333$

$\bullet$ $a=b \neq c$, $a, b, c \neq 0$: abbiamo $9\cdot 8=72$ possibili scelte di $a, b, c$, e per ogni terna abbiamo $3$ numeri la cui somma vale $222a+111c$, per cui $\sum f(x_i)=\frac{222a+111c}{2a+c}=111$. In tutto $72 \cdot 111=7992$

$\bullet$ $b=c=0$: $9$ possibili scelte di $a$, e per ogni $x=100a$ che rispetta queste condizioni $f(x)=100$. La somma in questo caso vale $100 \cdot 9=900$

$\bullet$ $a=b$, $c=0$; $9$ possibili scelte per $a, b$, per ognuna $\sum f(x_i)=\frac{211a}{2a}=\frac{211}{2}$. Quindi in tutto $9\cdot \frac{211}{2}=949,5$

$\bullet$ $a\neq b$, $c=0$: $\displaystyle\binom{9}{2}=36$ possibilità per $a, b$; per ognuna $\sum f(x_i)=\frac{211a+211b}{a+b}=211$. In tutto $36 \cdot 211=7596$

La somma richiesta è quindi uguale a $18648+333+7992+900+949,5+7596=36418,5$. Ci interessano le ultime quattro cifre della parte intera.
(Valeria Martinelli)

Problema 89 [134]
Sia $2^k$ la massima potenza di $2$ nell'intervallo $\{1, ..., n\}$. Si osserva facilmente che $f(n)$ non può mai essere maggiore di $f(2^k)$, pertanto $f(n)$ è massima quando $n=2^k$, e $g(2^k)=1$.

Dato che $5>4=2^2$ e $9=3^2>2^3=8$, se un intero contiene nella sua fattorizzazione primi diversi da $2$ e $3$, o se $3$ compare con un esponente maggiore o uguale a $2$, $g(n)=g(n-1)$ in quanto $n$ ha sicuramente meno fattori di $2^k$.
Inoltre, se $n=2^{k-1}\cdot 3$, $g(n)=g(n-1)+1$, in quanto n contiene lo stesso numero di fattori di $2^k$.

Dunque, se $2^{k-1}\cdot 3\le n <2^{k}$, $g(n)=2$, altrimenti $g(n)=1$.

Possiamo quindi trovare tutti gli interi minori o uguali a $99$ per cui $g(n)=2$, che sono i valori compresi tra i tripli delle potenze di $2$ e la potenza di $2$ successiva.

Dunque la somma richiesta è uguale a $99+(4-3)+(8-6)+(16-12)+(32-24)+(64-48)+(100-96)=99+1+2+4+8+16+4=99+31+4=134$.
(Valeria Martinelli)

Problema 90 [8784]
Chiamiamo $f(n)=x^n+\frac{1}{x^n}$, il problema ci chiede di calcolare $|f(1)\cdot f(2)\cdot\dots\cdot f(2022)|$.
Esplorando un po’ la situazione si può notare che $f(1)=1,f(2)=-1,f(3)=-2,f(4)=-1,f(5)=1$ e $f(6)=2$ e che questi valori potrebbero formare un ciclo che si ripete ogni $6$.
Dimostriamolo:
consideriamo $f(n) \cdot f(1)= \left( x^n+\frac{1}{x^n} \right) \cdot \left(x+ \frac{1}{x} \right) = x^{n+1} + x^{n-1} + \frac{1}{x^{n-1}} + \frac{1}{x^{n+1}} = f(n+1) + f(n-1)$, essendo $f(1)=1$ otteniamo che $f(n) \cdot f(1) =f(n)= f(n+1) + f(n-1)$ da cui $f(n+1)=f(n)-f(n-1)$. L’ultima relazione ci indica che $f(n+1)$ dipende unicamente da $f(n)$ e $f(n-1)$ ovvero se troviamo un ciclo quest’ultimo si ripeterà. Controllando si può vedere che $f(7)=1$ e $f(8)=-1$ quindi il ciclo si ripeterà. Analizzando il nostro ciclo si nota facilmente che gli unici fattori che contribuiranno nel nostro prodotto saranno quelli con $n$ multiplo di $3$, che in valore assoluto valgono $2$. Il nostro prodotto sarà quindi del tipo $2^k$ dove $k$ è il numero di multipli di $3$ tra $1$ e $2022$ ovvero $674$ $(\frac{2022}{3}=674)$. Il problema si è trasformato in un quesito di teoria dei numeri poiché dobbiamo dare come risposta le ultime $4$ cifre di $2^{674}$ quindi calcolare $2^{674}$ $(\mathrm{mod}\,10000)$. Riducendo con un po’ di conti l’enorme numero si trova $8784$.
(Lorenzo Bastioni)

Problema 91

Problema 92

Problema 93 [279]
$x$,$y$ e $z$ sono interscambiabili. Per cui è possibile, per facilitarci nella risoluzione, porre $x \geq y \geq z$.

Adesso andiamo in ordine:\\
abbiamo

$x^2<x^2+2y+z \leq x^2+3x < x^2+4x+4=(x+2)^2$

e siccome lo vogliamo come quadrato

$x^2+2y+z=(x+1)^2 \Rightarrow 2y+z=2x+1 \Rightarrow x=\frac{2y+z-1}{2}$.

Ora usiamo la seconda equazione con le nuove sostituzioni.\\
$y^2<y^2+2z+x=y^2+2z+\frac{2y+z-1}{2}=y^2+y+2.5(z)-0.5<y^2+y+3y+4=(y+2)^2$.

Per lo stesso motivo di prima:

$y^2+y+2.5(z)-0.5=(y+1)^2 \Rightarrow 5z-3=2y \Rightarrow y=\frac{5z-3}{2}$\\
Ora è il momento di usare l'ultima equazione, anche qui sostituendo $y$ e $x$:

$z^2<z^2+2x+y=z^2+2y+z-1+y=z^2+z+3(\frac{5z-3}{2})-1=z^2+8.5z-4.5$.

Dopo vari tentativi si nota che gli unici quadrati possibili per questa quantità sono $(x+1)^2$ e $(x+4)^2$.\\

Infatti si avrà $z^2+8.5z-4.5=z^2+8z+16 \Rightarrow z=43$, trovandoti poi $y=106$ e $x=127$ e $z^2+8.5z-4.5=z^2+2z+1$ trovando quindi $x=z=y=1$.
(Alessandro Avellino)

Problema 94 [6460]
Possiamo riscrivere il testo del problema nel seguente modo:
$$\sum_{n=1}^{2020} \lfloor \frac{\sqrt{n+2}}{\sqrt{n+1} - \sqrt{n}} \rfloor$$
Andiamo ad analizzare il nostro addendo generale privo della funzione floor:
$ \frac{\sqrt{n+2}}{\sqrt{n+1} - \sqrt{n}} = \frac{\sqrt{n+2}}{\sqrt{n+1} - \sqrt{n}} \cdot \frac{\sqrt{n+1} + \sqrt{n}}{\sqrt{n+1} + \sqrt{n}} = \sqrt{(n+2) \cdot (n+1) } + \sqrt{ (n+2) \cdot n } = \sqrt{n^2 + 3n+ 2} + \sqrt{n^2 +2n} $.
Esplorando un po' la situazione ci si potrebbe convincere che $2n+2 \leq \sqrt{n^2 + 3n+ 2} + \sqrt{n^2 +2n} < 2n+3$ (con $n$ nell'intervallo che interessa a noi ovvero $[1,2020]$).
Dimostriamolo:
La seconda disuguaglianza è molto semplice infatti $\sqrt{n^2 + 3n+ 2} + \sqrt{n^2 +2n} = \sqrt{(n+2)^2 -n-2} + \sqrt{(n+1)^2 - 1} < \sqrt{(n+2)^2} + \sqrt{(n+1)^2} = n+2 +n+1 = 2n+3$ quindi $\sqrt{n^2 + 3n+ 2} + \sqrt{n^2 +2n} < 2n+3$.
La prima disuguaglianza è leggermente più fastidiosa:
vogliamo $2n+2 \leq \sqrt{n^2 + 3n+ 2} + \sqrt{n^2 +2n}$ che è molto lungo da risolvere a mano dunque proviamo a scomporre in maniera intelligente il problema:
ad esempio, se dimostrassimo che $\sqrt{n^2 + 3n+ 2} \geq n + \frac{4}{3} \wedge \sqrt{n^2 +2n} \geq n+\frac{2}{3}$ avremmo finito poiché sommando le disuguaglianze si otterrebbe la tesi (la veridicità delle due disuguaglianze implica la tesi ma non viceversa).
Le due disuguaglianze sono due semplici disequazioni irrazionali e non ci dovremmo preoccupare di condizioni di esistenza o simili poiché nel nostro intervallo è tutto positivo:
1)
$ \sqrt{n^2 + 3n+ 2} \geq n + \frac{4}{3} \rightarrow n^2+3n+2 \geq (n + \frac{4}{3})^2 \rightarrow n \geq - \frac{2}{9}$, nel nostro intervallo la disuguaglianza vale.
2)
$ \sqrt{n^2 +2n} \geq n+\frac{2}{3} \rightarrow n^2+2n \geq (n+\frac{2}{3})^2 \rightarrow n \geq \frac{2}{3} $, nel nostro intervallo la disuguaglianza vale.
La tesi è dunque dimostrata, $2n+2 \leq \sqrt{n^2 + 3n+ 2} + \sqrt{n^2 +2n} < 2n+3$. Ciò implica che $\lfloor \frac{\sqrt{n+2}}{\sqrt{n+1} - \sqrt{n}} \rfloor = 2n+2$ (nel nostro intervallo) e possiamo riscrivere la nostra somma:
$$\sum_{n=1}^{2020} 2n+2 = \left( 2 \sum_{n=1}^{2020} n \right) + 2 \cdot 2020 = 2020 \cdot 2021 + 4040 = 4086460$$
(La seconda uguaglianza si ha con la formula di Gauss per la somma dei primi $k$ numeri naturali) Prendendo le ultime $4$ cifre del risultato abbiamo la soluzione: $6460$.
(Lorenzo Bastioni)

Problema 95 [21]
Mettiamo in un piano cartesiano i punti [math]. Si ha che [math] è il coffieciente angolare della retta per i due punti [math]. Quindi una sequenza è non genovese se tutti i coefficienti angolari sono non negativi; ovvero se e solo se essa è debolmente crescente. Supponiamo di avere una sequenza debolmente crescente [math]. Vi è una bigezione fra le sequenze di questo tipo e quelle del tipo [math], che sono tutte le sequenze strettamente crescenti con elementi positivi minori di [math]. Esse a loro volta corrispondono con i modi di scegliere [math] elementi distinti fra [math]. Il numero totale di sequenze (genovesi e non) è infine [math]; quindi il numero di quelle genovesi è [math] che è congruo a [math] modulo [math], essendo che [math] modulo [math].
(Federico Volpe)

Problema 96 [2029]
I numeri curiosi sono esattamente i liberi dai quadrati.\\
($\Leftarrow$) Se $n$ è un numero libero da quadrati allora $\tau(n)$ è una potenza di due, inoltre ogni divisore $d$ di $n$ è un numero libero da quadrati, quindi anche $\tau(d)$ è una potenza di $2$ e dato che $\tau(d) \le \tau(n)$ abbiamo che $\tau(d)|\tau(n)$.\\
($\Rightarrow$) Supponiamo che $n = p_1^{a_1}p_2^{a_2}\cdots p_m^{a_m}$ sia un numero libero da quadrati. Allora abbiamo
\begin{align*}
&\tau(n/p_i)|\tau(n)\\
\Rightarrow &(a_1+1)\cdots(a_i)\cdots(a_m+1)|(a_1+1)\cdots(a_i+1)\cdots(a_m+1)\\
\Rightarrow &a_i | a_i+1 \Rightarrow a_i = 1.
\end{align*}
Dunque $n$ è libero da quadrati.\\
\\
Inoltre notiamo che $f(a,b)$ è semplicemente $MCD(a,b)$. A questo punto scriviamo
$$S(n) = \sum_{j=1}^n f(j,n) = \sum_{j=1}^n MCD(j,n),$$
dimostreremo che la funzione $S$ è moltiplicativa, cioè per ogni $m,n \in \mathbb{Z}^+$ coprimi $S(m)S(n) = S(mn)$. Infatti, segue dal teorema del resto cinese, possiamo rimpiazzare $j \text{ mod }\,mn$ con due numeri $j_m \text{ mod }\,m$ e $j_n \text{ mod }n$ in modo che:
\begin{align*}
S(mn) &= \sum_{j=1}^{mn} MCD(j,mn) = \sum_{j=1}^{mn} MCD(j,m)MCD(j,n)\\
&= \sum_{j_m=1}^{m} \sum_{j_n =1}^{n} MCD(j_m,m)MCD(j_n,n)\\
&= \left(\sum_{j_m=1}^m MCD(j_m,m)\right)\left(\sum_{j_n=1}^n MCD(j_n,n)\right) = S(m)S(n).
\end{align*}
Ma quindi dato che $C$ è un prodotto tra primi ed è facile verificare che $S(p) > 1$ per ogni primo $p$, abbiamo che $C$ stesso deve essere primo. Un po' di brute-force dà la risposta.
(Michele Tomasi)

Problema 97

Problema 98

Problema 99

Problema 100 [1339]
Premetto innanzitutto che questo geometrico è più tecnico degli altri ed è necessario avere qualche prerequisito sulle simmediane e il cerchio di Feuerbach. Tutti i fatti noti di cui farò uso si trovano sul libro di Evan Chen ma sono certo che si possano trovare anche girovagando su internet.

La soluzione di questo problema si può suddividere in tre claim.

Primo claim: La tangente di $\Gamma$ in $N_a$, la retta $BC$ e la tangente di $\Gamma$ in $A$ concorrono nel punto $X_a$.\\
Qua c'è solo da usare qualche fatto noto delle simmediane. Infatti $\triangle BAN_a \sim \triangle CAM_a \Rightarrow \angle BAN_a= \angle CAM_a \Rightarrow AN_a$ è la $A-simmediana$ di $\triangle ABC$. Dopodiché basta usare che $CB$ è la $C-simmediana$ di $\triangle ACN_a$ e quindi le tangenti in $A$ e in $N_a$ concorrono con essa.

Secondo claim:
$Y_a$ e simili stanno sulla circonferenza di Feuerbach, ovvero $\gamma$ e $\omega$ coincidono.
Per essere capaci di dimostrarlo però è necessario notare qualcosa di più subdolo.
Sia $H$ l'ortocentro di $\triangle ABC$, $V$ il punto medio di $AH$ e $Q$ l'intersezione tra la retta $AH$ e $\Gamma$. Siano inoltre $R$ e $r$ i raggi di $\Gamma$ e di $\gamma$.
Lemma: $V$ è L'ortocentro di $\triangle AM_{a}X_{a}$.
Proof: Dimostriamo innanzitutto che la retta $VM_a$ è perpendicolare a $AX_a$. CIò è equivalente a dimostrare che $\angle X_{a}M_{a}V = 90^{\circ}- \angle AX_{a}M_a=\angle X_{a}AP_{a} \Leftrightarrow \angle P_{a}M_{a}V=\angle X_{a}AQ \Leftrightarrow \frac{VP_{a}}{AQ}=\frac{r}{R}=\frac{1}{2}$.
Poiché $Q$ è il simmetrico di $H$ su $BC$ (si dimostra con angle chasing), $HQ+HA=2HP_{a}+2HV \Rightarrow AQ=2VP_{a} \Rightarrow \frac{VP_{a}}{AQ}=\frac{1}{2}$.
$AV \perp X_{a}M_{a}$ e $M_{a}V \perp AX_{a} \Rightarrow V$ ortocentro di $\triangle AM_{a}X_{a}$.
A questo punto per dimostrare il claim iniziale basta dire che, poichè $V$ è ortocentro, $X_{a},V,Y_{a}$ sono allineati e quindi $\angle VY_{A}M_{a}=90^{\circ}=\angle VP_{a}M_{a} \Rightarrow VY_{a}M_{a}P_{a}$ ciclico $\Rightarrow Y_{a}$ sta su $\gamma$.
Similarmente si può dimostrare che anche $Y_{b}$ e $Y_{c}$ stanno su $\gamma$.

Terzo claim: le rette $P_{a}P_{b}, DE, CH$ concorrono in $F$.
$\angle BHC=180^{\circ}- \angle BAC \Rightarrow H$ sta su $\Gamma'$. Ora basta osservare che $P_{a}P_{b}, DE, CH$ sono gli assi radicali di $\gamma, \Gamma', (P_{a}HP_{b}C)$.
Similarmente si dimostra che $P_{a}P_{c}, DE, BH$ concorrono in $G$.

Ora che abbiamo delle informazioni "decenti" sui punti $F$ e $G$ possiamo fare del segment chasing per trovare i valori di $P_{a}F$ e $P_{a}G$. $\triangle P_{a}P_{b}P_{c}$ è il famoso triangolo ortico e $P_{c}G$ e $P_{b}F$ sono le bisettrici, quindi l'idea sarebbe quella di trovare i tre lati e poi usare il teorema delle bisettrici.
$BC^{2}-P_{b}C^{2}=AB^{2}-(AC-P_{b}C)^{2} \Rightarrow P_{b}C=\frac{84}{5}$.
$\triangle BCA \sim \triangle P_{b}CP_{a} \Rightarrow \frac{P_{a}P_{b}}{P_{b}C}=\frac{AB}{BC} \Rightarrow P_{a}P_{b}=\frac{78}{5}$.
Similarmente si trovano $P_{b}P_{c}=\frac{924}{65}$ e $P_{a}P_{c}=\frac{150}{13}$
$\frac{P_{a}F}{P_{a}P_{c}-P_{a}F}=\frac{P_{b}P_{a}}{P_{b}P_{c}} \Rightarrow P_{a}F=\frac{1950}{323}$
$\frac{P_{a}G}{P_{a}P_{b}-P_{a}G}=\frac{P_{c}P_{a}}{P_{c}P_{b}} \Rightarrow P_{a}G=\frac{650}{93}$
$P_{a}F+P_{a}G=\frac{391300}{30039}$. La risposta è quindi $1339$.
(Denis Tusca)
[/quote]
[/quote]
[/quote]
Godl3x04
Messaggi: 12
Iscritto il: 27 lug 2019, 22:03

Re: #Proviamoci - Soluzioni commentate OH6

Messaggio da Godl3x04 »

Problema 1 [3]
$n$ il numero di commensali e $d$ la distanza tra loro, i commensali sono i punti ${A_1, A_2,..., A_n}$ mettiamo $A_1$ in un punto qualsiasi del piano. $A_2$ potrà stare ovunque nella circonferenza di raggio $d$ e centro $A_1$. Il terzo punto ha solo $2$ posizioni in cui poter stare, che sono le intersezioni delle $2$ circonferenze di centro $A_1$ e $A_2$, entrambe di raggio $d$. Posizionato il terzo punto e tracciata la terza circonferenza noteremo che non ci sarà nessun punto in comune a tutte e $3$ le circonferenze. Di conseguenza non ci può essere un quarto punto che rispetta le condizioni.
(Alessandro Avellino)

Problema 2 [6]
Si tratta di disporre tre diversi ingredienti, che si può fare semplicemente in $3!=6$ modi diversi. La risposta è quindi $6$.
(Matteo Salicandro)

Problema 3 [228]
Il massimo numero ottenibile lanciando gli $n$ dadi è $n\cdot k$, il minimo è $n\cdot 1=n$. Pertanto $n\cdot k-n=168$, cioè $n(k-1)=168$. Questo vuol dire che $n$ è un qualsiasi divisore positivo di $168$, esclusi $168$ e $84$ visto che $k \geq 4$. $168=2^3\cdot 3\cdot 7$, quindi la somma dei suoi divisori è $(2^4-1)(3+1)(7+1)=480$. La somma dei possibili valori di $n$ è pertanto $480-168-84=228$.
(Federico Borasio)

Problema 4

Problema 5

Problema 6 [36]
Un rettangolo con le diagonali perpendicolari è un quadrato. L'area è quindi $6^2=36$.
(Filippo Prandina)

Problema 7 [18]
La successione è definita, per $n \geq 1$, come
$\begin{cases}
x_{n+2}=x_{n+1}+x_n\\
x_6=76\\
x_5=47
\end{cases}$
Da qui ricaviamo che $x_n=x_{n+2}-x_{n+1}$, pertanto $x_4=76-49=29$ e $x_3=47-29=18$, che è il terzo termine della successione.
(Federico Borasio)

Problema 8 [2022]
Supponiamo che ci siano $2022$ fisici. Tutti dicono la verità, cioè che non sono matematici: non c'è alcuna contraddizione. $2022$ è anche il numero di abitanti dell'isola, quindi è il massimo numero di fisici.

Problema 9 [8]
$\frac{4n}{32}=k$ con $k\in N\geq 0$. Quindi $n=8k$. Il minimo si ha quando $k=1$ e $n=8$.
(Filippo Prandina)

Problema 10

Problema 11 [9999]
Ci sono più soluzioni in quanto i primi due termini possono essere $1,3$, oppure $2,2$ oppure $3,1$. La risposta è quindi $9999$.
(Matteo Salicandro)

Problema 12

Problema 13 [28]
$X$ appartiene alla perpendicolare di $BC$ passante per $A$. Inoltre $X$ e $2$ punti su $BC$ formano lo stesso angolo che forma $A$ con quei $2$ punti, di conseguenza $X$ è il simmetrico di $A$. Da questo deduciamo che $AX=2AD$, dove $AD$ è l'altezza.
(Alessandro Avellino)

Problema 14 [1011]
La quantità in questione, $(3n+2)^{5n+8}$, è un quadrato perfetto se lo è la base, cioè $3n+2$, oppure se l'esponente, $5n+8$, è pari. Tuttavia la prima possibilità non si verifica mai poiché un quadrato non è mai congruo a $2\,(\mathrm{mod}\,3)$. Dunque la risposta è semplicemente il numero di interi positivi $1\leq n\leq 2022$ tali che la quantità $5n+8$ sia pari, ovvero tutti e soli i numeri pari in questo intervallo, che sono $\frac{2022}{2}=1011$.
(Lorenzo Weiss)

Problema 15

Problema 16 [2019]
Il problema chiede per quanti interi positivi minori o uguali a $2022$ accade che $\lfloor \frac{\pi +n}{n}\rfloor =\lfloor \frac{\pi}{n}+1\rfloor =\lfloor \frac{\pi}{n}\rfloor +1=1\Longrightarrow \lfloor \frac{\pi}{n}\rfloor =0$, il che succede solo quando $\pi\approx 3,14\leq n$, i.e. in $2022-(4-1)=2019$ casi.
(Lorenzo Weiss)

Problema 17

Problema 18

Problema 19 [14]
Disegnando la circonferenza $x^2+y^2=5$ risulta evidente che gli unici punti a coordinate intere che vi appartengono, che sono $k=8$, sono quelli della forma $P_i(\pm 1,\pm 2)$, $P_i(\pm 2,\pm 1)$, $P_i(\pm 1,\mp 2)$, $P_i(\pm 2,\mp 1)$. In alternativa, è possibile risolvere l'equazione della stessa circonferenza sugli interi $x,y$. Da ciò si ricava l'ottagono $P_1P_2\dots P_8$, la cui area, come si vede facilmente, è $16-4\cdot\frac{1}{2}=14$.
(Lorenzo Weiss)

Problema 20

Problema 21 [210]
Notiamo che, comunque presi $6$ elementi distinti dall'insieme $X=\{1, 2, 3, 4, 5, 6, 7, 8, 9, 10\}$, c'è uno e un solo modo di disporli in ordine crescente, dunque la richiesta del problema è equivalente al trovare il numero di sottoinsiemi di $6$ elementi di $X$, che sono $\displaystyle\binom{10}{6}=210$.
(Valeria Martinelli)

Problema 22 [2022]
Si può ottenere $2$ solo quando una delle cifre è $2$ e le altre sono nulle, oppure quando due cifre sono uguali a $1$ e le altre sono nulle. Nel primo caso, la prima cifra sarà necessariamente $2$, altrimenti il numero non avrà $2022$ cifre: si ottiene solo una possibilità.
Nel secondo caso, invece, la prima cifra sarà $1$, mentre l'altro $1$ occuperà una delle altre $2021$ posizioni disponibili, ottenendo $2021$ possibilità.
Sommando le possibilità, si arriva a $2022$.
(Daniele Prisco)

Problema 23

Problema 24 [4042]
Se $x$ è la lunghezza del segmento $PA$ e $r$ è il raggio della circonferenza,
$OP=OA+PA=r+x=2021$; $PB=OA+OB+PA=r+r+x$. Di conseguenza, $PA+PB=x+r+r+x=2(r+x)=2×2021=4042$.
(Daniele Prisco)

Problema 25 [66]
Poniamo $a+b=x$ e $c+d=y$.
La disuguaglianza adesso è
$x+y\leq \sqrt{x^2+y^2} \Rightarrow x^2+2xy+y^2\leq x^2+y^2$
Da cui $2xy\leq0$ ma siccome $x$ e $y$ sono somme di interi maggiori o uguali a $0$, l'unico modo per cui $xy=0$ si ha quando uno dei $2$, o entrambi, è uguale a $0$. Se $c+d=y=0$ da cui $c=d=0$, quindi $(a,b,0,0)$. Ricordiamo che $a \geq b$ quindi i possibili valori sono $66$.
(Alessandro Avellino)

Problema 26 [6856]
Sia $n$ il numero di circonferenze sulla prima riga. Avendo le circonferenze diametro $2$, la base del rettangolo è uguale a $2n$. Invece, essendo tutte le circonferenze tangenti, i centri di quella più alta e delle due agli estremi della base sono disposte sui vertici di un triangolo equilatero, di lato $2n-2$. L'altezza del rettangolo è quindi uguale all'altezza del triangolo aumentata di $2$, ossia $\sqrt{3}(n-1)+2$.
Quindi vogliamo $4n+2\sqrt{3}(n-1)+4>2022$, che ci dà immediatamente $n=271$, quindi $k=\frac{271\cdot 272}{2}=36856$.
La risposta voluta sono le ultime quattro cifre di $k$.
(Valeria Martinelli)

Problema 27 [250]
Intanto notiamo che $78125$ è $5^7$.
La somma di $M$ termini consecutivi con primo termine $a$ è del tipo:
$a+(a+1)+(a+2)+...+(a+M-1)$ nel quale compaiono infatti $M$ termini consecutivi.
È quindi possibile raccogliere come $a \cdot M + (1+2+3+...+M-1)=a \cdot M+ \frac{M(M-1)}{2}=M(a+\frac{M-1}{2})=78125 \Rightarrow M(2a-1+M)=5^7\cdot2$, $M<2a-1+M$ quindi al massimo $M=2 \cdot 5^3$.
(Alessandro Avellino)

Problema 28

Problema 29

Problema 30

Problema 31 [20]
Il doppio delle cifre di $n$ dovrà necessariamente essere un numero di $3$ cifre ognuna delle quali minore di $5$. Una volta individuato il numero più piccolo $n$ come $101\,(2n=202)$ e il massimo $n$ come $444\,(2n=888)$ i restanti saranno compresi tra essi. Ci saranno $18$ numeri palindromi tra questi due estremi, con la cifra intermedia contente al più il $4$ e gli estremi uguali da $1$ a $4$. I numeri saranno $101$, $111$, $121$, $131$, $141$, $202$, $222$… fino a $444$. Il totale è di $20$ numeri bipalindromi.
(Irene Mancone)


Problema 32

Problema 33 [74]
Il costo di un pallone ($C$) può essere pari o dispari. Se è pari, può essere pagato con $\frac{C}{2}$ monete da $2$ orue. Poiché $\frac{C}{2}$ è pari, $C$ sarà un multiplo di $4$, quindi potrà assumere $37$ valori.
Se $C$ è dispari, invece, il pallone può essere pagato con $\frac{C-1}{2}$ monete da $2$ orue e una moneta da un orue. Di conseguenza, $C-1$ è un multiplo di $2$, ma $\frac{C-1}{2}$ è dispari (aggiungendo una moneta da un orue, il numero di monete usate è pari). Di conseguenza, $C-1$ è multiplo di $2$ ma non di $4$, quindi può assumere altri $37$ valori.
La somma dei valori possibili è dunque $74$.
(Daniele Prisco)

Problema 34 [2023]
Notiamo che possiamo riscrivere il testo nella seguente maniera: $P((x-y)(x+y))=P(x-y)P(x+y)$.
Notiamo anche che sostituendo $x-y \mapsto z$ e $x+y \mapsto w$ la coppia $(z,w)$ può assumere qualsiasi coppia di valori in $\mathbb{R}^2$ poiché il sistema
$\begin{cases}
x-y=z\\
x+y=w
\end{cases}$
ha soluzioni per ogni coppia $(z,w)$ fissata.
Dunque il nostro problema è equivalente a $P(zw)=P(z)P(y)$ per ogni $z$ e $w$ in $\mathbb{R}$. Questa relazione è molto simile ad una delle famose equazioni funzionali di Cauchy, $f(xy)=f(x)f(y)$. Quest'ultima equazione citata ha soluzione $f(x)=x^n$ o $f(x)=0$ (con $n$ naturale) se la funzione è continua (la continuità è una delle tante condizioni sufficienti), ma un polinomio come $P$ è continuo dunque le possibili soluzioni sono quelle riportate con $n$ che va da $0$ a $2021$ per $P(x)=x^n$ più $P(x)=0$. La soluzione è quindi $2023$ polinomi possibili.
(Lorenzo Bastioni)

Problema 35

Problema 36 [2020]
Si noti che $abc=2022-ab-a$, quindi per massimizzare il prodotto bisogna minimizzare $a$ e $b$. $a,b,c$ sono $\geq 1$ per ipotesi, dunque si pone $a=b=1$ dando come soluzione $c=2020$. Quindi $(a,b,c)=(1,1,2020)$ e $abc=2020$.
(Filippo Prandina)

Problema 37 [201]
Poniamo [math] e notiamo che [math] per [math]. Ora possiamo costruire il seguente polinomio: [math] di cui conosciamo gli zeri [math] e poiché [math] è chiaramente monico possiamo affermare che [math]. Sostituendo infine [math] con [math] e ricavando [math] otteniamo che [math] da cui possiamo trovare la soluzione al problema calcolando [math]. La risposta è quindi [math].
(Lorenzo Bastioni)

Problema 38

Problema 39

Problema 40

Problema 41

Problema 42

Problema 43 [4238]
Se si devono prendere almeno $n$ dolci per essere sicuri di prenderne almeno $1$ di un determinato tipo, questo vuol dire che ci sono $n-1$ dolci non di quel tipo. In particolare:
$\begin{cases}
S+B=55\\
B+C=70\\
S+C=91
\end{cases}$
Il sistema è di immediata risoluzione e porta come soluzione $(S,C,B)=(38,53,17)$ da cui $SCB=34238$. Sono richieste solo le ultime $4$ cifre.
(Filippo Prandina)

Problema 44 [4]
La configurazione è unica, infatti se prendiamo il punto $A_1$ e lo collegassimo con un punto diverso da $B_{2022}$, come per esempio $B_{2021}$, $B_{2022}$ sarà collegato con un punto diverso da $A_1$, per esempio $A_2$. Allora $A_1B_{2021}$ non interseca $A_2B_{2022}$. Iterando questo ragionamento ad ogni punto $A_i$, si ottiene un'unica configurazione: segmenti di estremi $A_iB_{2023-i}$. Quindi i coefficienti angolari saranno $\pm 2/(2023-2i)$. Per il prodotto, il numeratore è sempre $2$ mentre il denominatore è sempre dispari. Trovare quindi ciò che il problema richiede equivale a trovare le ultime $2$ cifre di $2^{2022}$.
(Alessandro Avellino)

Problema 45 [18]
Notiamo che, comunque presa una casella, il numero di giorni che Alberto o Barbara impiegano per raggiungerla non dipende dal percorso scelto, e le uniche caselle che hanno la stessa distanza sia da Alberto che da Barbara sono quelle sulla diagonale del quadrato diversa da quella di partenza, dunque i due si incontrano se e solo se entrambi passano per una di queste tre caselle.
Dato che un percorso è valido se e solo se fa $2$ passi in una direzione e $2$ nell'altra, tutte le coppie di percorsi possibili sono $\frac{4!}{2!\cdot 2!}\cdot \frac{4!}{2!\cdot 2!}=36$, quelli in cui entrambi passano per il centro sono $(2!\cdot 2!)(2!\cdot 2!)=16$, mentre quelli in cui entrambi passano in uno dei due angoli sono $2$.
Le coppie di percorsi con cui Alberto e Barbara non si incontrano sono quindi $36-16-2=18$.
(Valeria Martinelli)

Problema 46

Problema 47 [171]
Poiché $wxy+2021$ deve dare un numero pari, $wxy$ dev'essere dispari, e di conseguenza $w$,$x$ e $y$ devono esserlo.

$w=2a+1, x=2b+1, z=2c+1$ quindi $2a+1+2b+1+2c+1=37 \Rightarrow 2(a+b+c)=34 \Rightarrow a+b+c=17$
Adesso è un semplice stars and bars, infatti $a$, $b$ e $c$ possono essere anche nulli. Distribuire $17$ elementi in $3$ contenitori, $\binom{17+3-1}{3-1}=\binom{19}{2}$.
(Alessandro Avellino)

Problema 48 [42]
Notiamo che $CQB~APB$, in quanto hanno $2$ angoli congruenti, entrambi pari a $74$. Da questo consegue anche che sono triangoli isosceli. $\measuredangle CQB=\measuredangle APB=180-2(74)=32$. Inoltre $S$ sta nell'asse di $BC$, di conseguenza anche nella bisettrice di $CBQ$. Ragionamento analogo per $APB$. Ora osserviamo il quadrilatero $PCAQ$. Questo ha gli angoli $APB$ e $CQB$ congruenti, da questo possiamo asserire che $PCAQ$ è ciclico. Questo permette di stabilire che $\measuredangle DAC=\measuredangle PQC$ e che $\measuredangle DCA=\measuredangle QPD$. Infine osserviamo il triangolo $QSP$. $\measuredangle QSP=180-\measuredangle QPS-\measuredangle PQS= 180-(16+\measuredangle PQC)-(16+\measuredangle QPA)=148-(\measuredangle PQC+\measuredangle APQ)=148-(\measuredangle DCA+\measuredangle DAC)=148-(180-\measuredangle ADC)=\measuredangle ADC-32.$ $\measuredangle ADC=360-(\measuredangle DCB+\measuredangle CBA+\measuredangle BAD)=360-3 \cdot 74=138$ da cui $\measuredangle QSP=138-132=106$. Essendo infine $S$ il circocentro di $ABC$, ne consegue che $\measuredangle ASC=2 \measuredangle ABC=148$. Quindi $\measuredangle ASQ + \measuredangle PSC= \measuredangle ASC- \measuredangle QSP=148-106=42$.
(Alessandro Avellino)

Problema 49 [15]
Dovendo esistere almeno un numero $k>1$ che elevato al cubo sia un divisore di $n$ si possono escludere i numeri da $2$ a $7$ in quanto l’unico loro divisore al cubo è $1$ e $k$ non può assumere tale valore. L’$8$ si può vedere come $2^3$ ed è un prigioniero di cubi, così come lo sono tutti i suoi multipli fino a $96$. È valido lo stesso discorso anche per i multipli di $27$ (essendo $3^3$) fino a $81$. Contandoli si arriva al risultato richiesto, $15$.
(Irene Mancone)

Problema 50

Problema 51

Problema 52[1872]
Chiamiamo le tre radici di p(x) a,b,c dove a<b<c . Essendoa,b e c in progressione aritmetica si ha che a = b-r e c = b+r dove r è la ragione della progressione aritmetica. Per le formule di Vietè a+b+c= 39 b-r + b+ b+ r= 39 da cui si ricava che b =13 . Sempre applicando le formule di Vietè, ab + ac + bc = 482 sostituendo i risultati ricavati in precedenza si trova che 13(13-r) + (13-r)(13+r) + 13(13+r) = 482 da cui sviluppando le parentesi 169 -13r +169 -r2 + 169 +13r = 482 ottenendo che r2 = 25 quindi poiché r è maggiore di 0 si ha che r=5. Quindi a=8 c=18 .
Applicando nuovamente le formule di Vietè, -m = -8*13*18 ovvero m= 1872
(Mattia Zunino)

Problema 53 [24]
Poniamo l'età di Marco al momento della prima affermazione uguale a $x$, quindi l'età del padre Luca è $4x$. Allora, $4$ anni dopo l'età di Marco è $x+4$ e quella del padre è $4x+4$, quindi vale l'equazione $3(x+4)=4x+4$; $x=12-4=8$ e infine $4x=32$.
La differenza di età tra i $2$ è $24$, quindi per rispondere alla domanda risolviamo $2y=y+24$, con $y$ l'età di Marco e $y+24$ l'età di Luca, $y=24$ che è quindi la risposta.
(Niso Cicalò)

Problema 54

Problema 55

Problema 56

Problema 57 [183]
Si può considerare il triangolo isoscele con i vertici di coordinate $B (0;0), C (20;0)$ e A$ (10; y)$ con $y > 0$
Il punto $H$ è l’intersezione tra le rette perpendicolari ai lati passanti per $A, B, C$. Sapendo che le distanze tra
$AH$ e $AD$ sono segmenti di lunghezza intera il punto $H$ si potrà trovare al minimo nella coordinata $(10;1)$. Calcolando il coefficiente angolare tra $H$ e i punti $B$ e $C$, le rette passanti per $B$ e $C$ con coefficiente angolare antireciproco si può ottenere l’ordinata di $A$ sostituendo in una delle due equazioni.
$m_{BH}$= $\frac{1}{10}$ $m^1$=$-10$ retta r passante per A e C: $y=-10x+200$
$m_{CH}$=$\frac{-1}{10}$ $m^1$=10 retta s passante per B e A : $y=10x$

Per $H (10,1)$ $A$ vale $(10, 100)$ e la distanza è intera ($99$).
Una volta stabilita l’ordinata massima di $A$ è sufficiente ripetere il procedimento. Per $H (10,2)$ $A (10, 50)$, per $H (10,4)$ $A(10,25)$. Nelle coordinate $(10, 10)$ $H$ e $A$ si sovrappongono. Poi si avrà che $A$ si trova in $(10,20)$, in $(10,2)$ e $(10,5)$. La soluzione si ottiene sommando le distanze trovate tra $H$ e $A$ senza ripetizioni.
(Irene Mancone)

Soluzione alternativa:
Essendo $ABC$ un triangolo isoscele la mediana $AD$ coincide con l'altezza rispetto a $BC$, chiamando $K$ il piede dell'altezza relativa ad $AC$ otteniamo che il triangolo $BKC$ è simile al triangolo $ABD$ poiché $\widehat{BKC} \cong \widehat{BDA} = 90^{\circ}$ e $\widehat{BCK} \cong \widehat{ABD}$ (essendo $ABC$ isoscele). Consideriamo ora i triangoli $BKC$ e $BDH$, anch'essi sono simili poiché $\widehat{BKC} \cong \widehat{BDH} = 90^{\circ}$ e $\widehat{CBK}$ è in comune. Per la proprietà transitiva $ABD \sim BKC$ e $BKC \sim BDH$ $\rightarrow$ $ABD \sim BDH$. Avendo dimostrato che i triangoli $ABD$ e $BDH$ sono simili vale la seguente uguaglianza: $\frac{BD}{AD}=\frac{HD}{BD} \rightarrow BD^2 = AD \cdot HD \rightarrow 100=AD \cdot HD$ (essendo $BD$ la metà di un segmento lungo $20$). Avendo che $AH$ e $AD$ sono di lunghezza intera anche $HD$ sarà di lunghezza intera (Poiché $HD$ è la differenza tra i due segmenti) ed avremo soluzioni limitate a $100=AD \cdot HD$ ovvero $(AD,HD)=(100,1),(50,2),(25,4),(20,5),(10,10),(5,20),(4,25),(2,50),(1,100)$. La soluzione $(AD,HD)=(10,10)$ non ci piace poiché $\widehat{ABC}$ varrebbe $90^{\circ}$ ($D$ sarebbe il centro della circonferenza circoscritta ad $ABC$ e $\widehat{ABC}$ sottenderebbe un diametro) dunque i possibili valori di $AH$ sono le diverse differenze fra le soluzioni ottenute ($AH=|AD-AH|$): $AH = 100-1= 99$, $AH = 50-2 = 48$, $AH = 25-4= 21$, $AH =20-5=15$ (le altre danno gli stessi valori poiché sono soluzioni "specchiate"). La risposta è quindi $99+48+21+15=183$.
(Lorenzo Bastioni)

Problema 58 [46]
La somma $S$ dei quadrati di delle radici un polinomio del tipo $ x^n + \alpha_{n-1} \cdot x^{n-1} + ... + \alpha_1 \cdot x + \alpha_0$ è

$S=\alpha_{n-1}^2-2 \cdot \alpha_{n-2}$

$$Q_n(x)=x^n+\sum_{i=0}^{n-1} \sqrt{i} \cdot x^i$$

Per cui, definiamo $S_n$ come la somma dei quadrati delle radici di $Q_n(x)$. Allora $S_n=n-1-2\sqrt{n-2}$ che è razionale solo se anche $2\sqrt{n-2}$ lo è. Questo accade solo quando $n-2$ è un quadrato perfetto, quindi ci sono $45$ valori. A questo aggiungiamo $n=1$, il cui polinomio è quindi $x$, che ha come radici un numero razionale. Quindi la soluzione è $46$.
(Alessandro Avellino)

Problema 59

Problema 60

Problema 61[9098]
Per il criterio di divisibilità per 9, s(n) ≡ n mod 9 dove s(n) è la somma delle cifre di n. Iterando il ragionamento su s(n) si ottiene che A(n)≡n mod9 . Poiché A(n) è un intero compreso fra 0 e 9, A(n) è uguale al resto della divisione di n per 9. I possibili resti della divisione delle potenze di 2021 per 9 sono 5, 7, 8, 4, 2, 1 e poi si ripetono ciclicamente con periodo 6. Perciò la somma richiesta è 2016/6 *(5+7+8+4+2+1) +5+7+8+4+2 = 9098.
(Mattia Zunino)
Problema 62 [1158]
Dette $H$ e $K$, rispettivamente, le proiezioni di $D$ e $C$ su $AB$ e $AH=x$ possiamo calcolare l'altezza del trapezio in due modi diversi servendoci del teorema di Pitagora ed eguagliare le espressioni $DH^2=AD^2-AH^2=33^2-x^2=CK^2=BC^2-BK^2=60^2-(69-6-x)^2$.

Risolvendo l'equazione, si trova l'unico valore $x=\frac{81}{7}$ e l'altezza si può quindi calcolare, ad esempio, come $h=DH=\sqrt{AD^2-AH^2}=\frac{60\sqrt{13}}{7}$. L'area del trapezio vale $A=\frac{(b_1+b_2)\cdot h}{2}=\frac{(AB+CD)\cdot AH}{2}=\frac{(69+6)\cdot (60\sqrt13)}{7\cdot2}\approx 1158,93$; la sua parte intera è $1158$ ed è la soluzione.
(Federico Magnolfi)

Problema 63

Problema 64

Problema 65 [5877]
$X$ si trova sulla superficie del triangolo $AMC$: infatti, se così non fosse, $\widehat{ABC}$ e $\widehat{AXM}$ sarebbero due angoli congruenti che insistono su uno stesso segmento ($AM$) e che appartengono allo stesso semipiano rispetto ad esso: ma ciò non è possibile in quanto significherebbe che $X$ appartiene alla circonferenza circoscritta al trangolo $ABM$, alla quale ovviamente non appartengono punti interni al triangolo $ABM$.

$AXMY$ è ciclico per ipotesi e $\widehat{AYM}$ e $\widehat{AXM}$ sono supplementari in quanto angoli opposti (per quanto prima dimostrato); dunque $\widehat{BYM}$, essendo anch'esso supplementare di $\widehat{AYM}$, è congruente a $\widehat{AXM}$, il quale a sua volta è congruente a $\widehat{ABC}$ per ipotesi.
Avremo allora che il triangolo $MBY$ è isoscele su base $BY$; essendo $M$ punto medio di $BC$, $MB\cong MY\cong MC$ e ciò implica che $BYC$ è rettangolo in $Y$. $CY$ è dunque l'altezza uscente da $C$ e si può calcolare con la formula inversa $CY=\frac{2A}{AB}$ dopo aver ricavato l'area con la formula di Erone o per via trigonometrica.
Si trova che quest'ultima vale $A=1200\sqrt{5}$ e dunque $CY^2=\left(\frac{2A}{AB}\right)^2=\frac{4\cdot1200^2\cdot 5}{70^2}\approx 5877,55$. La sua parte intera è $5877$ ed è la soluzione.
(Federico Magnolfi)

Problema 66 [1152]
Prendiamo l'equazione del problema e consideriamola $(\text{mod}\,7)$: $x^2 \equiv 7y + z \,(\text{mod}\,7) \Rightarrow x^2 \equiv z \, (\text{mod}\,7)$. I quadrati come $x^2$ modulo $7$ possono assumere come valori solo $0,1,2,4$ quindi $z$ potrà assumere tutti e i soli valori $0,1,2,4$ modulo $7$. Nell'intervallo $[1,2022]$ ci sono esattamente $288$ numeri congrui a $0 \, (mod 7)$, $288$ congrui a $1$, $288$ congrui a $2$ e $288$ congrui a $4$. La risposta è quindi $288 \cdot 4=1152$.
(Lorenzo Bastioni)

Problema 67 [3026]
Scriviamo il numeratore come $\prod_{n=2}^{2021}((n-1)+n+(n+1))=\prod_{n=2}^{2021}(3n)=3^{2021}\prod_{n=2}^{2021}(n)$. Si può quindi raccogliere un $3^{2021}$ al numeratore. Inoltre si raccolgono anche una potenza di $3^{1005}$ dato che $n$ può avere $3$ come fattore primo (elevato alla $1,\,2,\,3,\,\dots,6$). Ci sono $ \Big \lfloor \frac{2021}{3} \Big \rfloor=673$ multipli di 3, $ \Big \lfloor \frac{2021}{9} \Big \rfloor =224$ multipli di $9$ e così via: $ \Big \lfloor \frac{2021}{3} \Big \rfloor+ \Big \lfloor \frac{2021}{9} \Big \rfloor + \Big \lfloor \frac{2021}{27} \Big \rfloor + \Big \lfloor \frac{2021}{81} \Big \rfloor + \Big \lfloor \frac{2021}{243} \Big \rfloor + \Big \lfloor \frac{2021}{729} \Big \rfloor =673+224+74+24+8+2=1005$. In tutto quindi si è raccolto $2021+1005=3026$.
(Filippo Prandina)

Problema 68 [1104]
Chiamiamo due insiemi $A$ e $B$ "gemelli di primo grado" se $|A \cap B|=1$ e $1 \in A,B$, in generale chiamiamo due insiemi $A$ e $B$ "gemelli di $n$-esimo grado" se $|A \cap B|=n$ e $n \in A,B$.
Possiamo ora affrontare il conteggio di tutte le coppie di sottoinsiemi gemelle di $\{1,2,3,...,2021,2022 \}$ considerando prima i gemelli di primo grado, di secondo grado fino a quelli di $2022$-esimo grado (Non ci sono coppie di sottoinsiemi di $2023$-esimo grado o di $0$-esimo grado).
Gemelli di primo grado:
se $A$ e $B$ sono di primo grado allora contengono entrambi l'elemento $1$ e non hanno nessun altro elemento in comune. Ci verra comodo per dopo scrivere che gli altri zero elementi in comune si possono scegliere in $\displaystyle \binom{2021}{0}$ modi. Ora tutti gli altri $2021$ elementi possono essere messi o in $A$ o in $B$ o in nessuno dei due (in questo caso la "o" è una o esclusiva), quindi abbiamo 3 possibilità per ognuno dei $2021$ elementi ovvero $3^{2021}$ possibilità. Avremo quindi $\displaystyle \binom{2021}{0} 3^{2021}$ coppie di sottoinsiemi gemelle di primo grado.
In generale siamo pronti ad affrontare il caso di gemelli di $n$-esimo grado:
se $A$ e $B$ sono gemelli di $n$-esimo grado allora contengono entrambi l'elemento $n$ e hanno altri $n-1$ elementi in comune. Gli altri $n-1$ elementi in comune si possono scegliere in $\displaystyle \binom{2021}{n-1}$ modi. E per i restanti $2021-(n-1)=2022-n$ elementi abbiamo $3^{2022-n}$ possibilità come spiegato precedentemente. Avremo quindi $\displaystyle \binom{2021}{n-1} 3^{2022-n}$ coppie di sottoinsiemi gemelle di $n$-esimo grado.
Sommando tutte le possibilità otteniamo:
$$\sum_{k=1}^{2022} \binom{2021}{k-1} 3^{2022-k}$$
Reimpostando la somma sostituendo $k-1 \mapsto k$ otteniamo:
$$\sum_{k=0}^{2021} \binom{2021}{k} 3^{2021-k}$$
Ma questo è esattamente uguale a $(3+1)^{2021}=4^{2021}$ (Per il binomio di Newton o teorema binomiale). Riducendo l'enorme numero $(\text{mod}\,10000)$ si trova che le ultime $4$ cifre che ci interessano sono $1104$, da cui la risposta.
(Lorenzo Bastioni)

Problema 69

Problema 70

Problema 71

Problema 72[247]
Disponiamo le 32 squadre in qualunque ordine, e costruiamo gli incontri nel seguente modo: ad ogni turno, le squadre in un posto dispari si scontrano con la successiva. Le squadre perdenti vengono eliminate. Disponiamo le squadre restanti nello stesso ordine in cui si trovano, e ripetiamo finché resta solo una squadra. Ad esempio, se ad un certo punto l'ordine è $(3, 5, 2, 1)$, la squadra $3$ vince contro $5$ e $1$ vince contro $2$, dunque al turno dopo l'ordine sarà $(3, 1)$. Poniamo per semplicità la squadra $S_3$ per prima, e consideriamo i restanti $31!$ ordinamenti possibili. La squadra $S_3$ arriva in semifinale se e solo se non viene eliminata nei 3 turni precedenti, e ciò accade se e solo se nessuna tra le squadre $S_1$ ed $S_2$ è presente tra le prime 8 squadre. Possiamo quindi scegliere le prime 8, inclusa $S_3$, in $\displaystyle\binom{29}{7}$ modi, e le restanti in $24!$ modi.
La probabilità cercata è quindi $\displaystyle\binom{29}{7}\cdot 24!\cdot \frac{1}{31!}=\frac{29 \cdot 28 \cdot 27 \cdot 26 \cdot 25 \cdot 24 \cdot 23}{31 \cdot 30 \cdot 29 \cdot 28 \cdot 27}=\frac{24 \cdot 23}{31 \cdot 30}=\frac{92}{155}$. La risposta è quindi $92+155=247$.
(Valeria Martinelli)

Problema 73 [6068]
Notiamo che
$(p+q+r)(pq+qr+pr)=p^2q+p^2r+pq^2+pr^2+q^2r+qr^2+3pqr=1+3\cdot 2022=6067$.
Dato che $p+q+r$ e $pq+qr+pr$ sono entrambi interi positivi, devono necessariamente dividere entrambi $6067$. $6067$ è un numero primo, quindi o $p+q+r=1$ o $p+q+r=6067$. La risposta è perciò $6067+1=6068$.
Si noti che non ci si deve preoccupare di trovare che esistano $p,q,r$ che rispettano le condizioni trovate, dato che questo è assicurato dal fatto che sono le tre soluzioni dell'equazione $x^3-(p+q+r)x^2+(pq+qr+pr)x-pqr=0$, che ha sempre tre soluzioni nei numeri complessi.
(Federico Borasio)

Problema 74

Problema 75 [78]
Elevando entrambi i termini al quadrato (lecito perché entrambi sicuramente positivi) si ottiene $x^2+y^2-x-y+1/2 \leq 1/4$ riarrangiando $(x-1/2)^2+(y-1/2)^2 \leq 1/4$. Si ha quindi l'equazione di un cerchio centrato in $(1/2,1/2)$ con raggio $1/2$. La probabilità che un punto interno al quadrato definito nel piano cartesiano che ha vertice nell'origine e lato $1$ sia dentro il cerchio è quindi il rapporto tra l'area del cerchio e quella del quadrato: $\pi (1/2)^2 \approx 78%$.
(Filippo Prandina)

Problema 76 [4253]
Iniziamo col dimostrare che $P$ si trova all'esterno di $ABC$.

Detto $K$ il piede dell'altezza uscente da $A$ si ha che, essendo $\widehat{APB}$ congruente a $\widehat{AKB}$ perché entrambi retti, e dunque $AKBP$ ciclico, $P$ appartiene alla circonferenza circoscritta a $AKB$ alla quale non appartengono punti interni a $AKB$; ma $P$ appartiene alla mediana $AM$, la quale è evidentemente contenuta nel triangolo $AKB$ (lo si potrebbe dimostrare con i dati del problema), e quindi $P$ si trova all'esterno di $AKB$ ma anche di $ABC$.

Ora, è noto che in un triangolo di lati di lunghezza $9k$, $8k$, $7k$, con $k$ intero positivo, la mediana relativa alla base di lunghezza $8k$ misura $7k$: questo è facilmente verificabile per via trigonometrica, col teorema della mediana o anche con Pitagora; in ogni caso, $AM\cong AC$ e in particolare $\widehat{AMC}\cong \widehat{ACM}=\gamma$.

Sia $\left\{D\right\}=BC\cap PT$. $BTD\sim BCH$ per il secondo criterio di similitudine in quanto hanno l'angolo in $B$ in comune e $\widehat{BDT}\cong \widehat{BHC}$ perché retti, in particolare $\widehat{ATD}=\gamma$.

$\widehat{BMP}\cong\widehat{AMC}$ perché opposi al vertice ed essendo $BPM$ rettangolo in $P$ e $PD$ l'altezza relativa all'ipotenusa, $BPD=\gamma$. Allora $BPT$ è isoscele sulla base $PT$, in altre parole $T$ è il simmetrico di $P$ rispetto a $BC$; da ciò deriva anche che $BMT\cong BMP$, in particolare che $\widehat{BTM}$ sia retto.

$BMT\sim BCH$ per il secondo criterio di similitudine in quanto hanno l'angolo in $B$ in comune e $\widehat{BTM}\cong \widehat{BHC}$ perché entrambi retti. Ma allora, essendo $M$ punto medio di $BC$, necessariamente $T$ è punto medio di $BH$.

Il rapporto $\frac{[ATB]}{[THC]}$ è uguale al rapporto fra le altezze $\frac{AH}{CH}$ in quanto i due triangoli hanno basi congruenti per quanto appena dimostrato. Esse si possono calcolare con pitagora, dopo aver ricavato $BH=\frac{2A}{AC}$ e l'area attraverso Erone o per via trigonometrica. Sviluppando i calcoli si trova $BH=\frac{24\sqrt{5}}{2}$, $CH=\sqrt{BC^2-BH^2}=\frac{16}{7}$ e $AH=AC-CH=\frac{33}{7}$
Il risultato cercato è $1000\cdot \left(\frac{AH}{CH}\right)^2=1000\cdot\left(\frac{33}{16}\right)^2\approx4253,90$ la cui parte intera è $4253$ che è la soluzione.
(Federico Magnolfi)

Problema 77

Problema 78 [4151]
Cominciamo col dimostrare che $BCPQ$ è un trapezio. Sappiamo che l'ortocentro e il circocentro sono coniugati isogonali: in pratica, le rette $AH$ e $AO$ formano coi lati $AB$ e $AC$ angoli congruenti. Avremo dunque $\widehat{BAH}\cong \widehat{CAO}$, ma anche $\widehat{HAI}\cong \widehat{OAI}$ essendo $AI$ bisettrice perché $I$ incentro di $ABC$.

Riferendoci ora alla circonferenza circoscritta ad $ABC$, che chiamiamo $\Gamma$, abbiamo che $\widehat{FCE}\cong \widehat{FAE}\cong\widehat{EAD}\cong \widehat{ECD}$ dove la prima e la terza uguaglianza sono date dal fatto che gli angoli insistono su uno stesso arco. Facendo attenzione, notiamo che $\widehat{QAP}$ e $\widehat{QCP}$ sono congruenti per l'uguaglianza precedente e insistono sullo stesso segmento $QP$, dunque $AQPC$ è ciclico e chiamiamo $\omega$ la sua circonferenza circoscritta. Essendo $\widehat{ACD}$ retto poiché insiste sul diametro $AD$ di $\Gamma$, $\widehat{AQP}$ sarà anch'esso retto poiché supplementare dell'angolo opposto $\widehat{ACP}$ (ricordando che ora abbiamo fatto rifereimento a $\omega$). $BC$ e $QP$ sono allora due rette perpendicolari alla stessa retta $AQ$, perciò sono parallele e $BCPQ$ è un trapezio.

Per calcolare la sua area abbiamo bisogno dell'altezza $QH$ e della base minore $QP$, dato che la base maggiore ci è nota ed è $BC$.

$\textbf{Prima soluzione (sintetica)}$
Detti $K$ e $J$ rispettivamente i piedi dell'altezza $AH$ e della bisettrice $AI$, analizziamo i triangoli $ABJ$ e $AEC$: essi hanno $\widehat{ABJ}\cong \widehat{AEC}$ perché angoli alla circonferenza che insistono su uno stesso arco; $\widehat{BAJ}\cong \widehat{EAC}$ perché $AI$ bisettrice: quindi sono simili per il secondo criterio di similitudine. Considerando ora i triangoli $AKJ$ e $AQP$: essi hanno: l'angolo in $A$ in comune; $\widehat{AKJ}\cong \widehat{AQP}$ perché retti: quindi sono anch'essi simili per il secondo criterio di similitudine. Impostiamo la proporzione $\frac{QP}{KJ}=\frac{AP}{AJ}$ $(1)$: troveremo $KJ$ e $AJ$ lavorando su $ABC$, mentre $AP$ attraverso il rapporto di similitudine fra i triangoli $ABJ$ e $AEC$. Calcoliamo $AK$ invertendo la formula dell'area di $ABC$ e $BK$ col teorema di Pitagora: $AK=\frac{2A}{BC}=\frac{2\cdot \sqrt{p(p-AB)(p-BC)(p-AC)}}{BC}=120 \quad$ $BK=\sqrt{AB^2-AK^2}=50$.
Applichiamo ora il teorema della bisettrice per trovare $BJ$: $\frac{BJ}{AB}=\frac{BC-BJ}{AC}$ da cui si ricava che $BJ=65$. Dunque $KJ=BJ-BK=15$ e $AJ=\sqrt{AK^2+KJ^2}=15\sqrt{65}$.

Impostiamo il rapporto di similitudine fra $ABJ$ e $AEC$: $\frac{AP}{AB}=\frac{AJ}{AC}$ da cui si ricava che $AP=\frac{150\sqrt{65}}{7}$ e per la proporzione $(1)$ $QP=\frac{AP\cdot KJ}{AJ}=\frac{150}{7}$. A questo punto si trova $AQ$ con pitagora e $QK$ per differenza: $AQ=\sqrt{AP^2-QP^2}=\frac{1200}{7} \quad QK=AQ-AK=\frac{360}{7}$
L'area del trapezio $BCPQ$ è dunque $$A=\frac{(140+\frac{150}{7})\cdot\frac{360}{7}}{2}\approx 4151,02$$.

$\textbf{Seconda soluzione (trigonometria)}$
Ponendo $\widehat{ABC}=\beta$, $\cos\beta=\frac{AB^2+BC^2-AC^2}{2AB\cdot BC}=\frac{5}{13}$ (teorema del coseno), $\sin\beta=\sqrt{1-\cos^2\beta}=\frac{12}{13}$ $\widehat{BAC}=\alpha$, $\widehat{CAI}=\frac{\alpha}{2}$ perché $AI$ bisettrice, $\cos\alpha=\frac{AB^2+AC^2-BC^2}{2AB\cdot AC}=\frac{33}{65}$ (teorema del coseno), $\cos\frac{\alpha}{2}=\sqrt{\frac{1+\cos\alpha}{2}}=\frac{7}{\sqrt{65}}$.

Adesso abbiamo abbastanza dati per calcolare $AP=\frac{AC}{\cos\frac{\alpha}{2}}=\frac{150\sqrt{65}}{7}$.
Ponendo $\widehat{HAI}=\widehat{OAI}=\theta$ si ha che, essendo $\widehat{BAH}=\frac{\pi}{2}-\beta$, $\theta=\frac{\alpha}{2}+\beta-\frac{\pi}{2}$; $\cos\theta=\sin\frac{\alpha}{2}\cos\beta+\cos\frac{\alpha}{2}\sin\beta=\frac{8}{\sqrt{65}}$

$AQ=AP\cos\theta=\frac{1200}{7}\quad QP=\sqrt{AP^2-AQ^2}=\frac{150}{7}$. Ci manca l'altezza $QK$ dove $K$ è il piede dell'altezza uscente da $A$. $AK=AB\sin\beta=120\qquad QK=AQ-AK=\frac{360}{7}$. A questo punto l'area si trova allo stesso modo della soluzione per via sintetica.
(Federico Magnolfi)

Problema 79 [2011]
Si ha che $p^{(2)}(x)$ ha lo stesso grado di $p^{(4)}(x)$ questo significa che $p(x)$ è di grado $1$ o grado $0$. Ma non può essere di grado $0$ perché non è costante. Allora è del tipo $p(x)=mx+q$. La funzione $p(x)$ è chiaramente iniettiva ed essendo $p^{(2)}(x)=p^{(2022)}(x)$ per ipotesi, si ha necessariamente che $p(x)=p^{(2021)}(x)$. Ma quindi $p(2021)=0$ e $p(p(2021))=p(0)=2021$. Da qui si ricava facilmente che il polinomio è $p(x)=-x+2021$ e quindi $p(10)=2011$.
(Filippo Prandina)

Problema 80


Problema 81

Problema 82 [4]
$45=9\cdot 5$ quindi scomponiamo il modulo in modulo $9$ e modulo $5$. Modulo $5$ è semplicemente $4$, in quanto l'ultima cifra è proprio $4$. Definiamo $\overline{a_1a_2a_3...a_k}$ la scrittura decimale in successione di $a_1,a_2,...,a_k$, per esempio $\overline{4^2 5^2}=1625$. Inoltre, chiamato $N$ il nostro numero, $N=\overline{1^22^23^24^2...2022^2}$. $\overline{a_1a_2a_3...a_k}\equiv a_1+a_2+a_3+...+a_k\,(\mathrm{mod}\,9)$, dimostrarlo è facile. I residui quadratici di $9$ sono $0,1,4,7$ mentre la somma dei quadrati da $1$ a $9\,\mathrm{mod}\,9$ è $24$, ($-3$). Quindi ogni $9$ numeri vi è un $-3$ da addizionare, quindi è $(-3)\cdot 224$ (che sarebbe il numero di multipli di $9$ minori di $2022$). $-672$, a questo devi aggiungere i numeri da $2017$ a $2022$, $-672+1+4+7+7=-653\equiv -14\equiv 4$.
Siccome $N$ è congruo a $4$ sia $\mathrm{mod}\,5$ che $\mathrm{mod}\,9$, sarà congruo a $4$ anche $\mathrm{mod}\,45$.
(Alessandro Avellino)

Problema 83 [21]
Sia $H$ il piede dell'altezza uscente da $A$; $AY=AH-HY$. L'altezza $AH$ si può calcolare, ad esempio, invertendo la formula dell'area $A=\frac{b\cdot h}{2}$ dopo aver trovato quest'ultima con la formula di Erone, oppure trovando il coseno dell'angolo in $B$ col teorema del coseno, quindi il suo seno e applicando il primo teorema dei triangoli rettangoli a $ABH$. Eseguendo i calcoli, si trova che essa vale $AH=3\sqrt{5}$. $BH=\sqrt{AB^2-AH^2}=2$.

Detto $M$ il punto medio di$BC$ si ha che $HM=HX=2$ e quindi esso è punto medio anche di $HX$; ma $OM\perp BC$ in quanto in un triangolo isoscele la mediana è anche altezza: allora, essendo $HYX$ e $MOX$ simili per il secondo criterio di similitudine, e avendo rapporto di similitudine $2$, $HY=2HO$

Attraverso il teorema della corda, oppure attraverso la formula nota, possiamo calcolare il raggio della circonferenza circoscritta ad $ABC$, $R=\frac{BC}{2\sin\alpha}=\frac{abc}{4A}=\frac{21}{2\sqrt{5}}$.
Ora possiamo trovare $HY=2HO=2\sqrt{R^2-MC^2}=\frac{11\sqrt{5}}{5}$ e infine $AY=AH-HY=3\sqrt{5}-\frac{11\sqrt{5}}{5}=\frac{4\sqrt{5}}{5}$.

Stiamo cercando $AY^2=\frac{16}{5}$ e la risposta è infine $16+5=21$.
(Alessandro Lombardo & Federico Magnolfi)

Problema 84

Problema 85 [1136]

Iniziamo col calcolarci alcuni oggetti che in futuro potranno esserci utili.

Perimetro: 2p=14+16+18=48

l'Area: usando la formula di Erone, A=$\sqrt{p(p-a)(p-b)(p-c)}=\sqrt{24\cdot10\cdot8\cdot6}=48\sqrt{5}$

Raggio circonferenza circoscritta: $R=\frac{abc}{4A}=\frac{4032}{192\sqrt{5}}=\frac{21}{\sqrt{5}}=\frac{21\sqrt{5}}{5}$

Altezza relativa a BC: $AD=2A/BC=6\sqrt{5}$

Altezza relativa a AB, che incontra AB in F: $CF=2A/AB=\frac{48\sqrt{5}}{7}$

Detto L il punto di incontro tra BC e la perpendicolare passante per O, L è il punto medio di BC. Per cui CL=8, OC= R, $OL=\sqrt{\frac{441}{5}-\frac{320}{5}}=\sqrt{\frac{121}{5}}=\frac{11\sqrt{5}}{5}$

$CD=\sqrt{AC^2-AD^2}=\sqrt{144}=12$

$AF=\sqrt{AC^2-CF^2}=\frac{66}{7}$

LD=CD-CL=12-8=4

adesso calcoliamo OD:

$OD=\sqrt{OL^2+LD^2}=\sqrt{\frac{201}{5}}$

Infine calcoliamo AH, dove H è l'ortocentro. per calcolare AH notiamo che, AHF è simile a ABD, per cui $\frac{AH}{AB}=\frac{AF}{AD} \Rightarrow AH=\frac{AF \cdot AB}{AD}=\frac{66 \cdot 14}{42\sqrt{5}}=\frac{22\sqrt{5}}{5}$.

Adesso che abbiamo tutto ciò che ci serve proseguiamo. O è il punto di Miquel relativo ad XYD. questo vuol dire che O appartiene alla circonferenza passante per X,Y,A. Detto questo, abbiamo 2 quadrilateri ciclici, per definizione, BDOX e CDOY. A questi 2 aggiungiamo il quadrilatero AYOX, anch'esso ciclico.

1° Claim: XYD è simile ad ABC.
è sufficiente un po' di angle chasing:

ricordiamo che OA, OB e OC sono i raggi di (ABC).
Di conseguenza OBA=OAB, OBC=OCB e OCA=OAC. Inoltre, per via della ciclicità dei 3 quadrilateri sopra elencati,
OAB=OBA=OBX=ODX, OAC=OCA=OCY=ODY, da questo segue che YDX=ODY+ODX=CAO+OAB=BAC.

Analogamente DYX=CBA e DXY=BCA. Per cui i 2 triangoli sono simili

2° Claim: O è l'ortocentro di XYD.
Anche qui è sufficiente un po' di Angle chasing:

DYX+YDO=(DYO+OYX)+OCY=DCO+OAX+OCA=(DCO+OCA)+OAX=ACB+OAX=ACB+(OAH+HAX)=ACB+(OAH+HAB)=
ACB+(OAH+DAB)=ACB+(OAH+OAC)=ACB+DAC=90.
Ne consegue che OD è perpendicolare ad XY. Analogamente si ottiene che , XO e YO sono perpendicolari, rispettivamente, a DY e DX. Per cui O è l'ortocentro.

Di conseguenza, detto K il rapporto di similitudine tra ABC e XYD, $K=\frac{AH}{OD}=\frac{22}{201}$, il rapporto vale anche per i raggi r e R rispettivamente di XYD, e di ABC, $K=\frac{R}{r} \Rightarrow r=\frac{R}{K}=\frac{21\sqrt{1005}}{110}$
(Alessandro Avellino)

Problema 86 [1483]
Per prima cosa notiamo che, dato che $\frac{x_1+x_2}{x_3+x_4}$ è intero, abbiamo che $x_1+x_2 \ge x_3+x_4$, e similmente $x_3+x_4 \ge x_5+x_6 \ge x_1+x_2$.
Dunque $x_1+x_2= x_3+x_4=x_5+x_6$. Analogamente $x_2+x_3=x_4+x_5=x_6+x_1$.

Sia ora $S=x_1+x_2= x_3+x_4=x_5+x_6$ e $T=x_2+x_3= x_4+x_5=x_6+x_1$.
Abbiamo quindi che $\sum x_i = 3S=3T$, quindi $S=T$, ossia $x_1+x_2=x_2+x_3$, da cui $x_1=x_3$. In modo analogo otteniamo $x_1=x_3=x_5$ e $x_2=x_4=x_6$.

Possiamo quindi scrivere la sestupla come $(a, b, a, b, a, b)$, con $a, b$ interi positivi. Si verifica facilmente che ogni sestupla siffatta rispetta le condizioni del testo, difatti la somma di due termini consecutivi è costante, dunque le frazioni sono tutte uguali a $1$ e quindi intere.
Dunque, fissati $a, b$, c'è una e una sola sestupla valida.

Consideriamo ora due casi:

$\bullet$ $M$ non è multiplo di $3$. Allora, dato che $M=3S$, non ci sono sestuple valide, e quindi $f(M)=0$, che è un multiplo di $5$. In questo caso abbiamo quindi $2022-\frac{2022}{3}=1348$ possibili valori di $M$.

$\bullet$ $M=3k$, per qualche $k$ intero positivo. Dato che $k=a+b$, per ogni valore di $k$, ci sono $k-1$ sestuple valide, poiché abbiamo $k-1$ possibili valori di $a$, dunque $f(M)=k-1$. Vogliamo quindi che $k-1$ sia multiplo di $5$. Se $k=1$, non ci sono sestuple valide in quanto $a+b>1$, altrimenti, potendo scegliere $k$ nell'intervallo $[2, 1348]$, ci sono ${\lfloor \frac{1347}{5} \rfloor}=134$ valori accettabili.

In totale abbiamo quindi $1348+1+134=1483$ possibili valori di $M$.
(Valeria Martinelli)

Problema 87

Problema 88 [6418]
Osserviamo che, prese $3$ cifre distinte $a, b, c$ dall'insieme $X=\{1, 2, 3, 4, 5, 6, 7, 8, ,9 \}$, ci sono esattamente $6$ numeri compresi tra $100$ e $999$ che si scrivono con tutte e sole le cifre $a, b, c$, siano questi $x_1, ..., x_6$. Abbiamo che la loro somma $x_1+...+x_6$ vale $222a+222b+222c$, quindi $\sum f(x_i)=\frac{222(a+b+c)}{a+b+c}=222$.
Ci sono $\displaystyle\binom{9}{3}=84$ modi di scegliere $a, b, c$, quindi la somma in questo caso vale $222\cdot 84=18648$

Dobbiamo ora distinguere i casi in cui $a, b, c$ contengono uno o più zeri o cifre ripetute.
$\bullet$ $a=b=c$: per ognuna delle $9$ possibili scelte di $a, b, c$, c'è uno e un solo numero $x=111a$, e $f(x)=\frac{111a}{3a}=37$. Dunque la somma in questo caso vale $37 \cdot 9=333$

$\bullet$ $a=b \neq c$, $a, b, c \neq 0$: abbiamo $9\cdot 8=72$ possibili scelte di $a, b, c$, e per ogni terna abbiamo $3$ numeri la cui somma vale $222a+111c$, per cui $\sum f(x_i)=\frac{222a+111c}{2a+c}=111$. In tutto $72 \cdot 111=7992$

$\bullet$ $b=c=0$: $9$ possibili scelte di $a$, e per ogni $x=100a$ che rispetta queste condizioni $f(x)=100$. La somma in questo caso vale $100 \cdot 9=900$

$\bullet$ $a=b$, $c=0$; $9$ possibili scelte per $a, b$, per ognuna $\sum f(x_i)=\frac{211a}{2a}=\frac{211}{2}$. Quindi in tutto $9\cdot \frac{211}{2}=949,5$

$\bullet$ $a\neq b$, $c=0$: $\displaystyle\binom{9}{2}=36$ possibilità per $a, b$; per ognuna $\sum f(x_i)=\frac{211a+211b}{a+b}=211$. In tutto $36 \cdot 211=7596$

La somma richiesta è quindi uguale a $18648+333+7992+900+949,5+7596=36418,5$. Ci interessano le ultime quattro cifre della parte intera.
(Valeria Martinelli)

Problema 89 [134]
Sia $2^k$ la massima potenza di $2$ nell'intervallo $\{1, ..., n\}$. Si osserva facilmente che $f(n)$ non può mai essere maggiore di $f(2^k)$, pertanto $f(n)$ è massima quando $n=2^k$, e $g(2^k)=1$.

Dato che $5>4=2^2$ e $9=3^2>2^3=8$, se un intero contiene nella sua fattorizzazione primi diversi da $2$ e $3$, o se $3$ compare con un esponente maggiore o uguale a $2$, $g(n)=g(n-1)$ in quanto $n$ ha sicuramente meno fattori di $2^k$.
Inoltre, se $n=2^{k-1}\cdot 3$, $g(n)=g(n-1)+1$, in quanto n contiene lo stesso numero di fattori di $2^k$.

Dunque, se $2^{k-1}\cdot 3\le n <2^{k}$, $g(n)=2$, altrimenti $g(n)=1$.

Possiamo quindi trovare tutti gli interi minori o uguali a $99$ per cui $g(n)=2$, che sono i valori compresi tra i tripli delle potenze di $2$ e la potenza di $2$ successiva.

Dunque la somma richiesta è uguale a $99+(4-3)+(8-6)+(16-12)+(32-24)+(64-48)+(100-96)=99+1+2+4+8+16+4=99+31+4=134$.
(Valeria Martinelli)

Problema 90 [8784]
Chiamiamo $f(n)=x^n+\frac{1}{x^n}$, il problema ci chiede di calcolare $|f(1)\cdot f(2)\cdot\dots\cdot f(2022)|$.
Esplorando un po’ la situazione si può notare che $f(1)=1,f(2)=-1,f(3)=-2,f(4)=-1,f(5)=1$ e $f(6)=2$ e che questi valori potrebbero formare un ciclo che si ripete ogni $6$.
Dimostriamolo:
consideriamo $f(n) \cdot f(1)= \left( x^n+\frac{1}{x^n} \right) \cdot \left(x+ \frac{1}{x} \right) = x^{n+1} + x^{n-1} + \frac{1}{x^{n-1}} + \frac{1}{x^{n+1}} = f(n+1) + f(n-1)$, essendo $f(1)=1$ otteniamo che $f(n) \cdot f(1) =f(n)= f(n+1) + f(n-1)$ da cui $f(n+1)=f(n)-f(n-1)$. L’ultima relazione ci indica che $f(n+1)$ dipende unicamente da $f(n)$ e $f(n-1)$ ovvero se troviamo un ciclo quest’ultimo si ripeterà. Controllando si può vedere che $f(7)=1$ e $f(8)=-1$ quindi il ciclo si ripeterà. Analizzando il nostro ciclo si nota facilmente che gli unici fattori che contribuiranno nel nostro prodotto saranno quelli con $n$ multiplo di $3$, che in valore assoluto valgono $2$. Il nostro prodotto sarà quindi del tipo $2^k$ dove $k$ è il numero di multipli di $3$ tra $1$ e $2022$ ovvero $674$ $(\frac{2022}{3}=674)$. Il problema si è trasformato in un quesito di teoria dei numeri poiché dobbiamo dare come risposta le ultime $4$ cifre di $2^{674}$ quindi calcolare $2^{674}$ $(\mathrm{mod}\,10000)$. Riducendo con un po’ di conti l’enorme numero si trova $8784$.
(Lorenzo Bastioni)

Problema 91

Problema 92

Problema 93 [279]
$x$,$y$ e $z$ sono interscambiabili. Per cui è possibile, per facilitarci nella risoluzione, porre $x \geq y \geq z$.

Adesso andiamo in ordine:\\
abbiamo

$x^2<x^2+2y+z \leq x^2+3x < x^2+4x+4=(x+2)^2$

e siccome lo vogliamo come quadrato

$x^2+2y+z=(x+1)^2 \Rightarrow 2y+z=2x+1 \Rightarrow x=\frac{2y+z-1}{2}$.

Ora usiamo la seconda equazione con le nuove sostituzioni.\\
$y^2<y^2+2z+x=y^2+2z+\frac{2y+z-1}{2}=y^2+y+2.5(z)-0.5<y^2+y+3y+4=(y+2)^2$.

Per lo stesso motivo di prima:

$y^2+y+2.5(z)-0.5=(y+1)^2 \Rightarrow 5z-3=2y \Rightarrow y=\frac{5z-3}{2}$\\
Ora è il momento di usare l'ultima equazione, anche qui sostituendo $y$ e $x$:

$z^2<z^2+2x+y=z^2+2y+z-1+y=z^2+z+3(\frac{5z-3}{2})-1=z^2+8.5z-4.5$.

Dopo vari tentativi si nota che gli unici quadrati possibili per questa quantità sono $(x+1)^2$ e $(x+4)^2$.\\

Infatti si avrà $z^2+8.5z-4.5=z^2+8z+16 \Rightarrow z=43$, trovandoti poi $y=106$ e $x=127$ e $z^2+8.5z-4.5=z^2+2z+1$ trovando quindi $x=z=y=1$.
(Alessandro Avellino)

Problema 94 [6460]
Possiamo riscrivere il testo del problema nel seguente modo:
$$\sum_{n=1}^{2020} \lfloor \frac{\sqrt{n+2}}{\sqrt{n+1} - \sqrt{n}} \rfloor$$
Andiamo ad analizzare il nostro addendo generale privo della funzione floor:
$ \frac{\sqrt{n+2}}{\sqrt{n+1} - \sqrt{n}} = \frac{\sqrt{n+2}}{\sqrt{n+1} - \sqrt{n}} \cdot \frac{\sqrt{n+1} + \sqrt{n}}{\sqrt{n+1} + \sqrt{n}} = \sqrt{(n+2) \cdot (n+1) } + \sqrt{ (n+2) \cdot n } = \sqrt{n^2 + 3n+ 2} + \sqrt{n^2 +2n} $.
Esplorando un po' la situazione ci si potrebbe convincere che $2n+2 \leq \sqrt{n^2 + 3n+ 2} + \sqrt{n^2 +2n} < 2n+3$ (con $n$ nell'intervallo che interessa a noi ovvero $[1,2020]$).
Dimostriamolo:
La seconda disuguaglianza è molto semplice infatti $\sqrt{n^2 + 3n+ 2} + \sqrt{n^2 +2n} = \sqrt{(n+2)^2 -n-2} + \sqrt{(n+1)^2 - 1} < \sqrt{(n+2)^2} + \sqrt{(n+1)^2} = n+2 +n+1 = 2n+3$ quindi $\sqrt{n^2 + 3n+ 2} + \sqrt{n^2 +2n} < 2n+3$.
La prima disuguaglianza è leggermente più fastidiosa:
vogliamo $2n+2 \leq \sqrt{n^2 + 3n+ 2} + \sqrt{n^2 +2n}$ che è molto lungo da risolvere a mano dunque proviamo a scomporre in maniera intelligente il problema:
ad esempio, se dimostrassimo che $\sqrt{n^2 + 3n+ 2} \geq n + \frac{4}{3} \wedge \sqrt{n^2 +2n} \geq n+\frac{2}{3}$ avremmo finito poiché sommando le disuguaglianze si otterrebbe la tesi (la veridicità delle due disuguaglianze implica la tesi ma non viceversa).
Le due disuguaglianze sono due semplici disequazioni irrazionali e non ci dovremmo preoccupare di condizioni di esistenza o simili poiché nel nostro intervallo è tutto positivo:
1)
$ \sqrt{n^2 + 3n+ 2} \geq n + \frac{4}{3} \rightarrow n^2+3n+2 \geq (n + \frac{4}{3})^2 \rightarrow n \geq - \frac{2}{9}$, nel nostro intervallo la disuguaglianza vale.
2)
$ \sqrt{n^2 +2n} \geq n+\frac{2}{3} \rightarrow n^2+2n \geq (n+\frac{2}{3})^2 \rightarrow n \geq \frac{2}{3} $, nel nostro intervallo la disuguaglianza vale.
La tesi è dunque dimostrata, $2n+2 \leq \sqrt{n^2 + 3n+ 2} + \sqrt{n^2 +2n} < 2n+3$. Ciò implica che $\lfloor \frac{\sqrt{n+2}}{\sqrt{n+1} - \sqrt{n}} \rfloor = 2n+2$ (nel nostro intervallo) e possiamo riscrivere la nostra somma:
$$\sum_{n=1}^{2020} 2n+2 = \left( 2 \sum_{n=1}^{2020} n \right) + 2 \cdot 2020 = 2020 \cdot 2021 + 4040 = 4086460$$
(La seconda uguaglianza si ha con la formula di Gauss per la somma dei primi $k$ numeri naturali) Prendendo le ultime $4$ cifre del risultato abbiamo la soluzione: $6460$.
(Lorenzo Bastioni)

Problema 95 [21]
Mettiamo in un piano cartesiano i punti [math]. Si ha che [math] è il coffieciente angolare della retta per i due punti [math]. Quindi una sequenza è non genovese se tutti i coefficienti angolari sono non negativi; ovvero se e solo se essa è debolmente crescente. Supponiamo di avere una sequenza debolmente crescente [math]. Vi è una bigezione fra le sequenze di questo tipo e quelle del tipo [math], che sono tutte le sequenze strettamente crescenti con elementi positivi minori di [math]. Esse a loro volta corrispondono con i modi di scegliere [math] elementi distinti fra [math]. Il numero totale di sequenze (genovesi e non) è infine [math]; quindi il numero di quelle genovesi è [math] che è congruo a [math] modulo [math], essendo che [math] modulo [math].
(Federico Volpe)

Problema 96 [2029]
I numeri curiosi sono esattamente i liberi dai quadrati.\\
($\Leftarrow$) Se $n$ è un numero libero da quadrati allora $\tau(n)$ è una potenza di due, inoltre ogni divisore $d$ di $n$ è un numero libero da quadrati, quindi anche $\tau(d)$ è una potenza di $2$ e dato che $\tau(d) \le \tau(n)$ abbiamo che $\tau(d)|\tau(n)$.\\
($\Rightarrow$) Supponiamo che $n = p_1^{a_1}p_2^{a_2}\cdots p_m^{a_m}$ sia un numero libero da quadrati. Allora abbiamo
\begin{align*}
&\tau(n/p_i)|\tau(n)\\
\Rightarrow &(a_1+1)\cdots(a_i)\cdots(a_m+1)|(a_1+1)\cdots(a_i+1)\cdots(a_m+1)\\
\Rightarrow &a_i | a_i+1 \Rightarrow a_i = 1.
\end{align*}
Dunque $n$ è libero da quadrati.\\
\\
Inoltre notiamo che $f(a,b)$ è semplicemente $MCD(a,b)$. A questo punto scriviamo
$$S(n) = \sum_{j=1}^n f(j,n) = \sum_{j=1}^n MCD(j,n),$$
dimostreremo che la funzione $S$ è moltiplicativa, cioè per ogni $m,n \in \mathbb{Z}^+$ coprimi $S(m)S(n) = S(mn)$. Infatti, segue dal teorema del resto cinese, possiamo rimpiazzare $j \text{ mod }\,mn$ con due numeri $j_m \text{ mod }\,m$ e $j_n \text{ mod }n$ in modo che:
\begin{align*}
S(mn) &= \sum_{j=1}^{mn} MCD(j,mn) = \sum_{j=1}^{mn} MCD(j,m)MCD(j,n)\\
&= \sum_{j_m=1}^{m} \sum_{j_n =1}^{n} MCD(j_m,m)MCD(j_n,n)\\
&= \left(\sum_{j_m=1}^m MCD(j_m,m)\right)\left(\sum_{j_n=1}^n MCD(j_n,n)\right) = S(m)S(n).
\end{align*}
Ma quindi dato che $C$ è un prodotto tra primi ed è facile verificare che $S(p) > 1$ per ogni primo $p$, abbiamo che $C$ stesso deve essere primo. Un po' di brute-force dà la risposta.
(Michele Tomasi)

Problema 97

Problema 98

Problema 99

Problema 100 [1339]
Premetto innanzitutto che questo geometrico è più tecnico degli altri ed è necessario avere qualche prerequisito sulle simmediane e il cerchio di Feuerbach. Tutti i fatti noti di cui farò uso si trovano sul libro di Evan Chen ma sono certo che si possano trovare anche girovagando su internet.

La soluzione di questo problema si può suddividere in tre claim.

Primo claim: La tangente di $\Gamma$ in $N_a$, la retta $BC$ e la tangente di $\Gamma$ in $A$ concorrono nel punto $X_a$.\\
Qua c'è solo da usare qualche fatto noto delle simmediane. Infatti $\triangle BAN_a \sim \triangle CAM_a \Rightarrow \angle BAN_a= \angle CAM_a \Rightarrow AN_a$ è la $A-simmediana$ di $\triangle ABC$. Dopodiché basta usare che $CB$ è la $C-simmediana$ di $\triangle ACN_a$ e quindi le tangenti in $A$ e in $N_a$ concorrono con essa.

Secondo claim:
$Y_a$ e simili stanno sulla circonferenza di Feuerbach, ovvero $\gamma$ e $\omega$ coincidono.
Per essere capaci di dimostrarlo però è necessario notare qualcosa di più subdolo.
Sia $H$ l'ortocentro di $\triangle ABC$, $V$ il punto medio di $AH$ e $Q$ l'intersezione tra la retta $AH$ e $\Gamma$. Siano inoltre $R$ e $r$ i raggi di $\Gamma$ e di $\gamma$.
Lemma: $V$ è L'ortocentro di $\triangle AM_{a}X_{a}$.
Proof: Dimostriamo innanzitutto che la retta $VM_a$ è perpendicolare a $AX_a$. CIò è equivalente a dimostrare che $\angle X_{a}M_{a}V = 90^{\circ}- \angle AX_{a}M_a=\angle X_{a}AP_{a} \Leftrightarrow \angle P_{a}M_{a}V=\angle X_{a}AQ \Leftrightarrow \frac{VP_{a}}{AQ}=\frac{r}{R}=\frac{1}{2}$.
Poiché $Q$ è il simmetrico di $H$ su $BC$ (si dimostra con angle chasing), $HQ+HA=2HP_{a}+2HV \Rightarrow AQ=2VP_{a} \Rightarrow \frac{VP_{a}}{AQ}=\frac{1}{2}$.
$AV \perp X_{a}M_{a}$ e $M_{a}V \perp AX_{a} \Rightarrow V$ ortocentro di $\triangle AM_{a}X_{a}$.
A questo punto per dimostrare il claim iniziale basta dire che, poichè $V$ è ortocentro, $X_{a},V,Y_{a}$ sono allineati e quindi $\angle VY_{A}M_{a}=90^{\circ}=\angle VP_{a}M_{a} \Rightarrow VY_{a}M_{a}P_{a}$ ciclico $\Rightarrow Y_{a}$ sta su $\gamma$.
Similarmente si può dimostrare che anche $Y_{b}$ e $Y_{c}$ stanno su $\gamma$.

Terzo claim: le rette $P_{a}P_{b}, DE, CH$ concorrono in $F$.
$\angle BHC=180^{\circ}- \angle BAC \Rightarrow H$ sta su $\Gamma'$. Ora basta osservare che $P_{a}P_{b}, DE, CH$ sono gli assi radicali di $\gamma, \Gamma', (P_{a}HP_{b}C)$.
Similarmente si dimostra che $P_{a}P_{c}, DE, BH$ concorrono in $G$.

Ora che abbiamo delle informazioni "decenti" sui punti $F$ e $G$ possiamo fare del segment chasing per trovare i valori di $P_{a}F$ e $P_{a}G$. $\triangle P_{a}P_{b}P_{c}$ è il famoso triangolo ortico e $P_{c}G$ e $P_{b}F$ sono le bisettrici, quindi l'idea sarebbe quella di trovare i tre lati e poi usare il teorema delle bisettrici.
$BC^{2}-P_{b}C^{2}=AB^{2}-(AC-P_{b}C)^{2} \Rightarrow P_{b}C=\frac{84}{5}$.
$\triangle BCA \sim \triangle P_{b}CP_{a} \Rightarrow \frac{P_{a}P_{b}}{P_{b}C}=\frac{AB}{BC} \Rightarrow P_{a}P_{b}=\frac{78}{5}$.
Similarmente si trovano $P_{b}P_{c}=\frac{924}{65}$ e $P_{a}P_{c}=\frac{150}{13}$
$\frac{P_{a}F}{P_{a}P_{c}-P_{a}F}=\frac{P_{b}P_{a}}{P_{b}P_{c}} \Rightarrow P_{a}F=\frac{1950}{323}$
$\frac{P_{a}G}{P_{a}P_{b}-P_{a}G}=\frac{P_{c}P_{a}}{P_{c}P_{b}} \Rightarrow P_{a}G=\frac{650}{93}$
$P_{a}F+P_{a}G=\frac{391300}{30039}$. La risposta è quindi $1339$.
(Denis Tusca)
[/quote]
[/quote]
[/quote]
[/quote]
Valemart
Messaggi: 5
Iscritto il: 17 apr 2022, 12:46

Re: #Proviamoci - Soluzioni commentate OH6

Messaggio da Valemart »

Problema 1 [3]
$n$ il numero di commensali e $d$ la distanza tra loro, i commensali sono i punti ${A_1, A_2,..., A_n}$ mettiamo $A_1$ in un punto qualsiasi del piano. $A_2$ potrà stare ovunque nella circonferenza di raggio $d$ e centro $A_1$. Il terzo punto ha solo $2$ posizioni in cui poter stare, che sono le intersezioni delle $2$ circonferenze di centro $A_1$ e $A_2$, entrambe di raggio $d$. Posizionato il terzo punto e tracciata la terza circonferenza noteremo che non ci sarà nessun punto in comune a tutte e $3$ le circonferenze. Di conseguenza non ci può essere un quarto punto che rispetta le condizioni.
(Alessandro Avellino)

Problema 2 [6]
Si tratta di disporre tre diversi ingredienti, che si può fare semplicemente in $3!=6$ modi diversi. La risposta è quindi $6$.
(Matteo Salicandro)

Problema 3 [228]
Il massimo numero ottenibile lanciando gli $n$ dadi è $n\cdot k$, il minimo è $n\cdot 1=n$. Pertanto $n\cdot k-n=168$, cioè $n(k-1)=168$. Questo vuol dire che $n$ è un qualsiasi divisore positivo di $168$, esclusi $168$ e $84$ visto che $k \geq 4$. $168=2^3\cdot 3\cdot 7$, quindi la somma dei suoi divisori è $(2^4-1)(3+1)(7+1)=480$. La somma dei possibili valori di $n$ è pertanto $480-168-84=228$.
(Federico Borasio)

Problema 4

Problema 5

Problema 6 [36]
Un rettangolo con le diagonali perpendicolari è un quadrato. L'area è quindi $6^2=36$.
(Filippo Prandina)

Problema 7 [18]
La successione è definita, per $n \geq 1$, come
$\begin{cases}
x_{n+2}=x_{n+1}+x_n\\
x_6=76\\
x_5=47
\end{cases}$
Da qui ricaviamo che $x_n=x_{n+2}-x_{n+1}$, pertanto $x_4=76-49=29$ e $x_3=47-29=18$, che è il terzo termine della successione.
(Federico Borasio)

Problema 8 [2022]
Supponiamo che ci siano $2022$ fisici. Tutti dicono la verità, cioè che non sono matematici: non c'è alcuna contraddizione. $2022$ è anche il numero di abitanti dell'isola, quindi è il massimo numero di fisici.

Problema 9 [8]
$\frac{4n}{32}=k$ con $k\in N\geq 0$. Quindi $n=8k$. Il minimo si ha quando $k=1$ e $n=8$.
(Filippo Prandina)

Problema 10[8]
Sono possibili due casi, in base a qual è la circonferenza con raggio minore. Tuttavia, r deve essere massimo, quindi consideriamo il caso in cui r$>$5. L'area della corona circolare sarà data da
$\pi{r}^{2}-{5}^{2}\pi=\pi({r}^{2}-25)=16\pi.$ Per cui ${r}^{2}-25=16$, ovvero $r=\sqrt{41}\approx{6.403}$, la cui parte intera è 6.
(Daniele Prisco)

Problema 11 [9999]
Ci sono più soluzioni in quanto i primi due termini possono essere $1,3$, oppure $2,2$ oppure $3,1$. La risposta è quindi $9999$.
(Matteo Salicandro)

Problema 12[1261]
Affinché una parola sia impronunciabile per il nostro Loenzo, bisogna che contenga almeno una R. Se c'è una sola R nella parola, questa potrà essere la prima, la seconda o la terza lettera, e in ognuno di questi casi ci saranno 20*20=400 parole possibili, dato che nei restanti due 'posti' vanno inserite le altre 20 lettere dell'alfabeto, che possono essere ripetute. Se nella parola ci sono due R, esse si possono disporre in 3 modi, e per ognuno si potranno formare 20 parole. Infine, va incluso l'unico caso in cui la parola è formata da tre R.
In totale, i casi sono $400*3+20*3+1=1261$.
(Daniele Prisco)

Problema 13 [28]
$X$ appartiene alla perpendicolare di $BC$ passante per $A$. Inoltre $X$ e $2$ punti su $BC$ formano lo stesso angolo che forma $A$ con quei $2$ punti, di conseguenza $X$ è il simmetrico di $A$. Da questo deduciamo che $AX=2AD$, dove $AD$ è l'altezza.
(Alessandro Avellino)

Problema 14 [1011]
La quantità in questione, $(3n+2)^{5n+8}$, è un quadrato perfetto se lo è la base, cioè $3n+2$, oppure se l'esponente, $5n+8$, è pari. Tuttavia la prima possibilità non si verifica mai poiché un quadrato non è mai congruo a $2\,(\mathrm{mod}\,3)$. Dunque la risposta è semplicemente il numero di interi positivi $1\leq n\leq 2022$ tali che la quantità $5n+8$ sia pari, ovvero tutti e soli i numeri pari in questo intervallo, che sono $\frac{2022}{2}=1011$.
(Lorenzo Weiss)

Problema 15

Problema 16 [2019]
Il problema chiede per quanti interi positivi minori o uguali a $2022$ accade che $\lfloor \frac{\pi +n}{n}\rfloor =\lfloor \frac{\pi}{n}+1\rfloor =\lfloor \frac{\pi}{n}\rfloor +1=1\Longrightarrow \lfloor \frac{\pi}{n}\rfloor =0$, il che succede solo quando $\pi\approx 3,14\leq n$, i.e. in $2022-(4-1)=2019$ casi.
(Lorenzo Weiss)

Problema 17

Problema 18

Problema 19 [14]
Disegnando la circonferenza $x^2+y^2=5$ risulta evidente che gli unici punti a coordinate intere che vi appartengono, che sono $k=8$, sono quelli della forma $P_i(\pm 1,\pm 2)$, $P_i(\pm 2,\pm 1)$, $P_i(\pm 1,\mp 2)$, $P_i(\pm 2,\mp 1)$. In alternativa, è possibile risolvere l'equazione della stessa circonferenza sugli interi $x,y$. Da ciò si ricava l'ottagono $P_1P_2\dots P_8$, la cui area, come si vede facilmente, è $16-4\cdot\frac{1}{2}=14$.
(Lorenzo Weiss)

Problema 20

Problema 21 [210]
Notiamo che, comunque presi $6$ elementi distinti dall'insieme $X=\{1, 2, 3, 4, 5, 6, 7, 8, 9, 10\}$, c'è uno e un solo modo di disporli in ordine crescente, dunque la richiesta del problema è equivalente al trovare il numero di sottoinsiemi di $6$ elementi di $X$, che sono $\displaystyle\binom{10}{6}=210$.
(Valeria Martinelli)

Problema 22 [2022]
Si può ottenere $2$ solo quando una delle cifre è $2$ e le altre sono nulle, oppure quando due cifre sono uguali a $1$ e le altre sono nulle. Nel primo caso, la prima cifra sarà necessariamente $2$, altrimenti il numero non avrà $2022$ cifre: si ottiene solo una possibilità.
Nel secondo caso, invece, la prima cifra sarà $1$, mentre l'altro $1$ occuperà una delle altre $2021$ posizioni disponibili, ottenendo $2021$ possibilità.
Sommando le possibilità, si arriva a $2022$.
(Daniele Prisco)

Problema 23

Problema 24 [4042]
Se $x$ è la lunghezza del segmento $PA$ e $r$ è il raggio della circonferenza,
$OP=OA+PA=r+x=2021$; $PB=OA+OB+PA=r+r+x$. Di conseguenza, $PA+PB=x+r+r+x=2(r+x)=2×2021=4042$.
(Daniele Prisco)

Problema 25 [66]
Poniamo $a+b=x$ e $c+d=y$.
La disuguaglianza adesso è
$x+y\leq \sqrt{x^2+y^2} \Rightarrow x^2+2xy+y^2\leq x^2+y^2$
Da cui $2xy\leq0$ ma siccome $x$ e $y$ sono somme di interi maggiori o uguali a $0$, l'unico modo per cui $xy=0$ si ha quando uno dei $2$, o entrambi, è uguale a $0$. Se $c+d=y=0$ da cui $c=d=0$, quindi $(a,b,0,0)$. Ricordiamo che $a \geq b$ quindi i possibili valori sono $66$.
(Alessandro Avellino)

Problema 26 [6856]
Sia $n$ il numero di circonferenze sulla prima riga. Avendo le circonferenze diametro $2$, la base del rettangolo è uguale a $2n$. Invece, essendo tutte le circonferenze tangenti, i centri di quella più alta e delle due agli estremi della base sono disposte sui vertici di un triangolo equilatero, di lato $2n-2$. L'altezza del rettangolo è quindi uguale all'altezza del triangolo aumentata di $2$, ossia $\sqrt{3}(n-1)+2$.
Quindi vogliamo $4n+2\sqrt{3}(n-1)+4>2022$, che ci dà immediatamente $n=271$, quindi $k=\frac{271\cdot 272}{2}=36856$.
La risposta voluta sono le ultime quattro cifre di $k$.
(Valeria Martinelli)

Problema 27 [250]
Intanto notiamo che $78125$ è $5^7$.
La somma di $M$ termini consecutivi con primo termine $a$ è del tipo:
$a+(a+1)+(a+2)+...+(a+M-1)$ nel quale compaiono infatti $M$ termini consecutivi.
È quindi possibile raccogliere come $a \cdot M + (1+2+3+...+M-1)=a \cdot M+ \frac{M(M-1)}{2}=M(a+\frac{M-1}{2})=78125 \Rightarrow M(2a-1+M)=5^7\cdot2$, $M<2a-1+M$ quindi al massimo $M=2 \cdot 5^3$.
(Alessandro Avellino)

Problema 28

Problema 29

Problema 30

Problema 31 [20]
Il doppio delle cifre di $n$ dovrà necessariamente essere un numero di $3$ cifre ognuna delle quali minore di $5$. Una volta individuato il numero più piccolo $n$ come $101\,(2n=202)$ e il massimo $n$ come $444\,(2n=888)$ i restanti saranno compresi tra essi. Ci saranno $18$ numeri palindromi tra questi due estremi, con la cifra intermedia contente al più il $4$ e gli estremi uguali da $1$ a $4$. I numeri saranno $101$, $111$, $121$, $131$, $141$, $202$, $222$… fino a $444$. Il totale è di $20$ numeri bipalindromi.
(Irene Mancone)


Problema 32

Problema 33 [74]
Il costo di un pallone ($C$) può essere pari o dispari. Se è pari, può essere pagato con $\frac{C}{2}$ monete da $2$ orue. Poiché $\frac{C}{2}$ è pari, $C$ sarà un multiplo di $4$, quindi potrà assumere $37$ valori.
Se $C$ è dispari, invece, il pallone può essere pagato con $\frac{C-1}{2}$ monete da $2$ orue e una moneta da un orue. Di conseguenza, $C-1$ è un multiplo di $2$, ma $\frac{C-1}{2}$ è dispari (aggiungendo una moneta da un orue, il numero di monete usate è pari). Di conseguenza, $C-1$ è multiplo di $2$ ma non di $4$, quindi può assumere altri $37$ valori.
La somma dei valori possibili è dunque $74$.
(Daniele Prisco)

Problema 34 [2023]
Notiamo che possiamo riscrivere il testo nella seguente maniera: $P((x-y)(x+y))=P(x-y)P(x+y)$.
Notiamo anche che sostituendo $x-y \mapsto z$ e $x+y \mapsto w$ la coppia $(z,w)$ può assumere qualsiasi coppia di valori in $\mathbb{R}^2$ poiché il sistema
$\begin{cases}
x-y=z\\
x+y=w
\end{cases}$
ha soluzioni per ogni coppia $(z,w)$ fissata.
Dunque il nostro problema è equivalente a $P(zw)=P(z)P(y)$ per ogni $z$ e $w$ in $\mathbb{R}$. Questa relazione è molto simile ad una delle famose equazioni funzionali di Cauchy, $f(xy)=f(x)f(y)$. Quest'ultima equazione citata ha soluzione $f(x)=x^n$ o $f(x)=0$ (con $n$ naturale) se la funzione è continua (la continuità è una delle tante condizioni sufficienti), ma un polinomio come $P$ è continuo dunque le possibili soluzioni sono quelle riportate con $n$ che va da $0$ a $2021$ per $P(x)=x^n$ più $P(x)=0$. La soluzione è quindi $2023$ polinomi possibili.
(Lorenzo Bastioni)

Problema 35

Problema 36 [2020]
Si noti che $abc=2022-ab-a$, quindi per massimizzare il prodotto bisogna minimizzare $a$ e $b$. $a,b,c$ sono $\geq 1$ per ipotesi, dunque si pone $a=b=1$ dando come soluzione $c=2020$. Quindi $(a,b,c)=(1,1,2020)$ e $abc=2020$.
(Filippo Prandina)

Problema 37 [201]
Poniamo [math] e notiamo che [math] per [math]. Ora possiamo costruire il seguente polinomio: [math] di cui conosciamo gli zeri [math] e poiché [math] è chiaramente monico possiamo affermare che [math]. Sostituendo infine [math] con [math] e ricavando [math] otteniamo che [math] da cui possiamo trovare la soluzione al problema calcolando [math]. La risposta è quindi [math].
(Lorenzo Bastioni)

Problema 38

Problema 39

Problema 40

Problema 41

Problema 42

Problema 43 [4238]
Se si devono prendere almeno $n$ dolci per essere sicuri di prenderne almeno $1$ di un determinato tipo, questo vuol dire che ci sono $n-1$ dolci non di quel tipo. In particolare:
$\begin{cases}
S+B=55\\
B+C=70\\
S+C=91
\end{cases}$
Il sistema è di immediata risoluzione e porta come soluzione $(S,C,B)=(38,53,17)$ da cui $SCB=34238$. Sono richieste solo le ultime $4$ cifre.
(Filippo Prandina)

Problema 44 [4]
La configurazione è unica, infatti se prendiamo il punto $A_1$ e lo collegassimo con un punto diverso da $B_{2022}$, come per esempio $B_{2021}$, $B_{2022}$ sarà collegato con un punto diverso da $A_1$, per esempio $A_2$. Allora $A_1B_{2021}$ non interseca $A_2B_{2022}$. Iterando questo ragionamento ad ogni punto $A_i$, si ottiene un'unica configurazione: segmenti di estremi $A_iB_{2023-i}$. Quindi i coefficienti angolari saranno $\pm 2/(2023-2i)$. Per il prodotto, il numeratore è sempre $2$ mentre il denominatore è sempre dispari. Trovare quindi ciò che il problema richiede equivale a trovare le ultime $2$ cifre di $2^{2022}$.
(Alessandro Avellino)

Problema 45 [18]
Notiamo che, comunque presa una casella, il numero di giorni che Alberto o Barbara impiegano per raggiungerla non dipende dal percorso scelto, e le uniche caselle che hanno la stessa distanza sia da Alberto che da Barbara sono quelle sulla diagonale del quadrato diversa da quella di partenza, dunque i due si incontrano se e solo se entrambi passano per una di queste tre caselle.
Dato che un percorso è valido se e solo se fa $2$ passi in una direzione e $2$ nell'altra, tutte le coppie di percorsi possibili sono $\frac{4!}{2!\cdot 2!}\cdot \frac{4!}{2!\cdot 2!}=36$, quelli in cui entrambi passano per il centro sono $(2!\cdot 2!)(2!\cdot 2!)=16$, mentre quelli in cui entrambi passano in uno dei due angoli sono $2$.
Le coppie di percorsi con cui Alberto e Barbara non si incontrano sono quindi $36-16-2=18$.
(Valeria Martinelli)

Problema 46

Problema 47 [171]
Poiché $wxy+2021$ deve dare un numero pari, $wxy$ dev'essere dispari, e di conseguenza $w$,$x$ e $y$ devono esserlo.

$w=2a+1, x=2b+1, z=2c+1$ quindi $2a+1+2b+1+2c+1=37 \Rightarrow 2(a+b+c)=34 \Rightarrow a+b+c=17$
Adesso è un semplice stars and bars, infatti $a$, $b$ e $c$ possono essere anche nulli. Distribuire $17$ elementi in $3$ contenitori, $\binom{17+3-1}{3-1}=\binom{19}{2}$.
(Alessandro Avellino)

Problema 48 [42]
Notiamo che $CQB~APB$, in quanto hanno $2$ angoli congruenti, entrambi pari a $74$. Da questo consegue anche che sono triangoli isosceli. $\measuredangle CQB=\measuredangle APB=180-2(74)=32$. Inoltre $S$ sta nell'asse di $BC$, di conseguenza anche nella bisettrice di $CBQ$. Ragionamento analogo per $APB$. Ora osserviamo il quadrilatero $PCAQ$. Questo ha gli angoli $APB$ e $CQB$ congruenti, da questo possiamo asserire che $PCAQ$ è ciclico. Questo permette di stabilire che $\measuredangle DAC=\measuredangle PQC$ e che $\measuredangle DCA=\measuredangle QPD$. Infine osserviamo il triangolo $QSP$. $\measuredangle QSP=180-\measuredangle QPS-\measuredangle PQS= 180-(16+\measuredangle PQC)-(16+\measuredangle QPA)=148-(\measuredangle PQC+\measuredangle APQ)=148-(\measuredangle DCA+\measuredangle DAC)=148-(180-\measuredangle ADC)=\measuredangle ADC-32.$ $\measuredangle ADC=360-(\measuredangle DCB+\measuredangle CBA+\measuredangle BAD)=360-3 \cdot 74=138$ da cui $\measuredangle QSP=138-132=106$. Essendo infine $S$ il circocentro di $ABC$, ne consegue che $\measuredangle ASC=2 \measuredangle ABC=148$. Quindi $\measuredangle ASQ + \measuredangle PSC= \measuredangle ASC- \measuredangle QSP=148-106=42$.
(Alessandro Avellino)

Problema 49 [15]
Dovendo esistere almeno un numero $k>1$ che elevato al cubo sia un divisore di $n$ si possono escludere i numeri da $2$ a $7$ in quanto l’unico loro divisore al cubo è $1$ e $k$ non può assumere tale valore. L’$8$ si può vedere come $2^3$ ed è un prigioniero di cubi, così come lo sono tutti i suoi multipli fino a $96$. È valido lo stesso discorso anche per i multipli di $27$ (essendo $3^3$) fino a $81$. Contandoli si arriva al risultato richiesto, $15$.
(Irene Mancone)

Problema 50

Problema 51

Problema 52[1872]
Chiamiamo le tre radici di p(x) a,b,c dove a<b<c . Essendoa,b e c in progressione aritmetica si ha che a = b-r e c = b+r dove r è la ragione della progressione aritmetica. Per le formule di Vietè a+b+c= 39 b-r + b+ b+ r= 39 da cui si ricava che b =13 . Sempre applicando le formule di Vietè, ab + ac + bc = 482 sostituendo i risultati ricavati in precedenza si trova che 13(13-r) + (13-r)(13+r) + 13(13+r) = 482 da cui sviluppando le parentesi 169 -13r +169 -r2 + 169 +13r = 482 ottenendo che r2 = 25 quindi poiché r è maggiore di 0 si ha che r=5. Quindi a=8 c=18 .
Applicando nuovamente le formule di Vietè, -m = -8*13*18 ovvero m= 1872
(Mattia Zunino)

Problema 53 [24]
Poniamo l'età di Marco al momento della prima affermazione uguale a $x$, quindi l'età del padre Luca è $4x$. Allora, $4$ anni dopo l'età di Marco è $x+4$ e quella del padre è $4x+4$, quindi vale l'equazione $3(x+4)=4x+4$; $x=12-4=8$ e infine $4x=32$.
La differenza di età tra i $2$ è $24$, quindi per rispondere alla domanda risolviamo $2y=y+24$, con $y$ l'età di Marco e $y+24$ l'età di Luca, $y=24$ che è quindi la risposta.
(Niso Cicalò)

Problema 54

Problema 55

Problema 56

Problema 57 [183]
Si può considerare il triangolo isoscele con i vertici di coordinate $B (0;0), C (20;0)$ e A$ (10; y)$ con $y > 0$
Il punto $H$ è l’intersezione tra le rette perpendicolari ai lati passanti per $A, B, C$. Sapendo che le distanze tra
$AH$ e $AD$ sono segmenti di lunghezza intera il punto $H$ si potrà trovare al minimo nella coordinata $(10;1)$. Calcolando il coefficiente angolare tra $H$ e i punti $B$ e $C$, le rette passanti per $B$ e $C$ con coefficiente angolare antireciproco si può ottenere l’ordinata di $A$ sostituendo in una delle due equazioni.
$m_{BH}$= $\frac{1}{10}$ $m^1$=$-10$ retta r passante per A e C: $y=-10x+200$
$m_{CH}$=$\frac{-1}{10}$ $m^1$=10 retta s passante per B e A : $y=10x$

Per $H (10,1)$ $A$ vale $(10, 100)$ e la distanza è intera ($99$).
Una volta stabilita l’ordinata massima di $A$ è sufficiente ripetere il procedimento. Per $H (10,2)$ $A (10, 50)$, per $H (10,4)$ $A(10,25)$. Nelle coordinate $(10, 10)$ $H$ e $A$ si sovrappongono. Poi si avrà che $A$ si trova in $(10,20)$, in $(10,2)$ e $(10,5)$. La soluzione si ottiene sommando le distanze trovate tra $H$ e $A$ senza ripetizioni.
(Irene Mancone)

Soluzione alternativa:
Essendo $ABC$ un triangolo isoscele la mediana $AD$ coincide con l'altezza rispetto a $BC$, chiamando $K$ il piede dell'altezza relativa ad $AC$ otteniamo che il triangolo $BKC$ è simile al triangolo $ABD$ poiché $\widehat{BKC} \cong \widehat{BDA} = 90^{\circ}$ e $\widehat{BCK} \cong \widehat{ABD}$ (essendo $ABC$ isoscele). Consideriamo ora i triangoli $BKC$ e $BDH$, anch'essi sono simili poiché $\widehat{BKC} \cong \widehat{BDH} = 90^{\circ}$ e $\widehat{CBK}$ è in comune. Per la proprietà transitiva $ABD \sim BKC$ e $BKC \sim BDH$ $\rightarrow$ $ABD \sim BDH$. Avendo dimostrato che i triangoli $ABD$ e $BDH$ sono simili vale la seguente uguaglianza: $\frac{BD}{AD}=\frac{HD}{BD} \rightarrow BD^2 = AD \cdot HD \rightarrow 100=AD \cdot HD$ (essendo $BD$ la metà di un segmento lungo $20$). Avendo che $AH$ e $AD$ sono di lunghezza intera anche $HD$ sarà di lunghezza intera (Poiché $HD$ è la differenza tra i due segmenti) ed avremo soluzioni limitate a $100=AD \cdot HD$ ovvero $(AD,HD)=(100,1),(50,2),(25,4),(20,5),(10,10),(5,20),(4,25),(2,50),(1,100)$. La soluzione $(AD,HD)=(10,10)$ non ci piace poiché $\widehat{ABC}$ varrebbe $90^{\circ}$ ($D$ sarebbe il centro della circonferenza circoscritta ad $ABC$ e $\widehat{ABC}$ sottenderebbe un diametro) dunque i possibili valori di $AH$ sono le diverse differenze fra le soluzioni ottenute ($AH=|AD-AH|$): $AH = 100-1= 99$, $AH = 50-2 = 48$, $AH = 25-4= 21$, $AH =20-5=15$ (le altre danno gli stessi valori poiché sono soluzioni "specchiate"). La risposta è quindi $99+48+21+15=183$.
(Lorenzo Bastioni)

Problema 58 [46]
La somma $S$ dei quadrati di delle radici un polinomio del tipo $ x^n + \alpha_{n-1} \cdot x^{n-1} + ... + \alpha_1 \cdot x + \alpha_0$ è

$S=\alpha_{n-1}^2-2 \cdot \alpha_{n-2}$

$$Q_n(x)=x^n+\sum_{i=0}^{n-1} \sqrt{i} \cdot x^i$$

Per cui, definiamo $S_n$ come la somma dei quadrati delle radici di $Q_n(x)$. Allora $S_n=n-1-2\sqrt{n-2}$ che è razionale solo se anche $2\sqrt{n-2}$ lo è. Questo accade solo quando $n-2$ è un quadrato perfetto, quindi ci sono $45$ valori. A questo aggiungiamo $n=1$, il cui polinomio è quindi $x$, che ha come radici un numero razionale. Quindi la soluzione è $46$.
(Alessandro Avellino)

Problema 59

Problema 60

Problema 61[9098]
Per il criterio di divisibilità per 9, s(n) ≡ n mod 9 dove s(n) è la somma delle cifre di n. Iterando il ragionamento su s(n) si ottiene che A(n)≡n mod9 . Poiché A(n) è un intero compreso fra 0 e 9, A(n) è uguale al resto della divisione di n per 9. I possibili resti della divisione delle potenze di 2021 per 9 sono 5, 7, 8, 4, 2, 1 e poi si ripetono ciclicamente con periodo 6. Perciò la somma richiesta è 2016/6 *(5+7+8+4+2+1) +5+7+8+4+2 = 9098.
(Mattia Zunino)
Problema 62 [1158]
Dette $H$ e $K$, rispettivamente, le proiezioni di $D$ e $C$ su $AB$ e $AH=x$ possiamo calcolare l'altezza del trapezio in due modi diversi servendoci del teorema di Pitagora ed eguagliare le espressioni $DH^2=AD^2-AH^2=33^2-x^2=CK^2=BC^2-BK^2=60^2-(69-6-x)^2$.

Risolvendo l'equazione, si trova l'unico valore $x=\frac{81}{7}$ e l'altezza si può quindi calcolare, ad esempio, come $h=DH=\sqrt{AD^2-AH^2}=\frac{60\sqrt{13}}{7}$. L'area del trapezio vale $A=\frac{(b_1+b_2)\cdot h}{2}=\frac{(AB+CD)\cdot AH}{2}=\frac{(69+6)\cdot (60\sqrt13)}{7\cdot2}\approx 1158,93$; la sua parte intera è $1158$ ed è la soluzione.
(Federico Magnolfi)

Problema 63

Problema 64

Problema 65 [5877]
$X$ si trova sulla superficie del triangolo $AMC$: infatti, se così non fosse, $\widehat{ABC}$ e $\widehat{AXM}$ sarebbero due angoli congruenti che insistono su uno stesso segmento ($AM$) e che appartengono allo stesso semipiano rispetto ad esso: ma ciò non è possibile in quanto significherebbe che $X$ appartiene alla circonferenza circoscritta al trangolo $ABM$, alla quale ovviamente non appartengono punti interni al triangolo $ABM$.

$AXMY$ è ciclico per ipotesi e $\widehat{AYM}$ e $\widehat{AXM}$ sono supplementari in quanto angoli opposti (per quanto prima dimostrato); dunque $\widehat{BYM}$, essendo anch'esso supplementare di $\widehat{AYM}$, è congruente a $\widehat{AXM}$, il quale a sua volta è congruente a $\widehat{ABC}$ per ipotesi.
Avremo allora che il triangolo $MBY$ è isoscele su base $BY$; essendo $M$ punto medio di $BC$, $MB\cong MY\cong MC$ e ciò implica che $BYC$ è rettangolo in $Y$. $CY$ è dunque l'altezza uscente da $C$ e si può calcolare con la formula inversa $CY=\frac{2A}{AB}$ dopo aver ricavato l'area con la formula di Erone o per via trigonometrica.
Si trova che quest'ultima vale $A=1200\sqrt{5}$ e dunque $CY^2=\left(\frac{2A}{AB}\right)^2=\frac{4\cdot1200^2\cdot 5}{70^2}\approx 5877,55$. La sua parte intera è $5877$ ed è la soluzione.
(Federico Magnolfi)

Problema 66 [1152]
Prendiamo l'equazione del problema e consideriamola $(\text{mod}\,7)$: $x^2 \equiv 7y + z \,(\text{mod}\,7) \Rightarrow x^2 \equiv z \, (\text{mod}\,7)$. I quadrati come $x^2$ modulo $7$ possono assumere come valori solo $0,1,2,4$ quindi $z$ potrà assumere tutti e i soli valori $0,1,2,4$ modulo $7$. Nell'intervallo $[1,2022]$ ci sono esattamente $288$ numeri congrui a $0 \, (mod 7)$, $288$ congrui a $1$, $288$ congrui a $2$ e $288$ congrui a $4$. La risposta è quindi $288 \cdot 4=1152$.
(Lorenzo Bastioni)

Problema 67 [3026]
Scriviamo il numeratore come $\prod_{n=2}^{2021}((n-1)+n+(n+1))=\prod_{n=2}^{2021}(3n)=3^{2021}\prod_{n=2}^{2021}(n)$. Si può quindi raccogliere un $3^{2021}$ al numeratore. Inoltre si raccolgono anche una potenza di $3^{1005}$ dato che $n$ può avere $3$ come fattore primo (elevato alla $1,\,2,\,3,\,\dots,6$). Ci sono $ \Big \lfloor \frac{2021}{3} \Big \rfloor=673$ multipli di 3, $ \Big \lfloor \frac{2021}{9} \Big \rfloor =224$ multipli di $9$ e così via: $ \Big \lfloor \frac{2021}{3} \Big \rfloor+ \Big \lfloor \frac{2021}{9} \Big \rfloor + \Big \lfloor \frac{2021}{27} \Big \rfloor + \Big \lfloor \frac{2021}{81} \Big \rfloor + \Big \lfloor \frac{2021}{243} \Big \rfloor + \Big \lfloor \frac{2021}{729} \Big \rfloor =673+224+74+24+8+2=1005$. In tutto quindi si è raccolto $2021+1005=3026$.
(Filippo Prandina)

Problema 68 [1104]
Chiamiamo due insiemi $A$ e $B$ "gemelli di primo grado" se $|A \cap B|=1$ e $1 \in A,B$, in generale chiamiamo due insiemi $A$ e $B$ "gemelli di $n$-esimo grado" se $|A \cap B|=n$ e $n \in A,B$.
Possiamo ora affrontare il conteggio di tutte le coppie di sottoinsiemi gemelle di $\{1,2,3,...,2021,2022 \}$ considerando prima i gemelli di primo grado, di secondo grado fino a quelli di $2022$-esimo grado (Non ci sono coppie di sottoinsiemi di $2023$-esimo grado o di $0$-esimo grado).
Gemelli di primo grado:
se $A$ e $B$ sono di primo grado allora contengono entrambi l'elemento $1$ e non hanno nessun altro elemento in comune. Ci verra comodo per dopo scrivere che gli altri zero elementi in comune si possono scegliere in $\displaystyle \binom{2021}{0}$ modi. Ora tutti gli altri $2021$ elementi possono essere messi o in $A$ o in $B$ o in nessuno dei due (in questo caso la "o" è una o esclusiva), quindi abbiamo 3 possibilità per ognuno dei $2021$ elementi ovvero $3^{2021}$ possibilità. Avremo quindi $\displaystyle \binom{2021}{0} 3^{2021}$ coppie di sottoinsiemi gemelle di primo grado.
In generale siamo pronti ad affrontare il caso di gemelli di $n$-esimo grado:
se $A$ e $B$ sono gemelli di $n$-esimo grado allora contengono entrambi l'elemento $n$ e hanno altri $n-1$ elementi in comune. Gli altri $n-1$ elementi in comune si possono scegliere in $\displaystyle \binom{2021}{n-1}$ modi. E per i restanti $2021-(n-1)=2022-n$ elementi abbiamo $3^{2022-n}$ possibilità come spiegato precedentemente. Avremo quindi $\displaystyle \binom{2021}{n-1} 3^{2022-n}$ coppie di sottoinsiemi gemelle di $n$-esimo grado.
Sommando tutte le possibilità otteniamo:
$$\sum_{k=1}^{2022} \binom{2021}{k-1} 3^{2022-k}$$
Reimpostando la somma sostituendo $k-1 \mapsto k$ otteniamo:
$$\sum_{k=0}^{2021} \binom{2021}{k} 3^{2021-k}$$
Ma questo è esattamente uguale a $(3+1)^{2021}=4^{2021}$ (Per il binomio di Newton o teorema binomiale). Riducendo l'enorme numero $(\text{mod}\,10000)$ si trova che le ultime $4$ cifre che ci interessano sono $1104$, da cui la risposta.
(Lorenzo Bastioni)

Problema 69

Problema 70

Problema 71

Problema 72[247]
Disponiamo le 32 squadre in qualunque ordine, e costruiamo gli incontri nel seguente modo: ad ogni turno, le squadre in un posto dispari si scontrano con la successiva. Le squadre perdenti vengono eliminate. Disponiamo le squadre restanti nello stesso ordine in cui si trovano, e ripetiamo finché resta solo una squadra. Ad esempio, se ad un certo punto l'ordine è $(3, 5, 2, 1)$, la squadra $3$ vince contro $5$ e $1$ vince contro $2$, dunque al turno dopo l'ordine sarà $(3, 1)$. Poniamo per semplicità la squadra $S_3$ per prima, e consideriamo i restanti $31!$ ordinamenti possibili. La squadra $S_3$ arriva in semifinale se e solo se non viene eliminata nei 3 turni precedenti, e ciò accade se e solo se nessuna tra le squadre $S_1$ ed $S_2$ è presente tra le prime 8 squadre. Possiamo quindi scegliere le prime 8, inclusa $S_3$, in $\displaystyle\binom{29}{7}$ modi, e le restanti in $24!$ modi.
La probabilità cercata è quindi $\displaystyle\binom{29}{7}\cdot 24!\cdot \frac{1}{31!}=\frac{29 \cdot 28 \cdot 27 \cdot 26 \cdot 25 \cdot 24 \cdot 23}{31 \cdot 30 \cdot 29 \cdot 28 \cdot 27}=\frac{24 \cdot 23}{31 \cdot 30}=\frac{92}{155}$. La risposta è quindi $92+155=247$.
(Valeria Martinelli)

Problema 73 [6068]
Notiamo che
$(p+q+r)(pq+qr+pr)=p^2q+p^2r+pq^2+pr^2+q^2r+qr^2+3pqr=1+3\cdot 2022=6067$.
Dato che $p+q+r$ e $pq+qr+pr$ sono entrambi interi positivi, devono necessariamente dividere entrambi $6067$. $6067$ è un numero primo, quindi o $p+q+r=1$ o $p+q+r=6067$. La risposta è perciò $6067+1=6068$.
Si noti che non ci si deve preoccupare di trovare che esistano $p,q,r$ che rispettano le condizioni trovate, dato che questo è assicurato dal fatto che sono le tre soluzioni dell'equazione $x^3-(p+q+r)x^2+(pq+qr+pr)x-pqr=0$, che ha sempre tre soluzioni nei numeri complessi.
(Federico Borasio)

Problema 74

Problema 75 [78]
Elevando entrambi i termini al quadrato (lecito perché entrambi sicuramente positivi) si ottiene $x^2+y^2-x-y+1/2 \leq 1/4$ riarrangiando $(x-1/2)^2+(y-1/2)^2 \leq 1/4$. Si ha quindi l'equazione di un cerchio centrato in $(1/2,1/2)$ con raggio $1/2$. La probabilità che un punto interno al quadrato definito nel piano cartesiano che ha vertice nell'origine e lato $1$ sia dentro il cerchio è quindi il rapporto tra l'area del cerchio e quella del quadrato: $\pi (1/2)^2 \approx 78%$.
(Filippo Prandina)

Problema 76 [4253]
Iniziamo col dimostrare che $P$ si trova all'esterno di $ABC$.

Detto $K$ il piede dell'altezza uscente da $A$ si ha che, essendo $\widehat{APB}$ congruente a $\widehat{AKB}$ perché entrambi retti, e dunque $AKBP$ ciclico, $P$ appartiene alla circonferenza circoscritta a $AKB$ alla quale non appartengono punti interni a $AKB$; ma $P$ appartiene alla mediana $AM$, la quale è evidentemente contenuta nel triangolo $AKB$ (lo si potrebbe dimostrare con i dati del problema), e quindi $P$ si trova all'esterno di $AKB$ ma anche di $ABC$.

Ora, è noto che in un triangolo di lati di lunghezza $9k$, $8k$, $7k$, con $k$ intero positivo, la mediana relativa alla base di lunghezza $8k$ misura $7k$: questo è facilmente verificabile per via trigonometrica, col teorema della mediana o anche con Pitagora; in ogni caso, $AM\cong AC$ e in particolare $\widehat{AMC}\cong \widehat{ACM}=\gamma$.

Sia $\left\{D\right\}=BC\cap PT$. $BTD\sim BCH$ per il secondo criterio di similitudine in quanto hanno l'angolo in $B$ in comune e $\widehat{BDT}\cong \widehat{BHC}$ perché retti, in particolare $\widehat{ATD}=\gamma$.

$\widehat{BMP}\cong\widehat{AMC}$ perché opposi al vertice ed essendo $BPM$ rettangolo in $P$ e $PD$ l'altezza relativa all'ipotenusa, $BPD=\gamma$. Allora $BPT$ è isoscele sulla base $PT$, in altre parole $T$ è il simmetrico di $P$ rispetto a $BC$; da ciò deriva anche che $BMT\cong BMP$, in particolare che $\widehat{BTM}$ sia retto.

$BMT\sim BCH$ per il secondo criterio di similitudine in quanto hanno l'angolo in $B$ in comune e $\widehat{BTM}\cong \widehat{BHC}$ perché entrambi retti. Ma allora, essendo $M$ punto medio di $BC$, necessariamente $T$ è punto medio di $BH$.

Il rapporto $\frac{[ATB]}{[THC]}$ è uguale al rapporto fra le altezze $\frac{AH}{CH}$ in quanto i due triangoli hanno basi congruenti per quanto appena dimostrato. Esse si possono calcolare con pitagora, dopo aver ricavato $BH=\frac{2A}{AC}$ e l'area attraverso Erone o per via trigonometrica. Sviluppando i calcoli si trova $BH=\frac{24\sqrt{5}}{2}$, $CH=\sqrt{BC^2-BH^2}=\frac{16}{7}$ e $AH=AC-CH=\frac{33}{7}$
Il risultato cercato è $1000\cdot \left(\frac{AH}{CH}\right)^2=1000\cdot\left(\frac{33}{16}\right)^2\approx4253,90$ la cui parte intera è $4253$ che è la soluzione.
(Federico Magnolfi)

Problema 77

Problema 78 [4151]
Cominciamo col dimostrare che $BCPQ$ è un trapezio. Sappiamo che l'ortocentro e il circocentro sono coniugati isogonali: in pratica, le rette $AH$ e $AO$ formano coi lati $AB$ e $AC$ angoli congruenti. Avremo dunque $\widehat{BAH}\cong \widehat{CAO}$, ma anche $\widehat{HAI}\cong \widehat{OAI}$ essendo $AI$ bisettrice perché $I$ incentro di $ABC$.

Riferendoci ora alla circonferenza circoscritta ad $ABC$, che chiamiamo $\Gamma$, abbiamo che $\widehat{FCE}\cong \widehat{FAE}\cong\widehat{EAD}\cong \widehat{ECD}$ dove la prima e la terza uguaglianza sono date dal fatto che gli angoli insistono su uno stesso arco. Facendo attenzione, notiamo che $\widehat{QAP}$ e $\widehat{QCP}$ sono congruenti per l'uguaglianza precedente e insistono sullo stesso segmento $QP$, dunque $AQPC$ è ciclico e chiamiamo $\omega$ la sua circonferenza circoscritta. Essendo $\widehat{ACD}$ retto poiché insiste sul diametro $AD$ di $\Gamma$, $\widehat{AQP}$ sarà anch'esso retto poiché supplementare dell'angolo opposto $\widehat{ACP}$ (ricordando che ora abbiamo fatto rifereimento a $\omega$). $BC$ e $QP$ sono allora due rette perpendicolari alla stessa retta $AQ$, perciò sono parallele e $BCPQ$ è un trapezio.

Per calcolare la sua area abbiamo bisogno dell'altezza $QH$ e della base minore $QP$, dato che la base maggiore ci è nota ed è $BC$.

$\textbf{Prima soluzione (sintetica)}$
Detti $K$ e $J$ rispettivamente i piedi dell'altezza $AH$ e della bisettrice $AI$, analizziamo i triangoli $ABJ$ e $AEC$: essi hanno $\widehat{ABJ}\cong \widehat{AEC}$ perché angoli alla circonferenza che insistono su uno stesso arco; $\widehat{BAJ}\cong \widehat{EAC}$ perché $AI$ bisettrice: quindi sono simili per il secondo criterio di similitudine. Considerando ora i triangoli $AKJ$ e $AQP$: essi hanno: l'angolo in $A$ in comune; $\widehat{AKJ}\cong \widehat{AQP}$ perché retti: quindi sono anch'essi simili per il secondo criterio di similitudine. Impostiamo la proporzione $\frac{QP}{KJ}=\frac{AP}{AJ}$ $(1)$: troveremo $KJ$ e $AJ$ lavorando su $ABC$, mentre $AP$ attraverso il rapporto di similitudine fra i triangoli $ABJ$ e $AEC$. Calcoliamo $AK$ invertendo la formula dell'area di $ABC$ e $BK$ col teorema di Pitagora: $AK=\frac{2A}{BC}=\frac{2\cdot \sqrt{p(p-AB)(p-BC)(p-AC)}}{BC}=120 \quad$ $BK=\sqrt{AB^2-AK^2}=50$.
Applichiamo ora il teorema della bisettrice per trovare $BJ$: $\frac{BJ}{AB}=\frac{BC-BJ}{AC}$ da cui si ricava che $BJ=65$. Dunque $KJ=BJ-BK=15$ e $AJ=\sqrt{AK^2+KJ^2}=15\sqrt{65}$.

Impostiamo il rapporto di similitudine fra $ABJ$ e $AEC$: $\frac{AP}{AB}=\frac{AJ}{AC}$ da cui si ricava che $AP=\frac{150\sqrt{65}}{7}$ e per la proporzione $(1)$ $QP=\frac{AP\cdot KJ}{AJ}=\frac{150}{7}$. A questo punto si trova $AQ$ con pitagora e $QK$ per differenza: $AQ=\sqrt{AP^2-QP^2}=\frac{1200}{7} \quad QK=AQ-AK=\frac{360}{7}$
L'area del trapezio $BCPQ$ è dunque $$A=\frac{(140+\frac{150}{7})\cdot\frac{360}{7}}{2}\approx 4151,02$$.

$\textbf{Seconda soluzione (trigonometria)}$
Ponendo $\widehat{ABC}=\beta$, $\cos\beta=\frac{AB^2+BC^2-AC^2}{2AB\cdot BC}=\frac{5}{13}$ (teorema del coseno), $\sin\beta=\sqrt{1-\cos^2\beta}=\frac{12}{13}$ $\widehat{BAC}=\alpha$, $\widehat{CAI}=\frac{\alpha}{2}$ perché $AI$ bisettrice, $\cos\alpha=\frac{AB^2+AC^2-BC^2}{2AB\cdot AC}=\frac{33}{65}$ (teorema del coseno), $\cos\frac{\alpha}{2}=\sqrt{\frac{1+\cos\alpha}{2}}=\frac{7}{\sqrt{65}}$.

Adesso abbiamo abbastanza dati per calcolare $AP=\frac{AC}{\cos\frac{\alpha}{2}}=\frac{150\sqrt{65}}{7}$.
Ponendo $\widehat{HAI}=\widehat{OAI}=\theta$ si ha che, essendo $\widehat{BAH}=\frac{\pi}{2}-\beta$, $\theta=\frac{\alpha}{2}+\beta-\frac{\pi}{2}$; $\cos\theta=\sin\frac{\alpha}{2}\cos\beta+\cos\frac{\alpha}{2}\sin\beta=\frac{8}{\sqrt{65}}$

$AQ=AP\cos\theta=\frac{1200}{7}\quad QP=\sqrt{AP^2-AQ^2}=\frac{150}{7}$. Ci manca l'altezza $QK$ dove $K$ è il piede dell'altezza uscente da $A$. $AK=AB\sin\beta=120\qquad QK=AQ-AK=\frac{360}{7}$. A questo punto l'area si trova allo stesso modo della soluzione per via sintetica.
(Federico Magnolfi)

Problema 79 [2011]
Si ha che $p^{(2)}(x)$ ha lo stesso grado di $p^{(4)}(x)$ questo significa che $p(x)$ è di grado $1$ o grado $0$. Ma non può essere di grado $0$ perché non è costante. Allora è del tipo $p(x)=mx+q$. La funzione $p(x)$ è chiaramente iniettiva ed essendo $p^{(2)}(x)=p^{(2022)}(x)$ per ipotesi, si ha necessariamente che $p(x)=p^{(2021)}(x)$. Ma quindi $p(2021)=0$ e $p(p(2021))=p(0)=2021$. Da qui si ricava facilmente che il polinomio è $p(x)=-x+2021$ e quindi $p(10)=2011$.
(Filippo Prandina)

Problema 80


Problema 81

Problema 82 [4]
$45=9\cdot 5$ quindi scomponiamo il modulo in modulo $9$ e modulo $5$. Modulo $5$ è semplicemente $4$, in quanto l'ultima cifra è proprio $4$. Definiamo $\overline{a_1a_2a_3...a_k}$ la scrittura decimale in successione di $a_1,a_2,...,a_k$, per esempio $\overline{4^2 5^2}=1625$. Inoltre, chiamato $N$ il nostro numero, $N=\overline{1^22^23^24^2...2022^2}$. $\overline{a_1a_2a_3...a_k}\equiv a_1+a_2+a_3+...+a_k\,(\mathrm{mod}\,9)$, dimostrarlo è facile. I residui quadratici di $9$ sono $0,1,4,7$ mentre la somma dei quadrati da $1$ a $9\,\mathrm{mod}\,9$ è $24$, ($-3$). Quindi ogni $9$ numeri vi è un $-3$ da addizionare, quindi è $(-3)\cdot 224$ (che sarebbe il numero di multipli di $9$ minori di $2022$). $-672$, a questo devi aggiungere i numeri da $2017$ a $2022$, $-672+1+4+7+7=-653\equiv -14\equiv 4$.
Siccome $N$ è congruo a $4$ sia $\mathrm{mod}\,5$ che $\mathrm{mod}\,9$, sarà congruo a $4$ anche $\mathrm{mod}\,45$.
(Alessandro Avellino)

Problema 83 [21]
Sia $H$ il piede dell'altezza uscente da $A$; $AY=AH-HY$. L'altezza $AH$ si può calcolare, ad esempio, invertendo la formula dell'area $A=\frac{b\cdot h}{2}$ dopo aver trovato quest'ultima con la formula di Erone, oppure trovando il coseno dell'angolo in $B$ col teorema del coseno, quindi il suo seno e applicando il primo teorema dei triangoli rettangoli a $ABH$. Eseguendo i calcoli, si trova che essa vale $AH=3\sqrt{5}$. $BH=\sqrt{AB^2-AH^2}=2$.

Detto $M$ il punto medio di$BC$ si ha che $HM=HX=2$ e quindi esso è punto medio anche di $HX$; ma $OM\perp BC$ in quanto in un triangolo isoscele la mediana è anche altezza: allora, essendo $HYX$ e $MOX$ simili per il secondo criterio di similitudine, e avendo rapporto di similitudine $2$, $HY=2HO$

Attraverso il teorema della corda, oppure attraverso la formula nota, possiamo calcolare il raggio della circonferenza circoscritta ad $ABC$, $R=\frac{BC}{2\sin\alpha}=\frac{abc}{4A}=\frac{21}{2\sqrt{5}}$.
Ora possiamo trovare $HY=2HO=2\sqrt{R^2-MC^2}=\frac{11\sqrt{5}}{5}$ e infine $AY=AH-HY=3\sqrt{5}-\frac{11\sqrt{5}}{5}=\frac{4\sqrt{5}}{5}$.

Stiamo cercando $AY^2=\frac{16}{5}$ e la risposta è infine $16+5=21$.
(Alessandro Lombardo & Federico Magnolfi)

Problema 84

Problema 85 [1136]

Iniziamo col calcolarci alcuni oggetti che in futuro potranno esserci utili.

Perimetro: 2p=14+16+18=48

l'Area: usando la formula di Erone, A=$\sqrt{p(p-a)(p-b)(p-c)}=\sqrt{24\cdot10\cdot8\cdot6}=48\sqrt{5}$

Raggio circonferenza circoscritta: $R=\frac{abc}{4A}=\frac{4032}{192\sqrt{5}}=\frac{21}{\sqrt{5}}=\frac{21\sqrt{5}}{5}$

Altezza relativa a BC: $AD=2A/BC=6\sqrt{5}$

Altezza relativa a AB, che incontra AB in F: $CF=2A/AB=\frac{48\sqrt{5}}{7}$

Detto L il punto di incontro tra BC e la perpendicolare passante per O, L è il punto medio di BC. Per cui CL=8, OC= R, $OL=\sqrt{\frac{441}{5}-\frac{320}{5}}=\sqrt{\frac{121}{5}}=\frac{11\sqrt{5}}{5}$

$CD=\sqrt{AC^2-AD^2}=\sqrt{144}=12$

$AF=\sqrt{AC^2-CF^2}=\frac{66}{7}$

LD=CD-CL=12-8=4

adesso calcoliamo OD:

$OD=\sqrt{OL^2+LD^2}=\sqrt{\frac{201}{5}}$

Infine calcoliamo AH, dove H è l'ortocentro. per calcolare AH notiamo che, AHF è simile a ABD, per cui $\frac{AH}{AB}=\frac{AF}{AD} \Rightarrow AH=\frac{AF \cdot AB}{AD}=\frac{66 \cdot 14}{42\sqrt{5}}=\frac{22\sqrt{5}}{5}$.

Adesso che abbiamo tutto ciò che ci serve proseguiamo. O è il punto di Miquel relativo ad XYD. questo vuol dire che O appartiene alla circonferenza passante per X,Y,A. Detto questo, abbiamo 2 quadrilateri ciclici, per definizione, BDOX e CDOY. A questi 2 aggiungiamo il quadrilatero AYOX, anch'esso ciclico.

1° Claim: XYD è simile ad ABC.
è sufficiente un po' di angle chasing:

ricordiamo che OA, OB e OC sono i raggi di (ABC).
Di conseguenza OBA=OAB, OBC=OCB e OCA=OAC. Inoltre, per via della ciclicità dei 3 quadrilateri sopra elencati,
OAB=OBA=OBX=ODX, OAC=OCA=OCY=ODY, da questo segue che YDX=ODY+ODX=CAO+OAB=BAC.

Analogamente DYX=CBA e DXY=BCA. Per cui i 2 triangoli sono simili

2° Claim: O è l'ortocentro di XYD.
Anche qui è sufficiente un po' di Angle chasing:

DYX+YDO=(DYO+OYX)+OCY=DCO+OAX+OCA=(DCO+OCA)+OAX=ACB+OAX=ACB+(OAH+HAX)=ACB+(OAH+HAB)=
ACB+(OAH+DAB)=ACB+(OAH+OAC)=ACB+DAC=90.
Ne consegue che OD è perpendicolare ad XY. Analogamente si ottiene che , XO e YO sono perpendicolari, rispettivamente, a DY e DX. Per cui O è l'ortocentro.

Di conseguenza, detto K il rapporto di similitudine tra ABC e XYD, $K=\frac{AH}{OD}=\frac{22}{201}$, il rapporto vale anche per i raggi r e R rispettivamente di XYD, e di ABC, $K=\frac{R}{r} \Rightarrow r=\frac{R}{K}=\frac{21\sqrt{1005}}{110}$
(Alessandro Avellino)

Problema 86 [1483]
Per prima cosa notiamo che, dato che $\frac{x_1+x_2}{x_3+x_4}$ è intero, abbiamo che $x_1+x_2 \ge x_3+x_4$, e similmente $x_3+x_4 \ge x_5+x_6 \ge x_1+x_2$.
Dunque $x_1+x_2= x_3+x_4=x_5+x_6$. Analogamente $x_2+x_3=x_4+x_5=x_6+x_1$.

Sia ora $S=x_1+x_2= x_3+x_4=x_5+x_6$ e $T=x_2+x_3= x_4+x_5=x_6+x_1$.
Abbiamo quindi che $\sum x_i = 3S=3T$, quindi $S=T$, ossia $x_1+x_2=x_2+x_3$, da cui $x_1=x_3$. In modo analogo otteniamo $x_1=x_3=x_5$ e $x_2=x_4=x_6$.

Possiamo quindi scrivere la sestupla come $(a, b, a, b, a, b)$, con $a, b$ interi positivi. Si verifica facilmente che ogni sestupla siffatta rispetta le condizioni del testo, difatti la somma di due termini consecutivi è costante, dunque le frazioni sono tutte uguali a $1$ e quindi intere.
Dunque, fissati $a, b$, c'è una e una sola sestupla valida.

Consideriamo ora due casi:

$\bullet$ $M$ non è multiplo di $3$. Allora, dato che $M=3S$, non ci sono sestuple valide, e quindi $f(M)=0$, che è un multiplo di $5$. In questo caso abbiamo quindi $2022-\frac{2022}{3}=1348$ possibili valori di $M$.

$\bullet$ $M=3k$, per qualche $k$ intero positivo. Dato che $k=a+b$, per ogni valore di $k$, ci sono $k-1$ sestuple valide, poiché abbiamo $k-1$ possibili valori di $a$, dunque $f(M)=k-1$. Vogliamo quindi che $k-1$ sia multiplo di $5$. Se $k=1$, non ci sono sestuple valide in quanto $a+b>1$, altrimenti, potendo scegliere $k$ nell'intervallo $[2, 1348]$, ci sono ${\lfloor \frac{1347}{5} \rfloor}=134$ valori accettabili.

In totale abbiamo quindi $1348+1+134=1483$ possibili valori di $M$.
(Valeria Martinelli)

Problema 87

Problema 88 [6418]
Osserviamo che, prese $3$ cifre distinte $a, b, c$ dall'insieme $X=\{1, 2, 3, 4, 5, 6, 7, 8, ,9 \}$, ci sono esattamente $6$ numeri compresi tra $100$ e $999$ che si scrivono con tutte e sole le cifre $a, b, c$, siano questi $x_1, ..., x_6$. Abbiamo che la loro somma $x_1+...+x_6$ vale $222a+222b+222c$, quindi $\sum f(x_i)=\frac{222(a+b+c)}{a+b+c}=222$.
Ci sono $\displaystyle\binom{9}{3}=84$ modi di scegliere $a, b, c$, quindi la somma in questo caso vale $222\cdot 84=18648$

Dobbiamo ora distinguere i casi in cui $a, b, c$ contengono uno o più zeri o cifre ripetute.
$\bullet$ $a=b=c$: per ognuna delle $9$ possibili scelte di $a, b, c$, c'è uno e un solo numero $x=111a$, e $f(x)=\frac{111a}{3a}=37$. Dunque la somma in questo caso vale $37 \cdot 9=333$

$\bullet$ $a=b \neq c$, $a, b, c \neq 0$: abbiamo $9\cdot 8=72$ possibili scelte di $a, b, c$, e per ogni terna abbiamo $3$ numeri la cui somma vale $222a+111c$, per cui $\sum f(x_i)=\frac{222a+111c}{2a+c}=111$. In tutto $72 \cdot 111=7992$

$\bullet$ $b=c=0$: $9$ possibili scelte di $a$, e per ogni $x=100a$ che rispetta queste condizioni $f(x)=100$. La somma in questo caso vale $100 \cdot 9=900$

$\bullet$ $a=b$, $c=0$; $9$ possibili scelte per $a, b$, per ognuna $\sum f(x_i)=\frac{211a}{2a}=\frac{211}{2}$. Quindi in tutto $9\cdot \frac{211}{2}=949,5$

$\bullet$ $a\neq b$, $c=0$: $\displaystyle\binom{9}{2}=36$ possibilità per $a, b$; per ognuna $\sum f(x_i)=\frac{211a+211b}{a+b}=211$. In tutto $36 \cdot 211=7596$

La somma richiesta è quindi uguale a $18648+333+7992+900+949,5+7596=36418,5$. Ci interessano le ultime quattro cifre della parte intera.
(Valeria Martinelli)

Problema 89 [134]
Sia $2^k$ la massima potenza di $2$ nell'intervallo $\{1, ..., n\}$. Si osserva facilmente che $f(n)$ non può mai essere maggiore di $f(2^k)$, pertanto $f(n)$ è massima quando $n=2^k$, e $g(2^k)=1$.

Dato che $5>4=2^2$ e $9=3^2>2^3=8$, se un intero contiene nella sua fattorizzazione primi diversi da $2$ e $3$, o se $3$ compare con un esponente maggiore o uguale a $2$, $g(n)=g(n-1)$ in quanto $n$ ha sicuramente meno fattori di $2^k$.
Inoltre, se $n=2^{k-1}\cdot 3$, $g(n)=g(n-1)+1$, in quanto n contiene lo stesso numero di fattori di $2^k$.

Dunque, se $2^{k-1}\cdot 3\le n <2^{k}$, $g(n)=2$, altrimenti $g(n)=1$.

Possiamo quindi trovare tutti gli interi minori o uguali a $99$ per cui $g(n)=2$, che sono i valori compresi tra i tripli delle potenze di $2$ e la potenza di $2$ successiva.

Dunque la somma richiesta è uguale a $99+(4-3)+(8-6)+(16-12)+(32-24)+(64-48)+(100-96)=99+1+2+4+8+16+4=99+31+4=134$.
(Valeria Martinelli)

Problema 90 [8784]
Chiamiamo $f(n)=x^n+\frac{1}{x^n}$, il problema ci chiede di calcolare $|f(1)\cdot f(2)\cdot\dots\cdot f(2022)|$.
Esplorando un po’ la situazione si può notare che $f(1)=1,f(2)=-1,f(3)=-2,f(4)=-1,f(5)=1$ e $f(6)=2$ e che questi valori potrebbero formare un ciclo che si ripete ogni $6$.
Dimostriamolo:
consideriamo $f(n) \cdot f(1)= \left( x^n+\frac{1}{x^n} \right) \cdot \left(x+ \frac{1}{x} \right) = x^{n+1} + x^{n-1} + \frac{1}{x^{n-1}} + \frac{1}{x^{n+1}} = f(n+1) + f(n-1)$, essendo $f(1)=1$ otteniamo che $f(n) \cdot f(1) =f(n)= f(n+1) + f(n-1)$ da cui $f(n+1)=f(n)-f(n-1)$. L’ultima relazione ci indica che $f(n+1)$ dipende unicamente da $f(n)$ e $f(n-1)$ ovvero se troviamo un ciclo quest’ultimo si ripeterà. Controllando si può vedere che $f(7)=1$ e $f(8)=-1$ quindi il ciclo si ripeterà. Analizzando il nostro ciclo si nota facilmente che gli unici fattori che contribuiranno nel nostro prodotto saranno quelli con $n$ multiplo di $3$, che in valore assoluto valgono $2$. Il nostro prodotto sarà quindi del tipo $2^k$ dove $k$ è il numero di multipli di $3$ tra $1$ e $2022$ ovvero $674$ $(\frac{2022}{3}=674)$. Il problema si è trasformato in un quesito di teoria dei numeri poiché dobbiamo dare come risposta le ultime $4$ cifre di $2^{674}$ quindi calcolare $2^{674}$ $(\mathrm{mod}\,10000)$. Riducendo con un po’ di conti l’enorme numero si trova $8784$.
(Lorenzo Bastioni)

Problema 91[4929]
Osserviamo che $g(n)=3^5 \cdot \frac{n}{v_{3}(n)}$, dunque la somma richiesta è uguale a $\sum 3^5 \cdot \frac{n}{v_{3}(n)}$. Definiamo quindi $h(n)=\frac{n}{v_{3}(n)}$ e calcoliamo separatamente le somme di $h(n)$ in base a $v_{3}(n)$.

$\bullet$ $v_{3}(n)=5$ \rightarrow $h(n)=1$: $n \in \{243 \cdot 1\}$, \rightarrow $\sum h(n)=1$

$\bullet$ $v_{3}(n)=4$: $n \in \{81 \cdot 2, ..., 81 \cdot 5\}$ esclusi i multipli di $243$, $\sum h(n)=(2+...+5)-1\cdot 3=14-3=11$

$\bullet$ $v_{3}(n)=3$: $n \in \{27 \cdot 5, ..., 27 \cdot 15\}$ esclusi i multipli di $81$, $\sum h(n)=(5+...+15)-14\cdot 3=110-42=68$

$\bullet$ $v_{3}(n)=2$: $n \in \{9 \cdot 15, ..., 9 \cdot 46\}$ esclusi i multipli di $27$, $\sum h(n)=(15+...+46)-110\cdot 3=976-330=646$

$\bullet$ $v_{3}(n)=1$: $n \in \{3 \cdot 45, ..., 3 \cdot 140\}$ esclusi i multipli di $9$, $\sum h(n)=(45+...+140)-976\cdot 3=8880-2928=5952$

$\bullet$ $v_{3}(n)=0$: $n \in \{1 \cdot 135, ..., 1 \cdot 420\}$ esclusi i multipli di $3$, $\sum h(n)=(135+...+420)-8880\cdot 3=79365-26640=52725$

Quindi, $\sum h(n)=52725+5952+646+68+11+1=59403$, da cui $\sum f(n)=3^5 \cdot \sum h(n)=14434929$. Ci interessano le ultime 4 cifre
(Valeria Martinelli)

Problema 92

Problema 93 [279]
$x$,$y$ e $z$ sono interscambiabili. Per cui è possibile, per facilitarci nella risoluzione, porre $x \geq y \geq z$.

Adesso andiamo in ordine:\\
abbiamo

$x^2<x^2+2y+z \leq x^2+3x < x^2+4x+4=(x+2)^2$

e siccome lo vogliamo come quadrato

$x^2+2y+z=(x+1)^2 \Rightarrow 2y+z=2x+1 \Rightarrow x=\frac{2y+z-1}{2}$.

Ora usiamo la seconda equazione con le nuove sostituzioni.\\
$y^2<y^2+2z+x=y^2+2z+\frac{2y+z-1}{2}=y^2+y+2.5(z)-0.5<y^2+y+3y+4=(y+2)^2$.

Per lo stesso motivo di prima:

$y^2+y+2.5(z)-0.5=(y+1)^2 \Rightarrow 5z-3=2y \Rightarrow y=\frac{5z-3}{2}$\\
Ora è il momento di usare l'ultima equazione, anche qui sostituendo $y$ e $x$:

$z^2<z^2+2x+y=z^2+2y+z-1+y=z^2+z+3(\frac{5z-3}{2})-1=z^2+8.5z-4.5$.

Dopo vari tentativi si nota che gli unici quadrati possibili per questa quantità sono $(x+1)^2$ e $(x+4)^2$.\\

Infatti si avrà $z^2+8.5z-4.5=z^2+8z+16 \Rightarrow z=43$, trovandoti poi $y=106$ e $x=127$ e $z^2+8.5z-4.5=z^2+2z+1$ trovando quindi $x=z=y=1$.
(Alessandro Avellino)

Problema 94 [6460]
Possiamo riscrivere il testo del problema nel seguente modo:
$$\sum_{n=1}^{2020} \lfloor \frac{\sqrt{n+2}}{\sqrt{n+1} - \sqrt{n}} \rfloor$$
Andiamo ad analizzare il nostro addendo generale privo della funzione floor:
$ \frac{\sqrt{n+2}}{\sqrt{n+1} - \sqrt{n}} = \frac{\sqrt{n+2}}{\sqrt{n+1} - \sqrt{n}} \cdot \frac{\sqrt{n+1} + \sqrt{n}}{\sqrt{n+1} + \sqrt{n}} = \sqrt{(n+2) \cdot (n+1) } + \sqrt{ (n+2) \cdot n } = \sqrt{n^2 + 3n+ 2} + \sqrt{n^2 +2n} $.
Esplorando un po' la situazione ci si potrebbe convincere che $2n+2 \leq \sqrt{n^2 + 3n+ 2} + \sqrt{n^2 +2n} < 2n+3$ (con $n$ nell'intervallo che interessa a noi ovvero $[1,2020]$).
Dimostriamolo:
La seconda disuguaglianza è molto semplice infatti $\sqrt{n^2 + 3n+ 2} + \sqrt{n^2 +2n} = \sqrt{(n+2)^2 -n-2} + \sqrt{(n+1)^2 - 1} < \sqrt{(n+2)^2} + \sqrt{(n+1)^2} = n+2 +n+1 = 2n+3$ quindi $\sqrt{n^2 + 3n+ 2} + \sqrt{n^2 +2n} < 2n+3$.
La prima disuguaglianza è leggermente più fastidiosa:
vogliamo $2n+2 \leq \sqrt{n^2 + 3n+ 2} + \sqrt{n^2 +2n}$ che è molto lungo da risolvere a mano dunque proviamo a scomporre in maniera intelligente il problema:
ad esempio, se dimostrassimo che $\sqrt{n^2 + 3n+ 2} \geq n + \frac{4}{3} \wedge \sqrt{n^2 +2n} \geq n+\frac{2}{3}$ avremmo finito poiché sommando le disuguaglianze si otterrebbe la tesi (la veridicità delle due disuguaglianze implica la tesi ma non viceversa).
Le due disuguaglianze sono due semplici disequazioni irrazionali e non ci dovremmo preoccupare di condizioni di esistenza o simili poiché nel nostro intervallo è tutto positivo:
1)
$ \sqrt{n^2 + 3n+ 2} \geq n + \frac{4}{3} \rightarrow n^2+3n+2 \geq (n + \frac{4}{3})^2 \rightarrow n \geq - \frac{2}{9}$, nel nostro intervallo la disuguaglianza vale.
2)
$ \sqrt{n^2 +2n} \geq n+\frac{2}{3} \rightarrow n^2+2n \geq (n+\frac{2}{3})^2 \rightarrow n \geq \frac{2}{3} $, nel nostro intervallo la disuguaglianza vale.
La tesi è dunque dimostrata, $2n+2 \leq \sqrt{n^2 + 3n+ 2} + \sqrt{n^2 +2n} < 2n+3$. Ciò implica che $\lfloor \frac{\sqrt{n+2}}{\sqrt{n+1} - \sqrt{n}} \rfloor = 2n+2$ (nel nostro intervallo) e possiamo riscrivere la nostra somma:
$$\sum_{n=1}^{2020} 2n+2 = \left( 2 \sum_{n=1}^{2020} n \right) + 2 \cdot 2020 = 2020 \cdot 2021 + 4040 = 4086460$$
(La seconda uguaglianza si ha con la formula di Gauss per la somma dei primi $k$ numeri naturali) Prendendo le ultime $4$ cifre del risultato abbiamo la soluzione: $6460$.
(Lorenzo Bastioni)

Problema 95 [21]
Mettiamo in un piano cartesiano i punti [math]. Si ha che [math] è il coffieciente angolare della retta per i due punti [math]. Quindi una sequenza è non genovese se tutti i coefficienti angolari sono non negativi; ovvero se e solo se essa è debolmente crescente. Supponiamo di avere una sequenza debolmente crescente [math]. Vi è una bigezione fra le sequenze di questo tipo e quelle del tipo [math], che sono tutte le sequenze strettamente crescenti con elementi positivi minori di [math]. Esse a loro volta corrispondono con i modi di scegliere [math] elementi distinti fra [math]. Il numero totale di sequenze (genovesi e non) è infine [math]; quindi il numero di quelle genovesi è [math] che è congruo a [math] modulo [math], essendo che [math] modulo [math].
(Federico Volpe)

Problema 96 [2029]
I numeri curiosi sono esattamente i liberi dai quadrati.\\
($\Leftarrow$) Se $n$ è un numero libero da quadrati allora $\tau(n)$ è una potenza di due, inoltre ogni divisore $d$ di $n$ è un numero libero da quadrati, quindi anche $\tau(d)$ è una potenza di $2$ e dato che $\tau(d) \le \tau(n)$ abbiamo che $\tau(d)|\tau(n)$.\\
($\Rightarrow$) Supponiamo che $n = p_1^{a_1}p_2^{a_2}\cdots p_m^{a_m}$ sia un numero libero da quadrati. Allora abbiamo
\begin{align*}
&\tau(n/p_i)|\tau(n)\\
\Rightarrow &(a_1+1)\cdots(a_i)\cdots(a_m+1)|(a_1+1)\cdots(a_i+1)\cdots(a_m+1)\\
\Rightarrow &a_i | a_i+1 \Rightarrow a_i = 1.
\end{align*}
Dunque $n$ è libero da quadrati.\\
\\
Inoltre notiamo che $f(a,b)$ è semplicemente $MCD(a,b)$. A questo punto scriviamo
$$S(n) = \sum_{j=1}^n f(j,n) = \sum_{j=1}^n MCD(j,n),$$
dimostreremo che la funzione $S$ è moltiplicativa, cioè per ogni $m,n \in \mathbb{Z}^+$ coprimi $S(m)S(n) = S(mn)$. Infatti, segue dal teorema del resto cinese, possiamo rimpiazzare $j \text{ mod }\,mn$ con due numeri $j_m \text{ mod }\,m$ e $j_n \text{ mod }n$ in modo che:
\begin{align*}
S(mn) &= \sum_{j=1}^{mn} MCD(j,mn) = \sum_{j=1}^{mn} MCD(j,m)MCD(j,n)\\
&= \sum_{j_m=1}^{m} \sum_{j_n =1}^{n} MCD(j_m,m)MCD(j_n,n)\\
&= \left(\sum_{j_m=1}^m MCD(j_m,m)\right)\left(\sum_{j_n=1}^n MCD(j_n,n)\right) = S(m)S(n).
\end{align*}
Ma quindi dato che $C$ è un prodotto tra primi ed è facile verificare che $S(p) > 1$ per ogni primo $p$, abbiamo che $C$ stesso deve essere primo. Un po' di brute-force dà la risposta.
(Michele Tomasi)

Problema 97

Problema 98

Problema 99

Problema 100 [1339]
Premetto innanzitutto che questo geometrico è più tecnico degli altri ed è necessario avere qualche prerequisito sulle simmediane e il cerchio di Feuerbach. Tutti i fatti noti di cui farò uso si trovano sul libro di Evan Chen ma sono certo che si possano trovare anche girovagando su internet.

La soluzione di questo problema si può suddividere in tre claim.

Primo claim: La tangente di $\Gamma$ in $N_a$, la retta $BC$ e la tangente di $\Gamma$ in $A$ concorrono nel punto $X_a$.\\
Qua c'è solo da usare qualche fatto noto delle simmediane. Infatti $\triangle BAN_a \sim \triangle CAM_a \Rightarrow \angle BAN_a= \angle CAM_a \Rightarrow AN_a$ è la $A-simmediana$ di $\triangle ABC$. Dopodiché basta usare che $CB$ è la $C-simmediana$ di $\triangle ACN_a$ e quindi le tangenti in $A$ e in $N_a$ concorrono con essa.

Secondo claim:
$Y_a$ e simili stanno sulla circonferenza di Feuerbach, ovvero $\gamma$ e $\omega$ coincidono.
Per essere capaci di dimostrarlo però è necessario notare qualcosa di più subdolo.
Sia $H$ l'ortocentro di $\triangle ABC$, $V$ il punto medio di $AH$ e $Q$ l'intersezione tra la retta $AH$ e $\Gamma$. Siano inoltre $R$ e $r$ i raggi di $\Gamma$ e di $\gamma$.
Lemma: $V$ è L'ortocentro di $\triangle AM_{a}X_{a}$.
Proof: Dimostriamo innanzitutto che la retta $VM_a$ è perpendicolare a $AX_a$. CIò è equivalente a dimostrare che $\angle X_{a}M_{a}V = 90^{\circ}- \angle AX_{a}M_a=\angle X_{a}AP_{a} \Leftrightarrow \angle P_{a}M_{a}V=\angle X_{a}AQ \Leftrightarrow \frac{VP_{a}}{AQ}=\frac{r}{R}=\frac{1}{2}$.
Poiché $Q$ è il simmetrico di $H$ su $BC$ (si dimostra con angle chasing), $HQ+HA=2HP_{a}+2HV \Rightarrow AQ=2VP_{a} \Rightarrow \frac{VP_{a}}{AQ}=\frac{1}{2}$.
$AV \perp X_{a}M_{a}$ e $M_{a}V \perp AX_{a} \Rightarrow V$ ortocentro di $\triangle AM_{a}X_{a}$.
A questo punto per dimostrare il claim iniziale basta dire che, poichè $V$ è ortocentro, $X_{a},V,Y_{a}$ sono allineati e quindi $\angle VY_{A}M_{a}=90^{\circ}=\angle VP_{a}M_{a} \Rightarrow VY_{a}M_{a}P_{a}$ ciclico $\Rightarrow Y_{a}$ sta su $\gamma$.
Similarmente si può dimostrare che anche $Y_{b}$ e $Y_{c}$ stanno su $\gamma$.

Terzo claim: le rette $P_{a}P_{b}, DE, CH$ concorrono in $F$.
$\angle BHC=180^{\circ}- \angle BAC \Rightarrow H$ sta su $\Gamma'$. Ora basta osservare che $P_{a}P_{b}, DE, CH$ sono gli assi radicali di $\gamma, \Gamma', (P_{a}HP_{b}C)$.
Similarmente si dimostra che $P_{a}P_{c}, DE, BH$ concorrono in $G$.

Ora che abbiamo delle informazioni "decenti" sui punti $F$ e $G$ possiamo fare del segment chasing per trovare i valori di $P_{a}F$ e $P_{a}G$. $\triangle P_{a}P_{b}P_{c}$ è il famoso triangolo ortico e $P_{c}G$ e $P_{b}F$ sono le bisettrici, quindi l'idea sarebbe quella di trovare i tre lati e poi usare il teorema delle bisettrici.
$BC^{2}-P_{b}C^{2}=AB^{2}-(AC-P_{b}C)^{2} \Rightarrow P_{b}C=\frac{84}{5}$.
$\triangle BCA \sim \triangle P_{b}CP_{a} \Rightarrow \frac{P_{a}P_{b}}{P_{b}C}=\frac{AB}{BC} \Rightarrow P_{a}P_{b}=\frac{78}{5}$.
Similarmente si trovano $P_{b}P_{c}=\frac{924}{65}$ e $P_{a}P_{c}=\frac{150}{13}$
$\frac{P_{a}F}{P_{a}P_{c}-P_{a}F}=\frac{P_{b}P_{a}}{P_{b}P_{c}} \Rightarrow P_{a}F=\frac{1950}{323}$
$\frac{P_{a}G}{P_{a}P_{b}-P_{a}G}=\frac{P_{c}P_{a}}{P_{c}P_{b}} \Rightarrow P_{a}G=\frac{650}{93}$
$P_{a}F+P_{a}G=\frac{391300}{30039}$. La risposta è quindi $1339$.
(Denis Tusca)
Godl3x04
Messaggi: 12
Iscritto il: 27 lug 2019, 22:03

Re: #Proviamoci - Soluzioni commentate OH6

Messaggio da Godl3x04 »

Problema 1 [3]
$n$ il numero di commensali e $d$ la distanza tra loro, i commensali sono i punti ${A_1, A_2,..., A_n}$ mettiamo $A_1$ in un punto qualsiasi del piano. $A_2$ potrà stare ovunque nella circonferenza di raggio $d$ e centro $A_1$. Il terzo punto ha solo $2$ posizioni in cui poter stare, che sono le intersezioni delle $2$ circonferenze di centro $A_1$ e $A_2$, entrambe di raggio $d$. Posizionato il terzo punto e tracciata la terza circonferenza noteremo che non ci sarà nessun punto in comune a tutte e $3$ le circonferenze. Di conseguenza non ci può essere un quarto punto che rispetta le condizioni.
(Alessandro Avellino)

Problema 2 [6]
Si tratta di disporre tre diversi ingredienti, che si può fare semplicemente in $3!=6$ modi diversi. La risposta è quindi $6$.
(Matteo Salicandro)

Problema 3 [228]
Il massimo numero ottenibile lanciando gli $n$ dadi è $n\cdot k$, il minimo è $n\cdot 1=n$. Pertanto $n\cdot k-n=168$, cioè $n(k-1)=168$. Questo vuol dire che $n$ è un qualsiasi divisore positivo di $168$, esclusi $168$ e $84$ visto che $k \geq 4$. $168=2^3\cdot 3\cdot 7$, quindi la somma dei suoi divisori è $(2^4-1)(3+1)(7+1)=480$. La somma dei possibili valori di $n$ è pertanto $480-168-84=228$.
(Federico Borasio)

Problema 4

Problema 5

Problema 6 [36]
Un rettangolo con le diagonali perpendicolari è un quadrato. L'area è quindi $6^2=36$.
(Filippo Prandina)

Problema 7 [18]
La successione è definita, per $n \geq 1$, come
$\begin{cases}
x_{n+2}=x_{n+1}+x_n\\
x_6=76\\
x_5=47
\end{cases}$
Da qui ricaviamo che $x_n=x_{n+2}-x_{n+1}$, pertanto $x_4=76-49=29$ e $x_3=47-29=18$, che è il terzo termine della successione.
(Federico Borasio)

Problema 8 [2022]
Supponiamo che ci siano $2022$ fisici. Tutti dicono la verità, cioè che non sono matematici: non c'è alcuna contraddizione. $2022$ è anche il numero di abitanti dell'isola, quindi è il massimo numero di fisici.

Problema 9 [8]
$\frac{4n}{32}=k$ con $k\in N\geq 0$. Quindi $n=8k$. Il minimo si ha quando $k=1$ e $n=8$.
(Filippo Prandina)

Problema 10[8]
Sono possibili due casi, in base a qual è la circonferenza con raggio minore. Tuttavia, r deve essere massimo, quindi consideriamo il caso in cui r$>$5. L'area della corona circolare sarà data da
$\pi{r}^{2}-{5}^{2}\pi=\pi({r}^{2}-25)=16\pi.$ Per cui ${r}^{2}-25=16$, ovvero $r=\sqrt{41}\approx{6.403}$, la cui parte intera è 6.
(Daniele Prisco)

Problema 11 [9999]
Ci sono più soluzioni in quanto i primi due termini possono essere $1,3$, oppure $2,2$ oppure $3,1$. La risposta è quindi $9999$.
(Matteo Salicandro)

Problema 12[1261]
Affinché una parola sia impronunciabile per il nostro Loenzo, bisogna che contenga almeno una R. Se c'è una sola R nella parola, questa potrà essere la prima, la seconda o la terza lettera, e in ognuno di questi casi ci saranno 20*20=400 parole possibili, dato che nei restanti due 'posti' vanno inserite le altre 20 lettere dell'alfabeto, che possono essere ripetute. Se nella parola ci sono due R, esse si possono disporre in 3 modi, e per ognuno si potranno formare 20 parole. Infine, va incluso l'unico caso in cui la parola è formata da tre R.
In totale, i casi sono $400*3+20*3+1=1261$.
(Daniele Prisco)

Problema 13 [28]
$X$ appartiene alla perpendicolare di $BC$ passante per $A$. Inoltre $X$ e $2$ punti su $BC$ formano lo stesso angolo che forma $A$ con quei $2$ punti, di conseguenza $X$ è il simmetrico di $A$. Da questo deduciamo che $AX=2AD$, dove $AD$ è l'altezza.
(Alessandro Avellino)

Problema 14 [1011]
La quantità in questione, $(3n+2)^{5n+8}$, è un quadrato perfetto se lo è la base, cioè $3n+2$, oppure se l'esponente, $5n+8$, è pari. Tuttavia la prima possibilità non si verifica mai poiché un quadrato non è mai congruo a $2\,(\mathrm{mod}\,3)$. Dunque la risposta è semplicemente il numero di interi positivi $1\leq n\leq 2022$ tali che la quantità $5n+8$ sia pari, ovvero tutti e soli i numeri pari in questo intervallo, che sono $\frac{2022}{2}=1011$.
(Lorenzo Weiss)

Problema 15

Problema 16 [2019]
Il problema chiede per quanti interi positivi minori o uguali a $2022$ accade che $\lfloor \frac{\pi +n}{n}\rfloor =\lfloor \frac{\pi}{n}+1\rfloor =\lfloor \frac{\pi}{n}\rfloor +1=1\Longrightarrow \lfloor \frac{\pi}{n}\rfloor =0$, il che succede solo quando $\pi\approx 3,14\leq n$, i.e. in $2022-(4-1)=2019$ casi.
(Lorenzo Weiss)

Problema 17

Problema 18

Problema 19 [14]
Disegnando la circonferenza $x^2+y^2=5$ risulta evidente che gli unici punti a coordinate intere che vi appartengono, che sono $k=8$, sono quelli della forma $P_i(\pm 1,\pm 2)$, $P_i(\pm 2,\pm 1)$, $P_i(\pm 1,\mp 2)$, $P_i(\pm 2,\mp 1)$. In alternativa, è possibile risolvere l'equazione della stessa circonferenza sugli interi $x,y$. Da ciò si ricava l'ottagono $P_1P_2\dots P_8$, la cui area, come si vede facilmente, è $16-4\cdot\frac{1}{2}=14$.
(Lorenzo Weiss)

Problema 20

Problema 21 [210]
Notiamo che, comunque presi $6$ elementi distinti dall'insieme $X=\{1, 2, 3, 4, 5, 6, 7, 8, 9, 10\}$, c'è uno e un solo modo di disporli in ordine crescente, dunque la richiesta del problema è equivalente al trovare il numero di sottoinsiemi di $6$ elementi di $X$, che sono $\displaystyle\binom{10}{6}=210$.
(Valeria Martinelli)

Problema 22 [2022]
Si può ottenere $2$ solo quando una delle cifre è $2$ e le altre sono nulle, oppure quando due cifre sono uguali a $1$ e le altre sono nulle. Nel primo caso, la prima cifra sarà necessariamente $2$, altrimenti il numero non avrà $2022$ cifre: si ottiene solo una possibilità.
Nel secondo caso, invece, la prima cifra sarà $1$, mentre l'altro $1$ occuperà una delle altre $2021$ posizioni disponibili, ottenendo $2021$ possibilità.
Sommando le possibilità, si arriva a $2022$.
(Daniele Prisco)

Problema 23

Problema 24 [4042]
Se $x$ è la lunghezza del segmento $PA$ e $r$ è il raggio della circonferenza,
$OP=OA+PA=r+x=2021$; $PB=OA+OB+PA=r+r+x$. Di conseguenza, $PA+PB=x+r+r+x=2(r+x)=2×2021=4042$.
(Daniele Prisco)

Problema 25 [66]
Poniamo $a+b=x$ e $c+d=y$.
La disuguaglianza adesso è
$x+y\leq \sqrt{x^2+y^2} \Rightarrow x^2+2xy+y^2\leq x^2+y^2$
Da cui $2xy\leq0$ ma siccome $x$ e $y$ sono somme di interi maggiori o uguali a $0$, l'unico modo per cui $xy=0$ si ha quando uno dei $2$, o entrambi, è uguale a $0$. Se $c+d=y=0$ da cui $c=d=0$, quindi $(a,b,0,0)$. Ricordiamo che $a \geq b$ quindi i possibili valori sono $66$.
(Alessandro Avellino)

Problema 26 [6856]
Sia $n$ il numero di circonferenze sulla prima riga. Avendo le circonferenze diametro $2$, la base del rettangolo è uguale a $2n$. Invece, essendo tutte le circonferenze tangenti, i centri di quella più alta e delle due agli estremi della base sono disposte sui vertici di un triangolo equilatero, di lato $2n-2$. L'altezza del rettangolo è quindi uguale all'altezza del triangolo aumentata di $2$, ossia $\sqrt{3}(n-1)+2$.
Quindi vogliamo $4n+2\sqrt{3}(n-1)+4>2022$, che ci dà immediatamente $n=271$, quindi $k=\frac{271\cdot 272}{2}=36856$.
La risposta voluta sono le ultime quattro cifre di $k$.
(Valeria Martinelli)

Problema 27 [250]
Intanto notiamo che $78125$ è $5^7$.
La somma di $M$ termini consecutivi con primo termine $a$ è del tipo:
$a+(a+1)+(a+2)+...+(a+M-1)$ nel quale compaiono infatti $M$ termini consecutivi.
È quindi possibile raccogliere come $a \cdot M + (1+2+3+...+M-1)=a \cdot M+ \frac{M(M-1)}{2}=M(a+\frac{M-1}{2})=78125 \Rightarrow M(2a-1+M)=5^7\cdot2$, $M<2a-1+M$ quindi al massimo $M=2 \cdot 5^3$.
(Alessandro Avellino)

Problema 28

Problema 29

Problema 30

Problema 31 [20]
Il doppio delle cifre di $n$ dovrà necessariamente essere un numero di $3$ cifre ognuna delle quali minore di $5$. Una volta individuato il numero più piccolo $n$ come $101\,(2n=202)$ e il massimo $n$ come $444\,(2n=888)$ i restanti saranno compresi tra essi. Ci saranno $18$ numeri palindromi tra questi due estremi, con la cifra intermedia contente al più il $4$ e gli estremi uguali da $1$ a $4$. I numeri saranno $101$, $111$, $121$, $131$, $141$, $202$, $222$… fino a $444$. Il totale è di $20$ numeri bipalindromi.
(Irene Mancone)


Problema 32

Problema 33 [74]
Il costo di un pallone ($C$) può essere pari o dispari. Se è pari, può essere pagato con $\frac{C}{2}$ monete da $2$ orue. Poiché $\frac{C}{2}$ è pari, $C$ sarà un multiplo di $4$, quindi potrà assumere $37$ valori.
Se $C$ è dispari, invece, il pallone può essere pagato con $\frac{C-1}{2}$ monete da $2$ orue e una moneta da un orue. Di conseguenza, $C-1$ è un multiplo di $2$, ma $\frac{C-1}{2}$ è dispari (aggiungendo una moneta da un orue, il numero di monete usate è pari). Di conseguenza, $C-1$ è multiplo di $2$ ma non di $4$, quindi può assumere altri $37$ valori.
La somma dei valori possibili è dunque $74$.
(Daniele Prisco)

Problema 34 [2023]
Notiamo che possiamo riscrivere il testo nella seguente maniera: $P((x-y)(x+y))=P(x-y)P(x+y)$.
Notiamo anche che sostituendo $x-y \mapsto z$ e $x+y \mapsto w$ la coppia $(z,w)$ può assumere qualsiasi coppia di valori in $\mathbb{R}^2$ poiché il sistema
$\begin{cases}
x-y=z\\
x+y=w
\end{cases}$
ha soluzioni per ogni coppia $(z,w)$ fissata.
Dunque il nostro problema è equivalente a $P(zw)=P(z)P(y)$ per ogni $z$ e $w$ in $\mathbb{R}$. Questa relazione è molto simile ad una delle famose equazioni funzionali di Cauchy, $f(xy)=f(x)f(y)$. Quest'ultima equazione citata ha soluzione $f(x)=x^n$ o $f(x)=0$ (con $n$ naturale) se la funzione è continua (la continuità è una delle tante condizioni sufficienti), ma un polinomio come $P$ è continuo dunque le possibili soluzioni sono quelle riportate con $n$ che va da $0$ a $2021$ per $P(x)=x^n$ più $P(x)=0$. La soluzione è quindi $2023$ polinomi possibili.
(Lorenzo Bastioni)

Problema 35

Problema 36 [2020]
Si noti che $abc=2022-ab-a$, quindi per massimizzare il prodotto bisogna minimizzare $a$ e $b$. $a,b,c$ sono $\geq 1$ per ipotesi, dunque si pone $a=b=1$ dando come soluzione $c=2020$. Quindi $(a,b,c)=(1,1,2020)$ e $abc=2020$.
(Filippo Prandina)

Problema 37 [201]
Poniamo [math] e notiamo che [math] per [math]. Ora possiamo costruire il seguente polinomio: [math] di cui conosciamo gli zeri [math] e poiché [math] è chiaramente monico possiamo affermare che [math]. Sostituendo infine [math] con [math] e ricavando [math] otteniamo che [math] da cui possiamo trovare la soluzione al problema calcolando [math]. La risposta è quindi [math].
(Lorenzo Bastioni)

Problema 38

Problema 39

Problema 40

Problema 41

Problema 42

Problema 43 [4238]
Se si devono prendere almeno $n$ dolci per essere sicuri di prenderne almeno $1$ di un determinato tipo, questo vuol dire che ci sono $n-1$ dolci non di quel tipo. In particolare:
$\begin{cases}
S+B=55\\
B+C=70\\
S+C=91
\end{cases}$
Il sistema è di immediata risoluzione e porta come soluzione $(S,C,B)=(38,53,17)$ da cui $SCB=34238$. Sono richieste solo le ultime $4$ cifre.
(Filippo Prandina)

Problema 44 [4]
La configurazione è unica, infatti se prendiamo il punto $A_1$ e lo collegassimo con un punto diverso da $B_{2022}$, come per esempio $B_{2021}$, $B_{2022}$ sarà collegato con un punto diverso da $A_1$, per esempio $A_2$. Allora $A_1B_{2021}$ non interseca $A_2B_{2022}$. Iterando questo ragionamento ad ogni punto $A_i$, si ottiene un'unica configurazione: segmenti di estremi $A_iB_{2023-i}$. Quindi i coefficienti angolari saranno $\pm 2/(2023-2i)$. Per il prodotto, il numeratore è sempre $2$ mentre il denominatore è sempre dispari. Trovare quindi ciò che il problema richiede equivale a trovare le ultime $2$ cifre di $2^{2022}$.
(Alessandro Avellino)

Problema 45 [18]
Notiamo che, comunque presa una casella, il numero di giorni che Alberto o Barbara impiegano per raggiungerla non dipende dal percorso scelto, e le uniche caselle che hanno la stessa distanza sia da Alberto che da Barbara sono quelle sulla diagonale del quadrato diversa da quella di partenza, dunque i due si incontrano se e solo se entrambi passano per una di queste tre caselle.
Dato che un percorso è valido se e solo se fa $2$ passi in una direzione e $2$ nell'altra, tutte le coppie di percorsi possibili sono $\frac{4!}{2!\cdot 2!}\cdot \frac{4!}{2!\cdot 2!}=36$, quelli in cui entrambi passano per il centro sono $(2!\cdot 2!)(2!\cdot 2!)=16$, mentre quelli in cui entrambi passano in uno dei due angoli sono $2$.
Le coppie di percorsi con cui Alberto e Barbara non si incontrano sono quindi $36-16-2=18$.
(Valeria Martinelli)

Problema 46

Problema 47 [171]
Poiché $wxy+2021$ deve dare un numero pari, $wxy$ dev'essere dispari, e di conseguenza $w$,$x$ e $y$ devono esserlo.

$w=2a+1, x=2b+1, z=2c+1$ quindi $2a+1+2b+1+2c+1=37 \Rightarrow 2(a+b+c)=34 \Rightarrow a+b+c=17$
Adesso è un semplice stars and bars, infatti $a$, $b$ e $c$ possono essere anche nulli. Distribuire $17$ elementi in $3$ contenitori, $\binom{17+3-1}{3-1}=\binom{19}{2}$.
(Alessandro Avellino)

Problema 48 [42]
Notiamo che $CQB~APB$, in quanto hanno $2$ angoli congruenti, entrambi pari a $74$. Da questo consegue anche che sono triangoli isosceli. $\measuredangle CQB=\measuredangle APB=180-2(74)=32$. Inoltre $S$ sta nell'asse di $BC$, di conseguenza anche nella bisettrice di $CBQ$. Ragionamento analogo per $APB$. Ora osserviamo il quadrilatero $PCAQ$. Questo ha gli angoli $APB$ e $CQB$ congruenti, da questo possiamo asserire che $PCAQ$ è ciclico. Questo permette di stabilire che $\measuredangle DAC=\measuredangle PQC$ e che $\measuredangle DCA=\measuredangle QPD$. Infine osserviamo il triangolo $QSP$. $\measuredangle QSP=180-\measuredangle QPS-\measuredangle PQS= 180-(16+\measuredangle PQC)-(16+\measuredangle QPA)=148-(\measuredangle PQC+\measuredangle APQ)=148-(\measuredangle DCA+\measuredangle DAC)=148-(180-\measuredangle ADC)=\measuredangle ADC-32.$ $\measuredangle ADC=360-(\measuredangle DCB+\measuredangle CBA+\measuredangle BAD)=360-3 \cdot 74=138$ da cui $\measuredangle QSP=138-132=106$. Essendo infine $S$ il circocentro di $ABC$, ne consegue che $\measuredangle ASC=2 \measuredangle ABC=148$. Quindi $\measuredangle ASQ + \measuredangle PSC= \measuredangle ASC- \measuredangle QSP=148-106=42$.
(Alessandro Avellino)

Problema 49 [15]
Dovendo esistere almeno un numero $k>1$ che elevato al cubo sia un divisore di $n$ si possono escludere i numeri da $2$ a $7$ in quanto l’unico loro divisore al cubo è $1$ e $k$ non può assumere tale valore. L’$8$ si può vedere come $2^3$ ed è un prigioniero di cubi, così come lo sono tutti i suoi multipli fino a $96$. È valido lo stesso discorso anche per i multipli di $27$ (essendo $3^3$) fino a $81$. Contandoli si arriva al risultato richiesto, $15$.
(Irene Mancone)

Problema 50

Problema 51

Problema 52[1872]
Chiamiamo le tre radici di p(x) a,b,c dove a<b<c . Essendoa,b e c in progressione aritmetica si ha che a = b-r e c = b+r dove r è la ragione della progressione aritmetica. Per le formule di Vietè a+b+c= 39 b-r + b+ b+ r= 39 da cui si ricava che b =13 . Sempre applicando le formule di Vietè, ab + ac + bc = 482 sostituendo i risultati ricavati in precedenza si trova che 13(13-r) + (13-r)(13+r) + 13(13+r) = 482 da cui sviluppando le parentesi 169 -13r +169 -r2 + 169 +13r = 482 ottenendo che r2 = 25 quindi poiché r è maggiore di 0 si ha che r=5. Quindi a=8 c=18 .
Applicando nuovamente le formule di Vietè, -m = -8*13*18 ovvero m= 1872
(Mattia Zunino)

Problema 53 [24]
Poniamo l'età di Marco al momento della prima affermazione uguale a $x$, quindi l'età del padre Luca è $4x$. Allora, $4$ anni dopo l'età di Marco è $x+4$ e quella del padre è $4x+4$, quindi vale l'equazione $3(x+4)=4x+4$; $x=12-4=8$ e infine $4x=32$.
La differenza di età tra i $2$ è $24$, quindi per rispondere alla domanda risolviamo $2y=y+24$, con $y$ l'età di Marco e $y+24$ l'età di Luca, $y=24$ che è quindi la risposta.
(Niso Cicalò)

Problema 54

Problema 55

Problema 56

Problema 57 [183]
Si può considerare il triangolo isoscele con i vertici di coordinate $B (0;0), C (20;0)$ e A$ (10; y)$ con $y > 0$
Il punto $H$ è l’intersezione tra le rette perpendicolari ai lati passanti per $A, B, C$. Sapendo che le distanze tra
$AH$ e $AD$ sono segmenti di lunghezza intera il punto $H$ si potrà trovare al minimo nella coordinata $(10;1)$. Calcolando il coefficiente angolare tra $H$ e i punti $B$ e $C$, le rette passanti per $B$ e $C$ con coefficiente angolare antireciproco si può ottenere l’ordinata di $A$ sostituendo in una delle due equazioni.
$m_{BH}$= $\frac{1}{10}$ $m^1$=$-10$ retta r passante per A e C: $y=-10x+200$
$m_{CH}$=$\frac{-1}{10}$ $m^1$=10 retta s passante per B e A : $y=10x$

Per $H (10,1)$ $A$ vale $(10, 100)$ e la distanza è intera ($99$).
Una volta stabilita l’ordinata massima di $A$ è sufficiente ripetere il procedimento. Per $H (10,2)$ $A (10, 50)$, per $H (10,4)$ $A(10,25)$. Nelle coordinate $(10, 10)$ $H$ e $A$ si sovrappongono. Poi si avrà che $A$ si trova in $(10,20)$, in $(10,2)$ e $(10,5)$. La soluzione si ottiene sommando le distanze trovate tra $H$ e $A$ senza ripetizioni.
(Irene Mancone)

Soluzione alternativa:
Essendo $ABC$ un triangolo isoscele la mediana $AD$ coincide con l'altezza rispetto a $BC$, chiamando $K$ il piede dell'altezza relativa ad $AC$ otteniamo che il triangolo $BKC$ è simile al triangolo $ABD$ poiché $\widehat{BKC} \cong \widehat{BDA} = 90^{\circ}$ e $\widehat{BCK} \cong \widehat{ABD}$ (essendo $ABC$ isoscele). Consideriamo ora i triangoli $BKC$ e $BDH$, anch'essi sono simili poiché $\widehat{BKC} \cong \widehat{BDH} = 90^{\circ}$ e $\widehat{CBK}$ è in comune. Per la proprietà transitiva $ABD \sim BKC$ e $BKC \sim BDH$ $\rightarrow$ $ABD \sim BDH$. Avendo dimostrato che i triangoli $ABD$ e $BDH$ sono simili vale la seguente uguaglianza: $\frac{BD}{AD}=\frac{HD}{BD} \rightarrow BD^2 = AD \cdot HD \rightarrow 100=AD \cdot HD$ (essendo $BD$ la metà di un segmento lungo $20$). Avendo che $AH$ e $AD$ sono di lunghezza intera anche $HD$ sarà di lunghezza intera (Poiché $HD$ è la differenza tra i due segmenti) ed avremo soluzioni limitate a $100=AD \cdot HD$ ovvero $(AD,HD)=(100,1),(50,2),(25,4),(20,5),(10,10),(5,20),(4,25),(2,50),(1,100)$. La soluzione $(AD,HD)=(10,10)$ non ci piace poiché $\widehat{ABC}$ varrebbe $90^{\circ}$ ($D$ sarebbe il centro della circonferenza circoscritta ad $ABC$ e $\widehat{ABC}$ sottenderebbe un diametro) dunque i possibili valori di $AH$ sono le diverse differenze fra le soluzioni ottenute ($AH=|AD-AH|$): $AH = 100-1= 99$, $AH = 50-2 = 48$, $AH = 25-4= 21$, $AH =20-5=15$ (le altre danno gli stessi valori poiché sono soluzioni "specchiate"). La risposta è quindi $99+48+21+15=183$.
(Lorenzo Bastioni)

Problema 58 [46]
La somma $S$ dei quadrati di delle radici un polinomio del tipo $ x^n + \alpha_{n-1} \cdot x^{n-1} + ... + \alpha_1 \cdot x + \alpha_0$ è

$S=\alpha_{n-1}^2-2 \cdot \alpha_{n-2}$

$$Q_n(x)=x^n+\sum_{i=0}^{n-1} \sqrt{i} \cdot x^i$$

Per cui, definiamo $S_n$ come la somma dei quadrati delle radici di $Q_n(x)$. Allora $S_n=n-1-2\sqrt{n-2}$ che è razionale solo se anche $2\sqrt{n-2}$ lo è. Questo accade solo quando $n-2$ è un quadrato perfetto, quindi ci sono $45$ valori. A questo aggiungiamo $n=1$, il cui polinomio è quindi $x$, che ha come radici un numero razionale. Quindi la soluzione è $46$.
(Alessandro Avellino)

Problema 59

Problema 60

Problema 61[9098]
Per il criterio di divisibilità per 9, s(n) ≡ n mod 9 dove s(n) è la somma delle cifre di n. Iterando il ragionamento su s(n) si ottiene che A(n)≡n mod9 . Poiché A(n) è un intero compreso fra 0 e 9, A(n) è uguale al resto della divisione di n per 9. I possibili resti della divisione delle potenze di 2021 per 9 sono 5, 7, 8, 4, 2, 1 e poi si ripetono ciclicamente con periodo 6. Perciò la somma richiesta è 2016/6 *(5+7+8+4+2+1) +5+7+8+4+2 = 9098.
(Mattia Zunino)
Problema 62 [1158]
Dette $H$ e $K$, rispettivamente, le proiezioni di $D$ e $C$ su $AB$ e $AH=x$ possiamo calcolare l'altezza del trapezio in due modi diversi servendoci del teorema di Pitagora ed eguagliare le espressioni $DH^2=AD^2-AH^2=33^2-x^2=CK^2=BC^2-BK^2=60^2-(69-6-x)^2$.

Risolvendo l'equazione, si trova l'unico valore $x=\frac{81}{7}$ e l'altezza si può quindi calcolare, ad esempio, come $h=DH=\sqrt{AD^2-AH^2}=\frac{60\sqrt{13}}{7}$. L'area del trapezio vale $A=\frac{(b_1+b_2)\cdot h}{2}=\frac{(AB+CD)\cdot AH}{2}=\frac{(69+6)\cdot (60\sqrt13)}{7\cdot2}\approx 1158,93$; la sua parte intera è $1158$ ed è la soluzione.
(Federico Magnolfi)

Problema 63

Problema 64

Problema 65 [5877]
$X$ si trova sulla superficie del triangolo $AMC$: infatti, se così non fosse, $\widehat{ABC}$ e $\widehat{AXM}$ sarebbero due angoli congruenti che insistono su uno stesso segmento ($AM$) e che appartengono allo stesso semipiano rispetto ad esso: ma ciò non è possibile in quanto significherebbe che $X$ appartiene alla circonferenza circoscritta al trangolo $ABM$, alla quale ovviamente non appartengono punti interni al triangolo $ABM$.

$AXMY$ è ciclico per ipotesi e $\widehat{AYM}$ e $\widehat{AXM}$ sono supplementari in quanto angoli opposti (per quanto prima dimostrato); dunque $\widehat{BYM}$, essendo anch'esso supplementare di $\widehat{AYM}$, è congruente a $\widehat{AXM}$, il quale a sua volta è congruente a $\widehat{ABC}$ per ipotesi.
Avremo allora che il triangolo $MBY$ è isoscele su base $BY$; essendo $M$ punto medio di $BC$, $MB\cong MY\cong MC$ e ciò implica che $BYC$ è rettangolo in $Y$. $CY$ è dunque l'altezza uscente da $C$ e si può calcolare con la formula inversa $CY=\frac{2A}{AB}$ dopo aver ricavato l'area con la formula di Erone o per via trigonometrica.
Si trova che quest'ultima vale $A=1200\sqrt{5}$ e dunque $CY^2=\left(\frac{2A}{AB}\right)^2=\frac{4\cdot1200^2\cdot 5}{70^2}\approx 5877,55$. La sua parte intera è $5877$ ed è la soluzione.
(Federico Magnolfi)

Problema 66 [1152]
Prendiamo l'equazione del problema e consideriamola $(\text{mod}\,7)$: $x^2 \equiv 7y + z \,(\text{mod}\,7) \Rightarrow x^2 \equiv z \, (\text{mod}\,7)$. I quadrati come $x^2$ modulo $7$ possono assumere come valori solo $0,1,2,4$ quindi $z$ potrà assumere tutti e i soli valori $0,1,2,4$ modulo $7$. Nell'intervallo $[1,2022]$ ci sono esattamente $288$ numeri congrui a $0 \, (mod 7)$, $288$ congrui a $1$, $288$ congrui a $2$ e $288$ congrui a $4$. La risposta è quindi $288 \cdot 4=1152$.
(Lorenzo Bastioni)

Problema 67 [3026]
Scriviamo il numeratore come $\prod_{n=2}^{2021}((n-1)+n+(n+1))=\prod_{n=2}^{2021}(3n)=3^{2021}\prod_{n=2}^{2021}(n)$. Si può quindi raccogliere un $3^{2021}$ al numeratore. Inoltre si raccolgono anche una potenza di $3^{1005}$ dato che $n$ può avere $3$ come fattore primo (elevato alla $1,\,2,\,3,\,\dots,6$). Ci sono $ \Big \lfloor \frac{2021}{3} \Big \rfloor=673$ multipli di 3, $ \Big \lfloor \frac{2021}{9} \Big \rfloor =224$ multipli di $9$ e così via: $ \Big \lfloor \frac{2021}{3} \Big \rfloor+ \Big \lfloor \frac{2021}{9} \Big \rfloor + \Big \lfloor \frac{2021}{27} \Big \rfloor + \Big \lfloor \frac{2021}{81} \Big \rfloor + \Big \lfloor \frac{2021}{243} \Big \rfloor + \Big \lfloor \frac{2021}{729} \Big \rfloor =673+224+74+24+8+2=1005$. In tutto quindi si è raccolto $2021+1005=3026$.
(Filippo Prandina)

Problema 68 [1104]
Chiamiamo due insiemi $A$ e $B$ "gemelli di primo grado" se $|A \cap B|=1$ e $1 \in A,B$, in generale chiamiamo due insiemi $A$ e $B$ "gemelli di $n$-esimo grado" se $|A \cap B|=n$ e $n \in A,B$.
Possiamo ora affrontare il conteggio di tutte le coppie di sottoinsiemi gemelle di $\{1,2,3,...,2021,2022 \}$ considerando prima i gemelli di primo grado, di secondo grado fino a quelli di $2022$-esimo grado (Non ci sono coppie di sottoinsiemi di $2023$-esimo grado o di $0$-esimo grado).
Gemelli di primo grado:
se $A$ e $B$ sono di primo grado allora contengono entrambi l'elemento $1$ e non hanno nessun altro elemento in comune. Ci verra comodo per dopo scrivere che gli altri zero elementi in comune si possono scegliere in $\displaystyle \binom{2021}{0}$ modi. Ora tutti gli altri $2021$ elementi possono essere messi o in $A$ o in $B$ o in nessuno dei due (in questo caso la "o" è una o esclusiva), quindi abbiamo 3 possibilità per ognuno dei $2021$ elementi ovvero $3^{2021}$ possibilità. Avremo quindi $\displaystyle \binom{2021}{0} 3^{2021}$ coppie di sottoinsiemi gemelle di primo grado.
In generale siamo pronti ad affrontare il caso di gemelli di $n$-esimo grado:
se $A$ e $B$ sono gemelli di $n$-esimo grado allora contengono entrambi l'elemento $n$ e hanno altri $n-1$ elementi in comune. Gli altri $n-1$ elementi in comune si possono scegliere in $\displaystyle \binom{2021}{n-1}$ modi. E per i restanti $2021-(n-1)=2022-n$ elementi abbiamo $3^{2022-n}$ possibilità come spiegato precedentemente. Avremo quindi $\displaystyle \binom{2021}{n-1} 3^{2022-n}$ coppie di sottoinsiemi gemelle di $n$-esimo grado.
Sommando tutte le possibilità otteniamo:
$$\sum_{k=1}^{2022} \binom{2021}{k-1} 3^{2022-k}$$
Reimpostando la somma sostituendo $k-1 \mapsto k$ otteniamo:
$$\sum_{k=0}^{2021} \binom{2021}{k} 3^{2021-k}$$
Ma questo è esattamente uguale a $(3+1)^{2021}=4^{2021}$ (Per il binomio di Newton o teorema binomiale). Riducendo l'enorme numero $(\text{mod}\,10000)$ si trova che le ultime $4$ cifre che ci interessano sono $1104$, da cui la risposta.
(Lorenzo Bastioni)

Problema 69

Problema 70

Problema 71

Problema 72[247]
Disponiamo le 32 squadre in qualunque ordine, e costruiamo gli incontri nel seguente modo: ad ogni turno, le squadre in un posto dispari si scontrano con la successiva. Le squadre perdenti vengono eliminate. Disponiamo le squadre restanti nello stesso ordine in cui si trovano, e ripetiamo finché resta solo una squadra. Ad esempio, se ad un certo punto l'ordine è $(3, 5, 2, 1)$, la squadra $3$ vince contro $5$ e $1$ vince contro $2$, dunque al turno dopo l'ordine sarà $(3, 1)$. Poniamo per semplicità la squadra $S_3$ per prima, e consideriamo i restanti $31!$ ordinamenti possibili. La squadra $S_3$ arriva in semifinale se e solo se non viene eliminata nei 3 turni precedenti, e ciò accade se e solo se nessuna tra le squadre $S_1$ ed $S_2$ è presente tra le prime 8 squadre. Possiamo quindi scegliere le prime 8, inclusa $S_3$, in $\displaystyle\binom{29}{7}$ modi, e le restanti in $24!$ modi.
La probabilità cercata è quindi $\displaystyle\binom{29}{7}\cdot 24!\cdot \frac{1}{31!}=\frac{29 \cdot 28 \cdot 27 \cdot 26 \cdot 25 \cdot 24 \cdot 23}{31 \cdot 30 \cdot 29 \cdot 28 \cdot 27}=\frac{24 \cdot 23}{31 \cdot 30}=\frac{92}{155}$. La risposta è quindi $92+155=247$.
(Valeria Martinelli)

Problema 73 [6068]
Notiamo che
$(p+q+r)(pq+qr+pr)=p^2q+p^2r+pq^2+pr^2+q^2r+qr^2+3pqr=1+3\cdot 2022=6067$.
Dato che $p+q+r$ e $pq+qr+pr$ sono entrambi interi positivi, devono necessariamente dividere entrambi $6067$. $6067$ è un numero primo, quindi o $p+q+r=1$ o $p+q+r=6067$. La risposta è perciò $6067+1=6068$.
Si noti che non ci si deve preoccupare di trovare che esistano $p,q,r$ che rispettano le condizioni trovate, dato che questo è assicurato dal fatto che sono le tre soluzioni dell'equazione $x^3-(p+q+r)x^2+(pq+qr+pr)x-pqr=0$, che ha sempre tre soluzioni nei numeri complessi.
(Federico Borasio)

Problema 74

Problema 75 [78]
Elevando entrambi i termini al quadrato (lecito perché entrambi sicuramente positivi) si ottiene $x^2+y^2-x-y+1/2 \leq 1/4$ riarrangiando $(x-1/2)^2+(y-1/2)^2 \leq 1/4$. Si ha quindi l'equazione di un cerchio centrato in $(1/2,1/2)$ con raggio $1/2$. La probabilità che un punto interno al quadrato definito nel piano cartesiano che ha vertice nell'origine e lato $1$ sia dentro il cerchio è quindi il rapporto tra l'area del cerchio e quella del quadrato: $\pi (1/2)^2 \approx 78%$.
(Filippo Prandina)

Problema 76 [4253]
Iniziamo col dimostrare che $P$ si trova all'esterno di $ABC$.

Detto $K$ il piede dell'altezza uscente da $A$ si ha che, essendo $\widehat{APB}$ congruente a $\widehat{AKB}$ perché entrambi retti, e dunque $AKBP$ ciclico, $P$ appartiene alla circonferenza circoscritta a $AKB$ alla quale non appartengono punti interni a $AKB$; ma $P$ appartiene alla mediana $AM$, la quale è evidentemente contenuta nel triangolo $AKB$ (lo si potrebbe dimostrare con i dati del problema), e quindi $P$ si trova all'esterno di $AKB$ ma anche di $ABC$.

Ora, è noto che in un triangolo di lati di lunghezza $9k$, $8k$, $7k$, con $k$ intero positivo, la mediana relativa alla base di lunghezza $8k$ misura $7k$: questo è facilmente verificabile per via trigonometrica, col teorema della mediana o anche con Pitagora; in ogni caso, $AM\cong AC$ e in particolare $\widehat{AMC}\cong \widehat{ACM}=\gamma$.

Sia $\left\{D\right\}=BC\cap PT$. $BTD\sim BCH$ per il secondo criterio di similitudine in quanto hanno l'angolo in $B$ in comune e $\widehat{BDT}\cong \widehat{BHC}$ perché retti, in particolare $\widehat{ATD}=\gamma$.

$\widehat{BMP}\cong\widehat{AMC}$ perché opposi al vertice ed essendo $BPM$ rettangolo in $P$ e $PD$ l'altezza relativa all'ipotenusa, $BPD=\gamma$. Allora $BPT$ è isoscele sulla base $PT$, in altre parole $T$ è il simmetrico di $P$ rispetto a $BC$; da ciò deriva anche che $BMT\cong BMP$, in particolare che $\widehat{BTM}$ sia retto.

$BMT\sim BCH$ per il secondo criterio di similitudine in quanto hanno l'angolo in $B$ in comune e $\widehat{BTM}\cong \widehat{BHC}$ perché entrambi retti. Ma allora, essendo $M$ punto medio di $BC$, necessariamente $T$ è punto medio di $BH$.

Il rapporto $\frac{[ATB]}{[THC]}$ è uguale al rapporto fra le altezze $\frac{AH}{CH}$ in quanto i due triangoli hanno basi congruenti per quanto appena dimostrato. Esse si possono calcolare con pitagora, dopo aver ricavato $BH=\frac{2A}{AC}$ e l'area attraverso Erone o per via trigonometrica. Sviluppando i calcoli si trova $BH=\frac{24\sqrt{5}}{2}$, $CH=\sqrt{BC^2-BH^2}=\frac{16}{7}$ e $AH=AC-CH=\frac{33}{7}$
Il risultato cercato è $1000\cdot \left(\frac{AH}{CH}\right)^2=1000\cdot\left(\frac{33}{16}\right)^2\approx4253,90$ la cui parte intera è $4253$ che è la soluzione.
(Federico Magnolfi)

Problema 77

Problema 78 [4151]
Cominciamo col dimostrare che $BCPQ$ è un trapezio. Sappiamo che l'ortocentro e il circocentro sono coniugati isogonali: in pratica, le rette $AH$ e $AO$ formano coi lati $AB$ e $AC$ angoli congruenti. Avremo dunque $\widehat{BAH}\cong \widehat{CAO}$, ma anche $\widehat{HAI}\cong \widehat{OAI}$ essendo $AI$ bisettrice perché $I$ incentro di $ABC$.

Riferendoci ora alla circonferenza circoscritta ad $ABC$, che chiamiamo $\Gamma$, abbiamo che $\widehat{FCE}\cong \widehat{FAE}\cong\widehat{EAD}\cong \widehat{ECD}$ dove la prima e la terza uguaglianza sono date dal fatto che gli angoli insistono su uno stesso arco. Facendo attenzione, notiamo che $\widehat{QAP}$ e $\widehat{QCP}$ sono congruenti per l'uguaglianza precedente e insistono sullo stesso segmento $QP$, dunque $AQPC$ è ciclico e chiamiamo $\omega$ la sua circonferenza circoscritta. Essendo $\widehat{ACD}$ retto poiché insiste sul diametro $AD$ di $\Gamma$, $\widehat{AQP}$ sarà anch'esso retto poiché supplementare dell'angolo opposto $\widehat{ACP}$ (ricordando che ora abbiamo fatto rifereimento a $\omega$). $BC$ e $QP$ sono allora due rette perpendicolari alla stessa retta $AQ$, perciò sono parallele e $BCPQ$ è un trapezio.

Per calcolare la sua area abbiamo bisogno dell'altezza $QH$ e della base minore $QP$, dato che la base maggiore ci è nota ed è $BC$.

$\textbf{Prima soluzione (sintetica)}$
Detti $K$ e $J$ rispettivamente i piedi dell'altezza $AH$ e della bisettrice $AI$, analizziamo i triangoli $ABJ$ e $AEC$: essi hanno $\widehat{ABJ}\cong \widehat{AEC}$ perché angoli alla circonferenza che insistono su uno stesso arco; $\widehat{BAJ}\cong \widehat{EAC}$ perché $AI$ bisettrice: quindi sono simili per il secondo criterio di similitudine. Considerando ora i triangoli $AKJ$ e $AQP$: essi hanno: l'angolo in $A$ in comune; $\widehat{AKJ}\cong \widehat{AQP}$ perché retti: quindi sono anch'essi simili per il secondo criterio di similitudine. Impostiamo la proporzione $\frac{QP}{KJ}=\frac{AP}{AJ}$ $(1)$: troveremo $KJ$ e $AJ$ lavorando su $ABC$, mentre $AP$ attraverso il rapporto di similitudine fra i triangoli $ABJ$ e $AEC$. Calcoliamo $AK$ invertendo la formula dell'area di $ABC$ e $BK$ col teorema di Pitagora: $AK=\frac{2A}{BC}=\frac{2\cdot \sqrt{p(p-AB)(p-BC)(p-AC)}}{BC}=120 \quad$ $BK=\sqrt{AB^2-AK^2}=50$.
Applichiamo ora il teorema della bisettrice per trovare $BJ$: $\frac{BJ}{AB}=\frac{BC-BJ}{AC}$ da cui si ricava che $BJ=65$. Dunque $KJ=BJ-BK=15$ e $AJ=\sqrt{AK^2+KJ^2}=15\sqrt{65}$.

Impostiamo il rapporto di similitudine fra $ABJ$ e $AEC$: $\frac{AP}{AB}=\frac{AJ}{AC}$ da cui si ricava che $AP=\frac{150\sqrt{65}}{7}$ e per la proporzione $(1)$ $QP=\frac{AP\cdot KJ}{AJ}=\frac{150}{7}$. A questo punto si trova $AQ$ con pitagora e $QK$ per differenza: $AQ=\sqrt{AP^2-QP^2}=\frac{1200}{7} \quad QK=AQ-AK=\frac{360}{7}$
L'area del trapezio $BCPQ$ è dunque $$A=\frac{(140+\frac{150}{7})\cdot\frac{360}{7}}{2}\approx 4151,02$$.

$\textbf{Seconda soluzione (trigonometria)}$
Ponendo $\widehat{ABC}=\beta$, $\cos\beta=\frac{AB^2+BC^2-AC^2}{2AB\cdot BC}=\frac{5}{13}$ (teorema del coseno), $\sin\beta=\sqrt{1-\cos^2\beta}=\frac{12}{13}$ $\widehat{BAC}=\alpha$, $\widehat{CAI}=\frac{\alpha}{2}$ perché $AI$ bisettrice, $\cos\alpha=\frac{AB^2+AC^2-BC^2}{2AB\cdot AC}=\frac{33}{65}$ (teorema del coseno), $\cos\frac{\alpha}{2}=\sqrt{\frac{1+\cos\alpha}{2}}=\frac{7}{\sqrt{65}}$.

Adesso abbiamo abbastanza dati per calcolare $AP=\frac{AC}{\cos\frac{\alpha}{2}}=\frac{150\sqrt{65}}{7}$.
Ponendo $\widehat{HAI}=\widehat{OAI}=\theta$ si ha che, essendo $\widehat{BAH}=\frac{\pi}{2}-\beta$, $\theta=\frac{\alpha}{2}+\beta-\frac{\pi}{2}$; $\cos\theta=\sin\frac{\alpha}{2}\cos\beta+\cos\frac{\alpha}{2}\sin\beta=\frac{8}{\sqrt{65}}$

$AQ=AP\cos\theta=\frac{1200}{7}\quad QP=\sqrt{AP^2-AQ^2}=\frac{150}{7}$. Ci manca l'altezza $QK$ dove $K$ è il piede dell'altezza uscente da $A$. $AK=AB\sin\beta=120\qquad QK=AQ-AK=\frac{360}{7}$. A questo punto l'area si trova allo stesso modo della soluzione per via sintetica.
(Federico Magnolfi)

Problema 79 [2011]
Si ha che $p^{(2)}(x)$ ha lo stesso grado di $p^{(4)}(x)$ questo significa che $p(x)$ è di grado $1$ o grado $0$. Ma non può essere di grado $0$ perché non è costante. Allora è del tipo $p(x)=mx+q$. La funzione $p(x)$ è chiaramente iniettiva ed essendo $p^{(2)}(x)=p^{(2022)}(x)$ per ipotesi, si ha necessariamente che $p(x)=p^{(2021)}(x)$. Ma quindi $p(2021)=0$ e $p(p(2021))=p(0)=2021$. Da qui si ricava facilmente che il polinomio è $p(x)=-x+2021$ e quindi $p(10)=2011$.
(Filippo Prandina)

Problema 80


Problema 81

Problema 82 [4]
$45=9\cdot 5$ quindi scomponiamo il modulo in modulo $9$ e modulo $5$. Modulo $5$ è semplicemente $4$, in quanto l'ultima cifra è proprio $4$. Definiamo $\overline{a_1a_2a_3...a_k}$ la scrittura decimale in successione di $a_1,a_2,...,a_k$, per esempio $\overline{4^2 5^2}=1625$. Inoltre, chiamato $N$ il nostro numero, $N=\overline{1^22^23^24^2...2022^2}$. $\overline{a_1a_2a_3...a_k}\equiv a_1+a_2+a_3+...+a_k\,(\mathrm{mod}\,9)$, dimostrarlo è facile. I residui quadratici di $9$ sono $0,1,4,7$ mentre la somma dei quadrati da $1$ a $9\,\mathrm{mod}\,9$ è $24$, ($-3$). Quindi ogni $9$ numeri vi è un $-3$ da addizionare, quindi è $(-3)\cdot 224$ (che sarebbe il numero di multipli di $9$ minori di $2022$). $-672$, a questo devi aggiungere i numeri da $2017$ a $2022$, $-672+1+4+7+7=-653\equiv -14\equiv 4$.
Siccome $N$ è congruo a $4$ sia $\mathrm{mod}\,5$ che $\mathrm{mod}\,9$, sarà congruo a $4$ anche $\mathrm{mod}\,45$.
(Alessandro Avellino)

Problema 83 [21]
Sia $H$ il piede dell'altezza uscente da $A$; $AY=AH-HY$. L'altezza $AH$ si può calcolare, ad esempio, invertendo la formula dell'area $A=\frac{b\cdot h}{2}$ dopo aver trovato quest'ultima con la formula di Erone, oppure trovando il coseno dell'angolo in $B$ col teorema del coseno, quindi il suo seno e applicando il primo teorema dei triangoli rettangoli a $ABH$. Eseguendo i calcoli, si trova che essa vale $AH=3\sqrt{5}$. $BH=\sqrt{AB^2-AH^2}=2$.

Detto $M$ il punto medio di$BC$ si ha che $HM=HX=2$ e quindi esso è punto medio anche di $HX$; ma $OM\perp BC$ in quanto in un triangolo isoscele la mediana è anche altezza: allora, essendo $HYX$ e $MOX$ simili per il secondo criterio di similitudine, e avendo rapporto di similitudine $2$, $HY=2HO$

Attraverso il teorema della corda, oppure attraverso la formula nota, possiamo calcolare il raggio della circonferenza circoscritta ad $ABC$, $R=\frac{BC}{2\sin\alpha}=\frac{abc}{4A}=\frac{21}{2\sqrt{5}}$.
Ora possiamo trovare $HY=2HO=2\sqrt{R^2-MC^2}=\frac{11\sqrt{5}}{5}$ e infine $AY=AH-HY=3\sqrt{5}-\frac{11\sqrt{5}}{5}=\frac{4\sqrt{5}}{5}$.

Stiamo cercando $AY^2=\frac{16}{5}$ e la risposta è infine $16+5=21$.
(Alessandro Lombardo & Federico Magnolfi)

Problema 84

Problema 85 [1136]

Iniziamo col calcolarci alcuni oggetti che in futuro potranno esserci utili.

Perimetro: 2p=14+16+18=48

l'Area: usando la formula di Erone, A=$\sqrt{p(p-a)(p-b)(p-c)}=\sqrt{24\cdot10\cdot8\cdot6}=48\sqrt{5}$

Raggio circonferenza circoscritta: $R=\frac{abc}{4A}=\frac{4032}{192\sqrt{5}}=\frac{21}{\sqrt{5}}=\frac{21\sqrt{5}}{5}$

Altezza relativa a BC: $AD=2A/BC=6\sqrt{5}$

Altezza relativa a AB, che incontra AB in F: $CF=2A/AB=\frac{48\sqrt{5}}{7}$

Detto L il punto di incontro tra BC e la perpendicolare passante per O, L è il punto medio di BC. Per cui CL=8, OC= R, $OL=\sqrt{\frac{441}{5}-\frac{320}{5}}=\sqrt{\frac{121}{5}}=\frac{11\sqrt{5}}{5}$

$CD=\sqrt{AC^2-AD^2}=\sqrt{144}=12$

$AF=\sqrt{AC^2-CF^2}=\frac{66}{7}$

LD=CD-CL=12-8=4

adesso calcoliamo OD:

$OD=\sqrt{OL^2+LD^2}=\sqrt{\frac{201}{5}}$

Infine calcoliamo AH, dove H è l'ortocentro. per calcolare AH notiamo che, AHF è simile a ABD, per cui $\frac{AH}{AB}=\frac{AF}{AD} \Rightarrow AH=\frac{AF \cdot AB}{AD}=\frac{66 \cdot 14}{42\sqrt{5}}=\frac{22\sqrt{5}}{5}$.

Adesso che abbiamo tutto ciò che ci serve proseguiamo. O è il punto di Miquel relativo ad XYD. questo vuol dire che O appartiene alla circonferenza passante per X,Y,A. Detto questo, abbiamo 2 quadrilateri ciclici, per definizione, BDOX e CDOY. A questi 2 aggiungiamo il quadrilatero AYOX, anch'esso ciclico.

1° Claim: XYD è simile ad ABC.
è sufficiente un po' di angle chasing:

ricordiamo che OA, OB e OC sono i raggi di (ABC).
Di conseguenza OBA=OAB, OBC=OCB e OCA=OAC. Inoltre, per via della ciclicità dei 3 quadrilateri sopra elencati,
OAB=OBA=OBX=ODX, OAC=OCA=OCY=ODY, da questo segue che YDX=ODY+ODX=CAO+OAB=BAC.

Analogamente DYX=CBA e DXY=BCA. Per cui i 2 triangoli sono simili

2° Claim: O è l'ortocentro di XYD.
Anche qui è sufficiente un po' di Angle chasing:

DYX+YDO=(DYO+OYX)+OCY=DCO+OAX+OCA=(DCO+OCA)+OAX=ACB+OAX=ACB+(OAH+HAX)=ACB+(OAH+HAB)=
ACB+(OAH+DAB)=ACB+(OAH+OAC)=ACB+DAC=90.
Ne consegue che OD è perpendicolare ad XY. Analogamente si ottiene che , XO e YO sono perpendicolari, rispettivamente, a DY e DX. Per cui O è l'ortocentro.

Di conseguenza, detto K il rapporto di similitudine tra ABC e XYD, $K=\frac{AH}{OD}=\frac{22}{201}$, il rapporto vale anche per i raggi r e R rispettivamente di XYD, e di ABC, $K=\frac{R}{r} \Rightarrow r=\frac{R}{K}=\frac{21\sqrt{1005}}{110}$
(Alessandro Avellino)

Problema 86 [1483]
Per prima cosa notiamo che, dato che $\frac{x_1+x_2}{x_3+x_4}$ è intero, abbiamo che $x_1+x_2 \ge x_3+x_4$, e similmente $x_3+x_4 \ge x_5+x_6 \ge x_1+x_2$.
Dunque $x_1+x_2= x_3+x_4=x_5+x_6$. Analogamente $x_2+x_3=x_4+x_5=x_6+x_1$.

Sia ora $S=x_1+x_2= x_3+x_4=x_5+x_6$ e $T=x_2+x_3= x_4+x_5=x_6+x_1$.
Abbiamo quindi che $\sum x_i = 3S=3T$, quindi $S=T$, ossia $x_1+x_2=x_2+x_3$, da cui $x_1=x_3$. In modo analogo otteniamo $x_1=x_3=x_5$ e $x_2=x_4=x_6$.

Possiamo quindi scrivere la sestupla come $(a, b, a, b, a, b)$, con $a, b$ interi positivi. Si verifica facilmente che ogni sestupla siffatta rispetta le condizioni del testo, difatti la somma di due termini consecutivi è costante, dunque le frazioni sono tutte uguali a $1$ e quindi intere.
Dunque, fissati $a, b$, c'è una e una sola sestupla valida.

Consideriamo ora due casi:

$\bullet$ $M$ non è multiplo di $3$. Allora, dato che $M=3S$, non ci sono sestuple valide, e quindi $f(M)=0$, che è un multiplo di $5$. In questo caso abbiamo quindi $2022-\frac{2022}{3}=1348$ possibili valori di $M$.

$\bullet$ $M=3k$, per qualche $k$ intero positivo. Dato che $k=a+b$, per ogni valore di $k$, ci sono $k-1$ sestuple valide, poiché abbiamo $k-1$ possibili valori di $a$, dunque $f(M)=k-1$. Vogliamo quindi che $k-1$ sia multiplo di $5$. Se $k=1$, non ci sono sestuple valide in quanto $a+b>1$, altrimenti, potendo scegliere $k$ nell'intervallo $[2, 1348]$, ci sono ${\lfloor \frac{1347}{5} \rfloor}=134$ valori accettabili.

In totale abbiamo quindi $1348+1+134=1483$ possibili valori di $M$.
(Valeria Martinelli)

Problema 87

Problema 88 [6418]
Osserviamo che, prese $3$ cifre distinte $a, b, c$ dall'insieme $X=\{1, 2, 3, 4, 5, 6, 7, 8, ,9 \}$, ci sono esattamente $6$ numeri compresi tra $100$ e $999$ che si scrivono con tutte e sole le cifre $a, b, c$, siano questi $x_1, ..., x_6$. Abbiamo che la loro somma $x_1+...+x_6$ vale $222a+222b+222c$, quindi $\sum f(x_i)=\frac{222(a+b+c)}{a+b+c}=222$.
Ci sono $\displaystyle\binom{9}{3}=84$ modi di scegliere $a, b, c$, quindi la somma in questo caso vale $222\cdot 84=18648$

Dobbiamo ora distinguere i casi in cui $a, b, c$ contengono uno o più zeri o cifre ripetute.
$\bullet$ $a=b=c$: per ognuna delle $9$ possibili scelte di $a, b, c$, c'è uno e un solo numero $x=111a$, e $f(x)=\frac{111a}{3a}=37$. Dunque la somma in questo caso vale $37 \cdot 9=333$

$\bullet$ $a=b \neq c$, $a, b, c \neq 0$: abbiamo $9\cdot 8=72$ possibili scelte di $a, b, c$, e per ogni terna abbiamo $3$ numeri la cui somma vale $222a+111c$, per cui $\sum f(x_i)=\frac{222a+111c}{2a+c}=111$. In tutto $72 \cdot 111=7992$

$\bullet$ $b=c=0$: $9$ possibili scelte di $a$, e per ogni $x=100a$ che rispetta queste condizioni $f(x)=100$. La somma in questo caso vale $100 \cdot 9=900$

$\bullet$ $a=b$, $c=0$; $9$ possibili scelte per $a, b$, per ognuna $\sum f(x_i)=\frac{211a}{2a}=\frac{211}{2}$. Quindi in tutto $9\cdot \frac{211}{2}=949,5$

$\bullet$ $a\neq b$, $c=0$: $\displaystyle\binom{9}{2}=36$ possibilità per $a, b$; per ognuna $\sum f(x_i)=\frac{211a+211b}{a+b}=211$. In tutto $36 \cdot 211=7596$

La somma richiesta è quindi uguale a $18648+333+7992+900+949,5+7596=36418,5$. Ci interessano le ultime quattro cifre della parte intera.
(Valeria Martinelli)

Problema 89 [134]
Sia $2^k$ la massima potenza di $2$ nell'intervallo $\{1, ..., n\}$. Si osserva facilmente che $f(n)$ non può mai essere maggiore di $f(2^k)$, pertanto $f(n)$ è massima quando $n=2^k$, e $g(2^k)=1$.

Dato che $5>4=2^2$ e $9=3^2>2^3=8$, se un intero contiene nella sua fattorizzazione primi diversi da $2$ e $3$, o se $3$ compare con un esponente maggiore o uguale a $2$, $g(n)=g(n-1)$ in quanto $n$ ha sicuramente meno fattori di $2^k$.
Inoltre, se $n=2^{k-1}\cdot 3$, $g(n)=g(n-1)+1$, in quanto n contiene lo stesso numero di fattori di $2^k$.

Dunque, se $2^{k-1}\cdot 3\le n <2^{k}$, $g(n)=2$, altrimenti $g(n)=1$.

Possiamo quindi trovare tutti gli interi minori o uguali a $99$ per cui $g(n)=2$, che sono i valori compresi tra i tripli delle potenze di $2$ e la potenza di $2$ successiva.

Dunque la somma richiesta è uguale a $99+(4-3)+(8-6)+(16-12)+(32-24)+(64-48)+(100-96)=99+1+2+4+8+16+4=99+31+4=134$.
(Valeria Martinelli)

Problema 90 [8784]
Chiamiamo $f(n)=x^n+\frac{1}{x^n}$, il problema ci chiede di calcolare $|f(1)\cdot f(2)\cdot\dots\cdot f(2022)|$.
Esplorando un po’ la situazione si può notare che $f(1)=1,f(2)=-1,f(3)=-2,f(4)=-1,f(5)=1$ e $f(6)=2$ e che questi valori potrebbero formare un ciclo che si ripete ogni $6$.
Dimostriamolo:
consideriamo $f(n) \cdot f(1)= \left( x^n+\frac{1}{x^n} \right) \cdot \left(x+ \frac{1}{x} \right) = x^{n+1} + x^{n-1} + \frac{1}{x^{n-1}} + \frac{1}{x^{n+1}} = f(n+1) + f(n-1)$, essendo $f(1)=1$ otteniamo che $f(n) \cdot f(1) =f(n)= f(n+1) + f(n-1)$ da cui $f(n+1)=f(n)-f(n-1)$. L’ultima relazione ci indica che $f(n+1)$ dipende unicamente da $f(n)$ e $f(n-1)$ ovvero se troviamo un ciclo quest’ultimo si ripeterà. Controllando si può vedere che $f(7)=1$ e $f(8)=-1$ quindi il ciclo si ripeterà. Analizzando il nostro ciclo si nota facilmente che gli unici fattori che contribuiranno nel nostro prodotto saranno quelli con $n$ multiplo di $3$, che in valore assoluto valgono $2$. Il nostro prodotto sarà quindi del tipo $2^k$ dove $k$ è il numero di multipli di $3$ tra $1$ e $2022$ ovvero $674$ $(\frac{2022}{3}=674)$. Il problema si è trasformato in un quesito di teoria dei numeri poiché dobbiamo dare come risposta le ultime $4$ cifre di $2^{674}$ quindi calcolare $2^{674}$ $(\mathrm{mod}\,10000)$. Riducendo con un po’ di conti l’enorme numero si trova $8784$.
(Lorenzo Bastioni)

Problema 91[4929]
Osserviamo che $g(n)=3^5 \cdot \frac{n}{v_{3}(n)}$, dunque la somma richiesta è uguale a $\sum 3^5 \cdot \frac{n}{v_{3}(n)}$. Definiamo quindi $h(n)=\frac{n}{v_{3}(n)}$ e calcoliamo separatamente le somme di $h(n)$ in base a $v_{3}(n)$.

$\bullet$ $v_{3}(n)=5$ \rightarrow $h(n)=1$: $n \in \{243 \cdot 1\}$, \rightarrow $\sum h(n)=1$

$\bullet$ $v_{3}(n)=4$: $n \in \{81 \cdot 2, ..., 81 \cdot 5\}$ esclusi i multipli di $243$, $\sum h(n)=(2+...+5)-1\cdot 3=14-3=11$

$\bullet$ $v_{3}(n)=3$: $n \in \{27 \cdot 5, ..., 27 \cdot 15\}$ esclusi i multipli di $81$, $\sum h(n)=(5+...+15)-14\cdot 3=110-42=68$

$\bullet$ $v_{3}(n)=2$: $n \in \{9 \cdot 15, ..., 9 \cdot 46\}$ esclusi i multipli di $27$, $\sum h(n)=(15+...+46)-110\cdot 3=976-330=646$

$\bullet$ $v_{3}(n)=1$: $n \in \{3 \cdot 45, ..., 3 \cdot 140\}$ esclusi i multipli di $9$, $\sum h(n)=(45+...+140)-976\cdot 3=8880-2928=5952$

$\bullet$ $v_{3}(n)=0$: $n \in \{1 \cdot 135, ..., 1 \cdot 420\}$ esclusi i multipli di $3$, $\sum h(n)=(135+...+420)-8880\cdot 3=79365-26640=52725$

Quindi, $\sum h(n)=52725+5952+646+68+11+1=59403$, da cui $\sum f(n)=3^5 \cdot \sum h(n)=14434929$. Ci interessano le ultime 4 cifre
(Valeria Martinelli)

Problema 92

Problema 93 [279]
$x$,$y$ e $z$ sono interscambiabili. Per cui è possibile, per facilitarci nella risoluzione, porre $x \geq y \geq z$.

Adesso andiamo in ordine:\\
abbiamo

$x^2<x^2+2y+z \leq x^2+3x < x^2+4x+4=(x+2)^2$

e siccome lo vogliamo come quadrato

$x^2+2y+z=(x+1)^2 \Rightarrow 2y+z=2x+1 \Rightarrow x=\frac{2y+z-1}{2}$.

Ora usiamo la seconda equazione con le nuove sostituzioni.\\
$y^2<y^2+2z+x=y^2+2z+\frac{2y+z-1}{2}=y^2+y+2.5(z)-0.5<y^2+y+3y+4=(y+2)^2$.

Per lo stesso motivo di prima:

$y^2+y+2.5(z)-0.5=(y+1)^2 \Rightarrow 5z-3=2y \Rightarrow y=\frac{5z-3}{2}$\\
Ora è il momento di usare l'ultima equazione, anche qui sostituendo $y$ e $x$:

$z^2<z^2+2x+y=z^2+2y+z-1+y=z^2+z+3(\frac{5z-3}{2})-1=z^2+8.5z-4.5$.

Dopo vari tentativi si nota che gli unici quadrati possibili per questa quantità sono $(x+1)^2$ e $(x+4)^2$.\\

Infatti si avrà $z^2+8.5z-4.5=z^2+8z+16 \Rightarrow z=43$, trovandoti poi $y=106$ e $x=127$ e $z^2+8.5z-4.5=z^2+2z+1$ trovando quindi $x=z=y=1$.
(Alessandro Avellino)

Problema 94 [6460]
Possiamo riscrivere il testo del problema nel seguente modo:
$$\sum_{n=1}^{2020} \lfloor \frac{\sqrt{n+2}}{\sqrt{n+1} - \sqrt{n}} \rfloor$$
Andiamo ad analizzare il nostro addendo generale privo della funzione floor:
$ \frac{\sqrt{n+2}}{\sqrt{n+1} - \sqrt{n}} = \frac{\sqrt{n+2}}{\sqrt{n+1} - \sqrt{n}} \cdot \frac{\sqrt{n+1} + \sqrt{n}}{\sqrt{n+1} + \sqrt{n}} = \sqrt{(n+2) \cdot (n+1) } + \sqrt{ (n+2) \cdot n } = \sqrt{n^2 + 3n+ 2} + \sqrt{n^2 +2n} $.
Esplorando un po' la situazione ci si potrebbe convincere che $2n+2 \leq \sqrt{n^2 + 3n+ 2} + \sqrt{n^2 +2n} < 2n+3$ (con $n$ nell'intervallo che interessa a noi ovvero $[1,2020]$).
Dimostriamolo:
La seconda disuguaglianza è molto semplice infatti $\sqrt{n^2 + 3n+ 2} + \sqrt{n^2 +2n} = \sqrt{(n+2)^2 -n-2} + \sqrt{(n+1)^2 - 1} < \sqrt{(n+2)^2} + \sqrt{(n+1)^2} = n+2 +n+1 = 2n+3$ quindi $\sqrt{n^2 + 3n+ 2} + \sqrt{n^2 +2n} < 2n+3$.
La prima disuguaglianza è leggermente più fastidiosa:
vogliamo $2n+2 \leq \sqrt{n^2 + 3n+ 2} + \sqrt{n^2 +2n}$ che è molto lungo da risolvere a mano dunque proviamo a scomporre in maniera intelligente il problema:
ad esempio, se dimostrassimo che $\sqrt{n^2 + 3n+ 2} \geq n + \frac{4}{3} \wedge \sqrt{n^2 +2n} \geq n+\frac{2}{3}$ avremmo finito poiché sommando le disuguaglianze si otterrebbe la tesi (la veridicità delle due disuguaglianze implica la tesi ma non viceversa).
Le due disuguaglianze sono due semplici disequazioni irrazionali e non ci dovremmo preoccupare di condizioni di esistenza o simili poiché nel nostro intervallo è tutto positivo:
1)
$ \sqrt{n^2 + 3n+ 2} \geq n + \frac{4}{3} \rightarrow n^2+3n+2 \geq (n + \frac{4}{3})^2 \rightarrow n \geq - \frac{2}{9}$, nel nostro intervallo la disuguaglianza vale.
2)
$ \sqrt{n^2 +2n} \geq n+\frac{2}{3} \rightarrow n^2+2n \geq (n+\frac{2}{3})^2 \rightarrow n \geq \frac{2}{3} $, nel nostro intervallo la disuguaglianza vale.
La tesi è dunque dimostrata, $2n+2 \leq \sqrt{n^2 + 3n+ 2} + \sqrt{n^2 +2n} < 2n+3$. Ciò implica che $\lfloor \frac{\sqrt{n+2}}{\sqrt{n+1} - \sqrt{n}} \rfloor = 2n+2$ (nel nostro intervallo) e possiamo riscrivere la nostra somma:
$$\sum_{n=1}^{2020} 2n+2 = \left( 2 \sum_{n=1}^{2020} n \right) + 2 \cdot 2020 = 2020 \cdot 2021 + 4040 = 4086460$$
(La seconda uguaglianza si ha con la formula di Gauss per la somma dei primi $k$ numeri naturali) Prendendo le ultime $4$ cifre del risultato abbiamo la soluzione: $6460$.
(Lorenzo Bastioni)

Problema 95 [21]
Mettiamo in un piano cartesiano i punti [math]. Si ha che [math] è il coffieciente angolare della retta per i due punti [math]. Quindi una sequenza è non genovese se tutti i coefficienti angolari sono non negativi; ovvero se e solo se essa è debolmente crescente. Supponiamo di avere una sequenza debolmente crescente [math]. Vi è una bigezione fra le sequenze di questo tipo e quelle del tipo [math], che sono tutte le sequenze strettamente crescenti con elementi positivi minori di [math]. Esse a loro volta corrispondono con i modi di scegliere [math] elementi distinti fra [math]. Il numero totale di sequenze (genovesi e non) è infine [math]; quindi il numero di quelle genovesi è [math] che è congruo a [math] modulo [math], essendo che [math] modulo [math].
(Federico Volpe)

Problema 96 [2029]
I numeri curiosi sono esattamente i liberi dai quadrati.\\
($\Leftarrow$) Se $n$ è un numero libero da quadrati allora $\tau(n)$ è una potenza di due, inoltre ogni divisore $d$ di $n$ è un numero libero da quadrati, quindi anche $\tau(d)$ è una potenza di $2$ e dato che $\tau(d) \le \tau(n)$ abbiamo che $\tau(d)|\tau(n)$.\\
($\Rightarrow$) Supponiamo che $n = p_1^{a_1}p_2^{a_2}\cdots p_m^{a_m}$ sia un numero libero da quadrati. Allora abbiamo
\begin{align*}
&\tau(n/p_i)|\tau(n)\\
\Rightarrow &(a_1+1)\cdots(a_i)\cdots(a_m+1)|(a_1+1)\cdots(a_i+1)\cdots(a_m+1)\\
\Rightarrow &a_i | a_i+1 \Rightarrow a_i = 1.
\end{align*}
Dunque $n$ è libero da quadrati.\\
\\
Inoltre notiamo che $f(a,b)$ è semplicemente $MCD(a,b)$. A questo punto scriviamo
$$S(n) = \sum_{j=1}^n f(j,n) = \sum_{j=1}^n MCD(j,n),$$
dimostreremo che la funzione $S$ è moltiplicativa, cioè per ogni $m,n \in \mathbb{Z}^+$ coprimi $S(m)S(n) = S(mn)$. Infatti, segue dal teorema del resto cinese, possiamo rimpiazzare $j \text{ mod }\,mn$ con due numeri $j_m \text{ mod }\,m$ e $j_n \text{ mod }n$ in modo che:
\begin{align*}
S(mn) &= \sum_{j=1}^{mn} MCD(j,mn) = \sum_{j=1}^{mn} MCD(j,m)MCD(j,n)\\
&= \sum_{j_m=1}^{m} \sum_{j_n =1}^{n} MCD(j_m,m)MCD(j_n,n)\\
&= \left(\sum_{j_m=1}^m MCD(j_m,m)\right)\left(\sum_{j_n=1}^n MCD(j_n,n)\right) = S(m)S(n).
\end{align*}
Ma quindi dato che $C$ è un prodotto tra primi ed è facile verificare che $S(p) > 1$ per ogni primo $p$, abbiamo che $C$ stesso deve essere primo. Un po' di brute-force dà la risposta.
(Michele Tomasi)

Problema 97 [118]

Problema 98

Problema 99

Problema 100 [1339]
Premetto innanzitutto che questo geometrico è più tecnico degli altri ed è necessario avere qualche prerequisito sulle simmediane e il cerchio di Feuerbach. Tutti i fatti noti di cui farò uso si trovano sul libro di Evan Chen ma sono certo che si possano trovare anche girovagando su internet.

La soluzione di questo problema si può suddividere in tre claim.

Primo claim: La tangente di $\Gamma$ in $N_a$, la retta $BC$ e la tangente di $\Gamma$ in $A$ concorrono nel punto $X_a$.\\
Qua c'è solo da usare qualche fatto noto delle simmediane. Infatti $\triangle BAN_a \sim \triangle CAM_a \Rightarrow \angle BAN_a= \angle CAM_a \Rightarrow AN_a$ è la $A-simmediana$ di $\triangle ABC$. Dopodiché basta usare che $CB$ è la $C-simmediana$ di $\triangle ACN_a$ e quindi le tangenti in $A$ e in $N_a$ concorrono con essa.

Secondo claim:
$Y_a$ e simili stanno sulla circonferenza di Feuerbach, ovvero $\gamma$ e $\omega$ coincidono.
Per essere capaci di dimostrarlo però è necessario notare qualcosa di più subdolo.
Sia $H$ l'ortocentro di $\triangle ABC$, $V$ il punto medio di $AH$ e $Q$ l'intersezione tra la retta $AH$ e $\Gamma$. Siano inoltre $R$ e $r$ i raggi di $\Gamma$ e di $\gamma$.
Lemma: $V$ è L'ortocentro di $\triangle AM_{a}X_{a}$.
Proof: Dimostriamo innanzitutto che la retta $VM_a$ è perpendicolare a $AX_a$. CIò è equivalente a dimostrare che $\angle X_{a}M_{a}V = 90^{\circ}- \angle AX_{a}M_a=\angle X_{a}AP_{a} \Leftrightarrow \angle P_{a}M_{a}V=\angle X_{a}AQ \Leftrightarrow \frac{VP_{a}}{AQ}=\frac{r}{R}=\frac{1}{2}$.
Poiché $Q$ è il simmetrico di $H$ su $BC$ (si dimostra con angle chasing), $HQ+HA=2HP_{a}+2HV \Rightarrow AQ=2VP_{a} \Rightarrow \frac{VP_{a}}{AQ}=\frac{1}{2}$.
$AV \perp X_{a}M_{a}$ e $M_{a}V \perp AX_{a} \Rightarrow V$ ortocentro di $\triangle AM_{a}X_{a}$.
A questo punto per dimostrare il claim iniziale basta dire che, poichè $V$ è ortocentro, $X_{a},V,Y_{a}$ sono allineati e quindi $\angle VY_{A}M_{a}=90^{\circ}=\angle VP_{a}M_{a} \Rightarrow VY_{a}M_{a}P_{a}$ ciclico $\Rightarrow Y_{a}$ sta su $\gamma$.
Similarmente si può dimostrare che anche $Y_{b}$ e $Y_{c}$ stanno su $\gamma$.

Terzo claim: le rette $P_{a}P_{b}, DE, CH$ concorrono in $F$.
$\angle BHC=180^{\circ}- \angle BAC \Rightarrow H$ sta su $\Gamma'$. Ora basta osservare che $P_{a}P_{b}, DE, CH$ sono gli assi radicali di $\gamma, \Gamma', (P_{a}HP_{b}C)$.
Similarmente si dimostra che $P_{a}P_{c}, DE, BH$ concorrono in $G$.

Ora che abbiamo delle informazioni "decenti" sui punti $F$ e $G$ possiamo fare del segment chasing per trovare i valori di $P_{a}F$ e $P_{a}G$. $\triangle P_{a}P_{b}P_{c}$ è il famoso triangolo ortico e $P_{c}G$ e $P_{b}F$ sono le bisettrici, quindi l'idea sarebbe quella di trovare i tre lati e poi usare il teorema delle bisettrici.
$BC^{2}-P_{b}C^{2}=AB^{2}-(AC-P_{b}C)^{2} \Rightarrow P_{b}C=\frac{84}{5}$.
$\triangle BCA \sim \triangle P_{b}CP_{a} \Rightarrow \frac{P_{a}P_{b}}{P_{b}C}=\frac{AB}{BC} \Rightarrow P_{a}P_{b}=\frac{78}{5}$.
Similarmente si trovano $P_{b}P_{c}=\frac{924}{65}$ e $P_{a}P_{c}=\frac{150}{13}$
$\frac{P_{a}F}{P_{a}P_{c}-P_{a}F}=\frac{P_{b}P_{a}}{P_{b}P_{c}} \Rightarrow P_{a}F=\frac{1950}{323}$
$\frac{P_{a}G}{P_{a}P_{b}-P_{a}G}=\frac{P_{c}P_{a}}{P_{c}P_{b}} \Rightarrow P_{a}G=\frac{650}{93}$
$P_{a}F+P_{a}G=\frac{391300}{30039}$. La risposta è quindi $1339$.
(Denis Tusca)
[/quote]
GranchioPodista
Messaggi: 13
Iscritto il: 17 apr 2022, 13:54

Re: #Proviamoci - Soluzioni commentate OH6

Messaggio da GranchioPodista »

Problema 1 [3]
$n$ il numero di commensali e $d$ la distanza tra loro, i commensali sono i punti ${A_1, A_2,..., A_n}$ mettiamo $A_1$ in un punto qualsiasi del piano. $A_2$ potrà stare ovunque nella circonferenza di raggio $d$ e centro $A_1$. Il terzo punto ha solo $2$ posizioni in cui poter stare, che sono le intersezioni delle $2$ circonferenze di centro $A_1$ e $A_2$, entrambe di raggio $d$. Posizionato il terzo punto e tracciata la terza circonferenza noteremo che non ci sarà nessun punto in comune a tutte e $3$ le circonferenze. Di conseguenza non ci può essere un quarto punto che rispetta le condizioni.
(Alessandro Avellino)

Problema 2 [6]
Si tratta di disporre tre diversi ingredienti, che si può fare semplicemente in $3!=6$ modi diversi. La risposta è quindi $6$.
(Matteo Salicandro)

Problema 3 [228]
Il massimo numero ottenibile lanciando gli $n$ dadi è $n\cdot k$, il minimo è $n\cdot 1=n$. Pertanto $n\cdot k-n=168$, cioè $n(k-1)=168$. Questo vuol dire che $n$ è un qualsiasi divisore positivo di $168$, esclusi $168$ e $84$ visto che $k \geq 4$. $168=2^3\cdot 3\cdot 7$, quindi la somma dei suoi divisori è $(2^4-1)(3+1)(7+1)=480$. La somma dei possibili valori di $n$ è pertanto $480-168-84=228$.
(Federico Borasio)

Problema 4

Problema 5

Problema 6 [36]
Un rettangolo con le diagonali perpendicolari è un quadrato. L'area è quindi $6^2=36$.
(Filippo Prandina)

Problema 7 [18]
La successione è definita, per $n \geq 1$, come
$\begin{cases}
x_{n+2}=x_{n+1}+x_n\\
x_6=76\\
x_5=47
\end{cases}$
Da qui ricaviamo che $x_n=x_{n+2}-x_{n+1}$, pertanto $x_4=76-49=29$ e $x_3=47-29=18$, che è il terzo termine della successione.
(Federico Borasio)

Problema 8 [2022]
Supponiamo che ci siano $2022$ fisici. Tutti dicono la verità, cioè che non sono matematici: non c'è alcuna contraddizione. $2022$ è anche il numero di abitanti dell'isola, quindi è il massimo numero di fisici.

Problema 9 [8]
$\frac{4n}{32}=k$ con $k\in N\geq 0$. Quindi $n=8k$. Il minimo si ha quando $k=1$ e $n=8$.
(Filippo Prandina)

Problema 10[8]
Sono possibili due casi, in base a qual è la circonferenza con raggio minore. Tuttavia, r deve essere massimo, quindi consideriamo il caso in cui r$>$5. L'area della corona circolare sarà data da
$\pi{r}^{2}-{5}^{2}\pi=\pi({r}^{2}-25)=16\pi.$ Per cui ${r}^{2}-25=16$, ovvero $r=\sqrt{41}\approx{6.403}$, la cui parte intera è 6.
(Daniele Prisco)

Problema 11 [9999]
Ci sono più soluzioni in quanto i primi due termini possono essere $1,3$, oppure $2,2$ oppure $3,1$. La risposta è quindi $9999$.
(Matteo Salicandro)

Problema 12[1261]
Affinché una parola sia impronunciabile per il nostro Loenzo, bisogna che contenga almeno una R. Se c'è una sola R nella parola, questa potrà essere la prima, la seconda o la terza lettera, e in ognuno di questi casi ci saranno 20*20=400 parole possibili, dato che nei restanti due 'posti' vanno inserite le altre 20 lettere dell'alfabeto, che possono essere ripetute. Se nella parola ci sono due R, esse si possono disporre in 3 modi, e per ognuno si potranno formare 20 parole. Infine, va incluso l'unico caso in cui la parola è formata da tre R.
In totale, i casi sono $400*3+20*3+1=1261$.
(Daniele Prisco)

Problema 13 [28]
$X$ appartiene alla perpendicolare di $BC$ passante per $A$. Inoltre $X$ e $2$ punti su $BC$ formano lo stesso angolo che forma $A$ con quei $2$ punti, di conseguenza $X$ è il simmetrico di $A$. Da questo deduciamo che $AX=2AD$, dove $AD$ è l'altezza.
(Alessandro Avellino)

Problema 14 [1011]
La quantità in questione, $(3n+2)^{5n+8}$, è un quadrato perfetto se lo è la base, cioè $3n+2$, oppure se l'esponente, $5n+8$, è pari. Tuttavia la prima possibilità non si verifica mai poiché un quadrato non è mai congruo a $2\,(\mathrm{mod}\,3)$. Dunque la risposta è semplicemente il numero di interi positivi $1\leq n\leq 2022$ tali che la quantità $5n+8$ sia pari, ovvero tutti e soli i numeri pari in questo intervallo, che sono $\frac{2022}{2}=1011$.
(Lorenzo Weiss)

Problema 15

Problema 16 [2019]
Il problema chiede per quanti interi positivi minori o uguali a $2022$ accade che $\lfloor \frac{\pi +n}{n}\rfloor =\lfloor \frac{\pi}{n}+1\rfloor =\lfloor \frac{\pi}{n}\rfloor +1=1\Longrightarrow \lfloor \frac{\pi}{n}\rfloor =0$, il che succede solo quando $\pi\approx 3,14\leq n$, i.e. in $2022-(4-1)=2019$ casi.
(Lorenzo Weiss)

Problema 17

Problema 18

Problema 19 [14]
Disegnando la circonferenza $x^2+y^2=5$ risulta evidente che gli unici punti a coordinate intere che vi appartengono, che sono $k=8$, sono quelli della forma $P_i(\pm 1,\pm 2)$, $P_i(\pm 2,\pm 1)$, $P_i(\pm 1,\mp 2)$, $P_i(\pm 2,\mp 1)$. In alternativa, è possibile risolvere l'equazione della stessa circonferenza sugli interi $x,y$. Da ciò si ricava l'ottagono $P_1P_2\dots P_8$, la cui area, come si vede facilmente, è $16-4\cdot\frac{1}{2}=14$.
(Lorenzo Weiss)

Problema 20

Problema 21 [210]
Notiamo che, comunque presi $6$ elementi distinti dall'insieme $X=\{1, 2, 3, 4, 5, 6, 7, 8, 9, 10\}$, c'è uno e un solo modo di disporli in ordine crescente, dunque la richiesta del problema è equivalente al trovare il numero di sottoinsiemi di $6$ elementi di $X$, che sono $\displaystyle\binom{10}{6}=210$.
(Valeria Martinelli)

Problema 22 [2022]
Si può ottenere $2$ solo quando una delle cifre è $2$ e le altre sono nulle, oppure quando due cifre sono uguali a $1$ e le altre sono nulle. Nel primo caso, la prima cifra sarà necessariamente $2$, altrimenti il numero non avrà $2022$ cifre: si ottiene solo una possibilità.
Nel secondo caso, invece, la prima cifra sarà $1$, mentre l'altro $1$ occuperà una delle altre $2021$ posizioni disponibili, ottenendo $2021$ possibilità.
Sommando le possibilità, si arriva a $2022$.
(Daniele Prisco)

Problema 23

Problema 24 [4042]
Se $x$ è la lunghezza del segmento $PA$ e $r$ è il raggio della circonferenza,
$OP=OA+PA=r+x=2021$; $PB=OA+OB+PA=r+r+x$. Di conseguenza, $PA+PB=x+r+r+x=2(r+x)=2×2021=4042$.
(Daniele Prisco)

Problema 25 [66]
Poniamo $a+b=x$ e $c+d=y$.
La disuguaglianza adesso è
$x+y\leq \sqrt{x^2+y^2} \Rightarrow x^2+2xy+y^2\leq x^2+y^2$
Da cui $2xy\leq0$ ma siccome $x$ e $y$ sono somme di interi maggiori o uguali a $0$, l'unico modo per cui $xy=0$ si ha quando uno dei $2$, o entrambi, è uguale a $0$. Se $c+d=y=0$ da cui $c=d=0$, quindi $(a,b,0,0)$. Ricordiamo che $a \geq b$ quindi i possibili valori sono $66$.
(Alessandro Avellino)

Problema 26 [6856]
Sia $n$ il numero di circonferenze sulla prima riga. Avendo le circonferenze diametro $2$, la base del rettangolo è uguale a $2n$. Invece, essendo tutte le circonferenze tangenti, i centri di quella più alta e delle due agli estremi della base sono disposte sui vertici di un triangolo equilatero, di lato $2n-2$. L'altezza del rettangolo è quindi uguale all'altezza del triangolo aumentata di $2$, ossia $\sqrt{3}(n-1)+2$.
Quindi vogliamo $4n+2\sqrt{3}(n-1)+4>2022$, che ci dà immediatamente $n=271$, quindi $k=\frac{271\cdot 272}{2}=36856$.
La risposta voluta sono le ultime quattro cifre di $k$.
(Valeria Martinelli)

Problema 27 [250]
Intanto notiamo che $78125$ è $5^7$.
La somma di $M$ termini consecutivi con primo termine $a$ è del tipo:
$a+(a+1)+(a+2)+...+(a+M-1)$ nel quale compaiono infatti $M$ termini consecutivi.
È quindi possibile raccogliere come $a \cdot M + (1+2+3+...+M-1)=a \cdot M+ \frac{M(M-1)}{2}=M(a+\frac{M-1}{2})=78125 \Rightarrow M(2a-1+M)=5^7\cdot2$, $M<2a-1+M$ quindi al massimo $M=2 \cdot 5^3$.
(Alessandro Avellino)

Problema 28

Problema 29

Problema 30

Problema 31 [20]
Il doppio delle cifre di $n$ dovrà necessariamente essere un numero di $3$ cifre ognuna delle quali minore di $5$. Una volta individuato il numero più piccolo $n$ come $101\,(2n=202)$ e il massimo $n$ come $444\,(2n=888)$ i restanti saranno compresi tra essi. Ci saranno $18$ numeri palindromi tra questi due estremi, con la cifra intermedia contente al più il $4$ e gli estremi uguali da $1$ a $4$. I numeri saranno $101$, $111$, $121$, $131$, $141$, $202$, $222$… fino a $444$. Il totale è di $20$ numeri bipalindromi.
(Irene Mancone)


Problema 32

Problema 33 [74]
Il costo di un pallone ($C$) può essere pari o dispari. Se è pari, può essere pagato con $\frac{C}{2}$ monete da $2$ orue. Poiché $\frac{C}{2}$ è pari, $C$ sarà un multiplo di $4$, quindi potrà assumere $37$ valori.
Se $C$ è dispari, invece, il pallone può essere pagato con $\frac{C-1}{2}$ monete da $2$ orue e una moneta da un orue. Di conseguenza, $C-1$ è un multiplo di $2$, ma $\frac{C-1}{2}$ è dispari (aggiungendo una moneta da un orue, il numero di monete usate è pari). Di conseguenza, $C-1$ è multiplo di $2$ ma non di $4$, quindi può assumere altri $37$ valori.
La somma dei valori possibili è dunque $74$.
(Daniele Prisco)

Problema 34 [2023]
Notiamo che possiamo riscrivere il testo nella seguente maniera: $P((x-y)(x+y))=P(x-y)P(x+y)$.
Notiamo anche che sostituendo $x-y \mapsto z$ e $x+y \mapsto w$ la coppia $(z,w)$ può assumere qualsiasi coppia di valori in $\mathbb{R}^2$ poiché il sistema
$\begin{cases}
x-y=z\\
x+y=w
\end{cases}$
ha soluzioni per ogni coppia $(z,w)$ fissata.
Dunque il nostro problema è equivalente a $P(zw)=P(z)P(y)$ per ogni $z$ e $w$ in $\mathbb{R}$. Questa relazione è molto simile ad una delle famose equazioni funzionali di Cauchy, $f(xy)=f(x)f(y)$. Quest'ultima equazione citata ha soluzione $f(x)=x^n$ o $f(x)=0$ (con $n$ naturale) se la funzione è continua (la continuità è una delle tante condizioni sufficienti), ma un polinomio come $P$ è continuo dunque le possibili soluzioni sono quelle riportate con $n$ che va da $0$ a $2021$ per $P(x)=x^n$ più $P(x)=0$. La soluzione è quindi $2023$ polinomi possibili.
(Lorenzo Bastioni)

Problema 35

Problema 36 [2020]
Si noti che $abc=2022-ab-a$, quindi per massimizzare il prodotto bisogna minimizzare $a$ e $b$. $a,b,c$ sono $\geq 1$ per ipotesi, dunque si pone $a=b=1$ dando come soluzione $c=2020$. Quindi $(a,b,c)=(1,1,2020)$ e $abc=2020$.
(Filippo Prandina)

Problema 37 [201]
Poniamo [math] e notiamo che [math] per [math]. Ora possiamo costruire il seguente polinomio: [math] di cui conosciamo gli zeri [math] e poiché [math] è chiaramente monico possiamo affermare che [math]. Sostituendo infine [math] con [math] e ricavando [math] otteniamo che [math] da cui possiamo trovare la soluzione al problema calcolando [math]. La risposta è quindi [math].
(Lorenzo Bastioni)

Problema 38

Problema 39

Problema 40

Problema 41

Problema 42

Problema 43 [4238]
Se si devono prendere almeno $n$ dolci per essere sicuri di prenderne almeno $1$ di un determinato tipo, questo vuol dire che ci sono $n-1$ dolci non di quel tipo. In particolare:
$\begin{cases}
S+B=55\\
B+C=70\\
S+C=91
\end{cases}$
Il sistema è di immediata risoluzione e porta come soluzione $(S,C,B)=(38,53,17)$ da cui $SCB=34238$. Sono richieste solo le ultime $4$ cifre.
(Filippo Prandina)

Problema 44 [4]
La configurazione è unica, infatti se prendiamo il punto $A_1$ e lo collegassimo con un punto diverso da $B_{2022}$, come per esempio $B_{2021}$, $B_{2022}$ sarà collegato con un punto diverso da $A_1$, per esempio $A_2$. Allora $A_1B_{2021}$ non interseca $A_2B_{2022}$. Iterando questo ragionamento ad ogni punto $A_i$, si ottiene un'unica configurazione: segmenti di estremi $A_iB_{2023-i}$. Quindi i coefficienti angolari saranno $\pm 2/(2023-2i)$. Per il prodotto, il numeratore è sempre $2$ mentre il denominatore è sempre dispari. Trovare quindi ciò che il problema richiede equivale a trovare le ultime $2$ cifre di $2^{2022}$.
(Alessandro Avellino)

Problema 45 [18]
Notiamo che, comunque presa una casella, il numero di giorni che Alberto o Barbara impiegano per raggiungerla non dipende dal percorso scelto, e le uniche caselle che hanno la stessa distanza sia da Alberto che da Barbara sono quelle sulla diagonale del quadrato diversa da quella di partenza, dunque i due si incontrano se e solo se entrambi passano per una di queste tre caselle.
Dato che un percorso è valido se e solo se fa $2$ passi in una direzione e $2$ nell'altra, tutte le coppie di percorsi possibili sono $\frac{4!}{2!\cdot 2!}\cdot \frac{4!}{2!\cdot 2!}=36$, quelli in cui entrambi passano per il centro sono $(2!\cdot 2!)(2!\cdot 2!)=16$, mentre quelli in cui entrambi passano in uno dei due angoli sono $2$.
Le coppie di percorsi con cui Alberto e Barbara non si incontrano sono quindi $36-16-2=18$.
(Valeria Martinelli)

Problema 46

Problema 47 [171]
Poiché $wxy+2021$ deve dare un numero pari, $wxy$ dev'essere dispari, e di conseguenza $w$,$x$ e $y$ devono esserlo.

$w=2a+1, x=2b+1, z=2c+1$ quindi $2a+1+2b+1+2c+1=37 \Rightarrow 2(a+b+c)=34 \Rightarrow a+b+c=17$
Adesso è un semplice stars and bars, infatti $a$, $b$ e $c$ possono essere anche nulli. Distribuire $17$ elementi in $3$ contenitori, $\binom{17+3-1}{3-1}=\binom{19}{2}$.
(Alessandro Avellino)

Problema 48 [42]
Notiamo che $CQB~APB$, in quanto hanno $2$ angoli congruenti, entrambi pari a $74$. Da questo consegue anche che sono triangoli isosceli. $\measuredangle CQB=\measuredangle APB=180-2(74)=32$. Inoltre $S$ sta nell'asse di $BC$, di conseguenza anche nella bisettrice di $CBQ$. Ragionamento analogo per $APB$. Ora osserviamo il quadrilatero $PCAQ$. Questo ha gli angoli $APB$ e $CQB$ congruenti, da questo possiamo asserire che $PCAQ$ è ciclico. Questo permette di stabilire che $\measuredangle DAC=\measuredangle PQC$ e che $\measuredangle DCA=\measuredangle QPD$. Infine osserviamo il triangolo $QSP$. $\measuredangle QSP=180-\measuredangle QPS-\measuredangle PQS= 180-(16+\measuredangle PQC)-(16+\measuredangle QPA)=148-(\measuredangle PQC+\measuredangle APQ)=148-(\measuredangle DCA+\measuredangle DAC)=148-(180-\measuredangle ADC)=\measuredangle ADC-32.$ $\measuredangle ADC=360-(\measuredangle DCB+\measuredangle CBA+\measuredangle BAD)=360-3 \cdot 74=138$ da cui $\measuredangle QSP=138-132=106$. Essendo infine $S$ il circocentro di $ABC$, ne consegue che $\measuredangle ASC=2 \measuredangle ABC=148$. Quindi $\measuredangle ASQ + \measuredangle PSC= \measuredangle ASC- \measuredangle QSP=148-106=42$.
(Alessandro Avellino)

Problema 49 [15]
Dovendo esistere almeno un numero $k>1$ che elevato al cubo sia un divisore di $n$ si possono escludere i numeri da $2$ a $7$ in quanto l’unico loro divisore al cubo è $1$ e $k$ non può assumere tale valore. L’$8$ si può vedere come $2^3$ ed è un prigioniero di cubi, così come lo sono tutti i suoi multipli fino a $96$. È valido lo stesso discorso anche per i multipli di $27$ (essendo $3^3$) fino a $81$. Contandoli si arriva al risultato richiesto, $15$.
(Irene Mancone)

Problema 50

Problema 51

Problema 52[1872]
Chiamiamo le tre radici di p(x) a,b,c dove a<b<c . Essendoa,b e c in progressione aritmetica si ha che a = b-r e c = b+r dove r è la ragione della progressione aritmetica. Per le formule di Vietè a+b+c= 39 b-r + b+ b+ r= 39 da cui si ricava che b =13 . Sempre applicando le formule di Vietè, ab + ac + bc = 482 sostituendo i risultati ricavati in precedenza si trova che 13(13-r) + (13-r)(13+r) + 13(13+r) = 482 da cui sviluppando le parentesi 169 -13r +169 -r2 + 169 +13r = 482 ottenendo che r2 = 25 quindi poiché r è maggiore di 0 si ha che r=5. Quindi a=8 c=18 .
Applicando nuovamente le formule di Vietè, -m = -8*13*18 ovvero m= 1872
(Mattia Zunino)

Problema 53 [24]
Poniamo l'età di Marco al momento della prima affermazione uguale a $x$, quindi l'età del padre Luca è $4x$. Allora, $4$ anni dopo l'età di Marco è $x+4$ e quella del padre è $4x+4$, quindi vale l'equazione $3(x+4)=4x+4$; $x=12-4=8$ e infine $4x=32$.
La differenza di età tra i $2$ è $24$, quindi per rispondere alla domanda risolviamo $2y=y+24$, con $y$ l'età di Marco e $y+24$ l'età di Luca, $y=24$ che è quindi la risposta.
(Niso Cicalò)

Problema 54

Problema 55

Problema 56

Problema 57 [183]
Si può considerare il triangolo isoscele con i vertici di coordinate $B (0;0), C (20;0)$ e A$ (10; y)$ con $y > 0$
Il punto $H$ è l’intersezione tra le rette perpendicolari ai lati passanti per $A, B, C$. Sapendo che le distanze tra
$AH$ e $AD$ sono segmenti di lunghezza intera il punto $H$ si potrà trovare al minimo nella coordinata $(10;1)$. Calcolando il coefficiente angolare tra $H$ e i punti $B$ e $C$, le rette passanti per $B$ e $C$ con coefficiente angolare antireciproco si può ottenere l’ordinata di $A$ sostituendo in una delle due equazioni.
$m_{BH}$= $\frac{1}{10}$ $m^1$=$-10$ retta r passante per A e C: $y=-10x+200$
$m_{CH}$=$\frac{-1}{10}$ $m^1$=10 retta s passante per B e A : $y=10x$

Per $H (10,1)$ $A$ vale $(10, 100)$ e la distanza è intera ($99$).
Una volta stabilita l’ordinata massima di $A$ è sufficiente ripetere il procedimento. Per $H (10,2)$ $A (10, 50)$, per $H (10,4)$ $A(10,25)$. Nelle coordinate $(10, 10)$ $H$ e $A$ si sovrappongono. Poi si avrà che $A$ si trova in $(10,20)$, in $(10,2)$ e $(10,5)$. La soluzione si ottiene sommando le distanze trovate tra $H$ e $A$ senza ripetizioni.
(Irene Mancone)

Soluzione alternativa:
Essendo $ABC$ un triangolo isoscele la mediana $AD$ coincide con l'altezza rispetto a $BC$, chiamando $K$ il piede dell'altezza relativa ad $AC$ otteniamo che il triangolo $BKC$ è simile al triangolo $ABD$ poiché $\widehat{BKC} \cong \widehat{BDA} = 90^{\circ}$ e $\widehat{BCK} \cong \widehat{ABD}$ (essendo $ABC$ isoscele). Consideriamo ora i triangoli $BKC$ e $BDH$, anch'essi sono simili poiché $\widehat{BKC} \cong \widehat{BDH} = 90^{\circ}$ e $\widehat{CBK}$ è in comune. Per la proprietà transitiva $ABD \sim BKC$ e $BKC \sim BDH$ $\rightarrow$ $ABD \sim BDH$. Avendo dimostrato che i triangoli $ABD$ e $BDH$ sono simili vale la seguente uguaglianza: $\frac{BD}{AD}=\frac{HD}{BD} \rightarrow BD^2 = AD \cdot HD \rightarrow 100=AD \cdot HD$ (essendo $BD$ la metà di un segmento lungo $20$). Avendo che $AH$ e $AD$ sono di lunghezza intera anche $HD$ sarà di lunghezza intera (Poiché $HD$ è la differenza tra i due segmenti) ed avremo soluzioni limitate a $100=AD \cdot HD$ ovvero $(AD,HD)=(100,1),(50,2),(25,4),(20,5),(10,10),(5,20),(4,25),(2,50),(1,100)$. La soluzione $(AD,HD)=(10,10)$ non ci piace poiché $\widehat{ABC}$ varrebbe $90^{\circ}$ ($D$ sarebbe il centro della circonferenza circoscritta ad $ABC$ e $\widehat{ABC}$ sottenderebbe un diametro) dunque i possibili valori di $AH$ sono le diverse differenze fra le soluzioni ottenute ($AH=|AD-AH|$): $AH = 100-1= 99$, $AH = 50-2 = 48$, $AH = 25-4= 21$, $AH =20-5=15$ (le altre danno gli stessi valori poiché sono soluzioni "specchiate"). La risposta è quindi $99+48+21+15=183$.
(Lorenzo Bastioni)

Problema 58 [46]
La somma $S$ dei quadrati di delle radici un polinomio del tipo $ x^n + \alpha_{n-1} \cdot x^{n-1} + ... + \alpha_1 \cdot x + \alpha_0$ è

$S=\alpha_{n-1}^2-2 \cdot \alpha_{n-2}$

$$Q_n(x)=x^n+\sum_{i=0}^{n-1} \sqrt{i} \cdot x^i$$

Per cui, definiamo $S_n$ come la somma dei quadrati delle radici di $Q_n(x)$. Allora $S_n=n-1-2\sqrt{n-2}$ che è razionale solo se anche $2\sqrt{n-2}$ lo è. Questo accade solo quando $n-2$ è un quadrato perfetto, quindi ci sono $45$ valori. A questo aggiungiamo $n=1$, il cui polinomio è quindi $x$, che ha come radici un numero razionale. Quindi la soluzione è $46$.
(Alessandro Avellino)

Problema 59

Problema 60

Problema 61[9098]
Per il criterio di divisibilità per 9, s(n) ≡ n mod 9 dove s(n) è la somma delle cifre di n. Iterando il ragionamento su s(n) si ottiene che A(n)≡n mod9 . Poiché A(n) è un intero compreso fra 0 e 9, A(n) è uguale al resto della divisione di n per 9. I possibili resti della divisione delle potenze di 2021 per 9 sono 5, 7, 8, 4, 2, 1 e poi si ripetono ciclicamente con periodo 6. Perciò la somma richiesta è 2016/6 *(5+7+8+4+2+1) +5+7+8+4+2 = 9098.
(Mattia Zunino)
Problema 62 [1158]
Dette $H$ e $K$, rispettivamente, le proiezioni di $D$ e $C$ su $AB$ e $AH=x$ possiamo calcolare l'altezza del trapezio in due modi diversi servendoci del teorema di Pitagora ed eguagliare le espressioni $DH^2=AD^2-AH^2=33^2-x^2=CK^2=BC^2-BK^2=60^2-(69-6-x)^2$.

Risolvendo l'equazione, si trova l'unico valore $x=\frac{81}{7}$ e l'altezza si può quindi calcolare, ad esempio, come $h=DH=\sqrt{AD^2-AH^2}=\frac{60\sqrt{13}}{7}$. L'area del trapezio vale $A=\frac{(b_1+b_2)\cdot h}{2}=\frac{(AB+CD)\cdot AH}{2}=\frac{(69+6)\cdot (60\sqrt13)}{7\cdot2}\approx 1158,93$; la sua parte intera è $1158$ ed è la soluzione.
(Federico Magnolfi)

Problema 63

Problema 64

Problema 65 [5877]
$X$ si trova sulla superficie del triangolo $AMC$: infatti, se così non fosse, $\widehat{ABC}$ e $\widehat{AXM}$ sarebbero due angoli congruenti che insistono su uno stesso segmento ($AM$) e che appartengono allo stesso semipiano rispetto ad esso: ma ciò non è possibile in quanto significherebbe che $X$ appartiene alla circonferenza circoscritta al trangolo $ABM$, alla quale ovviamente non appartengono punti interni al triangolo $ABM$.

$AXMY$ è ciclico per ipotesi e $\widehat{AYM}$ e $\widehat{AXM}$ sono supplementari in quanto angoli opposti (per quanto prima dimostrato); dunque $\widehat{BYM}$, essendo anch'esso supplementare di $\widehat{AYM}$, è congruente a $\widehat{AXM}$, il quale a sua volta è congruente a $\widehat{ABC}$ per ipotesi.
Avremo allora che il triangolo $MBY$ è isoscele su base $BY$; essendo $M$ punto medio di $BC$, $MB\cong MY\cong MC$ e ciò implica che $BYC$ è rettangolo in $Y$. $CY$ è dunque l'altezza uscente da $C$ e si può calcolare con la formula inversa $CY=\frac{2A}{AB}$ dopo aver ricavato l'area con la formula di Erone o per via trigonometrica.
Si trova che quest'ultima vale $A=1200\sqrt{5}$ e dunque $CY^2=\left(\frac{2A}{AB}\right)^2=\frac{4\cdot1200^2\cdot 5}{70^2}\approx 5877,55$. La sua parte intera è $5877$ ed è la soluzione.
(Federico Magnolfi)

Problema 66 [1155]
Prendiamo l'equazione del problema e consideriamola $(\text{mod}\,7)$: $x^2 \equiv 7y + z \,(\text{mod}\,7) \Rightarrow x^2 \equiv z \, (\text{mod}\,7)$. I quadrati come $x^2$ modulo $7$ possono assumere come valori solo $0,1,2,4$ quindi $z$ potrà assumere tutti e i soli valori $0,1,2,4$ modulo $7$. Nell'intervallo $[1,2022]$ ci sono esattamente $288$ numeri congrui a $0 \, (mod 7)$, $289$ congrui a $1$, $289$ congrui a $2$ e $289$ congrui a $4$. La risposta è quindi $288+289 \cdot 3=1155$.
(Lorenzo Bastioni)

Problema 67 [3026]
Scriviamo il numeratore come $\prod_{n=2}^{2021}((n-1)+n+(n+1))=\prod_{n=2}^{2021}(3n)=3^{2021}\prod_{n=2}^{2021}(n)$. Si può quindi raccogliere un $3^{2021}$ al numeratore. Inoltre si raccolgono anche una potenza di $3^{1005}$ dato che $n$ può avere $3$ come fattore primo (elevato alla $1,\,2,\,3,\,\dots,6$). Ci sono $ \Big \lfloor \frac{2021}{3} \Big \rfloor=673$ multipli di 3, $ \Big \lfloor \frac{2021}{9} \Big \rfloor =224$ multipli di $9$ e così via: $ \Big \lfloor \frac{2021}{3} \Big \rfloor+ \Big \lfloor \frac{2021}{9} \Big \rfloor + \Big \lfloor \frac{2021}{27} \Big \rfloor + \Big \lfloor \frac{2021}{81} \Big \rfloor + \Big \lfloor \frac{2021}{243} \Big \rfloor + \Big \lfloor \frac{2021}{729} \Big \rfloor =673+224+74+24+8+2=1005$. In tutto quindi si è raccolto $2021+1005=3026$.
(Filippo Prandina)

Problema 68 [1104]
Chiamiamo due insiemi $A$ e $B$ "gemelli di primo grado" se $|A \cap B|=1$ e $1 \in A,B$, in generale chiamiamo due insiemi $A$ e $B$ "gemelli di $n$-esimo grado" se $|A \cap B|=n$ e $n \in A,B$.
Possiamo ora affrontare il conteggio di tutte le coppie di sottoinsiemi gemelle di $\{1,2,3,...,2021,2022 \}$ considerando prima i gemelli di primo grado, di secondo grado fino a quelli di $2022$-esimo grado (Non ci sono coppie di sottoinsiemi di $2023$-esimo grado o di $0$-esimo grado).
Gemelli di primo grado:
se $A$ e $B$ sono di primo grado allora contengono entrambi l'elemento $1$ e non hanno nessun altro elemento in comune. Ci verra comodo per dopo scrivere che gli altri zero elementi in comune si possono scegliere in $\displaystyle \binom{2021}{0}$ modi. Ora tutti gli altri $2021$ elementi possono essere messi o in $A$ o in $B$ o in nessuno dei due (in questo caso la "o" è una o esclusiva), quindi abbiamo 3 possibilità per ognuno dei $2021$ elementi ovvero $3^{2021}$ possibilità. Avremo quindi $\displaystyle \binom{2021}{0} 3^{2021}$ coppie di sottoinsiemi gemelle di primo grado.
In generale siamo pronti ad affrontare il caso di gemelli di $n$-esimo grado:
se $A$ e $B$ sono gemelli di $n$-esimo grado allora contengono entrambi l'elemento $n$ e hanno altri $n-1$ elementi in comune. Gli altri $n-1$ elementi in comune si possono scegliere in $\displaystyle \binom{2021}{n-1}$ modi. E per i restanti $2021-(n-1)=2022-n$ elementi abbiamo $3^{2022-n}$ possibilità come spiegato precedentemente. Avremo quindi $\displaystyle \binom{2021}{n-1} 3^{2022-n}$ coppie di sottoinsiemi gemelle di $n$-esimo grado.
Sommando tutte le possibilità otteniamo:
$$\sum_{k=1}^{2022} \binom{2021}{k-1} 3^{2022-k}$$
Reimpostando la somma sostituendo $k-1 \mapsto k$ otteniamo:
$$\sum_{k=0}^{2021} \binom{2021}{k} 3^{2021-k}$$
Ma questo è esattamente uguale a $(3+1)^{2021}=4^{2021}$ (Per il binomio di Newton o teorema binomiale). Riducendo l'enorme numero $(\text{mod}\,10000)$ si trova che le ultime $4$ cifre che ci interessano sono $1104$, da cui la risposta.
(Lorenzo Bastioni)

Problema 69

Problema 70

Problema 71

Problema 72[247]
Disponiamo le 32 squadre in qualunque ordine, e costruiamo gli incontri nel seguente modo: ad ogni turno, le squadre in un posto dispari si scontrano con la successiva. Le squadre perdenti vengono eliminate. Disponiamo le squadre restanti nello stesso ordine in cui si trovano, e ripetiamo finché resta solo una squadra. Ad esempio, se ad un certo punto l'ordine è $(3, 5, 2, 1)$, la squadra $3$ vince contro $5$ e $1$ vince contro $2$, dunque al turno dopo l'ordine sarà $(3, 1)$. Poniamo per semplicità la squadra $S_3$ per prima, e consideriamo i restanti $31!$ ordinamenti possibili. La squadra $S_3$ arriva in semifinale se e solo se non viene eliminata nei 3 turni precedenti, e ciò accade se e solo se nessuna tra le squadre $S_1$ ed $S_2$ è presente tra le prime 8 squadre. Possiamo quindi scegliere le prime 8, inclusa $S_3$, in $\displaystyle\binom{29}{7}$ modi, e le restanti in $24!$ modi.
La probabilità cercata è quindi $\displaystyle\binom{29}{7}\cdot 24!\cdot \frac{1}{31!}=\frac{29 \cdot 28 \cdot 27 \cdot 26 \cdot 25 \cdot 24 \cdot 23}{31 \cdot 30 \cdot 29 \cdot 28 \cdot 27}=\frac{24 \cdot 23}{31 \cdot 30}=\frac{92}{155}$. La risposta è quindi $92+155=247$.
(Valeria Martinelli)

Problema 73 [6068]
Notiamo che
$(p+q+r)(pq+qr+pr)=p^2q+p^2r+pq^2+pr^2+q^2r+qr^2+3pqr=1+3\cdot 2022=6067$.
Dato che $p+q+r$ e $pq+qr+pr$ sono entrambi interi positivi, devono necessariamente dividere entrambi $6067$. $6067$ è un numero primo, quindi o $p+q+r=1$ o $p+q+r=6067$. La risposta è perciò $6067+1=6068$.
Si noti che non ci si deve preoccupare di trovare che esistano $p,q,r$ che rispettano le condizioni trovate, dato che questo è assicurato dal fatto che sono le tre soluzioni dell'equazione $x^3-(p+q+r)x^2+(pq+qr+pr)x-pqr=0$, che ha sempre tre soluzioni nei numeri complessi.
(Federico Borasio)

Problema 74

Problema 75 [78]
Elevando entrambi i termini al quadrato (lecito perché entrambi sicuramente positivi) si ottiene $x^2+y^2-x-y+1/2 \leq 1/4$ riarrangiando $(x-1/2)^2+(y-1/2)^2 \leq 1/4$. Si ha quindi l'equazione di un cerchio centrato in $(1/2,1/2)$ con raggio $1/2$. La probabilità che un punto interno al quadrato definito nel piano cartesiano che ha vertice nell'origine e lato $1$ sia dentro il cerchio è quindi il rapporto tra l'area del cerchio e quella del quadrato: $\pi (1/2)^2 \approx 78%$.
(Filippo Prandina)

Problema 76 [4253]
Iniziamo col dimostrare che $P$ si trova all'esterno di $ABC$.

Detto $K$ il piede dell'altezza uscente da $A$ si ha che, essendo $\widehat{APB}$ congruente a $\widehat{AKB}$ perché entrambi retti, e dunque $AKBP$ ciclico, $P$ appartiene alla circonferenza circoscritta a $AKB$ alla quale non appartengono punti interni a $AKB$; ma $P$ appartiene alla mediana $AM$, la quale è evidentemente contenuta nel triangolo $AKB$ (lo si potrebbe dimostrare con i dati del problema), e quindi $P$ si trova all'esterno di $AKB$ ma anche di $ABC$.

Ora, è noto che in un triangolo di lati di lunghezza $9k$, $8k$, $7k$, con $k$ intero positivo, la mediana relativa alla base di lunghezza $8k$ misura $7k$: questo è facilmente verificabile per via trigonometrica, col teorema della mediana o anche con Pitagora; in ogni caso, $AM\cong AC$ e in particolare $\widehat{AMC}\cong \widehat{ACM}=\gamma$.

Sia $\left\{D\right\}=BC\cap PT$. $BTD\sim BCH$ per il secondo criterio di similitudine in quanto hanno l'angolo in $B$ in comune e $\widehat{BDT}\cong \widehat{BHC}$ perché retti, in particolare $\widehat{ATD}=\gamma$.

$\widehat{BMP}\cong\widehat{AMC}$ perché opposi al vertice ed essendo $BPM$ rettangolo in $P$ e $PD$ l'altezza relativa all'ipotenusa, $BPD=\gamma$. Allora $BPT$ è isoscele sulla base $PT$, in altre parole $T$ è il simmetrico di $P$ rispetto a $BC$; da ciò deriva anche che $BMT\cong BMP$, in particolare che $\widehat{BTM}$ sia retto.

$BMT\sim BCH$ per il secondo criterio di similitudine in quanto hanno l'angolo in $B$ in comune e $\widehat{BTM}\cong \widehat{BHC}$ perché entrambi retti. Ma allora, essendo $M$ punto medio di $BC$, necessariamente $T$ è punto medio di $BH$.

Il rapporto $\frac{[ATB]}{[THC]}$ è uguale al rapporto fra le altezze $\frac{AH}{CH}$ in quanto i due triangoli hanno basi congruenti per quanto appena dimostrato. Esse si possono calcolare con pitagora, dopo aver ricavato $BH=\frac{2A}{AC}$ e l'area attraverso Erone o per via trigonometrica. Sviluppando i calcoli si trova $BH=\frac{24\sqrt{5}}{2}$, $CH=\sqrt{BC^2-BH^2}=\frac{16}{7}$ e $AH=AC-CH=\frac{33}{7}$
Il risultato cercato è $1000\cdot \left(\frac{AH}{CH}\right)^2=1000\cdot\left(\frac{33}{16}\right)^2\approx4253,90$ la cui parte intera è $4253$ che è la soluzione.
(Federico Magnolfi)

Problema 77

Problema 78 [4151]
Cominciamo col dimostrare che $BCPQ$ è un trapezio. Sappiamo che l'ortocentro e il circocentro sono coniugati isogonali: in pratica, le rette $AH$ e $AO$ formano coi lati $AB$ e $AC$ angoli congruenti. Avremo dunque $\widehat{BAH}\cong \widehat{CAO}$, ma anche $\widehat{HAI}\cong \widehat{OAI}$ essendo $AI$ bisettrice perché $I$ incentro di $ABC$.

Riferendoci ora alla circonferenza circoscritta ad $ABC$, che chiamiamo $\Gamma$, abbiamo che $\widehat{FCE}\cong \widehat{FAE}\cong\widehat{EAD}\cong \widehat{ECD}$ dove la prima e la terza uguaglianza sono date dal fatto che gli angoli insistono su uno stesso arco. Facendo attenzione, notiamo che $\widehat{QAP}$ e $\widehat{QCP}$ sono congruenti per l'uguaglianza precedente e insistono sullo stesso segmento $QP$, dunque $AQPC$ è ciclico e chiamiamo $\omega$ la sua circonferenza circoscritta. Essendo $\widehat{ACD}$ retto poiché insiste sul diametro $AD$ di $\Gamma$, $\widehat{AQP}$ sarà anch'esso retto poiché supplementare dell'angolo opposto $\widehat{ACP}$ (ricordando che ora abbiamo fatto rifereimento a $\omega$). $BC$ e $QP$ sono allora due rette perpendicolari alla stessa retta $AQ$, perciò sono parallele e $BCPQ$ è un trapezio.

Per calcolare la sua area abbiamo bisogno dell'altezza $QH$ e della base minore $QP$, dato che la base maggiore ci è nota ed è $BC$.

$\textbf{Prima soluzione (sintetica)}$
Detti $K$ e $J$ rispettivamente i piedi dell'altezza $AH$ e della bisettrice $AI$, analizziamo i triangoli $ABJ$ e $AEC$: essi hanno $\widehat{ABJ}\cong \widehat{AEC}$ perché angoli alla circonferenza che insistono su uno stesso arco; $\widehat{BAJ}\cong \widehat{EAC}$ perché $AI$ bisettrice: quindi sono simili per il secondo criterio di similitudine. Considerando ora i triangoli $AKJ$ e $AQP$: essi hanno: l'angolo in $A$ in comune; $\widehat{AKJ}\cong \widehat{AQP}$ perché retti: quindi sono anch'essi simili per il secondo criterio di similitudine. Impostiamo la proporzione $\frac{QP}{KJ}=\frac{AP}{AJ}$ $(1)$: troveremo $KJ$ e $AJ$ lavorando su $ABC$, mentre $AP$ attraverso il rapporto di similitudine fra i triangoli $ABJ$ e $AEC$. Calcoliamo $AK$ invertendo la formula dell'area di $ABC$ e $BK$ col teorema di Pitagora: $AK=\frac{2A}{BC}=\frac{2\cdot \sqrt{p(p-AB)(p-BC)(p-AC)}}{BC}=120 \quad$ $BK=\sqrt{AB^2-AK^2}=50$.
Applichiamo ora il teorema della bisettrice per trovare $BJ$: $\frac{BJ}{AB}=\frac{BC-BJ}{AC}$ da cui si ricava che $BJ=65$. Dunque $KJ=BJ-BK=15$ e $AJ=\sqrt{AK^2+KJ^2}=15\sqrt{65}$.

Impostiamo il rapporto di similitudine fra $ABJ$ e $AEC$: $\frac{AP}{AB}=\frac{AJ}{AC}$ da cui si ricava che $AP=\frac{150\sqrt{65}}{7}$ e per la proporzione $(1)$ $QP=\frac{AP\cdot KJ}{AJ}=\frac{150}{7}$. A questo punto si trova $AQ$ con pitagora e $QK$ per differenza: $AQ=\sqrt{AP^2-QP^2}=\frac{1200}{7} \quad QK=AQ-AK=\frac{360}{7}$
L'area del trapezio $BCPQ$ è dunque $$A=\frac{(140+\frac{150}{7})\cdot\frac{360}{7}}{2}\approx 4151,02$$.

$\textbf{Seconda soluzione (trigonometria)}$
Ponendo $\widehat{ABC}=\beta$, $\cos\beta=\frac{AB^2+BC^2-AC^2}{2AB\cdot BC}=\frac{5}{13}$ (teorema del coseno), $\sin\beta=\sqrt{1-\cos^2\beta}=\frac{12}{13}$ $\widehat{BAC}=\alpha$, $\widehat{CAI}=\frac{\alpha}{2}$ perché $AI$ bisettrice, $\cos\alpha=\frac{AB^2+AC^2-BC^2}{2AB\cdot AC}=\frac{33}{65}$ (teorema del coseno), $\cos\frac{\alpha}{2}=\sqrt{\frac{1+\cos\alpha}{2}}=\frac{7}{\sqrt{65}}$.

Adesso abbiamo abbastanza dati per calcolare $AP=\frac{AC}{\cos\frac{\alpha}{2}}=\frac{150\sqrt{65}}{7}$.
Ponendo $\widehat{HAI}=\widehat{OAI}=\theta$ si ha che, essendo $\widehat{BAH}=\frac{\pi}{2}-\beta$, $\theta=\frac{\alpha}{2}+\beta-\frac{\pi}{2}$; $\cos\theta=\sin\frac{\alpha}{2}\cos\beta+\cos\frac{\alpha}{2}\sin\beta=\frac{8}{\sqrt{65}}$

$AQ=AP\cos\theta=\frac{1200}{7}\quad QP=\sqrt{AP^2-AQ^2}=\frac{150}{7}$. Ci manca l'altezza $QK$ dove $K$ è il piede dell'altezza uscente da $A$. $AK=AB\sin\beta=120\qquad QK=AQ-AK=\frac{360}{7}$. A questo punto l'area si trova allo stesso modo della soluzione per via sintetica.
(Federico Magnolfi)

Problema 79 [2011]
Si ha che $p^{(2)}(x)$ ha lo stesso grado di $p^{(4)}(x)$ questo significa che $p(x)$ è di grado $1$ o grado $0$. Ma non può essere di grado $0$ perché non è costante. Allora è del tipo $p(x)=mx+q$. La funzione $p(x)$ è chiaramente iniettiva ed essendo $p^{(2)}(x)=p^{(2022)}(x)$ per ipotesi, si ha necessariamente che $p(x)=p^{(2021)}(x)$. Ma quindi $p(2021)=0$ e $p(p(2021))=p(0)=2021$. Da qui si ricava facilmente che il polinomio è $p(x)=-x+2021$ e quindi $p(10)=2011$.
(Filippo Prandina)

Problema 80


Problema 81

Problema 82 [4]
$45=9\cdot 5$ quindi scomponiamo il modulo in modulo $9$ e modulo $5$. Modulo $5$ è semplicemente $4$, in quanto l'ultima cifra è proprio $4$. Definiamo $\overline{a_1a_2a_3...a_k}$ la scrittura decimale in successione di $a_1,a_2,...,a_k$, per esempio $\overline{4^2 5^2}=1625$. Inoltre, chiamato $N$ il nostro numero, $N=\overline{1^22^23^24^2...2022^2}$. $\overline{a_1a_2a_3...a_k}\equiv a_1+a_2+a_3+...+a_k\,(\mathrm{mod}\,9)$, dimostrarlo è facile. I residui quadratici di $9$ sono $0,1,4,7$ mentre la somma dei quadrati da $1$ a $9\,\mathrm{mod}\,9$ è $24$, ($-3$). Quindi ogni $9$ numeri vi è un $-3$ da addizionare, quindi è $(-3)\cdot 224$ (che sarebbe il numero di multipli di $9$ minori di $2022$). $-672$, a questo devi aggiungere i numeri da $2017$ a $2022$, $-672+1+4+7+7=-653\equiv -14\equiv 4$.
Siccome $N$ è congruo a $4$ sia $\mathrm{mod}\,5$ che $\mathrm{mod}\,9$, sarà congruo a $4$ anche $\mathrm{mod}\,45$.
(Alessandro Avellino)

Problema 83 [21]
Sia $H$ il piede dell'altezza uscente da $A$; $AY=AH-HY$. L'altezza $AH$ si può calcolare, ad esempio, invertendo la formula dell'area $A=\frac{b\cdot h}{2}$ dopo aver trovato quest'ultima con la formula di Erone, oppure trovando il coseno dell'angolo in $B$ col teorema del coseno, quindi il suo seno e applicando il primo teorema dei triangoli rettangoli a $ABH$. Eseguendo i calcoli, si trova che essa vale $AH=3\sqrt{5}$. $BH=\sqrt{AB^2-AH^2}=2$.

Detto $M$ il punto medio di$BC$ si ha che $HM=HX=2$ e quindi esso è punto medio anche di $HX$; ma $OM\perp BC$ in quanto in un triangolo isoscele la mediana è anche altezza: allora, essendo $HYX$ e $MOX$ simili per il secondo criterio di similitudine, e avendo rapporto di similitudine $2$, $HY=2HO$

Attraverso il teorema della corda, oppure attraverso la formula nota, possiamo calcolare il raggio della circonferenza circoscritta ad $ABC$, $R=\frac{BC}{2\sin\alpha}=\frac{abc}{4A}=\frac{21}{2\sqrt{5}}$.
Ora possiamo trovare $HY=2HO=2\sqrt{R^2-MC^2}=\frac{11\sqrt{5}}{5}$ e infine $AY=AH-HY=3\sqrt{5}-\frac{11\sqrt{5}}{5}=\frac{4\sqrt{5}}{5}$.

Stiamo cercando $AY^2=\frac{16}{5}$ e la risposta è infine $16+5=21$.
(Alessandro Lombardo & Federico Magnolfi)

Problema 84

Problema 85 [1136]

Iniziamo col calcolarci alcuni oggetti che in futuro potranno esserci utili.

Perimetro: 2p=14+16+18=48

l'Area: usando la formula di Erone, A=$\sqrt{p(p-a)(p-b)(p-c)}=\sqrt{24\cdot10\cdot8\cdot6}=48\sqrt{5}$

Raggio circonferenza circoscritta: $R=\frac{abc}{4A}=\frac{4032}{192\sqrt{5}}=\frac{21}{\sqrt{5}}=\frac{21\sqrt{5}}{5}$

Altezza relativa a BC: $AD=2A/BC=6\sqrt{5}$

Altezza relativa a AB, che incontra AB in F: $CF=2A/AB=\frac{48\sqrt{5}}{7}$

Detto L il punto di incontro tra BC e la perpendicolare passante per O, L è il punto medio di BC. Per cui CL=8, OC= R, $OL=\sqrt{\frac{441}{5}-\frac{320}{5}}=\sqrt{\frac{121}{5}}=\frac{11\sqrt{5}}{5}$

$CD=\sqrt{AC^2-AD^2}=\sqrt{144}=12$

$AF=\sqrt{AC^2-CF^2}=\frac{66}{7}$

LD=CD-CL=12-8=4

adesso calcoliamo OD:

$OD=\sqrt{OL^2+LD^2}=\sqrt{\frac{201}{5}}$

Infine calcoliamo AH, dove H è l'ortocentro. per calcolare AH notiamo che, AHF è simile a ABD, per cui $\frac{AH}{AB}=\frac{AF}{AD} \Rightarrow AH=\frac{AF \cdot AB}{AD}=\frac{66 \cdot 14}{42\sqrt{5}}=\frac{22\sqrt{5}}{5}$.

Adesso che abbiamo tutto ciò che ci serve proseguiamo. O è il punto di Miquel relativo ad XYD. questo vuol dire che O appartiene alla circonferenza passante per X,Y,A. Detto questo, abbiamo 2 quadrilateri ciclici, per definizione, BDOX e CDOY. A questi 2 aggiungiamo il quadrilatero AYOX, anch'esso ciclico.

1° Claim: XYD è simile ad ABC.
è sufficiente un po' di angle chasing:

ricordiamo che OA, OB e OC sono i raggi di (ABC).
Di conseguenza OBA=OAB, OBC=OCB e OCA=OAC. Inoltre, per via della ciclicità dei 3 quadrilateri sopra elencati,
OAB=OBA=OBX=ODX, OAC=OCA=OCY=ODY, da questo segue che YDX=ODY+ODX=CAO+OAB=BAC.

Analogamente DYX=CBA e DXY=BCA. Per cui i 2 triangoli sono simili

2° Claim: O è l'ortocentro di XYD.
Anche qui è sufficiente un po' di Angle chasing:

DYX+YDO=(DYO+OYX)+OCY=DCO+OAX+OCA=(DCO+OCA)+OAX=ACB+OAX=ACB+(OAH+HAX)=ACB+(OAH+HAB)=
ACB+(OAH+DAB)=ACB+(OAH+OAC)=ACB+DAC=90.
Ne consegue che OD è perpendicolare ad XY. Analogamente si ottiene che , XO e YO sono perpendicolari, rispettivamente, a DY e DX. Per cui O è l'ortocentro.

Di conseguenza, detto K il rapporto di similitudine tra ABC e XYD, $K=\frac{AH}{OD}=\frac{22}{201}$, il rapporto vale anche per i raggi r e R rispettivamente di XYD, e di ABC, $K=\frac{R}{r} \Rightarrow r=\frac{R}{K}=\frac{21\sqrt{1005}}{110}$
(Alessandro Avellino)

Problema 86 [1483]
Per prima cosa notiamo che, dato che $\frac{x_1+x_2}{x_3+x_4}$ è intero, abbiamo che $x_1+x_2 \ge x_3+x_4$, e similmente $x_3+x_4 \ge x_5+x_6 \ge x_1+x_2$.
Dunque $x_1+x_2= x_3+x_4=x_5+x_6$. Analogamente $x_2+x_3=x_4+x_5=x_6+x_1$.

Sia ora $S=x_1+x_2= x_3+x_4=x_5+x_6$ e $T=x_2+x_3= x_4+x_5=x_6+x_1$.
Abbiamo quindi che $\sum x_i = 3S=3T$, quindi $S=T$, ossia $x_1+x_2=x_2+x_3$, da cui $x_1=x_3$. In modo analogo otteniamo $x_1=x_3=x_5$ e $x_2=x_4=x_6$.

Possiamo quindi scrivere la sestupla come $(a, b, a, b, a, b)$, con $a, b$ interi positivi. Si verifica facilmente che ogni sestupla siffatta rispetta le condizioni del testo, difatti la somma di due termini consecutivi è costante, dunque le frazioni sono tutte uguali a $1$ e quindi intere.
Dunque, fissati $a, b$, c'è una e una sola sestupla valida.

Consideriamo ora due casi:

$\bullet$ $M$ non è multiplo di $3$. Allora, dato che $M=3S$, non ci sono sestuple valide, e quindi $f(M)=0$, che è un multiplo di $5$. In questo caso abbiamo quindi $2022-\frac{2022}{3}=1348$ possibili valori di $M$.

$\bullet$ $M=3k$, per qualche $k$ intero positivo. Dato che $k=a+b$, per ogni valore di $k$, ci sono $k-1$ sestuple valide, poiché abbiamo $k-1$ possibili valori di $a$, dunque $f(M)=k-1$. Vogliamo quindi che $k-1$ sia multiplo di $5$. Se $k=1$, non ci sono sestuple valide in quanto $a+b>1$, altrimenti, potendo scegliere $k$ nell'intervallo $[2, 1348]$, ci sono ${\lfloor \frac{1347}{5} \rfloor}=134$ valori accettabili.

In totale abbiamo quindi $1348+1+134=1483$ possibili valori di $M$.
(Valeria Martinelli)

Problema 87

Problema 88 [6418]
Osserviamo che, prese $3$ cifre distinte $a, b, c$ dall'insieme $X=\{1, 2, 3, 4, 5, 6, 7, 8, ,9 \}$, ci sono esattamente $6$ numeri compresi tra $100$ e $999$ che si scrivono con tutte e sole le cifre $a, b, c$, siano questi $x_1, ..., x_6$. Abbiamo che la loro somma $x_1+...+x_6$ vale $222a+222b+222c$, quindi $\sum f(x_i)=\frac{222(a+b+c)}{a+b+c}=222$.
Ci sono $\displaystyle\binom{9}{3}=84$ modi di scegliere $a, b, c$, quindi la somma in questo caso vale $222\cdot 84=18648$

Dobbiamo ora distinguere i casi in cui $a, b, c$ contengono uno o più zeri o cifre ripetute.
$\bullet$ $a=b=c$: per ognuna delle $9$ possibili scelte di $a, b, c$, c'è uno e un solo numero $x=111a$, e $f(x)=\frac{111a}{3a}=37$. Dunque la somma in questo caso vale $37 \cdot 9=333$

$\bullet$ $a=b \neq c$, $a, b, c \neq 0$: abbiamo $9\cdot 8=72$ possibili scelte di $a, b, c$, e per ogni terna abbiamo $3$ numeri la cui somma vale $222a+111c$, per cui $\sum f(x_i)=\frac{222a+111c}{2a+c}=111$. In tutto $72 \cdot 111=7992$

$\bullet$ $b=c=0$: $9$ possibili scelte di $a$, e per ogni $x=100a$ che rispetta queste condizioni $f(x)=100$. La somma in questo caso vale $100 \cdot 9=900$

$\bullet$ $a=b$, $c=0$; $9$ possibili scelte per $a, b$, per ognuna $\sum f(x_i)=\frac{211a}{2a}=\frac{211}{2}$. Quindi in tutto $9\cdot \frac{211}{2}=949,5$

$\bullet$ $a\neq b$, $c=0$: $\displaystyle\binom{9}{2}=36$ possibilità per $a, b$; per ognuna $\sum f(x_i)=\frac{211a+211b}{a+b}=211$. In tutto $36 \cdot 211=7596$

La somma richiesta è quindi uguale a $18648+333+7992+900+949,5+7596=36418,5$. Ci interessano le ultime quattro cifre della parte intera.
(Valeria Martinelli)

Problema 89 [134]
Sia $2^k$ la massima potenza di $2$ nell'intervallo $\{1, ..., n\}$. Si osserva facilmente che $f(n)$ non può mai essere maggiore di $f(2^k)$, pertanto $f(n)$ è massima quando $n=2^k$, e $g(2^k)=1$.

Dato che $5>4=2^2$ e $9=3^2>2^3=8$, se un intero contiene nella sua fattorizzazione primi diversi da $2$ e $3$, o se $3$ compare con un esponente maggiore o uguale a $2$, $g(n)=g(n-1)$ in quanto $n$ ha sicuramente meno fattori di $2^k$.
Inoltre, se $n=2^{k-1}\cdot 3$, $g(n)=g(n-1)+1$, in quanto n contiene lo stesso numero di fattori di $2^k$.

Dunque, se $2^{k-1}\cdot 3\le n <2^{k}$, $g(n)=2$, altrimenti $g(n)=1$.

Possiamo quindi trovare tutti gli interi minori o uguali a $99$ per cui $g(n)=2$, che sono i valori compresi tra i tripli delle potenze di $2$ e la potenza di $2$ successiva.

Dunque la somma richiesta è uguale a $99+(4-3)+(8-6)+(16-12)+(32-24)+(64-48)+(100-96)=99+1+2+4+8+16+4=99+31+4=134$.
(Valeria Martinelli)

Problema 90 [8784]
Chiamiamo $f(n)=x^n+\frac{1}{x^n}$, il problema ci chiede di calcolare $|f(1)\cdot f(2)\cdot\dots\cdot f(2022)|$.
Esplorando un po’ la situazione si può notare che $f(1)=1,f(2)=-1,f(3)=-2,f(4)=-1,f(5)=1$ e $f(6)=2$ e che questi valori potrebbero formare un ciclo che si ripete ogni $6$.
Dimostriamolo:
consideriamo $f(n) \cdot f(1)= \left( x^n+\frac{1}{x^n} \right) \cdot \left(x+ \frac{1}{x} \right) = x^{n+1} + x^{n-1} + \frac{1}{x^{n-1}} + \frac{1}{x^{n+1}} = f(n+1) + f(n-1)$, essendo $f(1)=1$ otteniamo che $f(n) \cdot f(1) =f(n)= f(n+1) + f(n-1)$ da cui $f(n+1)=f(n)-f(n-1)$. L’ultima relazione ci indica che $f(n+1)$ dipende unicamente da $f(n)$ e $f(n-1)$ ovvero se troviamo un ciclo quest’ultimo si ripeterà. Controllando si può vedere che $f(7)=1$ e $f(8)=-1$ quindi il ciclo si ripeterà. Analizzando il nostro ciclo si nota facilmente che gli unici fattori che contribuiranno nel nostro prodotto saranno quelli con $n$ multiplo di $3$, che in valore assoluto valgono $2$. Il nostro prodotto sarà quindi del tipo $2^k$ dove $k$ è il numero di multipli di $3$ tra $1$ e $2022$ ovvero $674$ $(\frac{2022}{3}=674)$. Il problema si è trasformato in un quesito di teoria dei numeri poiché dobbiamo dare come risposta le ultime $4$ cifre di $2^{674}$ quindi calcolare $2^{674}$ $(\mathrm{mod}\,10000)$. Riducendo con un po’ di conti l’enorme numero si trova $8784$.
(Lorenzo Bastioni)

Problema 91[4929]
Osserviamo che $g(n)=3^5 \cdot \frac{n}{v_{3}(n)}$, dunque la somma richiesta è uguale a $\sum 3^5 \cdot \frac{n}{v_{3}(n)}$. Definiamo quindi $h(n)=\frac{n}{v_{3}(n)}$ e calcoliamo separatamente le somme di $h(n)$ in base a $v_{3}(n)$.

$\bullet$ $v_{3}(n)=5$ \rightarrow $h(n)=1$: $n \in \{243 \cdot 1\}$, \rightarrow $\sum h(n)=1$

$\bullet$ $v_{3}(n)=4$: $n \in \{81 \cdot 2, ..., 81 \cdot 5\}$ esclusi i multipli di $243$, $\sum h(n)=(2+...+5)-1\cdot 3=14-3=11$

$\bullet$ $v_{3}(n)=3$: $n \in \{27 \cdot 5, ..., 27 \cdot 15\}$ esclusi i multipli di $81$, $\sum h(n)=(5+...+15)-14\cdot 3=110-42=68$

$\bullet$ $v_{3}(n)=2$: $n \in \{9 \cdot 15, ..., 9 \cdot 46\}$ esclusi i multipli di $27$, $\sum h(n)=(15+...+46)-110\cdot 3=976-330=646$

$\bullet$ $v_{3}(n)=1$: $n \in \{3 \cdot 45, ..., 3 \cdot 140\}$ esclusi i multipli di $9$, $\sum h(n)=(45+...+140)-976\cdot 3=8880-2928=5952$

$\bullet$ $v_{3}(n)=0$: $n \in \{1 \cdot 135, ..., 1 \cdot 420\}$ esclusi i multipli di $3$, $\sum h(n)=(135+...+420)-8880\cdot 3=79365-26640=52725$

Quindi, $\sum h(n)=52725+5952+646+68+11+1=59403$, da cui $\sum f(n)=3^5 \cdot \sum h(n)=14434929$. Ci interessano le ultime 4 cifre
(Valeria Martinelli)

Problema 92

Problema 93 [279]
$x$,$y$ e $z$ sono interscambiabili. Per cui è possibile, per facilitarci nella risoluzione, porre $x \geq y \geq z$.

Adesso andiamo in ordine:\\
abbiamo

$x^2<x^2+2y+z \leq x^2+3x < x^2+4x+4=(x+2)^2$

e siccome lo vogliamo come quadrato

$x^2+2y+z=(x+1)^2 \Rightarrow 2y+z=2x+1 \Rightarrow x=\frac{2y+z-1}{2}$.

Ora usiamo la seconda equazione con le nuove sostituzioni.\\
$y^2<y^2+2z+x=y^2+2z+\frac{2y+z-1}{2}=y^2+y+2.5(z)-0.5<y^2+y+3y+4=(y+2)^2$.

Per lo stesso motivo di prima:

$y^2+y+2.5(z)-0.5=(y+1)^2 \Rightarrow 5z-3=2y \Rightarrow y=\frac{5z-3}{2}$\\
Ora è il momento di usare l'ultima equazione, anche qui sostituendo $y$ e $x$:

$z^2<z^2+2x+y=z^2+2y+z-1+y=z^2+z+3(\frac{5z-3}{2})-1=z^2+8.5z-4.5$.

Dopo vari tentativi si nota che gli unici quadrati possibili per questa quantità sono $(x+1)^2$ e $(x+4)^2$.\\

Infatti si avrà $z^2+8.5z-4.5=z^2+8z+16 \Rightarrow z=43$, trovandoti poi $y=106$ e $x=127$ e $z^2+8.5z-4.5=z^2+2z+1$ trovando quindi $x=z=y=1$.
(Alessandro Avellino)

Problema 94 [6460]
Possiamo riscrivere il testo del problema nel seguente modo:
$$\sum_{n=1}^{2020} \lfloor \frac{\sqrt{n+2}}{\sqrt{n+1} - \sqrt{n}} \rfloor$$
Andiamo ad analizzare il nostro addendo generale privo della funzione floor:
$ \frac{\sqrt{n+2}}{\sqrt{n+1} - \sqrt{n}} = \frac{\sqrt{n+2}}{\sqrt{n+1} - \sqrt{n}} \cdot \frac{\sqrt{n+1} + \sqrt{n}}{\sqrt{n+1} + \sqrt{n}} = \sqrt{(n+2) \cdot (n+1) } + \sqrt{ (n+2) \cdot n } = \sqrt{n^2 + 3n+ 2} + \sqrt{n^2 +2n} $.
Esplorando un po' la situazione ci si potrebbe convincere che $2n+2 \leq \sqrt{n^2 + 3n+ 2} + \sqrt{n^2 +2n} < 2n+3$ (con $n$ nell'intervallo che interessa a noi ovvero $[1,2020]$).
Dimostriamolo:
La seconda disuguaglianza è molto semplice infatti $\sqrt{n^2 + 3n+ 2} + \sqrt{n^2 +2n} = \sqrt{(n+2)^2 -n-2} + \sqrt{(n+1)^2 - 1} < \sqrt{(n+2)^2} + \sqrt{(n+1)^2} = n+2 +n+1 = 2n+3$ quindi $\sqrt{n^2 + 3n+ 2} + \sqrt{n^2 +2n} < 2n+3$.
La prima disuguaglianza è leggermente più fastidiosa:
vogliamo $2n+2 \leq \sqrt{n^2 + 3n+ 2} + \sqrt{n^2 +2n}$ che è molto lungo da risolvere a mano dunque proviamo a scomporre in maniera intelligente il problema:
ad esempio, se dimostrassimo che $\sqrt{n^2 + 3n+ 2} \geq n + \frac{4}{3} \wedge \sqrt{n^2 +2n} \geq n+\frac{2}{3}$ avremmo finito poiché sommando le disuguaglianze si otterrebbe la tesi (la veridicità delle due disuguaglianze implica la tesi ma non viceversa).
Le due disuguaglianze sono due semplici disequazioni irrazionali e non ci dovremmo preoccupare di condizioni di esistenza o simili poiché nel nostro intervallo è tutto positivo:
1)
$ \sqrt{n^2 + 3n+ 2} \geq n + \frac{4}{3} \rightarrow n^2+3n+2 \geq (n + \frac{4}{3})^2 \rightarrow n \geq - \frac{2}{9}$, nel nostro intervallo la disuguaglianza vale.
2)
$ \sqrt{n^2 +2n} \geq n+\frac{2}{3} \rightarrow n^2+2n \geq (n+\frac{2}{3})^2 \rightarrow n \geq \frac{2}{3} $, nel nostro intervallo la disuguaglianza vale.
La tesi è dunque dimostrata, $2n+2 \leq \sqrt{n^2 + 3n+ 2} + \sqrt{n^2 +2n} < 2n+3$. Ciò implica che $\lfloor \frac{\sqrt{n+2}}{\sqrt{n+1} - \sqrt{n}} \rfloor = 2n+2$ (nel nostro intervallo) e possiamo riscrivere la nostra somma:
$$\sum_{n=1}^{2020} 2n+2 = \left( 2 \sum_{n=1}^{2020} n \right) + 2 \cdot 2020 = 2020 \cdot 2021 + 4040 = 4086460$$
(La seconda uguaglianza si ha con la formula di Gauss per la somma dei primi $k$ numeri naturali) Prendendo le ultime $4$ cifre del risultato abbiamo la soluzione: $6460$.
(Lorenzo Bastioni)

Problema 95 [21]
Mettiamo in un piano cartesiano i punti [math]. Si ha che [math] è il coffieciente angolare della retta per i due punti [math]. Quindi una sequenza è non genovese se tutti i coefficienti angolari sono non negativi; ovvero se e solo se essa è debolmente crescente. Supponiamo di avere una sequenza debolmente crescente [math]. Vi è una bigezione fra le sequenze di questo tipo e quelle del tipo [math], che sono tutte le sequenze strettamente crescenti con elementi positivi minori di [math]. Esse a loro volta corrispondono con i modi di scegliere [math] elementi distinti fra [math]. Il numero totale di sequenze (genovesi e non) è infine [math]; quindi il numero di quelle genovesi è [math] che è congruo a [math] modulo [math], essendo che [math] modulo [math].
(Federico Volpe)

Problema 96 [2029]
I numeri curiosi sono esattamente i liberi dai quadrati.\\
($\Leftarrow$) Se $n$ è un numero libero da quadrati allora $\tau(n)$ è una potenza di due, inoltre ogni divisore $d$ di $n$ è un numero libero da quadrati, quindi anche $\tau(d)$ è una potenza di $2$ e dato che $\tau(d) \le \tau(n)$ abbiamo che $\tau(d)|\tau(n)$.\\
($\Rightarrow$) Supponiamo che $n = p_1^{a_1}p_2^{a_2}\cdots p_m^{a_m}$ sia un numero libero da quadrati. Allora abbiamo
\begin{align*}
&\tau(n/p_i)|\tau(n)\\
\Rightarrow &(a_1+1)\cdots(a_i)\cdots(a_m+1)|(a_1+1)\cdots(a_i+1)\cdots(a_m+1)\\
\Rightarrow &a_i | a_i+1 \Rightarrow a_i = 1.
\end{align*}
Dunque $n$ è libero da quadrati.\\
\\
Inoltre notiamo che $f(a,b)$ è semplicemente $MCD(a,b)$. A questo punto scriviamo
$$S(n) = \sum_{j=1}^n f(j,n) = \sum_{j=1}^n MCD(j,n),$$
dimostreremo che la funzione $S$ è moltiplicativa, cioè per ogni $m,n \in \mathbb{Z}^+$ coprimi $S(m)S(n) = S(mn)$. Infatti, segue dal teorema del resto cinese, possiamo rimpiazzare $j \text{ mod }\,mn$ con due numeri $j_m \text{ mod }\,m$ e $j_n \text{ mod }n$ in modo che:
\begin{align*}
S(mn) &= \sum_{j=1}^{mn} MCD(j,mn) = \sum_{j=1}^{mn} MCD(j,m)MCD(j,n)\\
&= \sum_{j_m=1}^{m} \sum_{j_n =1}^{n} MCD(j_m,m)MCD(j_n,n)\\
&= \left(\sum_{j_m=1}^m MCD(j_m,m)\right)\left(\sum_{j_n=1}^n MCD(j_n,n)\right) = S(m)S(n).
\end{align*}
Ma quindi dato che $C$ è un prodotto tra primi ed è facile verificare che $S(p) > 1$ per ogni primo $p$, abbiamo che $C$ stesso deve essere primo. Un po' di brute-force dà la risposta.
(Michele Tomasi)

Problema 97 [118]

Problema 98

Problema 99

Problema 100 [1339]
Premetto innanzitutto che questo geometrico è più tecnico degli altri ed è necessario avere qualche prerequisito sulle simmediane e il cerchio di Feuerbach. Tutti i fatti noti di cui farò uso si trovano sul libro di Evan Chen ma sono certo che si possano trovare anche girovagando su internet.

La soluzione di questo problema si può suddividere in tre claim.

Primo claim: La tangente di $\Gamma$ in $N_a$, la retta $BC$ e la tangente di $\Gamma$ in $A$ concorrono nel punto $X_a$.\\
Qua c'è solo da usare qualche fatto noto delle simmediane. Infatti $\triangle BAN_a \sim \triangle CAM_a \Rightarrow \angle BAN_a= \angle CAM_a \Rightarrow AN_a$ è la $A-simmediana$ di $\triangle ABC$. Dopodiché basta usare che $CB$ è la $C-simmediana$ di $\triangle ACN_a$ e quindi le tangenti in $A$ e in $N_a$ concorrono con essa.

Secondo claim:
$Y_a$ e simili stanno sulla circonferenza di Feuerbach, ovvero $\gamma$ e $\omega$ coincidono.
Per essere capaci di dimostrarlo però è necessario notare qualcosa di più subdolo.
Sia $H$ l'ortocentro di $\triangle ABC$, $V$ il punto medio di $AH$ e $Q$ l'intersezione tra la retta $AH$ e $\Gamma$. Siano inoltre $R$ e $r$ i raggi di $\Gamma$ e di $\gamma$.
Lemma: $V$ è L'ortocentro di $\triangle AM_{a}X_{a}$.
Proof: Dimostriamo innanzitutto che la retta $VM_a$ è perpendicolare a $AX_a$. CIò è equivalente a dimostrare che $\angle X_{a}M_{a}V = 90^{\circ}- \angle AX_{a}M_a=\angle X_{a}AP_{a} \Leftrightarrow \angle P_{a}M_{a}V=\angle X_{a}AQ \Leftrightarrow \frac{VP_{a}}{AQ}=\frac{r}{R}=\frac{1}{2}$.
Poiché $Q$ è il simmetrico di $H$ su $BC$ (si dimostra con angle chasing), $HQ+HA=2HP_{a}+2HV \Rightarrow AQ=2VP_{a} \Rightarrow \frac{VP_{a}}{AQ}=\frac{1}{2}$.
$AV \perp X_{a}M_{a}$ e $M_{a}V \perp AX_{a} \Rightarrow V$ ortocentro di $\triangle AM_{a}X_{a}$.
A questo punto per dimostrare il claim iniziale basta dire che, poichè $V$ è ortocentro, $X_{a},V,Y_{a}$ sono allineati e quindi $\angle VY_{A}M_{a}=90^{\circ}=\angle VP_{a}M_{a} \Rightarrow VY_{a}M_{a}P_{a}$ ciclico $\Rightarrow Y_{a}$ sta su $\gamma$.
Similarmente si può dimostrare che anche $Y_{b}$ e $Y_{c}$ stanno su $\gamma$.

Terzo claim: le rette $P_{a}P_{b}, DE, CH$ concorrono in $F$.
$\angle BHC=180^{\circ}- \angle BAC \Rightarrow H$ sta su $\Gamma'$. Ora basta osservare che $P_{a}P_{b}, DE, CH$ sono gli assi radicali di $\gamma, \Gamma', (P_{a}HP_{b}C)$.
Similarmente si dimostra che $P_{a}P_{c}, DE, BH$ concorrono in $G$.

Ora che abbiamo delle informazioni "decenti" sui punti $F$ e $G$ possiamo fare del segment chasing per trovare i valori di $P_{a}F$ e $P_{a}G$. $\triangle P_{a}P_{b}P_{c}$ è il famoso triangolo ortico e $P_{c}G$ e $P_{b}F$ sono le bisettrici, quindi l'idea sarebbe quella di trovare i tre lati e poi usare il teorema delle bisettrici.
$BC^{2}-P_{b}C^{2}=AB^{2}-(AC-P_{b}C)^{2} \Rightarrow P_{b}C=\frac{84}{5}$.
$\triangle BCA \sim \triangle P_{b}CP_{a} \Rightarrow \frac{P_{a}P_{b}}{P_{b}C}=\frac{AB}{BC} \Rightarrow P_{a}P_{b}=\frac{78}{5}$.
Similarmente si trovano $P_{b}P_{c}=\frac{924}{65}$ e $P_{a}P_{c}=\frac{150}{13}$
$\frac{P_{a}F}{P_{a}P_{c}-P_{a}F}=\frac{P_{b}P_{a}}{P_{b}P_{c}} \Rightarrow P_{a}F=\frac{1950}{323}$
$\frac{P_{a}G}{P_{a}P_{b}-P_{a}G}=\frac{P_{c}P_{a}}{P_{c}P_{b}} \Rightarrow P_{a}G=\frac{650}{93}$
$P_{a}F+P_{a}G=\frac{391300}{30039}$. La risposta è quindi $1339$.
(Denis Tusca)
[/quote]
[/quote]
Avatar utente
Feuergas
Messaggi: 1
Iscritto il: 08 nov 2019, 23:29

Re: #Proviamoci - Soluzioni commentate OH6

Messaggio da Feuergas »

Dato che si è raggiunto il limite di caratteri, fino a che non si trova una nuova soluzione penso che intanto si possano postare un po' di soluzioni da sole.
Ecco la mia per il 98:


Problema 98 [1]
Sia $C_n:=\{k\in\{1,2,\dots,2n\}:2n\:|\:k(k+1)\}$.
In particolare si ha che $g(n)=\frac{|C_n|}{|A_n|}=\frac{|C_n|}{2n}$
Lemma 1
Se $n=2^m\in\mathbb{N}$ con $m\in\mathbb{N},m\geq 1$ allora $C_{2^m}=\{2^{m+1}-1,2^{m+1}\}$ (e in particolare $|C_{2^m}|=2$).
Dimostrazione 1
Dato $k\in\{1,2,\dots,2\cdot2^m\}$ allora $2\cdot 2^m=2^{m+1}\:|\:k(k+1)\iff 2^{m+1}\:|\:k\vee 2^{m+1}\:|\:k+1\iff k=2^{m+1}-1,2^{m+1}$.
Lemma 2
Se $n=p^m$ con $p\in\mathbb{P}=\{p\in\mathbb{N}:p$ è primo$,p>2\}$ e $m\in\mathbb{N},m\geq 1$ allora $C_{p^m}=\{p^m-1,p^m,2p^m-1,2p^m\}$ (e quindi in particolare $|C_{p^m}|=4$).
Dimostazione 2
Si ha infatti che, dato $k\in\{1,2,\dots,2p^m\}$, allora $2p^m\:|\:k(k+1)\iff p^m\:|\:k(k+1)\iff p^m\:|\:k\vee p^m\:|\:k+1\iff k=p^m,2p^m\vee k+1=p^m,2p^m\iff k=p^m-1,p^m,2p^m-1,2p^m$.
Lemma 3
Siano $a,b\in\mathbb{N}$ tali che $\gcd(a,b)=1$. Allora
  • posto $E_a=\{k\in C_a:2\:|\:k\}$ si ha $2|E_a|=|C_a|$, ovvero esattamente metà degli elementi di $C_a$ sono pari;
  • $|E_{ab}|=|E_a||E_b|$, ovvero la funzione $n\mapsto |E_n|$ è moltiplicativa.
Dimostrazione 3
Per mostrare la tesi si considerino i seguenti casi:
  • Se $a=1$ allora $E_1=\{2\}$ e $C_1=\{1,2\}$, da cui la tesi.
  • Se $a=2^m,m\geq 1$ la tesi segue dal Lemma 1.
  • Se $a=p^m,p\in\mathbb{P},m\geq 1$ la tesi segue dal Lemma 2.
  • Se $a=p^m\alpha$, con $p\in\mathbb{P}\cup\{2\},m\geq 1$ e $\alpha\in\mathbb{N}$ tale che $\gcd(p,\alpha)=1$.
    Sia $\bar{k}\in C_a$. Allora esistono $k_{p^m}\in C_{p^m},k_{\alpha}\in C_{\alpha}$ tali che $\bar{k}\equiv k_{p^m} (\mathrm{mod}\:2p^m)$, $\bar{k}\equiv k_{\alpha} (\mathrm{mod}\:2\alpha)$, infatti $2p^m\alpha=a\:|\:\bar{k}(\bar{k}+1)$.
    In particolare ogni $\bar{k}\in C_a$ è univocamente determinato dai corrispettivi $k_{p^m},k_{\alpha}$, infatti il sistema
    \begin{equation}
    \begin{cases}
    \bar{k}\equiv k_{p^m} (\mathrm{mod}\:2p^m)
    \\
    \bar{k}\equiv k_{\alpha} (\mathrm{mod}\:2\alpha)
    \end{cases}
    \end{equation}
    ha un'unica soluzione se $k_{p^m}\equiv k_{\alpha} (\mathrm{mod}\:2)$ e nessuna altrimenti.
    Segue che a ogni elemento $\tilde{k}\in E_a$ corrisponde un'unica coppia $(k_{p^m},k_{\alpha})\in E_{p^m}\times E_{\alpha}$ e a ogni elemento $\tilde{k}\in C_a \setminus E_a$ corrisponde un'unica coppia $(k_{p^m},k_{\alpha})\in (C_{p^m}\setminus E_{p^m})\times (C_{\alpha}\setminus E_{\alpha})$. Sia $O_a=C_a\setminus E_a$.
    Si ottiene quindi che $|E_a|=|E_{p^m}||E_{\alpha}|$, $|O_a|=|O_{p^m}||O_{\alpha}|$,
    \begin{equation}
    |C_a|=|E_a|+|O_a|=|E_{p^m}||E_{\alpha}|+|O_{p^m}||O_{\alpha}|=2|E_{p^m}||E_{\alpha}|=2|E_a|
    \end{equation}
    dove si è usata l'ipotesi induttiva $2|E_{p^m}|=|C_{p^m}|=|E_{p^m}|+|O_{p^m}|$ (analogamente per $E_{\alpha}$).
    Si noti come si è mostrata in questo modo anche la moltiplicatività di $n\mapsto E_n$.
Lemma 4
Siano $a,b\in\mathbb{N}$ tali che $\gcd(a,b)=1$. Allora $g(ab)=g(a)g(b)$, ovvero $g$ è moltiplicativa.
Dimostrazione 4
Per il Lemma 3 si ottiene $g(ab)=\frac{|C_{ab}|}{2ab}=\frac{|E_{ab}|}{ab}=\frac{|E_a|}{a}\frac{|E_b|}{b}=\frac{|C_a|}{2a}\frac{|C_b|}{2b}=g(a)g(b)$.

Per il Lemma 1 si ha che $g(2^m)=\frac{|C_{2^m}|}{2\cdot2^{m}}=\frac{2}{2^{m+1}}=\frac{1}{2^m}$.
Analogamente per il Lemma 2 abbiamo $g(p^m)=\frac{|C_{p^m}|}{2p^m}=\frac{4}{2p^m}=\frac{2}{p^m}$.
Si ottiene quindi che dati $p,q\in\mathbb{P}\cup\{2\},m,n\in\mathbb{N}$ tali che $g(p^m)=g(q^n)\Rightarrow p^m=q^n\Rightarrow p=q,m=n$.
Dato che ogni numero può essere scritto univocamente come prodotto di fattori primi (a meno dell'ordine), dalla moltiplicatività di $g(n)$ otteniamo che $\forall n,m\in\mathbb{N}$ si ha $g(n)=g(m)\Rightarrow n=m$, ovvero $g$ è iniettiva.
Segue che $|S_i|=1\:\forall i$ e il risultato del problema è perciò $1$.
(Leonardo Gagliardoni)
Mattysal
Messaggi: 254
Iscritto il: 06 feb 2018, 14:54
Località: Torino
Contatta:

Re: #Proviamoci - Soluzioni commentate OH6

Messaggio da Mattysal »

Visto il raggiungimento del limite di caratteri, facciamo così: ogni nuovo aggiornamento consisterà di due messaggi: dall'1 al 70 e dal 71 al 100
Grazie a tutti per la collaborazione finora, sta andando meglio del previsto
Mattysal
Messaggi: 254
Iscritto il: 06 feb 2018, 14:54
Località: Torino
Contatta:

Re: #Proviamoci - Soluzioni commentate OH6

Messaggio da Mattysal »

Problema 1 [3]
$n$ il numero di commensali e $d$ la distanza tra loro, i commensali sono i punti ${A_1, A_2,..., A_n}$ mettiamo $A_1$ in un punto qualsiasi del piano. $A_2$ potrà stare ovunque nella circonferenza di raggio $d$ e centro $A_1$. Il terzo punto ha solo $2$ posizioni in cui poter stare, che sono le intersezioni delle $2$ circonferenze di centro $A_1$ e $A_2$, entrambe di raggio $d$. Posizionato il terzo punto e tracciata la terza circonferenza noteremo che non ci sarà nessun punto in comune a tutte e $3$ le circonferenze. Di conseguenza non ci può essere un quarto punto che rispetta le condizioni.
(Alessandro Avellino)

Problema 2 [6]
Si tratta di disporre tre diversi ingredienti, che si può fare semplicemente in $3!=6$ modi diversi. La risposta è quindi $6$.
(Matteo Salicandro)

Problema 3 [228]
Il massimo numero ottenibile lanciando gli $n$ dadi è $n\cdot k$, il minimo è $n\cdot 1=n$. Pertanto $n\cdot k-n=168$, cioè $n(k-1)=168$. Questo vuol dire che $n$ è un qualsiasi divisore positivo di $168$, esclusi $168$ e $84$ visto che $k \geq 4$. $168=2^3\cdot 3\cdot 7$, quindi la somma dei suoi divisori è $(2^4-1)(3+1)(7+1)=480$. La somma dei possibili valori di $n$ è pertanto $480-168-84=228$.
(Federico Borasio)

Problema 4

Problema 5 [7]
Banalmente (tanto la calcolatrice era lecita) $5^1,=5, 5^2=25, 5^3=125, 5^4=625, 5^5=3125, 5^6=15625, 5^7=78125, 5^8=390625, 5^9=1953125, 5^{10}=9765625, 5^{11}=48828125, 5^{12}=244140625, 5^{13}=1220703125, 5^{14}=6103515625$. Non è necessario andare oltre, perché per il principio dei cassetti, un numero con 11 cifre contiene necessariamente almeno due cifre identiche. Si vede facilmente, confrontando i numeri scritti, che la risposta è $7$.
(Matteo Salicandro)

Problema 6 [36]
Un rettangolo con le diagonali perpendicolari è un quadrato. L'area è quindi $6^2=36$.
(Filippo Prandina)

Problema 7 [18]
La successione è definita, per $n \geq 1$, come
$\begin{cases}
x_{n+2}=x_{n+1}+x_n\\
x_6=76\\
x_5=47
\end{cases}$
Da qui ricaviamo che $x_n=x_{n+2}-x_{n+1}$, pertanto $x_4=76-49=29$ e $x_3=47-29=18$, che è il terzo termine della successione.
(Federico Borasio)

Problema 8 [2022]
Supponiamo che ci siano $2022$ fisici. Tutti dicono la verità, cioè che non sono matematici: non c'è alcuna contraddizione. $2022$ è anche il numero di abitanti dell'isola, quindi è il massimo numero di fisici.

Problema 9 [8]
$\frac{4n}{32}=k$ con $k\in N\geq 0$. Quindi $n=8k$. Il minimo si ha quando $k=1$ e $n=8$.
(Filippo Prandina)

Problema 10[8]
Sono possibili due casi, in base a qual è la circonferenza con raggio minore. Tuttavia, r deve essere massimo, quindi consideriamo il caso in cui r$>$5. L'area della corona circolare sarà data da
$\pi{r}^{2}-{5}^{2}\pi=\pi({r}^{2}-25)=16\pi.$ Per cui ${r}^{2}-25=16$, ovvero $r=\sqrt{41}\approx{6.403}$, la cui parte intera è 6.
(Daniele Prisco)

Problema 11 [9999]
Ci sono più soluzioni in quanto i primi due termini possono essere $1,3$, oppure $2,2$ oppure $3,1$. La risposta è quindi $9999$.
(Matteo Salicandro)

Problema 12[1261]
Affinché una parola sia impronunciabile per il nostro Loenzo, bisogna che contenga almeno una R. Se c'è una sola R nella parola, questa potrà essere la prima, la seconda o la terza lettera, e in ognuno di questi casi ci saranno 20*20=400 parole possibili, dato che nei restanti due 'posti' vanno inserite le altre 20 lettere dell'alfabeto, che possono essere ripetute. Se nella parola ci sono due R, esse si possono disporre in 3 modi, e per ognuno si potranno formare 20 parole. Infine, va incluso l'unico caso in cui la parola è formata da tre R.
In totale, i casi sono $400*3+20*3+1=1261$.
(Daniele Prisco)

Problema 13 [28]
$X$ appartiene alla perpendicolare di $BC$ passante per $A$. Inoltre $X$ e $2$ punti su $BC$ formano lo stesso angolo che forma $A$ con quei $2$ punti, di conseguenza $X$ è il simmetrico di $A$. Da questo deduciamo che $AX=2AD$, dove $AD$ è l'altezza.
(Alessandro Avellino)

Problema 14 [1011]
La quantità in questione, $(3n+2)^{5n+8}$, è un quadrato perfetto se lo è la base, cioè $3n+2$, oppure se l'esponente, $5n+8$, è pari. Tuttavia la prima possibilità non si verifica mai poiché un quadrato non è mai congruo a $2\,(\mathrm{mod}\,3)$. Dunque la risposta è semplicemente il numero di interi positivi $1\leq n\leq 2022$ tali che la quantità $5n+8$ sia pari, ovvero tutti e soli i numeri pari in questo intervallo, che sono $\frac{2022}{2}=1011$.
(Lorenzo Weiss)

Problema 15

Problema 16 [2019]
Il problema chiede per quanti interi positivi minori o uguali a $2022$ accade che $\lfloor \frac{\pi +n}{n}\rfloor =\lfloor \frac{\pi}{n}+1\rfloor =\lfloor \frac{\pi}{n}\rfloor +1=1\Longrightarrow \lfloor \frac{\pi}{n}\rfloor =0$, il che succede solo quando $\pi\approx 3,14\leq n$, i.e. in $2022-(4-1)=2019$ casi.
(Lorenzo Weiss)

Problema 17

Problema 18

Problema 19 [14]
Disegnando la circonferenza $x^2+y^2=5$ risulta evidente che gli unici punti a coordinate intere che vi appartengono, che sono $k=8$, sono quelli della forma $P_i(\pm 1,\pm 2)$, $P_i(\pm 2,\pm 1)$, $P_i(\pm 1,\mp 2)$, $P_i(\pm 2,\mp 1)$. In alternativa, è possibile risolvere l'equazione della stessa circonferenza sugli interi $x,y$. Da ciò si ricava l'ottagono $P_1P_2\dots P_8$, la cui area, come si vede facilmente, è $16-4\cdot\frac{1}{2}=14$.
(Lorenzo Weiss)

Problema 20

Problema 21 [210]
Notiamo che, comunque presi $6$ elementi distinti dall'insieme $X=\{1, 2, 3, 4, 5, 6, 7, 8, 9, 10\}$, c'è uno e un solo modo di disporli in ordine crescente, dunque la richiesta del problema è equivalente al trovare il numero di sottoinsiemi di $6$ elementi di $X$, che sono $\displaystyle\binom{10}{6}=210$.
(Valeria Martinelli)

Problema 22 [2022]
Si può ottenere $2$ solo quando una delle cifre è $2$ e le altre sono nulle, oppure quando due cifre sono uguali a $1$ e le altre sono nulle. Nel primo caso, la prima cifra sarà necessariamente $2$, altrimenti il numero non avrà $2022$ cifre: si ottiene solo una possibilità.
Nel secondo caso, invece, la prima cifra sarà $1$, mentre l'altro $1$ occuperà una delle altre $2021$ posizioni disponibili, ottenendo $2021$ possibilità.
Sommando le possibilità, si arriva a $2022$.
(Daniele Prisco)

Problema 23

Problema 24 [4042]
Se $x$ è la lunghezza del segmento $PA$ e $r$ è il raggio della circonferenza,
$OP=OA+PA=r+x=2021$; $PB=OA+OB+PA=r+r+x$. Di conseguenza, $PA+PB=x+r+r+x=2(r+x)=2×2021=4042$.
(Daniele Prisco)

Problema 25 [66]
Poniamo $a+b=x$ e $c+d=y$.
La disuguaglianza adesso è
$x+y\leq \sqrt{x^2+y^2} \Rightarrow x^2+2xy+y^2\leq x^2+y^2$
Da cui $2xy\leq0$ ma siccome $x$ e $y$ sono somme di interi maggiori o uguali a $0$, l'unico modo per cui $xy=0$ si ha quando uno dei $2$, o entrambi, è uguale a $0$. Se $c+d=y=0$ da cui $c=d=0$, quindi $(a,b,0,0)$. Ricordiamo che $a \geq b$ quindi i possibili valori sono $66$.
(Alessandro Avellino)

Problema 26 [6856]
Sia $n$ il numero di circonferenze sulla prima riga. Avendo le circonferenze diametro $2$, la base del rettangolo è uguale a $2n$. Invece, essendo tutte le circonferenze tangenti, i centri di quella più alta e delle due agli estremi della base sono disposte sui vertici di un triangolo equilatero, di lato $2n-2$. L'altezza del rettangolo è quindi uguale all'altezza del triangolo aumentata di $2$, ossia $\sqrt{3}(n-1)+2$.
Quindi vogliamo $4n+2\sqrt{3}(n-1)+4>2022$, che ci dà immediatamente $n=271$, quindi $k=\frac{271\cdot 272}{2}=36856$.
La risposta voluta sono le ultime quattro cifre di $k$.
(Valeria Martinelli)

Problema 27 [250]
Intanto notiamo che $78125$ è $5^7$.
La somma di $M$ termini consecutivi con primo termine $a$ è del tipo:
$a+(a+1)+(a+2)+...+(a+M-1)$ nel quale compaiono infatti $M$ termini consecutivi.
È quindi possibile raccogliere come $a \cdot M + (1+2+3+...+M-1)=a \cdot M+ \frac{M(M-1)}{2}=M(a+\frac{M-1}{2})=78125 \Rightarrow M(2a-1+M)=5^7\cdot2$, $M<2a-1+M$ quindi al massimo $M=2 \cdot 5^3$.
(Alessandro Avellino)

Problema 28

Problema 29

Problema 30

Problema 31 [20]
Il doppio delle cifre di $n$ dovrà necessariamente essere un numero di $3$ cifre ognuna delle quali minore di $5$. Una volta individuato il numero più piccolo $n$ come $101\,(2n=202)$ e il massimo $n$ come $444\,(2n=888)$ i restanti saranno compresi tra essi. Ci saranno $18$ numeri palindromi tra questi due estremi, con la cifra intermedia contente al più il $4$ e gli estremi uguali da $1$ a $4$. I numeri saranno $101$, $111$, $121$, $131$, $141$, $202$, $222$… fino a $444$. Il totale è di $20$ numeri bipalindromi.
(Irene Mancone)


Problema 32

Problema 33 [74]
Il costo di un pallone ($C$) può essere pari o dispari. Se è pari, può essere pagato con $\frac{C}{2}$ monete da $2$ orue. Poiché $\frac{C}{2}$ è pari, $C$ sarà un multiplo di $4$, quindi potrà assumere $37$ valori.
Se $C$ è dispari, invece, il pallone può essere pagato con $\frac{C-1}{2}$ monete da $2$ orue e una moneta da un orue. Di conseguenza, $C-1$ è un multiplo di $2$, ma $\frac{C-1}{2}$ è dispari (aggiungendo una moneta da un orue, il numero di monete usate è pari). Di conseguenza, $C-1$ è multiplo di $2$ ma non di $4$, quindi può assumere altri $37$ valori.
La somma dei valori possibili è dunque $74$.
(Daniele Prisco)

Problema 34 [2023]
Notiamo che possiamo riscrivere il testo nella seguente maniera: $P((x-y)(x+y))=P(x-y)P(x+y)$.
Notiamo anche che sostituendo $x-y \mapsto z$ e $x+y \mapsto w$ la coppia $(z,w)$ può assumere qualsiasi coppia di valori in $\mathbb{R}^2$ poiché il sistema
$\begin{cases}
x-y=z\\
x+y=w
\end{cases}$
ha soluzioni per ogni coppia $(z,w)$ fissata.
Dunque il nostro problema è equivalente a $P(zw)=P(z)P(y)$ per ogni $z$ e $w$ in $\mathbb{R}$. Questa relazione è molto simile ad una delle famose equazioni funzionali di Cauchy, $f(xy)=f(x)f(y)$. Quest'ultima equazione citata ha soluzione $f(x)=x^n$ o $f(x)=0$ (con $n$ naturale) se la funzione è continua (la continuità è una delle tante condizioni sufficienti), ma un polinomio come $P$ è continuo dunque le possibili soluzioni sono quelle riportate con $n$ che va da $0$ a $2021$ per $P(x)=x^n$ più $P(x)=0$. La soluzione è quindi $2023$ polinomi possibili.
(Lorenzo Bastioni)

Problema 35

Problema 36 [2020]
Si noti che $abc=2022-ab-a$, quindi per massimizzare il prodotto bisogna minimizzare $a$ e $b$. $a,b,c$ sono $\geq 1$ per ipotesi, dunque si pone $a=b=1$ dando come soluzione $c=2020$. Quindi $(a,b,c)=(1,1,2020)$ e $abc=2020$.
(Filippo Prandina)

Problema 37 [201]
Poniamo [math] e notiamo che [math] per [math]. Ora possiamo costruire il seguente polinomio: [math] di cui conosciamo gli zeri [math] e poiché [math] è chiaramente monico possiamo affermare che [math]. Sostituendo infine [math] con [math] e ricavando [math] otteniamo che [math] da cui possiamo trovare la soluzione al problema calcolando [math]. La risposta è quindi [math].
(Lorenzo Bastioni)

Problema 38

Problema 39

Problema 40

Problema 41

Problema 42

Problema 43 [4238]
Se si devono prendere almeno $n$ dolci per essere sicuri di prenderne almeno $1$ di un determinato tipo, questo vuol dire che ci sono $n-1$ dolci non di quel tipo. In particolare:
$\begin{cases}
S+B=55\\
B+C=70\\
S+C=91
\end{cases}$
Il sistema è di immediata risoluzione e porta come soluzione $(S,C,B)=(38,53,17)$ da cui $SCB=34238$. Sono richieste solo le ultime $4$ cifre.
(Filippo Prandina)

Problema 44 [4]
La configurazione è unica, infatti se prendiamo il punto $A_1$ e lo collegassimo con un punto diverso da $B_{2022}$, come per esempio $B_{2021}$, $B_{2022}$ sarà collegato con un punto diverso da $A_1$, per esempio $A_2$. Allora $A_1B_{2021}$ non interseca $A_2B_{2022}$. Iterando questo ragionamento ad ogni punto $A_i$, si ottiene un'unica configurazione: segmenti di estremi $A_iB_{2023-i}$. Quindi i coefficienti angolari saranno $\pm 2/(2023-2i)$. Per il prodotto, il numeratore è sempre $2$ mentre il denominatore è sempre dispari. Trovare quindi ciò che il problema richiede equivale a trovare le ultime $2$ cifre di $2^{2022}$.
(Alessandro Avellino)

Problema 45 [18]
Notiamo che, comunque presa una casella, il numero di giorni che Alberto o Barbara impiegano per raggiungerla non dipende dal percorso scelto, e le uniche caselle che hanno la stessa distanza sia da Alberto che da Barbara sono quelle sulla diagonale del quadrato diversa da quella di partenza, dunque i due si incontrano se e solo se entrambi passano per una di queste tre caselle.
Dato che un percorso è valido se e solo se fa $2$ passi in una direzione e $2$ nell'altra, tutte le coppie di percorsi possibili sono $\frac{4!}{2!\cdot 2!}\cdot \frac{4!}{2!\cdot 2!}=36$, quelli in cui entrambi passano per il centro sono $(2!\cdot 2!)(2!\cdot 2!)=16$, mentre quelli in cui entrambi passano in uno dei due angoli sono $2$.
Le coppie di percorsi con cui Alberto e Barbara non si incontrano sono quindi $36-16-2=18$.
(Valeria Martinelli)

Problema 46

Problema 47 [171]
Poiché $wxy+2021$ deve dare un numero pari, $wxy$ dev'essere dispari, e di conseguenza $w$,$x$ e $y$ devono esserlo.

$w=2a+1, x=2b+1, z=2c+1$ quindi $2a+1+2b+1+2c+1=37 \Rightarrow 2(a+b+c)=34 \Rightarrow a+b+c=17$
Adesso è un semplice stars and bars, infatti $a$, $b$ e $c$ possono essere anche nulli. Distribuire $17$ elementi in $3$ contenitori, $\binom{17+3-1}{3-1}=\binom{19}{2}$.
(Alessandro Avellino)

Problema 48 [42]
Notiamo che $CQB~APB$, in quanto hanno $2$ angoli congruenti, entrambi pari a $74$. Da questo consegue anche che sono triangoli isosceli. $\measuredangle CQB=\measuredangle APB=180-2(74)=32$. Inoltre $S$ sta nell'asse di $BC$, di conseguenza anche nella bisettrice di $CBQ$. Ragionamento analogo per $APB$. Ora osserviamo il quadrilatero $PCAQ$. Questo ha gli angoli $APB$ e $CQB$ congruenti, da questo possiamo asserire che $PCAQ$ è ciclico. Questo permette di stabilire che $\measuredangle DAC=\measuredangle PQC$ e che $\measuredangle DCA=\measuredangle QPD$. Infine osserviamo il triangolo $QSP$. $\measuredangle QSP=180-\measuredangle QPS-\measuredangle PQS= 180-(16+\measuredangle PQC)-(16+\measuredangle QPA)=148-(\measuredangle PQC+\measuredangle APQ)=148-(\measuredangle DCA+\measuredangle DAC)=148-(180-\measuredangle ADC)=\measuredangle ADC-32.$ $\measuredangle ADC=360-(\measuredangle DCB+\measuredangle CBA+\measuredangle BAD)=360-3 \cdot 74=138$ da cui $\measuredangle QSP=138-132=106$. Essendo infine $S$ il circocentro di $ABC$, ne consegue che $\measuredangle ASC=2 \measuredangle ABC=148$. Quindi $\measuredangle ASQ + \measuredangle PSC= \measuredangle ASC- \measuredangle QSP=148-106=42$.
(Alessandro Avellino)

Problema 49 [15]
Dovendo esistere almeno un numero $k>1$ che elevato al cubo sia un divisore di $n$ si possono escludere i numeri da $2$ a $7$ in quanto l’unico loro divisore al cubo è $1$ e $k$ non può assumere tale valore. L’$8$ si può vedere come $2^3$ ed è un prigioniero di cubi, così come lo sono tutti i suoi multipli fino a $96$. È valido lo stesso discorso anche per i multipli di $27$ (essendo $3^3$) fino a $81$. Contandoli si arriva al risultato richiesto, $15$.
(Irene Mancone)

Problema 50

Problema 51

Problema 52[1872]
Chiamiamo le tre radici di p(x) a,b,c dove a<b<c . Essendoa,b e c in progressione aritmetica si ha che a = b-r e c = b+r dove r è la ragione della progressione aritmetica. Per le formule di Vietè a+b+c= 39 b-r + b+ b+ r= 39 da cui si ricava che b =13 . Sempre applicando le formule di Vietè, ab + ac + bc = 482 sostituendo i risultati ricavati in precedenza si trova che 13(13-r) + (13-r)(13+r) + 13(13+r) = 482 da cui sviluppando le parentesi 169 -13r +169 -r2 + 169 +13r = 482 ottenendo che r2 = 25 quindi poiché r è maggiore di 0 si ha che r=5. Quindi a=8 c=18 .
Applicando nuovamente le formule di Vietè, -m = -8*13*18 ovvero m= 1872
(Mattia Zunino)

Problema 53 [24]
Poniamo l'età di Marco al momento della prima affermazione uguale a $x$, quindi l'età del padre Luca è $4x$. Allora, $4$ anni dopo l'età di Marco è $x+4$ e quella del padre è $4x+4$, quindi vale l'equazione $3(x+4)=4x+4$; $x=12-4=8$ e infine $4x=32$.
La differenza di età tra i $2$ è $24$, quindi per rispondere alla domanda risolviamo $2y=y+24$, con $y$ l'età di Marco e $y+24$ l'età di Luca, $y=24$ che è quindi la risposta.
(Niso Cicalò)

Problema 54

Problema 55

Problema 56

Problema 57 [183]
Si può considerare il triangolo isoscele con i vertici di coordinate $B (0;0), C (20;0)$ e A$ (10; y)$ con $y > 0$
Il punto $H$ è l’intersezione tra le rette perpendicolari ai lati passanti per $A, B, C$. Sapendo che le distanze tra
$AH$ e $AD$ sono segmenti di lunghezza intera il punto $H$ si potrà trovare al minimo nella coordinata $(10;1)$. Calcolando il coefficiente angolare tra $H$ e i punti $B$ e $C$, le rette passanti per $B$ e $C$ con coefficiente angolare antireciproco si può ottenere l’ordinata di $A$ sostituendo in una delle due equazioni.
$m_{BH}$= $\frac{1}{10}$ $m^1$=$-10$ retta r passante per A e C: $y=-10x+200$
$m_{CH}$=$\frac{-1}{10}$ $m^1$=10 retta s passante per B e A : $y=10x$

Per $H (10,1)$ $A$ vale $(10, 100)$ e la distanza è intera ($99$).
Una volta stabilita l’ordinata massima di $A$ è sufficiente ripetere il procedimento. Per $H (10,2)$ $A (10, 50)$, per $H (10,4)$ $A(10,25)$. Nelle coordinate $(10, 10)$ $H$ e $A$ si sovrappongono. Poi si avrà che $A$ si trova in $(10,20)$, in $(10,2)$ e $(10,5)$. La soluzione si ottiene sommando le distanze trovate tra $H$ e $A$ senza ripetizioni.
(Irene Mancone)

Soluzione alternativa:
Essendo $ABC$ un triangolo isoscele la mediana $AD$ coincide con l'altezza rispetto a $BC$, chiamando $K$ il piede dell'altezza relativa ad $AC$ otteniamo che il triangolo $BKC$ è simile al triangolo $ABD$ poiché $\widehat{BKC} \cong \widehat{BDA} = 90^{\circ}$ e $\widehat{BCK} \cong \widehat{ABD}$ (essendo $ABC$ isoscele). Consideriamo ora i triangoli $BKC$ e $BDH$, anch'essi sono simili poiché $\widehat{BKC} \cong \widehat{BDH} = 90^{\circ}$ e $\widehat{CBK}$ è in comune. Per la proprietà transitiva $ABD \sim BKC$ e $BKC \sim BDH$ $\rightarrow$ $ABD \sim BDH$. Avendo dimostrato che i triangoli $ABD$ e $BDH$ sono simili vale la seguente uguaglianza: $\frac{BD}{AD}=\frac{HD}{BD} \rightarrow BD^2 = AD \cdot HD \rightarrow 100=AD \cdot HD$ (essendo $BD$ la metà di un segmento lungo $20$). Avendo che $AH$ e $AD$ sono di lunghezza intera anche $HD$ sarà di lunghezza intera (Poiché $HD$ è la differenza tra i due segmenti) ed avremo soluzioni limitate a $100=AD \cdot HD$ ovvero $(AD,HD)=(100,1),(50,2),(25,4),(20,5),(10,10),(5,20),(4,25),(2,50),(1,100)$. La soluzione $(AD,HD)=(10,10)$ non ci piace poiché $\widehat{ABC}$ varrebbe $90^{\circ}$ ($D$ sarebbe il centro della circonferenza circoscritta ad $ABC$ e $\widehat{ABC}$ sottenderebbe un diametro) dunque i possibili valori di $AH$ sono le diverse differenze fra le soluzioni ottenute ($AH=|AD-AH|$): $AH = 100-1= 99$, $AH = 50-2 = 48$, $AH = 25-4= 21$, $AH =20-5=15$ (le altre danno gli stessi valori poiché sono soluzioni "specchiate"). La risposta è quindi $99+48+21+15=183$.
(Lorenzo Bastioni)

Problema 58 [46]
La somma $S$ dei quadrati di delle radici un polinomio del tipo $ x^n + \alpha_{n-1} \cdot x^{n-1} + ... + \alpha_1 \cdot x + \alpha_0$ è

$S=\alpha_{n-1}^2-2 \cdot \alpha_{n-2}$

$$Q_n(x)=x^n+\sum_{i=0}^{n-1} \sqrt{i} \cdot x^i$$

Per cui, definiamo $S_n$ come la somma dei quadrati delle radici di $Q_n(x)$. Allora $S_n=n-1-2\sqrt{n-2}$ che è razionale solo se anche $2\sqrt{n-2}$ lo è. Questo accade solo quando $n-2$ è un quadrato perfetto, quindi ci sono $45$ valori. A questo aggiungiamo $n=1$, il cui polinomio è quindi $x$, che ha come radici un numero razionale. Quindi la soluzione è $46$.
(Alessandro Avellino)

Problema 59

Problema 60

Problema 61[9098]
Per il criterio di divisibilità per 9, s(n) ≡ n mod 9 dove s(n) è la somma delle cifre di n. Iterando il ragionamento su s(n) si ottiene che A(n)≡n mod9 . Poiché A(n) è un intero compreso fra 0 e 9, A(n) è uguale al resto della divisione di n per 9. I possibili resti della divisione delle potenze di 2021 per 9 sono 5, 7, 8, 4, 2, 1 e poi si ripetono ciclicamente con periodo 6. Perciò la somma richiesta è 2016/6 *(5+7+8+4+2+1) +5+7+8+4+2 = 9098.
(Mattia Zunino)
Problema 62 [1158]
Dette $H$ e $K$, rispettivamente, le proiezioni di $D$ e $C$ su $AB$ e $AH=x$ possiamo calcolare l'altezza del trapezio in due modi diversi servendoci del teorema di Pitagora ed eguagliare le espressioni $DH^2=AD^2-AH^2=33^2-x^2=CK^2=BC^2-BK^2=60^2-(69-6-x)^2$.

Risolvendo l'equazione, si trova l'unico valore $x=\frac{81}{7}$ e l'altezza si può quindi calcolare, ad esempio, come $h=DH=\sqrt{AD^2-AH^2}=\frac{60\sqrt{13}}{7}$. L'area del trapezio vale $A=\frac{(b_1+b_2)\cdot h}{2}=\frac{(AB+CD)\cdot AH}{2}=\frac{(69+6)\cdot (60\sqrt13)}{7\cdot2}\approx 1158,93$; la sua parte intera è $1158$ ed è la soluzione.
(Federico Magnolfi)

Problema 63

Problema 64

Problema 65 [5877]
$X$ si trova sulla superficie del triangolo $AMC$: infatti, se così non fosse, $\widehat{ABC}$ e $\widehat{AXM}$ sarebbero due angoli congruenti che insistono su uno stesso segmento ($AM$) e che appartengono allo stesso semipiano rispetto ad esso: ma ciò non è possibile in quanto significherebbe che $X$ appartiene alla circonferenza circoscritta al trangolo $ABM$, alla quale ovviamente non appartengono punti interni al triangolo $ABM$.

$AXMY$ è ciclico per ipotesi e $\widehat{AYM}$ e $\widehat{AXM}$ sono supplementari in quanto angoli opposti (per quanto prima dimostrato); dunque $\widehat{BYM}$, essendo anch'esso supplementare di $\widehat{AYM}$, è congruente a $\widehat{AXM}$, il quale a sua volta è congruente a $\widehat{ABC}$ per ipotesi.
Avremo allora che il triangolo $MBY$ è isoscele su base $BY$; essendo $M$ punto medio di $BC$, $MB\cong MY\cong MC$ e ciò implica che $BYC$ è rettangolo in $Y$. $CY$ è dunque l'altezza uscente da $C$ e si può calcolare con la formula inversa $CY=\frac{2A}{AB}$ dopo aver ricavato l'area con la formula di Erone o per via trigonometrica.
Si trova che quest'ultima vale $A=1200\sqrt{5}$ e dunque $CY^2=\left(\frac{2A}{AB}\right)^2=\frac{4\cdot1200^2\cdot 5}{70^2}\approx 5877,55$. La sua parte intera è $5877$ ed è la soluzione.
(Federico Magnolfi)

Problema 66 [1155]
Prendiamo l'equazione del problema e consideriamola $(\text{mod}\,7)$: $x^2 \equiv 7y + z \,(\text{mod}\,7) \Rightarrow x^2 \equiv z \, (\text{mod}\,7)$. I quadrati come $x^2$ modulo $7$ possono assumere come valori solo $0,1,2,4$ quindi $z$ potrà assumere tutti e i soli valori $0,1,2,4$ modulo $7$. Nell'intervallo $[1,2022]$ ci sono esattamente $288$ numeri congrui a $0 \, (mod 7)$, $289$ congrui a $1$, $289$ congrui a $2$ e $289$ congrui a $4$. La risposta è quindi $288+289 \cdot 3=1155$.
(Lorenzo Bastioni)

Problema 67 [3026]
Scriviamo il numeratore come $\prod_{n=2}^{2021}((n-1)+n+(n+1))=\prod_{n=2}^{2021}(3n)=3^{2021}\prod_{n=2}^{2021}(n)$. Si può quindi raccogliere un $3^{2021}$ al numeratore. Inoltre si raccolgono anche una potenza di $3^{1005}$ dato che $n$ può avere $3$ come fattore primo (elevato alla $1,\,2,\,3,\,\dots,6$). Ci sono $ \Big \lfloor \frac{2021}{3} \Big \rfloor=673$ multipli di 3, $ \Big \lfloor \frac{2021}{9} \Big \rfloor =224$ multipli di $9$ e così via: $ \Big \lfloor \frac{2021}{3} \Big \rfloor+ \Big \lfloor \frac{2021}{9} \Big \rfloor + \Big \lfloor \frac{2021}{27} \Big \rfloor + \Big \lfloor \frac{2021}{81} \Big \rfloor + \Big \lfloor \frac{2021}{243} \Big \rfloor + \Big \lfloor \frac{2021}{729} \Big \rfloor =673+224+74+24+8+2=1005$. In tutto quindi si è raccolto $2021+1005=3026$.
(Filippo Prandina)

Problema 68 [1104]
Chiamiamo due insiemi $A$ e $B$ "gemelli di primo grado" se $|A \cap B|=1$ e $1 \in A,B$, in generale chiamiamo due insiemi $A$ e $B$ "gemelli di $n$-esimo grado" se $|A \cap B|=n$ e $n \in A,B$.
Possiamo ora affrontare il conteggio di tutte le coppie di sottoinsiemi gemelle di $\{1,2,3,...,2021,2022 \}$ considerando prima i gemelli di primo grado, di secondo grado fino a quelli di $2022$-esimo grado (Non ci sono coppie di sottoinsiemi di $2023$-esimo grado o di $0$-esimo grado).
Gemelli di primo grado:
se $A$ e $B$ sono di primo grado allora contengono entrambi l'elemento $1$ e non hanno nessun altro elemento in comune. Ci verra comodo per dopo scrivere che gli altri zero elementi in comune si possono scegliere in $\displaystyle \binom{2021}{0}$ modi. Ora tutti gli altri $2021$ elementi possono essere messi o in $A$ o in $B$ o in nessuno dei due (in questo caso la "o" è una o esclusiva), quindi abbiamo 3 possibilità per ognuno dei $2021$ elementi ovvero $3^{2021}$ possibilità. Avremo quindi $\displaystyle \binom{2021}{0} 3^{2021}$ coppie di sottoinsiemi gemelle di primo grado.
In generale siamo pronti ad affrontare il caso di gemelli di $n$-esimo grado:
se $A$ e $B$ sono gemelli di $n$-esimo grado allora contengono entrambi l'elemento $n$ e hanno altri $n-1$ elementi in comune. Gli altri $n-1$ elementi in comune si possono scegliere in $\displaystyle \binom{2021}{n-1}$ modi. E per i restanti $2021-(n-1)=2022-n$ elementi abbiamo $3^{2022-n}$ possibilità come spiegato precedentemente. Avremo quindi $\displaystyle \binom{2021}{n-1} 3^{2022-n}$ coppie di sottoinsiemi gemelle di $n$-esimo grado.
Sommando tutte le possibilità otteniamo:
$$\sum_{k=1}^{2022} \binom{2021}{k-1} 3^{2022-k}$$
Reimpostando la somma sostituendo $k-1 \mapsto k$ otteniamo:
$$\sum_{k=0}^{2021} \binom{2021}{k} 3^{2021-k}$$
Ma questo è esattamente uguale a $(3+1)^{2021}=4^{2021}$ (Per il binomio di Newton o teorema binomiale). Riducendo l'enorme numero $(\text{mod}\,10000)$ si trova che le ultime $4$ cifre che ci interessano sono $1104$, da cui la risposta.
(Lorenzo Bastioni)

Problema 69

Problema 70
Mattysal
Messaggi: 254
Iscritto il: 06 feb 2018, 14:54
Località: Torino
Contatta:

Re: #Proviamoci - Soluzioni commentate OH6

Messaggio da Mattysal »

Problema 71

Problema 72[247]
Disponiamo le 32 squadre in qualunque ordine, e costruiamo gli incontri nel seguente modo: ad ogni turno, le squadre in un posto dispari si scontrano con la successiva. Le squadre perdenti vengono eliminate. Disponiamo le squadre restanti nello stesso ordine in cui si trovano, e ripetiamo finché resta solo una squadra. Ad esempio, se ad un certo punto l'ordine è $(3, 5, 2, 1)$, la squadra $3$ vince contro $5$ e $1$ vince contro $2$, dunque al turno dopo l'ordine sarà $(3, 1)$. Poniamo per semplicità la squadra $S_3$ per prima, e consideriamo i restanti $31!$ ordinamenti possibili. La squadra $S_3$ arriva in semifinale se e solo se non viene eliminata nei 3 turni precedenti, e ciò accade se e solo se nessuna tra le squadre $S_1$ ed $S_2$ è presente tra le prime 8 squadre. Possiamo quindi scegliere le prime 8, inclusa $S_3$, in $\displaystyle\binom{29}{7}$ modi, e le restanti in $24!$ modi.
La probabilità cercata è quindi $\displaystyle\binom{29}{7}\cdot 24!\cdot \frac{1}{31!}=\frac{29 \cdot 28 \cdot 27 \cdot 26 \cdot 25 \cdot 24 \cdot 23}{31 \cdot 30 \cdot 29 \cdot 28 \cdot 27}=\frac{24 \cdot 23}{31 \cdot 30}=\frac{92}{155}$. La risposta è quindi $92+155=247$.
(Valeria Martinelli)

Problema 73 [6068]
Notiamo che
$(p+q+r)(pq+qr+pr)=p^2q+p^2r+pq^2+pr^2+q^2r+qr^2+3pqr=1+3\cdot 2022=6067$.
Dato che $p+q+r$ e $pq+qr+pr$ sono entrambi interi positivi, devono necessariamente dividere entrambi $6067$. $6067$ è un numero primo, quindi o $p+q+r=1$ o $p+q+r=6067$. La risposta è perciò $6067+1=6068$.
Si noti che non ci si deve preoccupare di trovare che esistano $p,q,r$ che rispettano le condizioni trovate, dato che questo è assicurato dal fatto che sono le tre soluzioni dell'equazione $x^3-(p+q+r)x^2+(pq+qr+pr)x-pqr=0$, che ha sempre tre soluzioni nei numeri complessi.
(Federico Borasio)

Problema 74

Problema 75 [78]
Elevando entrambi i termini al quadrato (lecito perché entrambi sicuramente positivi) si ottiene $x^2+y^2-x-y+1/2 \leq 1/4$ riarrangiando $(x-1/2)^2+(y-1/2)^2 \leq 1/4$. Si ha quindi l'equazione di un cerchio centrato in $(1/2,1/2)$ con raggio $1/2$. La probabilità che un punto interno al quadrato definito nel piano cartesiano che ha vertice nell'origine e lato $1$ sia dentro il cerchio è quindi il rapporto tra l'area del cerchio e quella del quadrato: $\pi (1/2)^2 \approx 78%$.
(Filippo Prandina)

Problema 76 [4253]
Iniziamo col dimostrare che $P$ si trova all'esterno di $ABC$.

Detto $K$ il piede dell'altezza uscente da $A$ si ha che, essendo $\widehat{APB}$ congruente a $\widehat{AKB}$ perché entrambi retti, e dunque $AKBP$ ciclico, $P$ appartiene alla circonferenza circoscritta a $AKB$ alla quale non appartengono punti interni a $AKB$; ma $P$ appartiene alla mediana $AM$, la quale è evidentemente contenuta nel triangolo $AKB$ (lo si potrebbe dimostrare con i dati del problema), e quindi $P$ si trova all'esterno di $AKB$ ma anche di $ABC$.

Ora, è noto che in un triangolo di lati di lunghezza $9k$, $8k$, $7k$, con $k$ intero positivo, la mediana relativa alla base di lunghezza $8k$ misura $7k$: questo è facilmente verificabile per via trigonometrica, col teorema della mediana o anche con Pitagora; in ogni caso, $AM\cong AC$ e in particolare $\widehat{AMC}\cong \widehat{ACM}=\gamma$.

Sia $\left\{D\right\}=BC\cap PT$. $BTD\sim BCH$ per il secondo criterio di similitudine in quanto hanno l'angolo in $B$ in comune e $\widehat{BDT}\cong \widehat{BHC}$ perché retti, in particolare $\widehat{ATD}=\gamma$.

$\widehat{BMP}\cong\widehat{AMC}$ perché opposi al vertice ed essendo $BPM$ rettangolo in $P$ e $PD$ l'altezza relativa all'ipotenusa, $BPD=\gamma$. Allora $BPT$ è isoscele sulla base $PT$, in altre parole $T$ è il simmetrico di $P$ rispetto a $BC$; da ciò deriva anche che $BMT\cong BMP$, in particolare che $\widehat{BTM}$ sia retto.

$BMT\sim BCH$ per il secondo criterio di similitudine in quanto hanno l'angolo in $B$ in comune e $\widehat{BTM}\cong \widehat{BHC}$ perché entrambi retti. Ma allora, essendo $M$ punto medio di $BC$, necessariamente $T$ è punto medio di $BH$.

Il rapporto $\frac{[ATB]}{[THC]}$ è uguale al rapporto fra le altezze $\frac{AH}{CH}$ in quanto i due triangoli hanno basi congruenti per quanto appena dimostrato. Esse si possono calcolare con pitagora, dopo aver ricavato $BH=\frac{2A}{AC}$ e l'area attraverso Erone o per via trigonometrica. Sviluppando i calcoli si trova $BH=\frac{24\sqrt{5}}{2}$, $CH=\sqrt{BC^2-BH^2}=\frac{16}{7}$ e $AH=AC-CH=\frac{33}{7}$
Il risultato cercato è $1000\cdot \left(\frac{AH}{CH}\right)^2=1000\cdot\left(\frac{33}{16}\right)^2\approx4253,90$ la cui parte intera è $4253$ che è la soluzione.
(Federico Magnolfi)

Problema 77

Problema 78 [4151]
Cominciamo col dimostrare che $BCPQ$ è un trapezio. Sappiamo che l'ortocentro e il circocentro sono coniugati isogonali: in pratica, le rette $AH$ e $AO$ formano coi lati $AB$ e $AC$ angoli congruenti. Avremo dunque $\widehat{BAH}\cong \widehat{CAO}$, ma anche $\widehat{HAI}\cong \widehat{OAI}$ essendo $AI$ bisettrice perché $I$ incentro di $ABC$.

Riferendoci ora alla circonferenza circoscritta ad $ABC$, che chiamiamo $\Gamma$, abbiamo che $\widehat{FCE}\cong \widehat{FAE}\cong\widehat{EAD}\cong \widehat{ECD}$ dove la prima e la terza uguaglianza sono date dal fatto che gli angoli insistono su uno stesso arco. Facendo attenzione, notiamo che $\widehat{QAP}$ e $\widehat{QCP}$ sono congruenti per l'uguaglianza precedente e insistono sullo stesso segmento $QP$, dunque $AQPC$ è ciclico e chiamiamo $\omega$ la sua circonferenza circoscritta. Essendo $\widehat{ACD}$ retto poiché insiste sul diametro $AD$ di $\Gamma$, $\widehat{AQP}$ sarà anch'esso retto poiché supplementare dell'angolo opposto $\widehat{ACP}$ (ricordando che ora abbiamo fatto rifereimento a $\omega$). $BC$ e $QP$ sono allora due rette perpendicolari alla stessa retta $AQ$, perciò sono parallele e $BCPQ$ è un trapezio.

Per calcolare la sua area abbiamo bisogno dell'altezza $QH$ e della base minore $QP$, dato che la base maggiore ci è nota ed è $BC$.

$\textbf{Prima soluzione (sintetica)}$
Detti $K$ e $J$ rispettivamente i piedi dell'altezza $AH$ e della bisettrice $AI$, analizziamo i triangoli $ABJ$ e $AEC$: essi hanno $\widehat{ABJ}\cong \widehat{AEC}$ perché angoli alla circonferenza che insistono su uno stesso arco; $\widehat{BAJ}\cong \widehat{EAC}$ perché $AI$ bisettrice: quindi sono simili per il secondo criterio di similitudine. Considerando ora i triangoli $AKJ$ e $AQP$: essi hanno: l'angolo in $A$ in comune; $\widehat{AKJ}\cong \widehat{AQP}$ perché retti: quindi sono anch'essi simili per il secondo criterio di similitudine. Impostiamo la proporzione $\frac{QP}{KJ}=\frac{AP}{AJ}$ $(1)$: troveremo $KJ$ e $AJ$ lavorando su $ABC$, mentre $AP$ attraverso il rapporto di similitudine fra i triangoli $ABJ$ e $AEC$. Calcoliamo $AK$ invertendo la formula dell'area di $ABC$ e $BK$ col teorema di Pitagora: $AK=\frac{2A}{BC}=\frac{2\cdot \sqrt{p(p-AB)(p-BC)(p-AC)}}{BC}=120 \quad$ $BK=\sqrt{AB^2-AK^2}=50$.
Applichiamo ora il teorema della bisettrice per trovare $BJ$: $\frac{BJ}{AB}=\frac{BC-BJ}{AC}$ da cui si ricava che $BJ=65$. Dunque $KJ=BJ-BK=15$ e $AJ=\sqrt{AK^2+KJ^2}=15\sqrt{65}$.

Impostiamo il rapporto di similitudine fra $ABJ$ e $AEC$: $\frac{AP}{AB}=\frac{AJ}{AC}$ da cui si ricava che $AP=\frac{150\sqrt{65}}{7}$ e per la proporzione $(1)$ $QP=\frac{AP\cdot KJ}{AJ}=\frac{150}{7}$. A questo punto si trova $AQ$ con pitagora e $QK$ per differenza: $AQ=\sqrt{AP^2-QP^2}=\frac{1200}{7} \quad QK=AQ-AK=\frac{360}{7}$
L'area del trapezio $BCPQ$ è dunque $$A=\frac{(140+\frac{150}{7})\cdot\frac{360}{7}}{2}\approx 4151,02$$.

$\textbf{Seconda soluzione (trigonometria)}$
Ponendo $\widehat{ABC}=\beta$, $\cos\beta=\frac{AB^2+BC^2-AC^2}{2AB\cdot BC}=\frac{5}{13}$ (teorema del coseno), $\sin\beta=\sqrt{1-\cos^2\beta}=\frac{12}{13}$ $\widehat{BAC}=\alpha$, $\widehat{CAI}=\frac{\alpha}{2}$ perché $AI$ bisettrice, $\cos\alpha=\frac{AB^2+AC^2-BC^2}{2AB\cdot AC}=\frac{33}{65}$ (teorema del coseno), $\cos\frac{\alpha}{2}=\sqrt{\frac{1+\cos\alpha}{2}}=\frac{7}{\sqrt{65}}$.

Adesso abbiamo abbastanza dati per calcolare $AP=\frac{AC}{\cos\frac{\alpha}{2}}=\frac{150\sqrt{65}}{7}$.
Ponendo $\widehat{HAI}=\widehat{OAI}=\theta$ si ha che, essendo $\widehat{BAH}=\frac{\pi}{2}-\beta$, $\theta=\frac{\alpha}{2}+\beta-\frac{\pi}{2}$; $\cos\theta=\sin\frac{\alpha}{2}\cos\beta+\cos\frac{\alpha}{2}\sin\beta=\frac{8}{\sqrt{65}}$

$AQ=AP\cos\theta=\frac{1200}{7}\quad QP=\sqrt{AP^2-AQ^2}=\frac{150}{7}$. Ci manca l'altezza $QK$ dove $K$ è il piede dell'altezza uscente da $A$. $AK=AB\sin\beta=120\qquad QK=AQ-AK=\frac{360}{7}$. A questo punto l'area si trova allo stesso modo della soluzione per via sintetica.
(Federico Magnolfi)

Problema 79 [2011]
Si ha che $p^{(2)}(x)$ ha lo stesso grado di $p^{(4)}(x)$ questo significa che $p(x)$ è di grado $1$ o grado $0$. Ma non può essere di grado $0$ perché non è costante. Allora è del tipo $p(x)=mx+q$. La funzione $p(x)$ è chiaramente iniettiva ed essendo $p^{(2)}(x)=p^{(2022)}(x)$ per ipotesi, si ha necessariamente che $p(x)=p^{(2021)}(x)$. Ma quindi $p(2021)=0$ e $p(p(2021))=p(0)=2021$. Da qui si ricava facilmente che il polinomio è $p(x)=-x+2021$ e quindi $p(10)=2011$.
(Filippo Prandina)

Problema 80


Problema 81

Problema 82 [4]
$45=9\cdot 5$ quindi scomponiamo il modulo in modulo $9$ e modulo $5$. Modulo $5$ è semplicemente $4$, in quanto l'ultima cifra è proprio $4$. Definiamo $\overline{a_1a_2a_3...a_k}$ la scrittura decimale in successione di $a_1,a_2,...,a_k$, per esempio $\overline{4^2 5^2}=1625$. Inoltre, chiamato $N$ il nostro numero, $N=\overline{1^22^23^24^2...2022^2}$. $\overline{a_1a_2a_3...a_k}\equiv a_1+a_2+a_3+...+a_k\,(\mathrm{mod}\,9)$, dimostrarlo è facile. I residui quadratici di $9$ sono $0,1,4,7$ mentre la somma dei quadrati da $1$ a $9\,\mathrm{mod}\,9$ è $24$, ($-3$). Quindi ogni $9$ numeri vi è un $-3$ da addizionare, quindi è $(-3)\cdot 224$ (che sarebbe il numero di multipli di $9$ minori di $2022$). $-672$, a questo devi aggiungere i numeri da $2017$ a $2022$, $-672+1+4+7+7=-653\equiv -14\equiv 4$.
Siccome $N$ è congruo a $4$ sia $\mathrm{mod}\,5$ che $\mathrm{mod}\,9$, sarà congruo a $4$ anche $\mathrm{mod}\,45$.
(Alessandro Avellino)

Problema 83 [21]
Sia $H$ il piede dell'altezza uscente da $A$; $AY=AH-HY$. L'altezza $AH$ si può calcolare, ad esempio, invertendo la formula dell'area $A=\frac{b\cdot h}{2}$ dopo aver trovato quest'ultima con la formula di Erone, oppure trovando il coseno dell'angolo in $B$ col teorema del coseno, quindi il suo seno e applicando il primo teorema dei triangoli rettangoli a $ABH$. Eseguendo i calcoli, si trova che essa vale $AH=3\sqrt{5}$. $BH=\sqrt{AB^2-AH^2}=2$.

Detto $M$ il punto medio di$BC$ si ha che $HM=HX=2$ e quindi esso è punto medio anche di $HX$; ma $OM\perp BC$ in quanto in un triangolo isoscele la mediana è anche altezza: allora, essendo $HYX$ e $MOX$ simili per il secondo criterio di similitudine, e avendo rapporto di similitudine $2$, $HY=2HO$

Attraverso il teorema della corda, oppure attraverso la formula nota, possiamo calcolare il raggio della circonferenza circoscritta ad $ABC$, $R=\frac{BC}{2\sin\alpha}=\frac{abc}{4A}=\frac{21}{2\sqrt{5}}$.
Ora possiamo trovare $HY=2HO=2\sqrt{R^2-MC^2}=\frac{11\sqrt{5}}{5}$ e infine $AY=AH-HY=3\sqrt{5}-\frac{11\sqrt{5}}{5}=\frac{4\sqrt{5}}{5}$.

Stiamo cercando $AY^2=\frac{16}{5}$ e la risposta è infine $16+5=21$.
(Alessandro Lombardo & Federico Magnolfi)

Problema 84

Problema 85 [1136]

Iniziamo col calcolarci alcuni oggetti che in futuro potranno esserci utili.

Perimetro: 2p=14+16+18=48

l'Area: usando la formula di Erone, A=$\sqrt{p(p-a)(p-b)(p-c)}=\sqrt{24\cdot10\cdot8\cdot6}=48\sqrt{5}$

Raggio circonferenza circoscritta: $R=\frac{abc}{4A}=\frac{4032}{192\sqrt{5}}=\frac{21}{\sqrt{5}}=\frac{21\sqrt{5}}{5}$

Altezza relativa a BC: $AD=2A/BC=6\sqrt{5}$

Altezza relativa a AB, che incontra AB in F: $CF=2A/AB=\frac{48\sqrt{5}}{7}$

Detto L il punto di incontro tra BC e la perpendicolare passante per O, L è il punto medio di BC. Per cui CL=8, OC= R, $OL=\sqrt{\frac{441}{5}-\frac{320}{5}}=\sqrt{\frac{121}{5}}=\frac{11\sqrt{5}}{5}$

$CD=\sqrt{AC^2-AD^2}=\sqrt{144}=12$

$AF=\sqrt{AC^2-CF^2}=\frac{66}{7}$

LD=CD-CL=12-8=4

adesso calcoliamo OD:

$OD=\sqrt{OL^2+LD^2}=\sqrt{\frac{201}{5}}$

Infine calcoliamo AH, dove H è l'ortocentro. per calcolare AH notiamo che, AHF è simile a ABD, per cui $\frac{AH}{AB}=\frac{AF}{AD} \Rightarrow AH=\frac{AF \cdot AB}{AD}=\frac{66 \cdot 14}{42\sqrt{5}}=\frac{22\sqrt{5}}{5}$.

Adesso che abbiamo tutto ciò che ci serve proseguiamo. O è il punto di Miquel relativo ad XYD. questo vuol dire che O appartiene alla circonferenza passante per X,Y,A. Detto questo, abbiamo 2 quadrilateri ciclici, per definizione, BDOX e CDOY. A questi 2 aggiungiamo il quadrilatero AYOX, anch'esso ciclico.

1° Claim: XYD è simile ad ABC.
è sufficiente un po' di angle chasing:

ricordiamo che OA, OB e OC sono i raggi di (ABC).
Di conseguenza OBA=OAB, OBC=OCB e OCA=OAC. Inoltre, per via della ciclicità dei 3 quadrilateri sopra elencati,
OAB=OBA=OBX=ODX, OAC=OCA=OCY=ODY, da questo segue che YDX=ODY+ODX=CAO+OAB=BAC.

Analogamente DYX=CBA e DXY=BCA. Per cui i 2 triangoli sono simili

2° Claim: O è l'ortocentro di XYD.
Anche qui è sufficiente un po' di Angle chasing:

DYX+YDO=(DYO+OYX)+OCY=DCO+OAX+OCA=(DCO+OCA)+OAX=ACB+OAX=ACB+(OAH+HAX)=ACB+(OAH+HAB)=
ACB+(OAH+DAB)=ACB+(OAH+OAC)=ACB+DAC=90.
Ne consegue che OD è perpendicolare ad XY. Analogamente si ottiene che , XO e YO sono perpendicolari, rispettivamente, a DY e DX. Per cui O è l'ortocentro.

Di conseguenza, detto K il rapporto di similitudine tra ABC e XYD, $K=\frac{AH}{OD}=\frac{22}{201}$, il rapporto vale anche per i raggi r e R rispettivamente di XYD, e di ABC, $K=\frac{R}{r} \Rightarrow r=\frac{R}{K}=\frac{21\sqrt{1005}}{110}$
(Alessandro Avellino)

Problema 86 [1483]
Per prima cosa notiamo che, dato che $\frac{x_1+x_2}{x_3+x_4}$ è intero, abbiamo che $x_1+x_2 \ge x_3+x_4$, e similmente $x_3+x_4 \ge x_5+x_6 \ge x_1+x_2$.
Dunque $x_1+x_2= x_3+x_4=x_5+x_6$. Analogamente $x_2+x_3=x_4+x_5=x_6+x_1$.

Sia ora $S=x_1+x_2= x_3+x_4=x_5+x_6$ e $T=x_2+x_3= x_4+x_5=x_6+x_1$.
Abbiamo quindi che $\sum x_i = 3S=3T$, quindi $S=T$, ossia $x_1+x_2=x_2+x_3$, da cui $x_1=x_3$. In modo analogo otteniamo $x_1=x_3=x_5$ e $x_2=x_4=x_6$.

Possiamo quindi scrivere la sestupla come $(a, b, a, b, a, b)$, con $a, b$ interi positivi. Si verifica facilmente che ogni sestupla siffatta rispetta le condizioni del testo, difatti la somma di due termini consecutivi è costante, dunque le frazioni sono tutte uguali a $1$ e quindi intere.
Dunque, fissati $a, b$, c'è una e una sola sestupla valida.

Consideriamo ora due casi:

$\bullet$ $M$ non è multiplo di $3$. Allora, dato che $M=3S$, non ci sono sestuple valide, e quindi $f(M)=0$, che è un multiplo di $5$. In questo caso abbiamo quindi $2022-\frac{2022}{3}=1348$ possibili valori di $M$.

$\bullet$ $M=3k$, per qualche $k$ intero positivo. Dato che $k=a+b$, per ogni valore di $k$, ci sono $k-1$ sestuple valide, poiché abbiamo $k-1$ possibili valori di $a$, dunque $f(M)=k-1$. Vogliamo quindi che $k-1$ sia multiplo di $5$. Se $k=1$, non ci sono sestuple valide in quanto $a+b>1$, altrimenti, potendo scegliere $k$ nell'intervallo $[2, 1348]$, ci sono ${\lfloor \frac{1347}{5} \rfloor}=134$ valori accettabili.

In totale abbiamo quindi $1348+1+134=1483$ possibili valori di $M$.
(Valeria Martinelli)

Problema 87

Problema 88 [6418]
Osserviamo che, prese $3$ cifre distinte $a, b, c$ dall'insieme $X=\{1, 2, 3, 4, 5, 6, 7, 8, ,9 \}$, ci sono esattamente $6$ numeri compresi tra $100$ e $999$ che si scrivono con tutte e sole le cifre $a, b, c$, siano questi $x_1, ..., x_6$. Abbiamo che la loro somma $x_1+...+x_6$ vale $222a+222b+222c$, quindi $\sum f(x_i)=\frac{222(a+b+c)}{a+b+c}=222$.
Ci sono $\displaystyle\binom{9}{3}=84$ modi di scegliere $a, b, c$, quindi la somma in questo caso vale $222\cdot 84=18648$

Dobbiamo ora distinguere i casi in cui $a, b, c$ contengono uno o più zeri o cifre ripetute.
$\bullet$ $a=b=c$: per ognuna delle $9$ possibili scelte di $a, b, c$, c'è uno e un solo numero $x=111a$, e $f(x)=\frac{111a}{3a}=37$. Dunque la somma in questo caso vale $37 \cdot 9=333$

$\bullet$ $a=b \neq c$, $a, b, c \neq 0$: abbiamo $9\cdot 8=72$ possibili scelte di $a, b, c$, e per ogni terna abbiamo $3$ numeri la cui somma vale $222a+111c$, per cui $\sum f(x_i)=\frac{222a+111c}{2a+c}=111$. In tutto $72 \cdot 111=7992$

$\bullet$ $b=c=0$: $9$ possibili scelte di $a$, e per ogni $x=100a$ che rispetta queste condizioni $f(x)=100$. La somma in questo caso vale $100 \cdot 9=900$

$\bullet$ $a=b$, $c=0$; $9$ possibili scelte per $a, b$, per ognuna $\sum f(x_i)=\frac{211a}{2a}=\frac{211}{2}$. Quindi in tutto $9\cdot \frac{211}{2}=949,5$

$\bullet$ $a\neq b$, $c=0$: $\displaystyle\binom{9}{2}=36$ possibilità per $a, b$; per ognuna $\sum f(x_i)=\frac{211a+211b}{a+b}=211$. In tutto $36 \cdot 211=7596$

La somma richiesta è quindi uguale a $18648+333+7992+900+949,5+7596=36418,5$. Ci interessano le ultime quattro cifre della parte intera.
(Valeria Martinelli)

Problema 89 [134]
Sia $2^k$ la massima potenza di $2$ nell'intervallo $\{1, ..., n\}$. Si osserva facilmente che $f(n)$ non può mai essere maggiore di $f(2^k)$, pertanto $f(n)$ è massima quando $n=2^k$, e $g(2^k)=1$.

Dato che $5>4=2^2$ e $9=3^2>2^3=8$, se un intero contiene nella sua fattorizzazione primi diversi da $2$ e $3$, o se $3$ compare con un esponente maggiore o uguale a $2$, $g(n)=g(n-1)$ in quanto $n$ ha sicuramente meno fattori di $2^k$.
Inoltre, se $n=2^{k-1}\cdot 3$, $g(n)=g(n-1)+1$, in quanto n contiene lo stesso numero di fattori di $2^k$.

Dunque, se $2^{k-1}\cdot 3\le n <2^{k}$, $g(n)=2$, altrimenti $g(n)=1$.

Possiamo quindi trovare tutti gli interi minori o uguali a $99$ per cui $g(n)=2$, che sono i valori compresi tra i tripli delle potenze di $2$ e la potenza di $2$ successiva.

Dunque la somma richiesta è uguale a $99+(4-3)+(8-6)+(16-12)+(32-24)+(64-48)+(100-96)=99+1+2+4+8+16+4=99+31+4=134$.
(Valeria Martinelli)

Problema 90 [8784]
Chiamiamo $f(n)=x^n+\frac{1}{x^n}$, il problema ci chiede di calcolare $|f(1)\cdot f(2)\cdot\dots\cdot f(2022)|$.
Esplorando un po’ la situazione si può notare che $f(1)=1,f(2)=-1,f(3)=-2,f(4)=-1,f(5)=1$ e $f(6)=2$ e che questi valori potrebbero formare un ciclo che si ripete ogni $6$.
Dimostriamolo:
consideriamo $f(n) \cdot f(1)= \left( x^n+\frac{1}{x^n} \right) \cdot \left(x+ \frac{1}{x} \right) = x^{n+1} + x^{n-1} + \frac{1}{x^{n-1}} + \frac{1}{x^{n+1}} = f(n+1) + f(n-1)$, essendo $f(1)=1$ otteniamo che $f(n) \cdot f(1) =f(n)= f(n+1) + f(n-1)$ da cui $f(n+1)=f(n)-f(n-1)$. L’ultima relazione ci indica che $f(n+1)$ dipende unicamente da $f(n)$ e $f(n-1)$ ovvero se troviamo un ciclo quest’ultimo si ripeterà. Controllando si può vedere che $f(7)=1$ e $f(8)=-1$ quindi il ciclo si ripeterà. Analizzando il nostro ciclo si nota facilmente che gli unici fattori che contribuiranno nel nostro prodotto saranno quelli con $n$ multiplo di $3$, che in valore assoluto valgono $2$. Il nostro prodotto sarà quindi del tipo $2^k$ dove $k$ è il numero di multipli di $3$ tra $1$ e $2022$ ovvero $674$ $(\frac{2022}{3}=674)$. Il problema si è trasformato in un quesito di teoria dei numeri poiché dobbiamo dare come risposta le ultime $4$ cifre di $2^{674}$ quindi calcolare $2^{674}$ $(\mathrm{mod}\,10000)$. Riducendo con un po’ di conti l’enorme numero si trova $8784$.
(Lorenzo Bastioni)

Problema 91[4929]
Osserviamo che $g(n)=3^5 \cdot \frac{n}{v_{3}(n)}$, dunque la somma richiesta è uguale a $\sum 3^5 \cdot \frac{n}{v_{3}(n)}$. Definiamo quindi $h(n)=\frac{n}{v_{3}(n)}$ e calcoliamo separatamente le somme di $h(n)$ in base a $v_{3}(n)$.

$\bullet$ $v_{3}(n)=5$ \rightarrow $h(n)=1$: $n \in \{243 \cdot 1\}$, \rightarrow $\sum h(n)=1$

$\bullet$ $v_{3}(n)=4$: $n \in \{81 \cdot 2, ..., 81 \cdot 5\}$ esclusi i multipli di $243$, $\sum h(n)=(2+...+5)-1\cdot 3=14-3=11$

$\bullet$ $v_{3}(n)=3$: $n \in \{27 \cdot 5, ..., 27 \cdot 15\}$ esclusi i multipli di $81$, $\sum h(n)=(5+...+15)-14\cdot 3=110-42=68$

$\bullet$ $v_{3}(n)=2$: $n \in \{9 \cdot 15, ..., 9 \cdot 46\}$ esclusi i multipli di $27$, $\sum h(n)=(15+...+46)-110\cdot 3=976-330=646$

$\bullet$ $v_{3}(n)=1$: $n \in \{3 \cdot 45, ..., 3 \cdot 140\}$ esclusi i multipli di $9$, $\sum h(n)=(45+...+140)-976\cdot 3=8880-2928=5952$

$\bullet$ $v_{3}(n)=0$: $n \in \{1 \cdot 135, ..., 1 \cdot 420\}$ esclusi i multipli di $3$, $\sum h(n)=(135+...+420)-8880\cdot 3=79365-26640=52725$

Quindi, $\sum h(n)=52725+5952+646+68+11+1=59403$, da cui $\sum f(n)=3^5 \cdot \sum h(n)=14434929$. Ci interessano le ultime 4 cifre
(Valeria Martinelli)

Problema 92

Problema 93 [279]
$x$,$y$ e $z$ sono interscambiabili. Per cui è possibile, per facilitarci nella risoluzione, porre $x \geq y \geq z$.

Adesso andiamo in ordine:\\
abbiamo

$x^2<x^2+2y+z \leq x^2+3x < x^2+4x+4=(x+2)^2$

e siccome lo vogliamo come quadrato

$x^2+2y+z=(x+1)^2 \Rightarrow 2y+z=2x+1 \Rightarrow x=\frac{2y+z-1}{2}$.

Ora usiamo la seconda equazione con le nuove sostituzioni.\\
$y^2<y^2+2z+x=y^2+2z+\frac{2y+z-1}{2}=y^2+y+2.5(z)-0.5<y^2+y+3y+4=(y+2)^2$.

Per lo stesso motivo di prima:

$y^2+y+2.5(z)-0.5=(y+1)^2 \Rightarrow 5z-3=2y \Rightarrow y=\frac{5z-3}{2}$\\
Ora è il momento di usare l'ultima equazione, anche qui sostituendo $y$ e $x$:

$z^2<z^2+2x+y=z^2+2y+z-1+y=z^2+z+3(\frac{5z-3}{2})-1=z^2+8.5z-4.5$.

Dopo vari tentativi si nota che gli unici quadrati possibili per questa quantità sono $(x+1)^2$ e $(x+4)^2$.\\

Infatti si avrà $z^2+8.5z-4.5=z^2+8z+16 \Rightarrow z=43$, trovandoti poi $y=106$ e $x=127$ e $z^2+8.5z-4.5=z^2+2z+1$ trovando quindi $x=z=y=1$.
(Alessandro Avellino)

Problema 94 [6460]
Possiamo riscrivere il testo del problema nel seguente modo:
$$\sum_{n=1}^{2020} \lfloor \frac{\sqrt{n+2}}{\sqrt{n+1} - \sqrt{n}} \rfloor$$
Andiamo ad analizzare il nostro addendo generale privo della funzione floor:
$ \frac{\sqrt{n+2}}{\sqrt{n+1} - \sqrt{n}} = \frac{\sqrt{n+2}}{\sqrt{n+1} - \sqrt{n}} \cdot \frac{\sqrt{n+1} + \sqrt{n}}{\sqrt{n+1} + \sqrt{n}} = \sqrt{(n+2) \cdot (n+1) } + \sqrt{ (n+2) \cdot n } = \sqrt{n^2 + 3n+ 2} + \sqrt{n^2 +2n} $.
Esplorando un po' la situazione ci si potrebbe convincere che $2n+2 \leq \sqrt{n^2 + 3n+ 2} + \sqrt{n^2 +2n} < 2n+3$ (con $n$ nell'intervallo che interessa a noi ovvero $[1,2020]$).
Dimostriamolo:
La seconda disuguaglianza è molto semplice infatti $\sqrt{n^2 + 3n+ 2} + \sqrt{n^2 +2n} = \sqrt{(n+2)^2 -n-2} + \sqrt{(n+1)^2 - 1} < \sqrt{(n+2)^2} + \sqrt{(n+1)^2} = n+2 +n+1 = 2n+3$ quindi $\sqrt{n^2 + 3n+ 2} + \sqrt{n^2 +2n} < 2n+3$.
La prima disuguaglianza è leggermente più fastidiosa:
vogliamo $2n+2 \leq \sqrt{n^2 + 3n+ 2} + \sqrt{n^2 +2n}$ che è molto lungo da risolvere a mano dunque proviamo a scomporre in maniera intelligente il problema:
ad esempio, se dimostrassimo che $\sqrt{n^2 + 3n+ 2} \geq n + \frac{4}{3} \wedge \sqrt{n^2 +2n} \geq n+\frac{2}{3}$ avremmo finito poiché sommando le disuguaglianze si otterrebbe la tesi (la veridicità delle due disuguaglianze implica la tesi ma non viceversa).
Le due disuguaglianze sono due semplici disequazioni irrazionali e non ci dovremmo preoccupare di condizioni di esistenza o simili poiché nel nostro intervallo è tutto positivo:
1)
$ \sqrt{n^2 + 3n+ 2} \geq n + \frac{4}{3} \rightarrow n^2+3n+2 \geq (n + \frac{4}{3})^2 \rightarrow n \geq - \frac{2}{9}$, nel nostro intervallo la disuguaglianza vale.
2)
$ \sqrt{n^2 +2n} \geq n+\frac{2}{3} \rightarrow n^2+2n \geq (n+\frac{2}{3})^2 \rightarrow n \geq \frac{2}{3} $, nel nostro intervallo la disuguaglianza vale.
La tesi è dunque dimostrata, $2n+2 \leq \sqrt{n^2 + 3n+ 2} + \sqrt{n^2 +2n} < 2n+3$. Ciò implica che $\lfloor \frac{\sqrt{n+2}}{\sqrt{n+1} - \sqrt{n}} \rfloor = 2n+2$ (nel nostro intervallo) e possiamo riscrivere la nostra somma:
$$\sum_{n=1}^{2020} 2n+2 = \left( 2 \sum_{n=1}^{2020} n \right) + 2 \cdot 2020 = 2020 \cdot 2021 + 4040 = 4086460$$
(La seconda uguaglianza si ha con la formula di Gauss per la somma dei primi $k$ numeri naturali) Prendendo le ultime $4$ cifre del risultato abbiamo la soluzione: $6460$.
(Lorenzo Bastioni)

Problema 95 [21]
Mettiamo in un piano cartesiano i punti [math]. Si ha che [math] è il coffieciente angolare della retta per i due punti [math]. Quindi una sequenza è non genovese se tutti i coefficienti angolari sono non negativi; ovvero se e solo se essa è debolmente crescente. Supponiamo di avere una sequenza debolmente crescente [math]. Vi è una bigezione fra le sequenze di questo tipo e quelle del tipo [math], che sono tutte le sequenze strettamente crescenti con elementi positivi minori di [math]. Esse a loro volta corrispondono con i modi di scegliere [math] elementi distinti fra [math]. Il numero totale di sequenze (genovesi e non) è infine [math]; quindi il numero di quelle genovesi è [math] che è congruo a [math] modulo [math], essendo che [math] modulo [math].
(Federico Volpe)

Problema 96 [2029]
I numeri curiosi sono esattamente i liberi dai quadrati.\\
($\Leftarrow$) Se $n$ è un numero libero da quadrati allora $\tau(n)$ è una potenza di due, inoltre ogni divisore $d$ di $n$ è un numero libero da quadrati, quindi anche $\tau(d)$ è una potenza di $2$ e dato che $\tau(d) \le \tau(n)$ abbiamo che $\tau(d)|\tau(n)$.\\
($\Rightarrow$) Supponiamo che $n = p_1^{a_1}p_2^{a_2}\cdots p_m^{a_m}$ sia un numero libero da quadrati. Allora abbiamo
\begin{align*}
&\tau(n/p_i)|\tau(n)\\
\Rightarrow &(a_1+1)\cdots(a_i)\cdots(a_m+1)|(a_1+1)\cdots(a_i+1)\cdots(a_m+1)\\
\Rightarrow &a_i | a_i+1 \Rightarrow a_i = 1.
\end{align*}
Dunque $n$ è libero da quadrati.\\
\\
Inoltre notiamo che $f(a,b)$ è semplicemente $MCD(a,b)$. A questo punto scriviamo
$$S(n) = \sum_{j=1}^n f(j,n) = \sum_{j=1}^n MCD(j,n),$$
dimostreremo che la funzione $S$ è moltiplicativa, cioè per ogni $m,n \in \mathbb{Z}^+$ coprimi $S(m)S(n) = S(mn)$. Infatti, segue dal teorema del resto cinese, possiamo rimpiazzare $j \text{ mod }\,mn$ con due numeri $j_m \text{ mod }\,m$ e $j_n \text{ mod }n$ in modo che:
\begin{align*}
S(mn) &= \sum_{j=1}^{mn} MCD(j,mn) = \sum_{j=1}^{mn} MCD(j,m)MCD(j,n)\\
&= \sum_{j_m=1}^{m} \sum_{j_n =1}^{n} MCD(j_m,m)MCD(j_n,n)\\
&= \left(\sum_{j_m=1}^m MCD(j_m,m)\right)\left(\sum_{j_n=1}^n MCD(j_n,n)\right) = S(m)S(n).
\end{align*}
Ma quindi dato che $C$ è un prodotto tra primi ed è facile verificare che $S(p) > 1$ per ogni primo $p$, abbiamo che $C$ stesso deve essere primo. Un po' di brute-force dà la risposta.
(Michele Tomasi)

Problema 97 [118]

Problema 98

Problema 99

Problema 100 [1339]
Premetto innanzitutto che questo geometrico è più tecnico degli altri ed è necessario avere qualche prerequisito sulle simmediane e il cerchio di Feuerbach. Tutti i fatti noti di cui farò uso si trovano sul libro di Evan Chen ma sono certo che si possano trovare anche girovagando su internet.

La soluzione di questo problema si può suddividere in tre claim.

Primo claim: La tangente di $\Gamma$ in $N_a$, la retta $BC$ e la tangente di $\Gamma$ in $A$ concorrono nel punto $X_a$.\\
Qua c'è solo da usare qualche fatto noto delle simmediane. Infatti $\triangle BAN_a \sim \triangle CAM_a \Rightarrow \angle BAN_a= \angle CAM_a \Rightarrow AN_a$ è la $A-simmediana$ di $\triangle ABC$. Dopodiché basta usare che $CB$ è la $C-simmediana$ di $\triangle ACN_a$ e quindi le tangenti in $A$ e in $N_a$ concorrono con essa.

Secondo claim:
$Y_a$ e simili stanno sulla circonferenza di Feuerbach, ovvero $\gamma$ e $\omega$ coincidono.
Per essere capaci di dimostrarlo però è necessario notare qualcosa di più subdolo.
Sia $H$ l'ortocentro di $\triangle ABC$, $V$ il punto medio di $AH$ e $Q$ l'intersezione tra la retta $AH$ e $\Gamma$. Siano inoltre $R$ e $r$ i raggi di $\Gamma$ e di $\gamma$.
Lemma: $V$ è L'ortocentro di $\triangle AM_{a}X_{a}$.
Proof: Dimostriamo innanzitutto che la retta $VM_a$ è perpendicolare a $AX_a$. CIò è equivalente a dimostrare che $\angle X_{a}M_{a}V = 90^{\circ}- \angle AX_{a}M_a=\angle X_{a}AP_{a} \Leftrightarrow \angle P_{a}M_{a}V=\angle X_{a}AQ \Leftrightarrow \frac{VP_{a}}{AQ}=\frac{r}{R}=\frac{1}{2}$.
Poiché $Q$ è il simmetrico di $H$ su $BC$ (si dimostra con angle chasing), $HQ+HA=2HP_{a}+2HV \Rightarrow AQ=2VP_{a} \Rightarrow \frac{VP_{a}}{AQ}=\frac{1}{2}$.
$AV \perp X_{a}M_{a}$ e $M_{a}V \perp AX_{a} \Rightarrow V$ ortocentro di $\triangle AM_{a}X_{a}$.
A questo punto per dimostrare il claim iniziale basta dire che, poichè $V$ è ortocentro, $X_{a},V,Y_{a}$ sono allineati e quindi $\angle VY_{A}M_{a}=90^{\circ}=\angle VP_{a}M_{a} \Rightarrow VY_{a}M_{a}P_{a}$ ciclico $\Rightarrow Y_{a}$ sta su $\gamma$.
Similarmente si può dimostrare che anche $Y_{b}$ e $Y_{c}$ stanno su $\gamma$.

Terzo claim: le rette $P_{a}P_{b}, DE, CH$ concorrono in $F$.
$\angle BHC=180^{\circ}- \angle BAC \Rightarrow H$ sta su $\Gamma'$. Ora basta osservare che $P_{a}P_{b}, DE, CH$ sono gli assi radicali di $\gamma, \Gamma', (P_{a}HP_{b}C)$.
Similarmente si dimostra che $P_{a}P_{c}, DE, BH$ concorrono in $G$.

Ora che abbiamo delle informazioni "decenti" sui punti $F$ e $G$ possiamo fare del segment chasing per trovare i valori di $P_{a}F$ e $P_{a}G$. $\triangle P_{a}P_{b}P_{c}$ è il famoso triangolo ortico e $P_{c}G$ e $P_{b}F$ sono le bisettrici, quindi l'idea sarebbe quella di trovare i tre lati e poi usare il teorema delle bisettrici.
$BC^{2}-P_{b}C^{2}=AB^{2}-(AC-P_{b}C)^{2} \Rightarrow P_{b}C=\frac{84}{5}$.
$\triangle BCA \sim \triangle P_{b}CP_{a} \Rightarrow \frac{P_{a}P_{b}}{P_{b}C}=\frac{AB}{BC} \Rightarrow P_{a}P_{b}=\frac{78}{5}$.
Similarmente si trovano $P_{b}P_{c}=\frac{924}{65}$ e $P_{a}P_{c}=\frac{150}{13}$
$\frac{P_{a}F}{P_{a}P_{c}-P_{a}F}=\frac{P_{b}P_{a}}{P_{b}P_{c}} \Rightarrow P_{a}F=\frac{1950}{323}$
$\frac{P_{a}G}{P_{a}P_{b}-P_{a}G}=\frac{P_{c}P_{a}}{P_{c}P_{b}} \Rightarrow P_{a}G=\frac{650}{93}$
$P_{a}F+P_{a}G=\frac{391300}{30039}$. La risposta è quindi $1339$.
(Denis Tusca)
Mattia Zunino
Messaggi: 2
Iscritto il: 19 apr 2022, 15:07

Re: #Proviamoci - Soluzioni commentate OH6

Messaggio da Mattia Zunino »

Mattysal ha scritto: 22 apr 2022, 18:41 Problema 1 [3]
$n$ il numero di commensali e $d$ la distanza tra loro, i commensali sono i punti ${A_1, A_2,..., A_n}$ mettiamo $A_1$ in un punto qualsiasi del piano. $A_2$ potrà stare ovunque nella circonferenza di raggio $d$ e centro $A_1$. Il terzo punto ha solo $2$ posizioni in cui poter stare, che sono le intersezioni delle $2$ circonferenze di centro $A_1$ e $A_2$, entrambe di raggio $d$. Posizionato il terzo punto e tracciata la terza circonferenza noteremo che non ci sarà nessun punto in comune a tutte e $3$ le circonferenze. Di conseguenza non ci può essere un quarto punto che rispetta le condizioni.
(Alessandro Avellino)

Problema 2 [6]
Si tratta di disporre tre diversi ingredienti, che si può fare semplicemente in $3!=6$ modi diversi. La risposta è quindi $6$.
(Matteo Salicandro)

Problema 3 [228]
Il massimo numero ottenibile lanciando gli $n$ dadi è $n\cdot k$, il minimo è $n\cdot 1=n$. Pertanto $n\cdot k-n=168$, cioè $n(k-1)=168$. Questo vuol dire che $n$ è un qualsiasi divisore positivo di $168$, esclusi $168$ e $84$ visto che $k \geq 4$. $168=2^3\cdot 3\cdot 7$, quindi la somma dei suoi divisori è $(2^4-1)(3+1)(7+1)=480$. La somma dei possibili valori di $n$ è pertanto $480-168-84=228$.
(Federico Borasio)

Problema 4

Problema 5 [7]
Banalmente (tanto la calcolatrice era lecita) $5^1,=5, 5^2=25, 5^3=125, 5^4=625, 5^5=3125, 5^6=15625, 5^7=78125, 5^8=390625, 5^9=1953125, 5^{10}=9765625, 5^{11}=48828125, 5^{12}=244140625, 5^{13}=1220703125, 5^{14}=6103515625$. Non è necessario andare oltre, perché per il principio dei cassetti, un numero con 11 cifre contiene necessariamente almeno due cifre identiche. Si vede facilmente, confrontando i numeri scritti, che la risposta è $7$.
(Matteo Salicandro)

Problema 6 [36]
Un rettangolo con le diagonali perpendicolari è un quadrato. L'area è quindi $6^2=36$.
(Filippo Prandina)

Problema 7 [18]
La successione è definita, per $n \geq 1$, come
$\begin{cases}
x_{n+2}=x_{n+1}+x_n\\
x_6=76\\
x_5=47
\end{cases}$
Da qui ricaviamo che $x_n=x_{n+2}-x_{n+1}$, pertanto $x_4=76-49=29$ e $x_3=47-29=18$, che è il terzo termine della successione.
(Federico Borasio)

Problema 8 [2022]
Supponiamo che ci siano $2022$ fisici. Tutti dicono la verità, cioè che non sono matematici: non c'è alcuna contraddizione. $2022$ è anche il numero di abitanti dell'isola, quindi è il massimo numero di fisici.

Problema 9 [8]
$\frac{4n}{32}=k$ con $k\in N\geq 0$. Quindi $n=8k$. Il minimo si ha quando $k=1$ e $n=8$.
(Filippo Prandina)

Problema 10[8]
Sono possibili due casi, in base a qual è la circonferenza con raggio minore. Tuttavia, r deve essere massimo, quindi consideriamo il caso in cui r$>$5. L'area della corona circolare sarà data da
$\pi{r}^{2}-{5}^{2}\pi=\pi({r}^{2}-25)=16\pi.$ Per cui ${r}^{2}-25=16$, ovvero $r=\sqrt{41}\approx{6.403}$, la cui parte intera è 6.
(Daniele Prisco)

Problema 11 [9999]
Ci sono più soluzioni in quanto i primi due termini possono essere $1,3$, oppure $2,2$ oppure $3,1$. La risposta è quindi $9999$.
(Matteo Salicandro)

Problema 12[1261]
Affinché una parola sia impronunciabile per il nostro Loenzo, bisogna che contenga almeno una R. Se c'è una sola R nella parola, questa potrà essere la prima, la seconda o la terza lettera, e in ognuno di questi casi ci saranno 20*20=400 parole possibili, dato che nei restanti due 'posti' vanno inserite le altre 20 lettere dell'alfabeto, che possono essere ripetute. Se nella parola ci sono due R, esse si possono disporre in 3 modi, e per ognuno si potranno formare 20 parole. Infine, va incluso l'unico caso in cui la parola è formata da tre R.
In totale, i casi sono $400*3+20*3+1=1261$.
(Daniele Prisco)

Problema 13 [28]
$X$ appartiene alla perpendicolare di $BC$ passante per $A$. Inoltre $X$ e $2$ punti su $BC$ formano lo stesso angolo che forma $A$ con quei $2$ punti, di conseguenza $X$ è il simmetrico di $A$. Da questo deduciamo che $AX=2AD$, dove $AD$ è l'altezza.
(Alessandro Avellino)

Problema 14 [1011]
La quantità in questione, $(3n+2)^{5n+8}$, è un quadrato perfetto se lo è la base, cioè $3n+2$, oppure se l'esponente, $5n+8$, è pari. Tuttavia la prima possibilità non si verifica mai poiché un quadrato non è mai congruo a $2\,(\mathrm{mod}\,3)$. Dunque la risposta è semplicemente il numero di interi positivi $1\leq n\leq 2022$ tali che la quantità $5n+8$ sia pari, ovvero tutti e soli i numeri pari in questo intervallo, che sono $\frac{2022}{2}=1011$.
(Lorenzo Weiss)

Problema 15

Problema 16 [2019]
Il problema chiede per quanti interi positivi minori o uguali a $2022$ accade che $\lfloor \frac{\pi +n}{n}\rfloor =\lfloor \frac{\pi}{n}+1\rfloor =\lfloor \frac{\pi}{n}\rfloor +1=1\Longrightarrow \lfloor \frac{\pi}{n}\rfloor =0$, il che succede solo quando $\pi\approx 3,14\leq n$, i.e. in $2022-(4-1)=2019$ casi.
(Lorenzo Weiss)

Problema 17

Problema 18

Problema 19 [14]
Disegnando la circonferenza $x^2+y^2=5$ risulta evidente che gli unici punti a coordinate intere che vi appartengono, che sono $k=8$, sono quelli della forma $P_i(\pm 1,\pm 2)$, $P_i(\pm 2,\pm 1)$, $P_i(\pm 1,\mp 2)$, $P_i(\pm 2,\mp 1)$. In alternativa, è possibile risolvere l'equazione della stessa circonferenza sugli interi $x,y$. Da ciò si ricava l'ottagono $P_1P_2\dots P_8$, la cui area, come si vede facilmente, è $16-4\cdot\frac{1}{2}=14$.
(Lorenzo Weiss)

Problema 20[0059]
I primi 13 numeri che sono invidiati da tutti i precedenti in ordine crescente sono: 2,3,4,5,6,7,8,9, 19, 29, 39, 49, 59. La risposta è dunque 59.
(Mattia Zunino)

Problema 21 [210]
Notiamo che, comunque presi $6$ elementi distinti dall'insieme $X=\{1, 2, 3, 4, 5, 6, 7, 8, 9, 10\}$, c'è uno e un solo modo di disporli in ordine crescente, dunque la richiesta del problema è equivalente al trovare il numero di sottoinsiemi di $6$ elementi di $X$, che sono $\displaystyle\binom{10}{6}=210$.
(Valeria Martinelli)

Problema 22 [2022]
Si può ottenere $2$ solo quando una delle cifre è $2$ e le altre sono nulle, oppure quando due cifre sono uguali a $1$ e le altre sono nulle. Nel primo caso, la prima cifra sarà necessariamente $2$, altrimenti il numero non avrà $2022$ cifre: si ottiene solo una possibilità.
Nel secondo caso, invece, la prima cifra sarà $1$, mentre l'altro $1$ occuperà una delle altre $2021$ posizioni disponibili, ottenendo $2021$ possibilità.
Sommando le possibilità, si arriva a $2022$.
(Daniele Prisco)

Problema 23

Problema 24 [4042]
Se $x$ è la lunghezza del segmento $PA$ e $r$ è il raggio della circonferenza,
$OP=OA+PA=r+x=2021$; $PB=OA+OB+PA=r+r+x$. Di conseguenza, $PA+PB=x+r+r+x=2(r+x)=2×2021=4042$.
(Daniele Prisco)

Problema 25 [66]
Poniamo $a+b=x$ e $c+d=y$.
La disuguaglianza adesso è
$x+y\leq \sqrt{x^2+y^2} \Rightarrow x^2+2xy+y^2\leq x^2+y^2$
Da cui $2xy\leq0$ ma siccome $x$ e $y$ sono somme di interi maggiori o uguali a $0$, l'unico modo per cui $xy=0$ si ha quando uno dei $2$, o entrambi, è uguale a $0$. Se $c+d=y=0$ da cui $c=d=0$, quindi $(a,b,0,0)$. Ricordiamo che $a \geq b$ quindi i possibili valori sono $66$.
(Alessandro Avellino)

Problema 26 [6856]
Sia $n$ il numero di circonferenze sulla prima riga. Avendo le circonferenze diametro $2$, la base del rettangolo è uguale a $2n$. Invece, essendo tutte le circonferenze tangenti, i centri di quella più alta e delle due agli estremi della base sono disposte sui vertici di un triangolo equilatero, di lato $2n-2$. L'altezza del rettangolo è quindi uguale all'altezza del triangolo aumentata di $2$, ossia $\sqrt{3}(n-1)+2$.
Quindi vogliamo $4n+2\sqrt{3}(n-1)+4>2022$, che ci dà immediatamente $n=271$, quindi $k=\frac{271\cdot 272}{2}=36856$.
La risposta voluta sono le ultime quattro cifre di $k$.
(Valeria Martinelli)

Problema 27 [250]
Intanto notiamo che $78125$ è $5^7$.
La somma di $M$ termini consecutivi con primo termine $a$ è del tipo:
$a+(a+1)+(a+2)+...+(a+M-1)$ nel quale compaiono infatti $M$ termini consecutivi.
È quindi possibile raccogliere come $a \cdot M + (1+2+3+...+M-1)=a \cdot M+ \frac{M(M-1)}{2}=M(a+\frac{M-1}{2})=78125 \Rightarrow M(2a-1+M)=5^7\cdot2$, $M<2a-1+M$ quindi al massimo $M=2 \cdot 5^3$.
(Alessandro Avellino)

Problema 28

Problema 29

Problema 30

Problema 31 [20]
Il doppio delle cifre di $n$ dovrà necessariamente essere un numero di $3$ cifre ognuna delle quali minore di $5$. Una volta individuato il numero più piccolo $n$ come $101\,(2n=202)$ e il massimo $n$ come $444\,(2n=888)$ i restanti saranno compresi tra essi. Ci saranno $18$ numeri palindromi tra questi due estremi, con la cifra intermedia contente al più il $4$ e gli estremi uguali da $1$ a $4$. I numeri saranno $101$, $111$, $121$, $131$, $141$, $202$, $222$… fino a $444$. Il totale è di $20$ numeri bipalindromi.
(Irene Mancone)


Problema 32

Problema 33 [74]
Il costo di un pallone ($C$) può essere pari o dispari. Se è pari, può essere pagato con $\frac{C}{2}$ monete da $2$ orue. Poiché $\frac{C}{2}$ è pari, $C$ sarà un multiplo di $4$, quindi potrà assumere $37$ valori.
Se $C$ è dispari, invece, il pallone può essere pagato con $\frac{C-1}{2}$ monete da $2$ orue e una moneta da un orue. Di conseguenza, $C-1$ è un multiplo di $2$, ma $\frac{C-1}{2}$ è dispari (aggiungendo una moneta da un orue, il numero di monete usate è pari). Di conseguenza, $C-1$ è multiplo di $2$ ma non di $4$, quindi può assumere altri $37$ valori.
La somma dei valori possibili è dunque $74$.
(Daniele Prisco)

Problema 34 [2023]
Notiamo che possiamo riscrivere il testo nella seguente maniera: $P((x-y)(x+y))=P(x-y)P(x+y)$.
Notiamo anche che sostituendo $x-y \mapsto z$ e $x+y \mapsto w$ la coppia $(z,w)$ può assumere qualsiasi coppia di valori in $\mathbb{R}^2$ poiché il sistema
$\begin{cases}
x-y=z\\
x+y=w
\end{cases}$
ha soluzioni per ogni coppia $(z,w)$ fissata.
Dunque il nostro problema è equivalente a $P(zw)=P(z)P(y)$ per ogni $z$ e $w$ in $\mathbb{R}$. Questa relazione è molto simile ad una delle famose equazioni funzionali di Cauchy, $f(xy)=f(x)f(y)$. Quest'ultima equazione citata ha soluzione $f(x)=x^n$ o $f(x)=0$ (con $n$ naturale) se la funzione è continua (la continuità è una delle tante condizioni sufficienti), ma un polinomio come $P$ è continuo dunque le possibili soluzioni sono quelle riportate con $n$ che va da $0$ a $2021$ per $P(x)=x^n$ più $P(x)=0$. La soluzione è quindi $2023$ polinomi possibili.
(Lorenzo Bastioni)

Problema 35

Problema 36 [2020]
Si noti che $abc=2022-ab-a$, quindi per massimizzare il prodotto bisogna minimizzare $a$ e $b$. $a,b,c$ sono $\geq 1$ per ipotesi, dunque si pone $a=b=1$ dando come soluzione $c=2020$. Quindi $(a,b,c)=(1,1,2020)$ e $abc=2020$.
(Filippo Prandina)

Problema 37 [201]
Poniamo [math] e notiamo che [math] per [math]. Ora possiamo costruire il seguente polinomio: [math] di cui conosciamo gli zeri [math] e poiché [math] è chiaramente monico possiamo affermare che [math]. Sostituendo infine [math] con [math] e ricavando [math] otteniamo che [math] da cui possiamo trovare la soluzione al problema calcolando [math]. La risposta è quindi [math].
(Lorenzo Bastioni)

Problema 38

Problema 39

Problema 40[0108]
Poiché BC e B'C' sono allineati e B' e C sono coincidenti e i triangoli ABC e A'B'C' sono equilateri, l' angolo AB'A' è di 60 gradi. Siano L il lato del triangolo A'B'C' ed l il lato del triangolo ABC. L'area del triangolo AB'A' è uguale al prodotto 1/2 * AB' * A'B' * sin60 = 1/2 * l*L*sin60 = l*L*√3 /4 = 6√3 Quindi l*L = 24 . Il prodotto delle aree dei due triangoli equilateri è l*l*√3/4 *L*L*√3/4= 24*24*3/16 = 108.
(Mattia Zunino)

Problema 41

Problema 42

Problema 43 [4238]
Se si devono prendere almeno $n$ dolci per essere sicuri di prenderne almeno $1$ di un determinato tipo, questo vuol dire che ci sono $n-1$ dolci non di quel tipo. In particolare:
$\begin{cases}
S+B=55\\
B+C=70\\
S+C=91
\end{cases}$
Il sistema è di immediata risoluzione e porta come soluzione $(S,C,B)=(38,53,17)$ da cui $SCB=34238$. Sono richieste solo le ultime $4$ cifre.
(Filippo Prandina)

Problema 44 [4]
La configurazione è unica, infatti se prendiamo il punto $A_1$ e lo collegassimo con un punto diverso da $B_{2022}$, come per esempio $B_{2021}$, $B_{2022}$ sarà collegato con un punto diverso da $A_1$, per esempio $A_2$. Allora $A_1B_{2021}$ non interseca $A_2B_{2022}$. Iterando questo ragionamento ad ogni punto $A_i$, si ottiene un'unica configurazione: segmenti di estremi $A_iB_{2023-i}$. Quindi i coefficienti angolari saranno $\pm 2/(2023-2i)$. Per il prodotto, il numeratore è sempre $2$ mentre il denominatore è sempre dispari. Trovare quindi ciò che il problema richiede equivale a trovare le ultime $2$ cifre di $2^{2022}$.
(Alessandro Avellino)

Problema 45 [18]
Notiamo che, comunque presa una casella, il numero di giorni che Alberto o Barbara impiegano per raggiungerla non dipende dal percorso scelto, e le uniche caselle che hanno la stessa distanza sia da Alberto che da Barbara sono quelle sulla diagonale del quadrato diversa da quella di partenza, dunque i due si incontrano se e solo se entrambi passano per una di queste tre caselle.
Dato che un percorso è valido se e solo se fa $2$ passi in una direzione e $2$ nell'altra, tutte le coppie di percorsi possibili sono $\frac{4!}{2!\cdot 2!}\cdot \frac{4!}{2!\cdot 2!}=36$, quelli in cui entrambi passano per il centro sono $(2!\cdot 2!)(2!\cdot 2!)=16$, mentre quelli in cui entrambi passano in uno dei due angoli sono $2$.
Le coppie di percorsi con cui Alberto e Barbara non si incontrano sono quindi $36-16-2=18$.
(Valeria Martinelli)

Problema 46

Problema 47 [171]
Poiché $wxy+2021$ deve dare un numero pari, $wxy$ dev'essere dispari, e di conseguenza $w$,$x$ e $y$ devono esserlo.

$w=2a+1, x=2b+1, z=2c+1$ quindi $2a+1+2b+1+2c+1=37 \Rightarrow 2(a+b+c)=34 \Rightarrow a+b+c=17$
Adesso è un semplice stars and bars, infatti $a$, $b$ e $c$ possono essere anche nulli. Distribuire $17$ elementi in $3$ contenitori, $\binom{17+3-1}{3-1}=\binom{19}{2}$.
(Alessandro Avellino)

Problema 48 [42]
Notiamo che $CQB~APB$, in quanto hanno $2$ angoli congruenti, entrambi pari a $74$. Da questo consegue anche che sono triangoli isosceli. $\measuredangle CQB=\measuredangle APB=180-2(74)=32$. Inoltre $S$ sta nell'asse di $BC$, di conseguenza anche nella bisettrice di $CBQ$. Ragionamento analogo per $APB$. Ora osserviamo il quadrilatero $PCAQ$. Questo ha gli angoli $APB$ e $CQB$ congruenti, da questo possiamo asserire che $PCAQ$ è ciclico. Questo permette di stabilire che $\measuredangle DAC=\measuredangle PQC$ e che $\measuredangle DCA=\measuredangle QPD$. Infine osserviamo il triangolo $QSP$. $\measuredangle QSP=180-\measuredangle QPS-\measuredangle PQS= 180-(16+\measuredangle PQC)-(16+\measuredangle QPA)=148-(\measuredangle PQC+\measuredangle APQ)=148-(\measuredangle DCA+\measuredangle DAC)=148-(180-\measuredangle ADC)=\measuredangle ADC-32.$ $\measuredangle ADC=360-(\measuredangle DCB+\measuredangle CBA+\measuredangle BAD)=360-3 \cdot 74=138$ da cui $\measuredangle QSP=138-132=106$. Essendo infine $S$ il circocentro di $ABC$, ne consegue che $\measuredangle ASC=2 \measuredangle ABC=148$. Quindi $\measuredangle ASQ + \measuredangle PSC= \measuredangle ASC- \measuredangle QSP=148-106=42$.
(Alessandro Avellino)

Problema 49 [15]
Dovendo esistere almeno un numero $k>1$ che elevato al cubo sia un divisore di $n$ si possono escludere i numeri da $2$ a $7$ in quanto l’unico loro divisore al cubo è $1$ e $k$ non può assumere tale valore. L’$8$ si può vedere come $2^3$ ed è un prigioniero di cubi, così come lo sono tutti i suoi multipli fino a $96$. È valido lo stesso discorso anche per i multipli di $27$ (essendo $3^3$) fino a $81$. Contandoli si arriva al risultato richiesto, $15$.
(Irene Mancone)

Problema 50

Problema 51

Problema 52[1872]
Chiamiamo le tre radici di p(x) a,b,c dove a<b<c . Essendoa,b e c in progressione aritmetica si ha che a = b-r e c = b+r dove r è la ragione della progressione aritmetica. Per le formule di Vietè a+b+c= 39 b-r + b+ b+ r= 39 da cui si ricava che b =13 . Sempre applicando le formule di Vietè, ab + ac + bc = 482 sostituendo i risultati ricavati in precedenza si trova che 13(13-r) + (13-r)(13+r) + 13(13+r) = 482 da cui sviluppando le parentesi 169 -13r +169 -r2 + 169 +13r = 482 ottenendo che r2 = 25 quindi poiché r è maggiore di 0 si ha che r=5. Quindi a=8 c=18 .
Applicando nuovamente le formule di Vietè, -m = -8*13*18 ovvero m= 1872
(Mattia Zunino)

Problema 53 [24]
Poniamo l'età di Marco al momento della prima affermazione uguale a $x$, quindi l'età del padre Luca è $4x$. Allora, $4$ anni dopo l'età di Marco è $x+4$ e quella del padre è $4x+4$, quindi vale l'equazione $3(x+4)=4x+4$; $x=12-4=8$ e infine $4x=32$.
La differenza di età tra i $2$ è $24$, quindi per rispondere alla domanda risolviamo $2y=y+24$, con $y$ l'età di Marco e $y+24$ l'età di Luca, $y=24$ che è quindi la risposta.
(Niso Cicalò)

Problema 54

Problema 55

Problema 56

Problema 57 [183]
Si può considerare il triangolo isoscele con i vertici di coordinate $B (0;0), C (20;0)$ e A$ (10; y)$ con $y > 0$
Il punto $H$ è l’intersezione tra le rette perpendicolari ai lati passanti per $A, B, C$. Sapendo che le distanze tra
$AH$ e $AD$ sono segmenti di lunghezza intera il punto $H$ si potrà trovare al minimo nella coordinata $(10;1)$. Calcolando il coefficiente angolare tra $H$ e i punti $B$ e $C$, le rette passanti per $B$ e $C$ con coefficiente angolare antireciproco si può ottenere l’ordinata di $A$ sostituendo in una delle due equazioni.
$m_{BH}$= $\frac{1}{10}$ $m^1$=$-10$ retta r passante per A e C: $y=-10x+200$
$m_{CH}$=$\frac{-1}{10}$ $m^1$=10 retta s passante per B e A : $y=10x$

Per $H (10,1)$ $A$ vale $(10, 100)$ e la distanza è intera ($99$).
Una volta stabilita l’ordinata massima di $A$ è sufficiente ripetere il procedimento. Per $H (10,2)$ $A (10, 50)$, per $H (10,4)$ $A(10,25)$. Nelle coordinate $(10, 10)$ $H$ e $A$ si sovrappongono. Poi si avrà che $A$ si trova in $(10,20)$, in $(10,2)$ e $(10,5)$. La soluzione si ottiene sommando le distanze trovate tra $H$ e $A$ senza ripetizioni.
(Irene Mancone)

Soluzione alternativa:
Essendo $ABC$ un triangolo isoscele la mediana $AD$ coincide con l'altezza rispetto a $BC$, chiamando $K$ il piede dell'altezza relativa ad $AC$ otteniamo che il triangolo $BKC$ è simile al triangolo $ABD$ poiché $\widehat{BKC} \cong \widehat{BDA} = 90^{\circ}$ e $\widehat{BCK} \cong \widehat{ABD}$ (essendo $ABC$ isoscele). Consideriamo ora i triangoli $BKC$ e $BDH$, anch'essi sono simili poiché $\widehat{BKC} \cong \widehat{BDH} = 90^{\circ}$ e $\widehat{CBK}$ è in comune. Per la proprietà transitiva $ABD \sim BKC$ e $BKC \sim BDH$ $\rightarrow$ $ABD \sim BDH$. Avendo dimostrato che i triangoli $ABD$ e $BDH$ sono simili vale la seguente uguaglianza: $\frac{BD}{AD}=\frac{HD}{BD} \rightarrow BD^2 = AD \cdot HD \rightarrow 100=AD \cdot HD$ (essendo $BD$ la metà di un segmento lungo $20$). Avendo che $AH$ e $AD$ sono di lunghezza intera anche $HD$ sarà di lunghezza intera (Poiché $HD$ è la differenza tra i due segmenti) ed avremo soluzioni limitate a $100=AD \cdot HD$ ovvero $(AD,HD)=(100,1),(50,2),(25,4),(20,5),(10,10),(5,20),(4,25),(2,50),(1,100)$. La soluzione $(AD,HD)=(10,10)$ non ci piace poiché $\widehat{ABC}$ varrebbe $90^{\circ}$ ($D$ sarebbe il centro della circonferenza circoscritta ad $ABC$ e $\widehat{ABC}$ sottenderebbe un diametro) dunque i possibili valori di $AH$ sono le diverse differenze fra le soluzioni ottenute ($AH=|AD-AH|$): $AH = 100-1= 99$, $AH = 50-2 = 48$, $AH = 25-4= 21$, $AH =20-5=15$ (le altre danno gli stessi valori poiché sono soluzioni "specchiate"). La risposta è quindi $99+48+21+15=183$.
(Lorenzo Bastioni)

Problema 58 [46]
La somma $S$ dei quadrati di delle radici un polinomio del tipo $ x^n + \alpha_{n-1} \cdot x^{n-1} + ... + \alpha_1 \cdot x + \alpha_0$ è

$S=\alpha_{n-1}^2-2 \cdot \alpha_{n-2}$

$$Q_n(x)=x^n+\sum_{i=0}^{n-1} \sqrt{i} \cdot x^i$$

Per cui, definiamo $S_n$ come la somma dei quadrati delle radici di $Q_n(x)$. Allora $S_n=n-1-2\sqrt{n-2}$ che è razionale solo se anche $2\sqrt{n-2}$ lo è. Questo accade solo quando $n-2$ è un quadrato perfetto, quindi ci sono $45$ valori. A questo aggiungiamo $n=1$, il cui polinomio è quindi $x$, che ha come radici un numero razionale. Quindi la soluzione è $46$.
(Alessandro Avellino)

Problema 59[1559]
Notiamo che se S(p)| p , allora S(p) =1 oppure S(p)=P . La prima è impossibile in quanto affinché la somma delle cifre di un numero sia 1 è necessario che il numero sia una potenza di 10, quindi un numero composto. Rimane la seconda relazione che, per un intero maggiore di 9 è impossibile. Rimangono quindi i casi in cui p=2,3,5,7. Se p|n e n è minore o uguale a 2021, basta calcolare quanti sono gli interi divisibili per 2 o per 3 o per 5 o per 7. Applicando il principio di inclusione-esclusione, si ottiene che il risultato è [2021/2]+[2021/3]+[2021/5]+[2021/7]-([2021/6]+[2021/10]+[2021/14]+[2021/15]+[2021/21]+[2021/35]) + [2021/30]+[2021/70]+[2021/42]+[2021/105]-[2021/210]=1559 dove [x/y] indica la parte intera del rapporto fra x e y.
(Mattia Zunino)
Problema 60[0728]
Notiamo che se k e k+1 non contengono zeri, allora la relazione è sempre vera. Supponiamo che solo uno fra k o k+1 contenga uno zero. Se così fosse, allora N_0 (k) = k oppure N_0 (k+1) = k+1 vera se e solo se k o k+1 non contengono zeri, il che è assurdo. Se ci fosse più di uno zero la relazione non sussisterebbe perché, come nel caso precedente, il primo membro sarebbe minore del secondo. Il problema si riduce dunque a contare quanti sono gli interi k minori o uguali a 1000 tali che né k né k+1 contengano zeri. Se k avesse tre cifre, allora lo si potrebbe scegliere in 9*9*8 (9 modi di scegliere la cifra delle centinaia e delle decine, mentre solo 8 per quella delle unità) modi. Se k avesse 2 cifre, allora ci sarebbero 9*8 modi (9 modi per le decine e 8 per le unità). Se k ne avesse soltanto 1 allora solo 8 modi. 9*9*8+9*8+8= 728
(Mattia Zunino)
Problema 61[9098]
Per il criterio di divisibilità per 9, s(n) ≡ n mod 9 dove s(n) è la somma delle cifre di n. Iterando il ragionamento su s(n) si ottiene che A(n)≡n mod9 . Poiché A(n) è un intero compreso fra 0 e 9, A(n) è uguale al resto della divisione di n per 9. I possibili resti della divisione delle potenze di 2021 per 9 sono 5, 7, 8, 4, 2, 1 e poi si ripetono ciclicamente con periodo 6. Perciò la somma richiesta è 2016/6 *(5+7+8+4+2+1) +5+7+8+4+2 = 9098.
(Mattia Zunino)
Problema 62 [1158]
Dette $H$ e $K$, rispettivamente, le proiezioni di $D$ e $C$ su $AB$ e $AH=x$ possiamo calcolare l'altezza del trapezio in due modi diversi servendoci del teorema di Pitagora ed eguagliare le espressioni $DH^2=AD^2-AH^2=33^2-x^2=CK^2=BC^2-BK^2=60^2-(69-6-x)^2$.

Risolvendo l'equazione, si trova l'unico valore $x=\frac{81}{7}$ e l'altezza si può quindi calcolare, ad esempio, come $h=DH=\sqrt{AD^2-AH^2}=\frac{60\sqrt{13}}{7}$. L'area del trapezio vale $A=\frac{(b_1+b_2)\cdot h}{2}=\frac{(AB+CD)\cdot AH}{2}=\frac{(69+6)\cdot (60\sqrt13)}{7\cdot2}\approx 1158,93$; la sua parte intera è $1158$ ed è la soluzione.
(Federico Magnolfi)

Problema 63

Problema 64

Problema 65 [5877]
$X$ si trova sulla superficie del triangolo $AMC$: infatti, se così non fosse, $\widehat{ABC}$ e $\widehat{AXM}$ sarebbero due angoli congruenti che insistono su uno stesso segmento ($AM$) e che appartengono allo stesso semipiano rispetto ad esso: ma ciò non è possibile in quanto significherebbe che $X$ appartiene alla circonferenza circoscritta al trangolo $ABM$, alla quale ovviamente non appartengono punti interni al triangolo $ABM$.

$AXMY$ è ciclico per ipotesi e $\widehat{AYM}$ e $\widehat{AXM}$ sono supplementari in quanto angoli opposti (per quanto prima dimostrato); dunque $\widehat{BYM}$, essendo anch'esso supplementare di $\widehat{AYM}$, è congruente a $\widehat{AXM}$, il quale a sua volta è congruente a $\widehat{ABC}$ per ipotesi.
Avremo allora che il triangolo $MBY$ è isoscele su base $BY$; essendo $M$ punto medio di $BC$, $MB\cong MY\cong MC$ e ciò implica che $BYC$ è rettangolo in $Y$. $CY$ è dunque l'altezza uscente da $C$ e si può calcolare con la formula inversa $CY=\frac{2A}{AB}$ dopo aver ricavato l'area con la formula di Erone o per via trigonometrica.
Si trova che quest'ultima vale $A=1200\sqrt{5}$ e dunque $CY^2=\left(\frac{2A}{AB}\right)^2=\frac{4\cdot1200^2\cdot 5}{70^2}\approx 5877,55$. La sua parte intera è $5877$ ed è la soluzione.
(Federico Magnolfi)

Problema 66 [1155]
Prendiamo l'equazione del problema e consideriamola $(\text{mod}\,7)$: $x^2 \equiv 7y + z \,(\text{mod}\,7) \Rightarrow x^2 \equiv z \, (\text{mod}\,7)$. I quadrati come $x^2$ modulo $7$ possono assumere come valori solo $0,1,2,4$ quindi $z$ potrà assumere tutti e i soli valori $0,1,2,4$ modulo $7$. Nell'intervallo $[1,2022]$ ci sono esattamente $288$ numeri congrui a $0 \, (mod 7)$, $289$ congrui a $1$, $289$ congrui a $2$ e $289$ congrui a $4$. La risposta è quindi $288+289 \cdot 3=1155$.
(Lorenzo Bastioni)

Problema 67 [3026]
Scriviamo il numeratore come $\prod_{n=2}^{2021}((n-1)+n+(n+1))=\prod_{n=2}^{2021}(3n)=3^{2021}\prod_{n=2}^{2021}(n)$. Si può quindi raccogliere un $3^{2021}$ al numeratore. Inoltre si raccolgono anche una potenza di $3^{1005}$ dato che $n$ può avere $3$ come fattore primo (elevato alla $1,\,2,\,3,\,\dots,6$). Ci sono $ \Big \lfloor \frac{2021}{3} \Big \rfloor=673$ multipli di 3, $ \Big \lfloor \frac{2021}{9} \Big \rfloor =224$ multipli di $9$ e così via: $ \Big \lfloor \frac{2021}{3} \Big \rfloor+ \Big \lfloor \frac{2021}{9} \Big \rfloor + \Big \lfloor \frac{2021}{27} \Big \rfloor + \Big \lfloor \frac{2021}{81} \Big \rfloor + \Big \lfloor \frac{2021}{243} \Big \rfloor + \Big \lfloor \frac{2021}{729} \Big \rfloor =673+224+74+24+8+2=1005$. In tutto quindi si è raccolto $2021+1005=3026$.
(Filippo Prandina)

Problema 68 [1104]
Chiamiamo due insiemi $A$ e $B$ "gemelli di primo grado" se $|A \cap B|=1$ e $1 \in A,B$, in generale chiamiamo due insiemi $A$ e $B$ "gemelli di $n$-esimo grado" se $|A \cap B|=n$ e $n \in A,B$.
Possiamo ora affrontare il conteggio di tutte le coppie di sottoinsiemi gemelle di $\{1,2,3,...,2021,2022 \}$ considerando prima i gemelli di primo grado, di secondo grado fino a quelli di $2022$-esimo grado (Non ci sono coppie di sottoinsiemi di $2023$-esimo grado o di $0$-esimo grado).
Gemelli di primo grado:
se $A$ e $B$ sono di primo grado allora contengono entrambi l'elemento $1$ e non hanno nessun altro elemento in comune. Ci verra comodo per dopo scrivere che gli altri zero elementi in comune si possono scegliere in $\displaystyle \binom{2021}{0}$ modi. Ora tutti gli altri $2021$ elementi possono essere messi o in $A$ o in $B$ o in nessuno dei due (in questo caso la "o" è una o esclusiva), quindi abbiamo 3 possibilità per ognuno dei $2021$ elementi ovvero $3^{2021}$ possibilità. Avremo quindi $\displaystyle \binom{2021}{0} 3^{2021}$ coppie di sottoinsiemi gemelle di primo grado.
In generale siamo pronti ad affrontare il caso di gemelli di $n$-esimo grado:
se $A$ e $B$ sono gemelli di $n$-esimo grado allora contengono entrambi l'elemento $n$ e hanno altri $n-1$ elementi in comune. Gli altri $n-1$ elementi in comune si possono scegliere in $\displaystyle \binom{2021}{n-1}$ modi. E per i restanti $2021-(n-1)=2022-n$ elementi abbiamo $3^{2022-n}$ possibilità come spiegato precedentemente. Avremo quindi $\displaystyle \binom{2021}{n-1} 3^{2022-n}$ coppie di sottoinsiemi gemelle di $n$-esimo grado.
Sommando tutte le possibilità otteniamo:
$$\sum_{k=1}^{2022} \binom{2021}{k-1} 3^{2022-k}$$
Reimpostando la somma sostituendo $k-1 \mapsto k$ otteniamo:
$$\sum_{k=0}^{2021} \binom{2021}{k} 3^{2021-k}$$
Ma questo è esattamente uguale a $(3+1)^{2021}=4^{2021}$ (Per il binomio di Newton o teorema binomiale). Riducendo l'enorme numero $(\text{mod}\,10000)$ si trova che le ultime $4$ cifre che ci interessano sono $1104$, da cui la risposta.
(Lorenzo Bastioni)

Problema 69

Problema 70
Rispondi